You are on page 1of 1136

https://www.edutvonline.

com

Cambridge Assessment International Education


Cambridge International General Certificate of Secondary Education
* 9 9 2 6 6 0 5 7 6 7 *

MATHEMATICS 0580/42
Paper 4 (Extended) February/March 2019
2 hours 30 minutes
Candidates answer on the Question Paper.
Additional Materials: Electronic calculator Geometrical instruments
Tracing paper (optional)

READ THESE INSTRUCTIONS FIRST

Write your centre number, candidate number and name on all the work you hand in.
Write in dark blue or black pen.
You may use an HB pencil for any diagrams or graphs.
Do not use staples, paper clips, glue or correction fluid.
DO NOT WRITE IN ANY BARCODES.

Answer all questions.


If working is needed for any question it must be shown below that question.
Electronic calculators should be used.
If the degree of accuracy is not specified in the question, and if the answer is not exact, give the answer to
three significant figures. Give answers in degrees to one decimal place.
For r, use either your calculator value or 3.142.

At the end of the examination, fasten all your work securely together.
The number of marks is given in brackets [ ] at the end of each question or part question.
The total of the marks for this paper is 130.

This document consists of 19 printed pages and 1 blank page.

DC (ST/SW) 164545/2
© UCLES 2019 [Turn over
2

1 Amol and Priya deliver 645 parcels in the ratio Amol : Priya = 11 : 4.

(a) Calculate the number of parcels Amol delivers.

................................................... [2]

(b) Amol drives his truck at an average speed of 50 km/h.


He leaves at 07 00 and arrives at 11 15.

Calculate the distance he drives.

............................................. km [2]

(c) Priya drives her van a distance of 54 km.


She leaves at 10 55 and arrives at 12 38.

Calculate her average speed.

.......................................... km/h [3]

(d) Priya has 50 identical parcels.


Each parcel has a mass of 17 kg, correct to the nearest kilogram.

Find the upper bound for the total mass of the 50 parcels.

.............................................. kg [1]

© UCLES 2019 0580/42/F/M/19


3

(e) 67 of the 645 parcels are damaged on the journey.

Calculate the percentage of parcels that are damaged.

............................................... % [1]

(f) (i) 29 parcels each have a value of $68.

By writing each of these numbers correct to 1 significant figure, find an estimate for the total
value of these 29 parcels.

$ .................................................. [1]

(ii) Without doing any calculation, complete this statement.

The actual total value of these 29 parcels is less than the answer to part (f)(i)

because ...................................................................................................................................... [1]

© UCLES 2019 0580/42/F/M/19 [Turn over


4

8
7
6
5
4
A B
3
2
1

–8 –7 –6 –5 –4 –3 –2 –1 0 1 2 3 4 5 6 7 x
–1
–2

C –3
–4
–5

(a) Describe fully the single transformation that maps

(i) triangle A onto triangle B,

....................................................................................................................................................

.................................................................................................................................................... [2]

(ii) triangle A onto triangle C.

....................................................................................................................................................

.................................................................................................................................................... [3]

(b) On the grid, draw the image of


1
(i) triangle A after an enlargement, scale factor - , centre (3, 0), [2]
2

-3
(ii) triangle A after a translation by the vector e o , [2]
1

0 1
(iii) triangle A after the transformation that is represented by the matrix e o.
1 0

[3]
© UCLES 2019 0580/42/F/M/19
5

3 Sushila, Ravi and Talika each have a bag of balls.


Each of the bags contains 10 red balls and 8 blue balls.

(a) Sushila takes one ball at random from her bag.

Find the probability that she takes a red ball.

................................................... [1]

(b) Ravi takes two balls at random from his bag, without replacement.

Find the probability that one ball is red and one ball is blue.

................................................... [3]

(c) Talika takes three balls at random from her bag, without replacement.

Calculate the probability that the three balls are the same colour.

................................................... [4]

© UCLES 2019 0580/42/F/M/19 [Turn over


6

4 The diagram shows an incomplete scale drawing of a market place, ABCD, where D is on CX.
The scale is 1 centimetre represents 5 metres.

A B Scale : 1 cm to 5 m

D lies on CX such that angle DAB = 75°.

(a) On the diagram, draw the line AD and mark the position of D. [2]

(b) Find the actual length of the side BC of the market place.

............................................... m [2]

© UCLES 2019 0580/42/F/M/19


7

(c) In this part, use a ruler and compasses only.

Street sellers are allowed in the part of the market place that is

• more than 35 metres from A


and
• nearer to C than to B
and
• nearer to CD than to BC.

On the diagram, construct and shade the region where street sellers are allowed. [7]

(d) Write the scale of the drawing in the form 1 : n.

1 : .................................................. [1]

© UCLES 2019 0580/42/F/M/19 [Turn over


8

3 3
5 The table shows some values for y = x - 2x for - 3 G x G 3.
10

x −3 −2 −1.5 −1 0 1 1.5 2 3
y 2.0 1.7 0 −2.0 −1.6

(a) Complete the table. [3]


3 3
(b) On the grid, draw the graph of y = x - 2x for - 3 G x G 3.
10

–3 –2 –1 0 1 2 3 x

–1

–2

[4]

© UCLES 2019 0580/42/F/M/19


9

3 3 1
(c) On the grid opposite, draw a suitable straight line to solve the equation x - 2x = (1 - x) for
10 2
- 3 G x G 3.

x = ....................... or x = ....................... or x = ....................... [4]


3 3
(d) For - 3 G x G 3, the equation x - 2x = 1 has n solutions.
10
Write down the value of n.

n = .................................................. [1]

© UCLES 2019 0580/42/F/M/19 [Turn over


10

6
C B

NOT TO
SCALE

1.5 cm

D E

0.25 cm

A
O 0.5 cm

The diagram shows a company logo made from a rectangle and a major sector of a circle.
The circle has centre O and radius OA.
OA = OD = 0.5 cm and AB = 1.5 cm.
E is a point on OC such that OE = 0.25 cm and angle OED = 90°.

(a) Calculate the perimeter of the logo.

............................................. cm [5]

© UCLES 2019 0580/42/F/M/19


11

(b) Calculate the area of the logo.

............................................ cm2 [3]

(c) A mathematically similar logo is drawn.


The area of this logo is 77.44 cm2.

(i) Calculate the radius of the major sector in this logo.

............................................. cm [3]

(ii) A gold model is made.


This model is a prism with a cross-section of area 77.44 cm2.

This gold model is 15 mm thick.


One cubic centimetre of gold has a mass of 19 grams.

Calculate the mass of the gold model in kilograms.

.............................................. kg [3]

© UCLES 2019 0580/42/F/M/19 [Turn over


12

7 (a) 20 students each record the mass, p grams, of their pencil case.
The table below shows the results.

Mass
0 1 p G 50 50 1 p G 100 100 1 p G 125 125 1 p G 150 150 1 p G 200
( p grams)
Frequency 2 5 4 6 3

(i) Calculate an estimate of the mean mass.

................................................ g [4]

(ii) Use the frequency table above to complete the cumulative frequency table.

Mass
p G 50 p G 100 p G 125 p G 150 p G 200
( p grams)
Cumulative
20
frequency
[2]

(iii) A student is chosen at random.

Find the probability that this student has a pencil case with a mass greater than 150 g.

................................................... [1]

(b) Some students each record the mass, m kg, of their school bag.
Adil wants to draw a histogram to show this information.

Complete the table below.

Mass (m kg) 01mG4 41mG6 61mG7 7 1 m G 10

Frequency 32 42

Height of bar on
1.6 2 1.2 2.8
histogram (cm)
[2]

© UCLES 2019 0580/42/F/M/19


13

(c) The frequency table below shows information about the number of books read by some students in a
reading marathon.

Number of
1 2 3 4 5 6 7 8
books read

Frequency 2 2 16 10 9 4 x 2

(i) The mean number of books read is 4.28 .

Find the value of x.

x = .................................................. [3]

(ii) Write down the mode.

................................................... [1]

(iii) Write down the median.

................................................... [1]

© UCLES 2019 0580/42/F/M/19 [Turn over


14

3
8 f (x) = , x !-2 g (x) = 8x - 5 h (x) = x 2 + 6
x+2

(a) Work out g e o .


1
4

................................................... [1]

(b) Work out ff(2).

................................................... [2]

(c) Find gg(x), giving your answer in its simplest form.

................................................... [2]

(d) Find g -1 (x) .

g -1 (x) = .................................................. [2]

(e) Write g (x) - f (x) as a single fraction in its simplest form.

................................................... [3]

© UCLES 2019 0580/42/F/M/19


15

(f) (i) Show that hg(x) = 19 simplifies to 16x 2 - 20x + 3 = 0 .

[3]

(ii) Use the quadratic formula to solve 16x 2 - 20x + 3 = 0 .


Show all your working and give your answers correct to 2 decimal places.

x = ......................... or x = ......................... [4]

© UCLES 2019 0580/42/F/M/19 [Turn over


16

9 (a) The Venn diagram shows two sets, A and B.

A h B
c
f
m
d
g k
e j

q p

(i) Use set notation to complete the statements.

(a) d ................... A [1]

(b) { f , g} = ............................... [1]

(ii) Complete the statement.

n (.........................) = 6 [1]

(b) In the Venn diagram below, shade C + D l .

C D

[1]

© UCLES 2019 0580/42/F/M/19


17

(c) 50 students study at least one of the subjects geography (G ), mathematics (M ) and history (H ).

18 study only mathematics.


19 study two or three of these subjects.
23 study geography.

The Venn diagram below is to be used to show this information.

G M

x
......... .........
7
x x

......... H

(i) Show that x = 4.

[2]

(ii) Complete the Venn diagram. [2]

(iii) Use set notation to complete this statement.

(G , M , H )l = ....................... [1]

(iv) Find n(G + (M , H )) .

................................................... [1]

© UCLES 2019 0580/42/F/M/19 [Turn over


18

10 (a) Solve the simultaneous equations.


You must show all your working.
6x + 5y = 27
5x - 3y = 44

x = ..................................................

y = .................................................. [4]

(b) y is inversely proportional to (x + 3) 2 .


When x = 2, y = 8.

Find y when x = 7.

y = .................................................. [3]

(c) Solve the inequality.


3 (x - 2) 1 7 (x + 2)

................................................... [3]

© UCLES 2019 0580/42/F/M/19


19

11 (a) The table shows the first five terms of sequence A and sequence B.

Term 1 2 3 4 5 6

Sequence A 7 13 23 37 55

Sequence B 1 3 9 27 81

(i) Complete the table for the 6th term of each sequence. [2]

(ii) Find the nth term of

(a) sequence A,

................................................... [2]

(b) sequence B.

................................................... [2]

(b) The nth term of another sequence is 4n 2 + n + 3.

Find

(i) the 2nd term,

................................................... [1]

(ii) the value of n when the nth term is 498.

n = .................................................. [3]

© UCLES 2019 0580/42/F/M/19


20

BLANK PAGE

Permission to reproduce items where third-party owned material protected by copyright is included has been sought and cleared where possible. Every
reasonable effort has been made by the publisher (UCLES) to trace copyright holders, but if any items requiring clearance have unwittingly been included, the
publisher will be pleased to make amends at the earliest possible opportunity.

To avoid the issue of disclosure of answer-related information to candidates, all copyright acknowledgements are reproduced online in the Cambridge
Assessment International Education Copyright Acknowledgements Booklet. This is produced for each series of examinations and is freely available to download
at www.cambridgeinternational.org after the live examination series.

Cambridge Assessment International Education is part of the Cambridge Assessment Group. Cambridge Assessment is the brand name of the University of
Cambridge Local Examinations Syndicate (UCLES), which itself is a department of the University of Cambridge.

© UCLES 2019 0580/42/F/M/19


https://www.edutvonline.com

Cambridge Assessment International Education


Cambridge International General Certificate of Secondary Education
* 1 4 0 2 5 2 8 9 1 3 *

MATHEMATICS 0580/41
Paper 4 (Extended) May/June 2019
2 hours 30 minutes
Candidates answer on the Question Paper.
Additional Materials: Electronic calculator Geometrical instruments
Tracing paper (optional)

READ THESE INSTRUCTIONS FIRST

Write your centre number, candidate number and name on all the work you hand in.
Write in dark blue or black pen.
You may use an HB pencil for any diagrams or graphs.
Do not use staples, paper clips, glue or correction fluid.
DO NOT WRITE IN ANY BARCODES.

Answer all questions.


If working is needed for any question it must be shown below that question.
Electronic calculators should be used.
If the degree of accuracy is not specified in the question, and if the answer is not exact, give the answer to
three significant figures. Give answers in degrees to one decimal place.
For r, use either your calculator value or 3.142.

At the end of the examination, fasten all your work securely together.
The number of marks is given in brackets [ ] at the end of each question or part question.
The total of the marks for this paper is 130.

This document consists of 16 printed pages.

DC (JP/SW) 164544/2
© UCLES 2019 [Turn over
https://www.edutvonline.com
2

1
y

10

3
T
2

0
1 2 3 4 5 6 7 8 9 10 x

-1
(a) (i) Translate shape T by the vector c m.
6
Label the image A. [2]

(ii) Rotate shape T about the point (5, 3) through 180°.


Label the image B. [2]

(iii) Describe fully the single transformation that maps shape A onto shape B.

....................................................................................................................................................

.................................................................................................................................................... [3]

(b) (i) Reflect shape T in the line y = x. [2]

(ii) Find the matrix that represents the transformation in part (b)(i).

f p [2]

© UCLES 2019 0580/41/M/J/19


https://www.edutvonline.com
3

2 The table shows some values for y = x 3 + 3x 2 + 2 .

x -3.5 -3 -2.5 -2 -1.5 -1 -0.5 0 0.5 1 1.5

y -4.1 5.1 6 5.4 4 2.6 2.9 12.1

(a) Complete the table. [3]

(b) On the grid, draw the graph of y = x 3 + 3x 2 + 2 for - 3.5 G x G 1.5 .

15

10

– 3.5 –3 – 2.5 –2 – 1.5 –1 – 0.5 0 0.5 1 1.5 x

–5

[4]

(c) Use your graph to solve the equation x 3 + 3x 2 + 2 = 0 for - 3.5 G x G 1.5 .

x = .................................................... [1]

(d) By drawing a suitable straight line, solve the equation x 3 + 3x 2 + 2x + 2 = 0 for - 3.5 G x G 1.5 .

x = .................................................... [2]

(e) For - 3.5 G x G 1.5 , the equation x 3 + 3x 2 + 2 = k has three solutions and k is an integer.

Write down a possible value of k.

k = .................................................... [1]

© UCLES 2019 0580/41/M/J/19 [Turn over


https://www.edutvonline.com
4

C North

170 m
120 m NOT TO
150 m SCALE

50 m

A
100 m
B

The diagram shows a field ABCDE.

(a) Calculate the perimeter of the field ABCDE.

................................................ m [4]

(b) Calculate angle ABD.

Angle ABD =  .......................................................... [4]


© UCLES 2019 0580/41/M/J/19
https://www.edutvonline.com
5

(c) (i) Calculate angle CBD.

Angle CBD =  .................................................... [2]

(ii) The point C is due north of the point B.

Find the bearing of D from B.

.................................................... [2]

(d) Calculate the area of the field ABCDE.


Give your answer in hectares.
[1 hectare = 10 000 m2]

...................................... hectares [4]

© UCLES 2019 0580/41/M/J/19 [Turn over


https://www.edutvonline.com
6

4 (a) The test scores of 14 students are shown below.

21 21 23 26 25 21 22 20 21 23 23 27 24 21

(i) Find the range, mode, median and mean of the test scores.

Range = ....................................................

Mode = ....................................................

Median = ....................................................

Mean = .................................................... [6]

(ii) A student is chosen at random.

Find the probability that this student has a test score of more than 24.

.................................................... [1]

(b) Petra records the score in each test she takes.

The mean of the first n scores is x.


The mean of the first (n – 1) scores is (x + 1).

Find the nth score in terms of n and x.


Give your answer in its simplest form.

.................................................... [3]

© UCLES 2019 0580/41/M/J/19


https://www.edutvonline.com
7

(c) During one year the midday temperatures, t°C, in Zedford were recorded.
The table shows the results.

Temperature (t°C) 0 1 t G 10 10 1 t G 15 15 1 t G 20 20 1 t G 25 25 1 t G 35
Number of days 50 85 100 120 10

(i) Calculate an estimate of the mean.

............................................... °C [4]

(ii) Complete the histogram to show the information in the table.

25

20

15
Frequency
density

10

0
0 5 10 15 20 25 30 35 t
Temperature (°C)
[4]

© UCLES 2019 0580/41/M/J/19 [Turn over


https://www.edutvonline.com
8

NOT TO
1.2 m SCALE

3m

The diagram shows the surface of a garden pond, made from a rectangle and two semicircles.
The rectangle measures 3 m by 1.2 m.

(a) Calculate the area of this surface.

............................................... m2 [3]

(b) The pond is a prism and the water in the pond has a depth of 20 cm.

Calculate the number of litres of water in the pond.

........................................... litres [3]

(c) After a rainfall, the number of litres of water in the pond is 1007.

Calculate the increase in the depth of water in the pond.


Give your answer in centimetres.

.............................................. cm [3]

© UCLES 2019 0580/41/M/J/19


https://www.edutvonline.com
9

6  = {students in a school}
F = {students who play football}
B = {students who play baseball}

There are 240 students in the school.

• 120 students play football


• 40 students play baseball
• 90 students play football but not baseball.

(a) Complete the Venn diagram to show this information.

 F B

.......... .......... ..........

..........

[2]

(b) Find n ^F l + Blh .

.................................................... [1]

(c) A student in the school is chosen at random.

Find the probability that this student plays baseball but not football.

.................................................... [1]

(d) Two students who play baseball are chosen at random.

Find the probability that they both also play football.

.................................................... [3]

© UCLES 2019 0580/41/M/J/19 [Turn over


https://www.edutvonline.com
10

1
7 (a) s = ut + at 2
2
(i) Find s when t = 26.5, u = 104.3 and a = -2.2 .
Give your answer in standard form, correct to 4 significant figures.

s =  .................................................... [4]

(ii) Rearrange the formula to write a in terms of u, t and s.

a =  ................................................... [3]

© UCLES 2019 0580/41/M/J/19


https://www.edutvonline.com
11

(b)

NOT TO
SCALE
(x – 1) cm
(x – 2) cm

(2x + 3) cm (x + 1) cm

The difference between the areas of the two rectangles is 62 cm2.

(i) Show that x 2 + 2x - 63 = 0 .

[3]

(ii) Factorise x 2 + 2x - 63 .

.................................................... [2]

(iii) Solve the equation x 2 + 2x - 63 = 0 to find the difference between the perimeters of the two
rectangles.

.............................................. cm [2]

© UCLES 2019 0580/41/M/J/19 [Turn over


https://www.edutvonline.com
12

8 (a) The price of a book increases from $2.50 to $2.65 .

Calculate the percentage increase.

............................................... % [3]

(b) Scott invests $500 for 7 years at a rate of 1.5% per year simple interest.

Calculate the value of his investment at the end of the 7 years.

$ .................................................... [3]

(c) In a city the population is increasing exponentially at a rate of 1.6% per year.

Find the overall percentage increase at the end of 20 years.

............................................... % [2]

(d) The population of a village is 6400.


The population is decreasing exponentially at a rate of r% per year.
After 22 years, the population will be 2607.

Find the value of r.

r = .................................................... [3]

© UCLES 2019 0580/41/M/J/19


https://www.edutvonline.com
13

9 f (x) = 7x - 2 g (x) = x 2 + 1 h (x) = 3 x

(a) Find gh(2).

.................................................... [2]

(b) Find f – 1(x).

f – 1(x) = .................................................... [2]

(c) gg (x) = ax 4 + bx 2 + c

Find the values of a, b and c.

a = ....................................................

b = ....................................................

c = .................................................... [3]

(d) Find x when hf(x) = 81.

x = .................................................... [3]

© UCLES 2019 0580/41/M/J/19 [Turn over


https://www.edutvonline.com
14

10 The volume of each of the following solids is 1000 cm3.

Calculate the value of x for each solid.

(a) A cube with side length x cm.

x =  .................................................... [1]

(b) A sphere with radius x cm.


4
[The volume, V, of a sphere with radius r is V = rr 3. ]
3

x =  ..........................................................   [3]

(c)

NOT TO
x 5 cm SCALE

x cm

A cone with radius x cm and slant height x 5cm.


1
[The volume, V, of a cone with radius r and height h is V = rr 2 h.]
3

x = .................................................... [4]

© UCLES 2019 0580/41/M/J/19


https://www.edutvonline.com
15

(d)

x NOT TO
cm
2 SCALE

x cm

27x
cm
2

A prism with a right-angled triangle as its cross-section.

x = .................................................... [4]

Question 11 is printed on the next page.

© UCLES 2019 0580/41/M/J/19 [Turn over


https://www.edutvonline.com
16

11 Brad travelled from his home in New York to Chamonix.

• He left his home at 16 30 and travelled by taxi to the airport in New York.
This journey took 55 minutes and had an average speed of 18 km/h.

• He then travelled by plane to Geneva, departing from New York at 22 15.


The flight path can be taken as an arc of a circle of radius 6400 km with a sector angle of 55.5°.
The local time in Geneva is 6 hours ahead of the local time in New York.
Brad arrived in Geneva at 11 25 the next day.

• To complete his journey, Brad travelled by bus from Geneva to Chamonix.


This journey started at 13 00 and took 1 hour 36 minutes.
The average speed was 65 km/h.
The local time in Chamonix is the same as the local time in Geneva.

Find the overall average speed of Brad’s journey from his home in New York to Chamonix.
Show all your working and give your answer in km/h.

.......................................... km/h [11]

Permission to reproduce items where third-party owned material protected by copyright is included has been sought and cleared where possible. Every
reasonable effort has been made by the publisher (UCLES) to trace copyright holders, but if any items requiring clearance have unwittingly been included, the
publisher will be pleased to make amends at the earliest possible opportunity.

To avoid the issue of disclosure of answer-related information to candidates, all copyright acknowledgements are reproduced online in the Cambridge
Assessment International Education Copyright Acknowledgements Booklet. This is produced for each series of examinations and is freely available to download
at www.cambridgeinternational.org after the live examination series.

Cambridge Assessment International Education is part of the Cambridge Assessment Group. Cambridge Assessment is the brand name of the University of
Cambridge Local Examinations Syndicate (UCLES), which itself is a department of the University of Cambridge.

© UCLES 2019 0580/41/M/J/19


https://www.edutvonline.com

Cambridge Assessment International Education


Cambridge International General Certificate of Secondary Education
* 0 1 4 1 1 5 9 4 2 8 *

MATHEMATICS 0580/42
Paper 4 (Extended) May/June 2019
2 hours 30 minutes
Candidates answer on the Question Paper.
Additional Materials: Electronic calculator Geometrical instruments
Tracing paper (optional)

READ THESE INSTRUCTIONS FIRST

Write your centre number, candidate number and name on all the work you hand in.
Write in dark blue or black pen.
You may use an HB pencil for any diagrams or graphs.
Do not use staples, paper clips, glue or correction fluid.
DO NOT WRITE IN ANY BARCODES.

Answer all questions.


If working is needed for any question it must be shown below that question.
Electronic calculators should be used.
If the degree of accuracy is not specified in the question, and if the answer is not exact, give the answer to
three significant figures. Give answers in degrees to one decimal place.
For π, use either your calculator value or 3.142.

At the end of the examination, fasten all your work securely together.
The number of marks is given in brackets [ ] at the end of each question or part question.
The total of the marks for this paper is 130.

This document consists of 19 printed pages and 1 blank page.

DC (SC/TP) 192445/3
© UCLES 2019 [Turn over
https://www.edutvonline.com
2

1 (a) The price of a newspaper increased from $0.97 to $1.13 .

Calculate the percentage increase.

........................................... % [3]

(b) One day, the newspaper had 60 pages of news and advertisements.
The ratio number of pages of news : number of pages of advertisements = 5 : 7.

(i) Calculate the number of pages of advertisements.

............................................... [2]

(ii) Write the number of pages of advertisements as a percentage of the number of pages of news.

........................................... % [1]

(c) On holiday Maria paid 2.25 euros for the newspaper when the exchange rate was $1 = 0.9416 euros.
At home Maria paid $1.13 for the newspaper.

Calculate the difference in price.


Give your answer in dollars, correct to the nearest cent.

$ .............................................. [3]

© UCLES 2019 0580/42/M/J/19


https://www.edutvonline.com
3

(d) The number of newspapers sold decreases exponentially by x% each year.


Over a period of 21 years the number of newspapers sold decreases from 1 763 000 to 58 000.

Calculate the value of x.

x = .............................................. [3]

(e) Every page of the newspaper is a rectangle measuring 43 cm by 28 cm, both correct to the nearest
centimetre.

Calculate the upper bound of the area of a page.

........................................ cm2 [2]

© UCLES 2019 0580/42/M/J/19 [Turn over


https://www.edutvonline.com
4

2 (a)
A C
26°
NOT TO
SCALE

F B x° D

AC is parallel to FBD, ABC is an isosceles triangle and CBE is a straight line.

Find the value of x.

x = .............................................. [3]

(b)
S
P

58°
17° T NOT TO
SCALE

The diagram shows a circle with diameter PQ.


SPT is a tangent to the circle at P.

Find the value of y.

y = .............................................. [5]

© UCLES 2019 0580/42/M/J/19


https://www.edutvonline.com
5

3 The probability that Andrei cycles to school is r.

(a) Write down, in terms of r, the probability that Andrei does not cycle to school.

............................................... [1]

(b) The probability that Benoit does not cycle to school is 1.3 - r.
The probability that both Andrei and Benoit do not cycle to school is 0.4 .

(i) Complete the equation in terms of r.

(.........................) # (.........................) = 0.4 [1]

(ii) Show that this equation simplifies to 10r 2 - 23r + 9 = 0 .

[3]

(iii) Solve by factorisation 10r 2 - 23r + 9 = 0 .

r = ................... or r = ................... [3]

(iv) Find the probability that Benoit does not cycle to school.

............................................... [1]

© UCLES 2019 0580/42/M/J/19 [Turn over


https://www.edutvonline.com
6

4 (a) The equation of a straight line is 2y = 3x + 4 .

(i) Find the gradient of this line.

............................................... [1]

(ii) Find the co-ordinates of the point where the line crosses the y-axis.

( ..................... , ..................... ) [1]

(b) The diagram shows a straight line L.

–2 0 2 4 6 x

L
–2

(i) Find the equation of line L.

............................................... [3]

(ii) Find the equation of the line perpendicular to line L that passes through (9, 3).

............................................... [3]

© UCLES 2019 0580/42/M/J/19


https://www.edutvonline.com
7

(c) A is the point (8, 5) and B is the point (- 4, 1).

(i) Calculate the length of AB.

............................................... [3]

(ii) Find the co-ordinates of the midpoint of AB.

( ..................... , ..................... ) [2]

© UCLES 2019 0580/42/M/J/19 [Turn over


https://www.edutvonline.com
8

1 x
5 The table shows some values of y = - for 0.15 G x G 3.5 .
2x 4

x 0.15 0.2 0.5 1 1.5 2 2.5 3 3.5

y 3.30 0.88 - 0.04 - 0.43 - 0.58 - 0.73

(a) Complete the table. [3]


1 x
(b) On the grid, draw the graph of y = - for 0.15 G x G 3.5 .
2x 4
The last two points have been plotted for you.

3.5

3.0

2.5

2.0

1.5

1.0

0.5

0
0.5 1.0 1.5 2.0 2.5 3.0 3.5 x

– 0.5

– 1.0
[4]
© UCLES 2019 0580/42/M/J/19
https://www.edutvonline.com
9

1 x 1
(c) Use your graph to solve the equation - = for 0.15 G x G 3.5 .
2x 4 2

x = .............................................. [1]

(d) (i) On the grid, draw the line y = 2 - x . [2]

(ii) Write down the x co-ordinates of the points where the line y = 2 - x crosses the graph of
1 x
y= - for 0.15 G x G 3.5 .
2x 4

x = .................... and x = .................... [2]


1 x
(e) Show that the graph of y = - can be used to find the value of 2 for 0.15 G x G 3.5 .
2x 4

[2]

© UCLES 2019 0580/42/M/J/19 [Turn over


https://www.edutvonline.com
10

6 (a) Expand and simplify.


(x + 7) (x - 3)

............................................... [2]

(b) Factorise completely.

(i) 15p 2 q 2 - 25q 3

............................................... [2]

(ii) 4fg + 6gh + 10fk + 15hk

............................................... [2]

(iii) 81k 2 - m 2

............................................... [2]

(c) Solve the equation.


x+2
3 (x - 4) + =6
5

x = .............................................. [4]

© UCLES 2019 0580/42/M/J/19


https://www.edutvonline.com
11

7 (a) Show that each interior angle of a regular pentagon is 108°.

[2]

(b)
D

E C NOT TO
O SCALE
M

X
A B

The diagram shows a regular pentagon ABCDE.


The vertices of the pentagon lie on a circle, centre O, radius 12 cm.
M is the midpoint of BC.

(i) Find BM.

BM = ........................................ cm [3]

(ii) OMX and ABX are straight lines.

(a) Find BX.

BX = ........................................ cm [3]

(b) Calculate the area of triangle AOX.

........................................ cm2 [3]

© UCLES 2019 0580/42/M/J/19 [Turn over


https://www.edutvonline.com
12

8 (a)
D

64° 53°

16.5 cm
NOT TO
SCALE
12.4 cm
A

95°
B

The diagram shows two triangles ABD and BCD.


AD = 16.5 cm and BD = 12.4 cm.
Angle ADB = 64°, angle BDC = 53° and angle DBC = 95°.

(i) Find AB.

AB = ........................................ cm [4]

(ii) Find BC.

BC = ........................................ cm [4]

© UCLES 2019 0580/42/M/J/19


https://www.edutvonline.com
13

(b)

3.8 cm
NOT TO
SCALE

7.7 cm

The diagram shows a sector of a circle of radius 3.8 cm.


The arc length is 7.7 cm.

(i) Calculate the value of y.

y = .............................................. [2]

(ii) Calculate the area of the sector.

........................................ cm2 [2]

© UCLES 2019 0580/42/M/J/19 [Turn over


https://www.edutvonline.com
14

9 100 students were each asked how much money, $m, they spent in one week.
The frequency table shows the results.

Amount ($m) 01mG5 5 1 m G 10 10 1 m G 20 20 1 m G 30 30 1 m G 50

Frequency 16 38 30 9 7

(a) Calculate an estimate of the mean.

$ .............................................. [4]

(b) Complete the cumulative frequency table below.

Amount ($m) mG5 m G 10 m G 20 m G 30 m G 50

Cumulative
16 100
frequency
[2]

© UCLES 2019 0580/42/M/J/19


https://www.edutvonline.com
15

(c) On the grid, draw the cumulative frequency diagram.

100

80

60
Cumulative
frequency

40

20

0
0 10 20 30 40 50 m
Amount ($)
[3]

(d) Use your cumulative frequency diagram to find an estimate for

(i) the median,


$ .............................................. [1]

(ii) the interquartile range,

$ .............................................. [2]

(iii) the number of students who spent more than $25.

............................................... [2]

© UCLES 2019 0580/42/M/J/19 [Turn over


https://www.edutvonline.com
16

10 (a) The volume of a solid metal sphere is 24 430 cm3.

(i) Calculate the radius of the sphere.


4
[The volume, V, of a sphere with radius r is V = rr 3. ]
3

......................................... cm [3]

(ii) The metal sphere is placed in an empty tank.


The tank is a cylinder with radius 50 cm, standing on its circular base.
Water is poured into the tank to a depth of 60 cm.

Calculate the number of litres of water needed.

...................................... litres [3]

(b) A different tank is a cuboid measuring 1.8 m by 1.5 m by 1.2 m.


Water flows from a pipe into this empty tank at a rate of 200 cm3 per second.

Find the time it takes to fill the tank.


Give your answer in hours and minutes.

........................ hours ..................... minutes [4]


© UCLES 2019 0580/42/M/J/19
https://www.edutvonline.com
17

(c)

NOT TO
SCALE
Area = 295 cm2 Area = 159.5 cm2

17 cm

The diagram shows two mathematically similar shapes with areas 295 cm2 and 159.5 cm2.
The width of the larger shape is 17 cm.

Calculate the width of the smaller shape.

......................................... cm [3]

© UCLES 2019 0580/42/M/J/19 [Turn over


https://www.edutvonline.com
18

11

Diagram 1 Diagram 2 Diagram 3 Diagram 4 Diagram 5

The sequence of diagrams above is made up of small lines and dots.

(a) Complete the table.

Diagram 1 Diagram 2 Diagram 3 Diagram 4 Diagram 5 Diagram 6

Number of
4 10 18 28
small lines
Number of
4 8 13 19
dots
[4]

(b) For Diagram n find an expression, in terms of n, for the number of small lines.

............................................... [2]

(c) Diagram r has 10 300 small lines.

Find the value of r.

r = .............................................. [2]

© UCLES 2019 0580/42/M/J/19


https://www.edutvonline.com
19

(d) The number of dots in Diagram n is an 2 + bn + 1.

Find the value of a and the value of b.

a = ..............................................

b = .............................................. [2]

© UCLES 2019 0580/42/M/J/19


https://www.edutvonline.com
20

BLANK PAGE

Permission to reproduce items where third-party owned material protected by copyright is included has been sought and cleared where possible. Every
reasonable effort has been made by the publisher (UCLES) to trace copyright holders, but if any items requiring clearance have unwittingly been included, the
publisher will be pleased to make amends at the earliest possible opportunity.

To avoid the issue of disclosure of answer-related information to candidates, all copyright acknowledgements are reproduced online in the Cambridge
Assessment International Education Copyright Acknowledgements Booklet. This is produced for each series of examinations and is freely available to download
at www.cambridgeinternational.org after the live examination series.

Cambridge Assessment International Education is part of the Cambridge Assessment Group. Cambridge Assessment is the brand name of the University of
Cambridge Local Examinations Syndicate (UCLES), which itself is a department of the University of Cambridge.

© UCLES 2019 0580/42/M/J/19


https://www.edutvonline.com

Cambridge Assessment International Education


Cambridge International General Certificate of Secondary Education
* 3 5 0 6 7 7 4 6 9 8 *

MATHEMATICS 0580/43
Paper 4 (Extended) May/June 2019
2 hours 30 minutes
Candidates answer on the Question Paper.
Additional Materials: Electronic calculator Geometrical instruments
Tracing paper (optional)

READ THESE INSTRUCTIONS FIRST

Write your centre number, candidate number and name on all the work you hand in.
Write in dark blue or black pen.
You may use an HB pencil for any diagrams or graphs.
Do not use staples, paper clips, glue or correction fluid.
DO NOT WRITE IN ANY BARCODES.

Answer all questions.


If working is needed for any question it must be shown below that question.
Electronic calculators should be used.
If the degree of accuracy is not specified in the question, and if the answer is not exact, give the answer to
three significant figures. Give answers in degrees to one decimal place.
For π, use either your calculator value or 3.142.

At the end of the examination, fasten all your work securely together.
The number of marks is given in brackets [ ] at the end of each question or part question.
The total of the marks for this paper is 130.

This document consists of 16 printed pages.

DC (KS/SW) 164542/3
© UCLES 2019 [Turn over
https://www.edutvonline.com
2

1 Here is part of a train timetable for a journey from London to Marseille.


All times given are in local time.
The local time in Marseille is 1 hour ahead of the local time in London.

London 07 19
Ashford 07 55
Lyon 13 00
Avignon 14 08
Marseille 14 46

(a) (i) Work out the total journey time from London to Marseille.
Give your answer in hours and minutes.

.................. h ...................... min [2]

(ii) The distance from London to Ashford is 90 km.


The local time in London is the same as the local time in Ashford.

Work out the average speed, in km/h, of the train between London and Ashford.

...................................... km/h [3]

(iii) During the journey, the train takes 35 seconds to completely cross a bridge.
The average speed of the train during this crossing is 90 km/h.
The length of the train is 95 metres.

Calculate the length, in metres, of this bridge.

........................................... m [4]

© UCLES 2019 0580/43/M/J/19


https://www.edutvonline.com
3

(b) The fares for the train journey are shown in the table below.

From London to Marseille Standard fare Premier fare

Adult $84 $140

Child $60 $96

(i) For the standard fare, write the ratio adult fare : child fare in its simplest form.

..................... : ..................... [1]

(ii) For an adult, find the percentage increase in the cost of the standard fare to the premier fare.

........................................... % [3]

(iii) For one journey from London to Marseille, the ratio

number of adults : number of children = 11 : 2.

There were 220 adults in total on this journey.


All of the children and 70% of the adults paid the standard fare.
The remaining adults paid the premier fare.

Calculate the total of the fares paid by the adults and the children.

$ .............................................. [5]

(c) There were 3.08 # 105 passengers that made this journey in 2018.
This was a 12% decrease in the number of passengers that made this journey in 2017.

Find the number of passengers that made this journey in 2017.


Give your answer in standard form.

............................................... [3]

© UCLES 2019 0580/43/M/J/19 [Turn over


https://www.edutvonline.com
4

2 (a) Solve.
5x - 17 = 7x + 3

x = .............................................. [2]

(b) Find the integer values of n that satisfy this inequality.

- 7 1 4n G 8

............................................... [3]

(c) Simplify.

(i) a3 # a6

............................................... [1]

(ii) (5xy 2) 3

............................................... [2]
1
-

f p
3
27x 12
(iii)
64y 3

............................................... [3]

© UCLES 2019 0580/43/M/J/19


https://www.edutvonline.com
5

3
y

3
A
2

–6 –5 –4 –3 –2 –1 0 1 2 3 4 5 6 x

–1

–2
B
–3

–4

–5

(a) On the grid, draw the image of


-3
(i) triangle A after a translation by the vector e o , [2]
2

(ii) triangle A after a reflection in the line y = x. [2]

(b) Describe fully the single transformation that maps triangle A onto triangle B.

............................................................................................................................................................

............................................................................................................................................................ [3]

(c) (i) Find the matrix that represents an enlargement, scale factor - 2, centre (0, 0).

f p [2]

(ii) Calculate the determinant of the matrix in part (c)(i).

............................................... [1]

© UCLES 2019 0580/43/M/J/19 [Turn over


https://www.edutvonline.com
6

4 (a)

5.6 cm
NOT TO
10 cm
SCALE

The diagram shows a hemispherical bowl of radius 5.6 cm and a cylindrical tin of height 10 cm.

(i) Show that the volume of the bowl is 368 cm3, correct to the nearest cm3.
4
[The volume, V, of a sphere with radius r is V = rr 3. ]
3

[2]

(ii) The tin is completely full of soup.


When all the soup is poured into the empty bowl, 80% of the volume of the bowl is filled.

Calculate the radius of the tin.

......................................... cm [4]

(b)
NOT TO
SCALE

6 cm

1.75 cm

The diagram shows a cone with radius 1.75 cm and height 6 cm.

(i) Calculate the total surface area of the cone.


[The curved surface area, A, of a cone with radius r and slant height l is A = rrl.]

........................................ cm2 [5]


© UCLES 2019 0580/43/M/J/19
https://www.edutvonline.com
7

(ii)
NOT TO
SCALE

4.5 cm
1.75 cm

The cone contains salt to a depth of 4.5 cm.


The top layer of the salt forms a circle that is parallel to the base of the cone.

(a) Show that the volume of the salt inside the cone is 18.9 cm3, correct to 1 decimal place.
1
[The volume, V, of a cone with radius r and height h is V = rr 2 h.]
3

[4]

(b) The salt is removed from the cone at a constant rate of 200 mm3 per second.

Calculate the time taken for the cone to be completely emptied.


Give your answer in seconds, correct to the nearest second.

............................................. s [3]

© UCLES 2019 0580/43/M/J/19 [Turn over


https://www.edutvonline.com
8

2
5 The diagram shows the graph of y = f (x) where f (x) = x 2 - - 2, x ! 0 .
x
y

12

10

–3 –2 –1 0 1 2 3 x

–2

–4

–6

–8

– 10

– 12

© UCLES 2019 0580/43/M/J/19


https://www.edutvonline.com
9

(a) Use the graph to find

(i) f (1) ,

............................................... [1]

(ii) ff (- 2) .

............................................... [2]

(b) On the grid opposite, draw a suitable straight line to solve the equation
2
x 2 - - 7 =- 3x for - 3 G x G 3.
x

x = ...................... or x = ........................... [4]

(c) By drawing a suitable tangent, find an estimate of the gradient of the curve at x = - 2.

............................................... [3]

(d) (i) Complete the table for y = g (x) where g (x) = 2 -x for - 3 G x G 3.

x -3 -2 -1 0 1 2 3

y 2 1 0.5 0.125
[3]

(ii) On the grid opposite, draw the graph of y = g (x) . [3]

(iii) Use your graph to find the positive solution to the equation f (x) = g (x) .

x = .............................................. [1]

© UCLES 2019 0580/43/M/J/19 [Turn over


https://www.edutvonline.com
10

6 The table shows the time, t seconds, taken by each of 120 boys to solve a puzzle.

Time
20 1 t G 30 30 1 t G 35 35 1 t G 40 40 1 t G 60 60 1 t G 100
(t seconds)
Frequency 38 27 21 16 18

(a) Calculate an estimate of the mean time.

............................................. s [4]

(b) On the grid, complete the histogram to show the information in the frequency table.

4
Frequency
density

0
20 30 40 50 60 70 80 90 100 t
Time (seconds)
[4]

© UCLES 2019 0580/43/M/J/19


https://www.edutvonline.com
11

7 A straight line joins the points A (-2, -3) and C (1, 9).

(a) Find the equation of the line AC in the form y = mx + c.

y = .............................................. [3]

(b) Calculate the acute angle between AC and the x-axis.

............................................... [2]

(c) ABCD is a kite, where AC is the longer diagonal of the kite.


B is the point (3.5, 2).

(i) Find the equation of the line BD in the form y = mx + c.

y = .............................................. [3]

(ii) The diagonals AC and BD intersect at (-0.5, 3).

Work out the co-ordinates of D.

(...................... , ....................) [2]

© UCLES 2019 0580/43/M/J/19 [Turn over


https://www.edutvonline.com
12

8 (a) Angelo has a bag containing 3 white counters and x black counters.
He takes two counters at random from the bag, without replacement.

(i) Complete the following statement.

The probability that Angelo takes two black counters is

x # .
x+3 [2]

7
(ii) The probability that Angelo takes two black counters is .
15
(a) Show that 4x2 - 25x - 21 = 0.

[4]

(b) Solve by factorisation.


4x2 - 25x - 21 = 0

x = .................... or x = ................. [3]

(c) Write down the number of black counters in the bag.

............................................... [1]

© UCLES 2019 0580/43/M/J/19


https://www.edutvonline.com
13

(b) Esme has a bag with 5 green counters and 4 red counters.
She takes three counters at random from the bag without replacement.

Work out the probability that the three counters are all the same colour.

............................................... [4]

© UCLES 2019 0580/43/M/J/19 [Turn over


https://www.edutvonline.com
14

9 (a)

C
NOT TO
SCALE

A D B
58 m

In the diagram, BC is a vertical wall standing on horizontal ground AB.


D is the point on AB where AD = 58 m.
The angle of elevation of C from A is 26°.
The angle of elevation of C from D is 72°.

(i) Show that AC = 76.7 m, correct to 1 decimal place.

[5]

(ii) Calculate BD.

BD = .......................................... m [3]

© UCLES 2019 0580/43/M/J/19


https://www.edutvonline.com
15

(b) Triangle EFG has an area of 70 m2.


EF : FG = 1 : 2 and angle EFG = 40°.

(i) Calculate EF.

EF = .......................................... m [4]

(ii) A different triangle PQR also has an area of 70 m2.


PQ : QR = 1 : 2 and PQ = EF.

Find angle PQR.

Angle PQR = .............................................. [1]

Question 10 is printed on the next page.

© UCLES 2019 0580/43/M/J/19 [Turn over


https://www.edutvonline.com
16

10 (a) 19, 15, 11, 7, ....

(i) Write down the next two terms of the sequence.

...................... , .................. [2]

(ii) Find the nth term of this sequence.

............................................... [2]

(iii) Find the value of n when the nth term is -65.

n = .............................................. [2]

(b) Another sequence has nth term 2n2 + 5n - 15.

Find the difference between the 4th term and the 5th term of this sequence.

............................................... [2]

Permission to reproduce items where third-party owned material protected by copyright is included has been sought and cleared where possible. Every
reasonable effort has been made by the publisher (UCLES) to trace copyright holders, but if any items requiring clearance have unwittingly been included, the
publisher will be pleased to make amends at the earliest possible opportunity.

To avoid the issue of disclosure of answer-related information to candidates, all copyright acknowledgements are reproduced online in the Cambridge
Assessment International Education Copyright Acknowledgements Booklet. This is produced for each series of examinations and is freely available to download
at www.cambridgeinternational.org after the live examination series.

Cambridge Assessment International Education is part of the Cambridge Assessment Group. Cambridge Assessment is the brand name of the University of
Cambridge Local Examinations Syndicate (UCLES), which itself is a department of the University of Cambridge.

© UCLES 2019 0580/43/M/J/19


www.dynamicpapers.com

Cambridge International Examinations


Cambridge International General Certificate of Secondary Education
* 2 8 1 6 8 6 3 4 6 0 *

MATHEMATICS 0580/41
Paper 4 (Extended) May/June 2018
2 hours 30 minutes
Candidates answer on the Question Paper.
Additional Materials: Electronic calculator Geometrical instruments
Tracing paper (optional)

READ THESE INSTRUCTIONS FIRST

Write your Centre number, candidate number and name on all the work you hand in.
Write in dark blue or black pen.
You may use an HB pencil for any diagrams or graphs.
Do not use staples, paper clips, glue or correction fluid.
DO NOT WRITE IN ANY BARCODES.

Answer all questions.


If working is needed for any question it must be shown below that question.
Electronic calculators should be used.
If the degree of accuracy is not specified in the question, and if the answer is not exact, give the answer to
three significant figures. Give answers in degrees to one decimal place.
For r, use either your calculator value or 3.142.

At the end of the examination, fasten all your work securely together.
The number of marks is given in brackets [ ] at the end of each question or part question.
The total of the marks for this paper is 130.

This document consists of 19 printed pages and 1 blank page.

DC (SC/CGW) 147701/2
© UCLES 2018 [Turn over
www.dynamicpapers.com
2

1 Adele, Barbara and Collette share $680 in the ratio 9 : 7 : 4.

(a) Show that Adele receives $306.

[1]

(b) Calculate the amount that Barbara and Collette each receives.

Barbara $ ...............................................

Collette $ ............................................... [3]

(c) Adele changes her $306 into euros (€) when the exchange rate is €1 = $1.125 .

Calculate the number of euros she receives.

€ ............................................... [2]

(d) Barbara spends a total of $17.56 on 5 kg of apples and 3 kg of bananas.


Apples cost $2.69 per kilogram.

Calculate the cost per kilogram of bananas.

$ ............................................... [3]
1
(e) Collette spends half of her share on clothes and of her share on books.
5
Calculate the amount she has left.

$ ............................................... [3]
© UCLES 2018 0580/41/M/J/18
www.dynamicpapers.com
3

2 The scale drawing shows two boundaries, AB and BC, of a field ABCD.
The scale of the drawing is 1 cm represents 8 m.

Scale: 1 cm to 8 m

(a) The boundaries CD and AD of the field are each 72 m long.

(i) Work out the length of CD and AD on the scale drawing.

.......................................... cm [1]

(ii) Using a ruler and compasses only, complete accurately the scale drawing of the field. [2]

(b) A tree in the field is

• equidistant from A and B


and
• equidistant from AB and BC.

On the scale drawing, construct two lines to find the position of the tree.
Use a straight edge and compasses only and leave in your construction arcs. [4]

© UCLES 2018 0580/41/M/J/18 [Turn over


www.dynamicpapers.com
4

3 (a) The price of a house decreased from $82 500 to $77 500.

Calculate the percentage decrease.

............................................ % [3]

(b) Roland invests $12 000 in an account that pays compound interest at a rate of 2.2% per year.

Calculate the value of his investment at the end of 6 years.


Give your answer correct to the nearest dollar.

$ ............................................... [3]

© UCLES 2018 0580/41/M/J/18


www.dynamicpapers.com
5

4
y

7
6

5
B
4

3
A
2
C
1
x
–6 –5 –4 –3 –2 –1 0 1 2 3 4 5 6 7 8
–1

–2

–3

–4
D
–5

–6

(a) Describe fully the single transformation that maps

(i) triangle A onto triangle B,

.....................................................................................................................................................

..................................................................................................................................................... [2]

(ii) triangle A onto triangle C,

.....................................................................................................................................................

..................................................................................................................................................... [3]

(iii) triangle A onto triangle D.

.....................................................................................................................................................

..................................................................................................................................................... [3]

(b) On the grid, draw the image of triangle A after an enlargement by scale factor 2, centre ^7, 3h . [2]

© UCLES 2018 0580/41/M/J/18 [Turn over


www.dynamicpapers.com
6

5 (a) Factorise.

(i) 2mn + m 2 - 6n - 3m

................................................ [2]

(ii) 4y 2 - 81

................................................ [1]

(iii) t 2 - 6t + 8

................................................ [2]

(b) Rearrange the formula to make x the subject.


2m - x
k=
x

x = ............................................... [4]

© UCLES 2018 0580/41/M/J/18


www.dynamicpapers.com
7

(c) Solve the simultaneous equations.


You must show all your working.
1
2 x - 3y=9
5x + y = 28

x = ..............................................

y = ............................................... [3]
3 4
(d) - =6
m+4 m
(i) Show that this equation can be written as 6m 2 + 25m + 16 = 0 .

[3]

(ii) Solve the equation 6m 2 + 25m + 16 = 0 .


Show all your working and give your answers correct to 2 decimal places.

m = ..................... or m = ..................... [4]

© UCLES 2018 0580/41/M/J/18 [Turn over


www.dynamicpapers.com
8

6 A solid hemisphere has volume 230 cm3.

(a) Calculate the radius of the hemisphere.


4
[The volume, V, of a sphere with radius r is V = rr 3 .]
3

.......................................... cm [3]

(b) A solid cylinder with radius 1.6 cm is attached to the hemisphere to make a toy.

NOT TO
SCALE

The total volume of the toy is 300 cm3.

(i) Calculate the height of the cylinder.

.......................................... cm [3]

© UCLES 2018 0580/41/M/J/18


www.dynamicpapers.com
9

(ii) A mathematically similar toy has volume 19 200 cm3.

Calculate the radius of the cylinder for this toy.

.......................................... cm [3]

© UCLES 2018 0580/41/M/J/18 [Turn over


www.dynamicpapers.com
10

7 The graph of y = 10 - 8x 2 for - 1.5 G x G 1.5 is drawn on the grid.

12

10

x
– 1.5 –1 – 0.5 0 0.5 1 1.5

–2

–4

–6

–8

© UCLES 2018 0580/41/M/J/18


www.dynamicpapers.com
11

(a) Write down the equation of the line of symmetry of the graph.

................................................ [1]

(b) On the grid opposite, draw the tangent to the curve at the point where x = 0.5 .
Find the gradient of this tangent.

................................................ [3]

(c) The table shows some values for y = x 3 + 3x + 4 .

x - 1.5 -1 - 0.5 0 0.5 1 1.5

y - 3.9 5.6 8 11.9

(i) Complete the table. [3]

(ii) On the grid opposite, draw the graph of y = x 3 + 3x + 4 for - 1.5 G x G 1.5 . [4]

(d) Show that the values of x where the two curves intersect are the solutions to the
equation x 3 + 8x 2 + 3x - 6 = 0 .

[1]

(e) By drawing a suitable straight line, solve the equation x 3 + 5x + 2 = 0 for - 1.5 G x G 1.5 .

x = ............................................... [3]

© UCLES 2018 0580/41/M/J/18 [Turn over


www.dynamicpapers.com
12

8 (a) The exterior angle of a regular polygon is x° and the interior angle is 8x°.

Calculate the number of sides of the polygon.

................................................ [3]

(b)
C

NOT TO
SCALE
O
D B

58°
A

A, B, C and D are points on the circumference of the circle, centre O.


DOB is a straight line and angle DAC = 58°.

Find angle CDB.

Angle CDB = ............................................... [3]

© UCLES 2018 0580/41/M/J/18


www.dynamicpapers.com
13

(c)

O R NOT TO
SCALE
48°

P
Q

P, Q and R are points on the circumference of the circle, centre O.


PO is parallel to QR and angle POQ = 48°.

(i) Find angle OPR.

Angle OPR = ............................................... [2]

(ii) The radius of the circle is 5.4 cm.

Calculate the length of the major arc PQ.

.......................................... cm [3]

© UCLES 2018 0580/41/M/J/18 [Turn over


www.dynamicpapers.com
14

5
9 The probability that it will rain tomorrow is .
8
1
If it rains, the probability that Rafael walks to school is .
6
7
If it does not rain, the probability that Rafael walks to school is .
10
(a) Complete the tree diagram.

Walks

........

Rains

........

........
Does not walk
Walks

........
........
Does not rain

........
Does not walk
[3]

(b) Calculate the probability that it will rain tomorrow and Rafael walks to school.

................................................ [2]

(c) Calculate the probability that Rafael does not walk to school.

................................................ [3]

© UCLES 2018 0580/41/M/J/18


www.dynamicpapers.com
15

10 (a) In 2017, the membership fee for a sports club was $79.50 .
This was an increase of 6% on the fee in 2016.

Calculate the fee in 2016.

$ ............................................... [3]

(b) On one day, the number of members using the exercise machines was 40, correct to the nearest 10.
Each member used a machine for 30 minutes, correct to the nearest 5 minutes.

Calculate the lower bound for the number of minutes the exercise machines were used on this day.

......................................... min [2]

(c) On another day, the number of members using the exercise machines (E), the swimming pool (S) and
the tennis courts (T ) is shown on the Venn diagram.


E S
20 5 33
4
7 8

16
T

(i) Find the number of members using only the tennis courts.

................................................ [1]

(ii) Find the number of members using the swimming pool.

................................................ [1]

(iii) A member using the swimming pool is chosen at random.

Find the probability that this member also uses the tennis courts and the exercise machines.

................................................ [2]

(iv) Find n ^T + ^E , Shh .

................................................ [1]
© UCLES 2018 0580/41/M/J/18 [Turn over
www.dynamicpapers.com
16

OA = c m AB = c m AC = c m
4 8 -3
11 (a)
3 -7 6
Find

(i) OB ,

OB = ............................................... [3]

(ii) BC .

BC = f p [2]

(b)
S R
NOT TO
SCALE
b
X

P Q
a

PQRS is a parallelogram with diagonals PR and SQ intersecting at X.


PQ = a and PS = b .

Find QX in terms of a and b.


Give your answer in its simplest form.

QX = ............................................... [2]

© UCLES 2018 0580/41/M/J/18


www.dynamicpapers.com
17

M =c m
2 5
(c)
1 8
Calculate

(i) M2 ,

M2 = f p [2]

(ii) M -1 .

M -1 = f p [2]

© UCLES 2018 0580/41/M/J/18 [Turn over


www.dynamicpapers.com
18

12 Marco is making patterns with grey and white circular mats.

Pattern 1 Pattern 2 Pattern 3 Pattern 4

The patterns form a sequence.


Marco makes a table to show some information about the patterns.

Pattern number 1 2 3 4 5

Number of grey mats 6 9 12 15

Total number of mats 6 10 15 21

(a) Complete the table for Pattern 5. [2]

(b) Find an expression, in terms of n, for the number of grey mats in Pattern n.

................................................ [2]

(c) Marco makes a pattern with 24 grey mats.

Find the total number of mats in this pattern.

................................................ [2]

© UCLES 2018 0580/41/M/J/18


www.dynamicpapers.com
19

(d) Marco needs a total of 6 mats to make the first pattern.


He needs a total of 16 mats to make the first two patterns.
1 3
He needs a total of n + an 2 + bn mats to make the first n patterns.
6
Find the value of a and the value of b.

a = ...............................................

b = ............................................... [6]

© UCLES 2018 0580/41/M/J/18


www.dynamicpapers.com
20

BLANK PAGE

Permission to reproduce items where third-party owned material protected by copyright is included has been sought and cleared where possible. Every
reasonable effort has been made by the publisher (UCLES) to trace copyright holders, but if any items requiring clearance have unwittingly been included, the
publisher will be pleased to make amends at the earliest possible opportunity.

To avoid the issue of disclosure of answer-related information to candidates, all copyright acknowledgements are reproduced online in the Cambridge International
Examinations Copyright Acknowledgements Booklet. This is produced for each series of examinations and is freely available to download at www.cie.org.uk after
the live examination series.

Cambridge International Examinations is part of the Cambridge Assessment Group. Cambridge Assessment is the brand name of University of Cambridge Local
Examinations Syndicate (UCLES), which is itself a department of the University of Cambridge.

© UCLES 2018 0580/41/M/J/18


www.dynamicpapers.com

Cambridge International Examinations


Cambridge International General Certificate of Secondary Education
* 5 1 3 4 4 2 6 0 4 2 *

MATHEMATICS 0580/42
Paper 4 (Extended) May/June 2018
2 hours 30 minutes
Candidates answer on the Question Paper.
Additional Materials: Electronic calculator Geometrical instruments
Tracing paper (optional)

READ THESE INSTRUCTIONS FIRST

Write your Centre number, candidate number and name on all the work you hand in.
Write in dark blue or black pen.
You may use an HB pencil for any diagrams or graphs.
Do not use staples, paper clips, glue or correction fluid.
DO NOT WRITE IN ANY BARCODES.

Answer all questions.


If working is needed for any question it must be shown below that question.
Electronic calculators should be used.
If the degree of accuracy is not specified in the question, and if the answer is not exact, give the answer to
three significant figures. Give answers in degrees to one decimal place.
For π, use either your calculator value or 3.142.

At the end of the examination, fasten all your work securely together.
The number of marks is given in brackets [ ] at the end of each question or part question.
The total of the marks for this paper is 130.

This document consists of 15 printed pages and 1 blank page.

DC (CE/SW) 148068/3
© UCLES 2018 [Turn over
www.dynamicpapers.com
2

1 (a) Here is a list of ingredients to make 20 biscuits.

260 g of butter
500 g of sugar
650 g of flour
425 g of rice

(i) Find the mass of rice as a percentage of the mass of sugar.

............................................ % [1]

(ii) Find the mass of butter needed to make 35 of these biscuits.

............................................. g [2]

(iii) Michel has 2 kg of each ingredient.

Work out the greatest number of these biscuits that he can make.

................................................. [3]

(b) A company makes these biscuits at a cost of $1.35 per packet.


These biscuits are sold for $1.89 per packet.

(i) Calculate the percentage profit the company makes on each packet.

............................................ % [3]

(ii) The selling price of $1.89 has increased by 8% from last year.

Calculate the selling price last year.

$ ................................................ [3]

© UCLES 2018 0580/42/M/J/18


www.dynamicpapers.com
3

(c) Over a period of 3 years, the company’s sales of biscuits increased from 15.6 million packets to
20.8 million packets.
The sales increased exponentially by the same percentage each year.

Calculate the percentage increase each year.

............................................ % [3]

(d) The people who work for the company are in the following age groups.

Group A Group B Group C

Under 30 years 30 to 50 years Over 50 years

The ratio of the number in group A to the number in group B is 7 : 10.


The ratio of the number in group B to the number in group C is 4 : 3.

(i) Find the ratio of the number in group A to the number in group C.
Give your answer in its simplest form.

....................... : ....................... [3]

(ii) There are 45 people in group C.

Find the total number of people who work for the company.

................................................. [3]

© UCLES 2018 0580/42/M/J/18 [Turn over


www.dynamicpapers.com
4

2 The time taken for each of 120 students to complete a cooking challenge is shown in the table.

Time (t minutes) 20 1 t G 25 25 1 t G 30 30 1 t G 35 35 1 t G 40 40 1 t G 45
Frequency 44 32 28 12 4

(a) (i) Write down the modal time interval.

................... 1 t G ................... [1]

(ii) Write down the interval containing the median time.

................... 1 t G ................... [1]

(iii) Calculate an estimate of the mean time.

......................................... min [4]

(iv) A student is chosen at random.

Find the probability that this student takes more than 40 minutes.

................................................. [1]

(b) (i) Complete the cumulative frequency table.

Time (t minutes) t G 20 t G 25 t G 30 t G 35 t G 40 t G 45
Cumulative
0 44
frequency

[2]

© UCLES 2018 0580/42/M/J/18


www.dynamicpapers.com
5

(ii) On the grid, draw a cumulative frequency diagram to show this information.

120

110

100

90

80

70
Cumulative
frequency 60

50

40

30

20

10

0 t
20 25 30 35 40 45
Time (minutes)

[3]

(iii) Find the median time.

......................................... min [1]

(iv) Find the interquartile range.

......................................... min [2]

(v) Find the number of students who took more than 37 minutes to complete the cooking challenge.

................................................. [2]

© UCLES 2018 0580/42/M/J/18 [Turn over


www.dynamicpapers.com
6

3
B
2

x
–7 –6 –5 –4 –3 –2 –1 0 1 2 3 4 5 6 7 8
–1

–2

–3
A
–4

–5

–6

(a) (i) Draw the image of triangle A after a reflection in the line x = 2. [2]

Draw the image of triangle A after a translation by the vector c m.


-2
(ii) [2]
4

1
(iii) Draw the image of triangle A after an enlargement by scale factor - , centre (3, 1). [3]
2
(b) Describe fully the single transformation that maps triangle A onto triangle B.

..............................................................................................................................................................

.............................................................................................................................................................. [3]

(c) Describe fully the single transformation represented by the matrix c m.


0 -1
-1 0

..............................................................................................................................................................

.............................................................................................................................................................. [2]

© UCLES 2018 0580/42/M/J/18


www.dynamicpapers.com
7

4 (a) Simplify.

(i) (3p2)5

................................................. [2]

(ii) 18x2y6 ' 2xy2

................................................. [2]
-2
c m
5
(iii)
m

................................................. [1]

(b) In this part, all measurements are in metres.

5x – 9

w NOT TO
SCALE

3x + 7

The diagram shows a rectangle.


The area of the rectangle is 310 m2.

Work out the value of w.

w = ................................................ [4]

© UCLES 2018 0580/42/M/J/18 [Turn over


www.dynamicpapers.com
8

5
A O

NOT TO
SCALE
8 cm 7 cm
78°
C
B

The diagram shows a design made from a triangle AOC joined to a sector OCB.
AC = 8 cm, OB = OC = 7 cm and angle ACO = 78°.

(a) Use the cosine rule to show that OA = 9.47 cm, correct to 2 decimal places.

[4]

(b) Calculate angle OAC.

Angle OAC = ................................................ [3]

© UCLES 2018 0580/42/M/J/18


www.dynamicpapers.com
9

(c) The perimeter of the design is 29.5 cm.

Show that angle COB = 41.2°, correct to 1 decimal place.

[5]

(d) Calculate the total area of the design.

......................................... cm2 [4]

© UCLES 2018 0580/42/M/J/18 [Turn over


www.dynamicpapers.com
10

x3 1
6 (a) Complete the table of values for y= - 2 , x ! 0.
3 2x

x -3 -2 -1 - 0.5 - 0.3 0.3 0.5 1 2 3


y - 9.1 - 2.8 - 0.8 - 5.6 - 5.5 - 2.0 8.9 [3]

x3 1
(b) On the grid, draw the graph of y= - 2 for - 3 G x G - 0.3 and 0.3 G x G 3.
3 2x
y

10

x
–3 –2 –1 0 1 2 3
–1

–2

–3

–4

–5

–6

–7

–8

–9

–10 [5]
© UCLES 2018 0580/42/M/J/18
www.dynamicpapers.com
11

(c) (i) By drawing a suitable tangent, find an estimate of the gradient of the curve at x = - 2.

................................................. [3]

(ii) Write down the equation of the tangent to the curve at x = - 2.


Give your answer in the form y = mx + c.

y = ................................................ [2]

(d) Use your graph to solve the equations.


x3 1
(i) - 2=0
3 2x
x = ................................................ [1]
x3 1
(ii) - 2 +4 = 0
3 2x

x = .................... or x = .................... or x = .................... [3]


x3 1
(e) The equation - + 4 = 0 can be written in the form axn + bxn -3 - 3 = 0.
3 2x 2
Find the value of a, the value of b and the value of n.

a = ................................................

b = ................................................

n = ................................................ [3]

© UCLES 2018 0580/42/M/J/18 [Turn over


www.dynamicpapers.com
12

7 In this question, all measurements are in metres.

6 NOT TO
x SCALE

2x – 3

The diagram shows a right-angled triangle.

(a) Show that 5x2 - 12x - 27 = 0.

[3]

(b) Solve 5x2 - 12x - 27 = 0.


Show all your working and give your answers correct to 2 decimal places.

x = ......................... or x = ......................... [4]

(c) Calculate the perimeter of the triangle.

............................................ m [2]

(d) Calculate the smallest angle of the triangle.

................................................. [2]

© UCLES 2018 0580/42/M/J/18


www.dynamicpapers.com
13

10
8 f (x) = 8 - 3x g (x) = , x !-1 h (x) = 2 x
x+1
(a) Find

hf c m ,
8
(i)
3

................................................. [2]

(ii) gh(-2),

................................................. [2]

(iii) g -1 (x) ,

g -1 (x) = ................................................ [3]

(iv) f -1 f (5) .

................................................. [1]

(b) Write f(x) + g(x) as a single fraction in its simplest form.

................................................. [3]

© UCLES 2018 0580/42/M/J/18 [Turn over


www.dynamicpapers.com
14

9 (a)

A B

C
109° NOT TO
SCALE
O

35°
28°
D

A, B, C, D and E lie on the circle, centre O.


Angle AEB = 35°, angle ODE = 28° and angle ACD = 109°.

(i) Work out the following angles, giving reasons for your answers.

(a) Angle EBD = ............................. because ...........................................................................

..............................................................................................................................................

.............................................................................................................................................. [3]

(b) Angle EAD = ............................. because ...........................................................................

.............................................................................................................................................. [2]

(ii) Work out angle BEO.

Angle BEO = ................................................ [3]

© UCLES 2018 0580/42/M/J/18


www.dynamicpapers.com
15

(b) In a regular polygon, the interior angle is 11 times the exterior angle.

(i) Work out the number of sides of this polygon.

................................................. [3]

(ii) Find the sum of the interior angles of this polygon.

................................................. [2]

© UCLES 2018 0580/42/M/J/18


www.dynamicpapers.com
16

BLANK PAGE

Permission to reproduce items where third-party owned material protected by copyright is included has been sought and cleared where possible. Every
reasonable effort has been made by the publisher (UCLES) to trace copyright holders, but if any items requiring clearance have unwittingly been included, the
publisher will be pleased to make amends at the earliest possible opportunity.

To avoid the issue of disclosure of answer-related information to candidates, all copyright acknowledgements are reproduced online in the Cambridge International
Examinations Copyright Acknowledgements Booklet. This is produced for each series of examinations and is freely available to download at www.cie.org.uk after
the live examination series.

Cambridge International Examinations is part of the Cambridge Assessment Group. Cambridge Assessment is the brand name of University of Cambridge Local
Examinations Syndicate (UCLES), which is itself a department of the University of Cambridge.

© UCLES 2018 0580/42/M/J/18


www.dynamicpapers.com

Cambridge International Examinations


Cambridge International General Certificate of Secondary Education
* 3 2 3 2 9 9 9 7 7 4 *

MATHEMATICS 0580/43
Paper 4 (Extended) May/June 2018
2 hours 30 minutes
Candidates answer on the Question Paper.
Additional Materials: Electronic calculator Geometrical instruments
Tracing paper (optional).

READ THESE INSTRUCTIONS FIRST

Write your Centre number, candidate number and name on all the work you hand in.
Write in dark blue or black pen.
You may use an HB pencil for any diagrams or graphs.
Do not use staples, paper clips, glue or correction fluid.
DO NOT WRITE IN ANY BARCODES.

Answer all questions.


If working is needed for any question it must be shown below that question.
Electronic calculators should be used.
If the degree of accuracy is not specified in the question, and if the answer is not exact, give the answer to
three significant figures. Give answers in degrees to one decimal place.
For π, use either your calculator value or 3.142.

At the end of the examination, fasten all your work securely together.
The number of marks is given in brackets [ ] at the end of each question or part question.
The total of the marks for this paper is 130.

This document consists of 19 printed pages and 1 blank page.

DC (SC/CGW) 148061/2
© UCLES 2018 [Turn over
www.dynamicpapers.com
2

1 (a) Rowena buys and sells clothes.

(i) She buys a jacket for $40 and sells it for $45.40 .

Calculate the percentage profit.

............................................ % [3]

(ii) She sells a dress for $42.60 after making a profit of 20% on the cost price.

Calculate the cost price.

$ ............................................... [3]

(b) Sara invests $500 for 15 years at a rate of 2% per year simple interest.

Calculate the total interest Sara receives.

$ ............................................... [2]

© UCLES 2018 0580/43/M/J/18


www.dynamicpapers.com
3

(c) Tomas has two cars.

(i) The value, today, of one car is $21 000.


The value of this car decreases exponentially by 18% each year.

Calculate the value of this car after 5 years.


Give your answer correct to the nearest hundred dollars.

$ ............................................... [3]

(ii) The value, today, of the other car is $15 000.


The value of this car increases exponentially by x % each year.
After 12 years the value of the car will be $42 190.

Calculate the value of x.

x = ............................................... [3]

© UCLES 2018 0580/43/M/J/18 [Turn over


www.dynamicpapers.com
4

2 (a) (i) y = 2x

Complete the table.

x 0 1 2 3 4

y 2 4 8
[2]

(ii) y = 14 - x 2

Complete the table.

x 0 1 2 3 4

y 13 10 5
[2]

(b) On the grid, draw the graphs of y = 2 x and y = 14 - x 2 for 0 G x G 4 .

16

14

12

10

0 x
1 2 3 4
−2
[6]

© UCLES 2018 0580/43/M/J/18


www.dynamicpapers.com
5

(c) Use your graphs to solve the equations.

(i) 2 x = 12

x = ............................................... [1]

(ii) 2 x = 14 - x 2

x = ............................................... [1]

(d) (i) On the grid, draw the line from the point (4, 2) that has a gradient of - 4 . [1]

(ii) Complete the statement.

This straight line is a .................................. to the graph of y = 14 - x 2

at the point ( .......... , .......... ). [2]

© UCLES 2018 0580/43/M/J/18 [Turn over


www.dynamicpapers.com
6

3 (a) The scatter diagram shows the physics mark and the chemistry mark for each of 12 students.

4
Chemistry
mark 3

0
0 1 2 3 4 5 6 7 8 9 10
Physics mark

(i) What type of correlation is shown in the scatter diagram?

................................................ [1]

(ii) On the scatter diagram, draw a line of best fit. [1]

(iii) Find an estimate of the chemistry mark for another student who has a physics mark of 4.

................................................ [1]

(b) A teacher records the number of days each of the 24 students in her class are absent.
The frequency table shows the results.

Number of days 0 1 2 3 4 5

Frequency 10 8 3 2 0 1

Find the mode, the median and the mean.

Mode = ...............................................

Median = ...............................................

Mean = ............................................... [5]

© UCLES 2018 0580/43/M/J/18


www.dynamicpapers.com
7

(c) Three sizes of eggs are sold in a shop.


The table shows the number of eggs of each size sold in one day.

Size Small Medium Large

Mass (m grams) 46 1 m G 52 52 1 m G 62 62 1 m G 80

Number of eggs sold 78 180 162

(i) Calculate an estimate of the mean mass.

............................................. g [4]

(ii) On the grid, draw a histogram to show the information in the table.

20

18

16

14

12
Frequency
density 10

0 m
40 50 60 70 80
Mass (grams)
[4]

© UCLES 2018 0580/43/M/J/18 [Turn over


www.dynamicpapers.com
8

4 (a) The diagram shows two sets of cards.

Set A 1 1 2 2 2

Set B 0 1 1 1 2

(i) Jojo chooses two cards at random from Set A without replacement.

Find the probability that the two cards have the same number.

................................................ [3]

(ii) Jojo replaces the two cards.


Kylie then chooses one card at random from Set A and one card at random from Set B.

Find the probability that the two cards have the same number.

................................................ [3]

(iii) Who is the most likely to choose two cards that have the same number?
Show all your working.

................................................ [1]

© UCLES 2018 0580/43/M/J/18


www.dynamicpapers.com
9

(b)
Set C 4 4 5 5 5

Lena chooses three cards at random from Set C without replacement.

Find the probability that the third card chosen is numbered 4.

................................................ [3]

© UCLES 2018 0580/43/M/J/18 [Turn over


www.dynamicpapers.com
10

5 (a) At a football match, the price of an adult ticket is $x and the price of a child ticket is $ ^x - 2.50h .
There are 18 500 adults and 2400 children attending the football match.
The total amount paid for the tickets is $320 040.

Find the price of an adult ticket.

$ ................................................ [4]

(b) (i) Factorise y 2 + 5y - 84 .

................................................ [2]

(ii)

NOT TO
y cm SCALE

(y + 5) cm

The area of the rectangle is 84 cm2.

Find the perimeter.

.......................................... cm [3]

© UCLES 2018 0580/43/M/J/18


www.dynamicpapers.com
11

(c) In a shop, the price of a monthly magazine is $m and the price of a weekly magazine is $ ^m - 0.75h .
One day, the shop receives
• $168 from selling monthly magazines
• $207 from selling weekly magazines.
The total number of these magazines sold during this day is 100.

(i) Show that 50m 2 - 225m + 63 = 0 .

[3]

(ii) Find the price of a monthly magazine.


Show all your working.

$ ............................................... [3]

© UCLES 2018 0580/43/M/J/18 [Turn over


www.dynamicpapers.com
12

6 (a)
E

6 cm

NOT TO
SCALE
A
12 cm

6 cm

In the pentagon ABCDE, angle ACB = angle AED = 90°.


Triangle ACD is equilateral with side length 12 cm.
DE = BC = 6 cm.

(i) Calculate angle BAE.

Angle BAE = ............................................... [4]

(ii) Calculate AB.

AB = ......................................... cm [2]

(iii) Calculate AE.

AE = ......................................... cm [3]
© UCLES 2018 0580/43/M/J/18
www.dynamicpapers.com
13

(iv) Calculate the area of the pentagon.

......................................... cm2 [4]

(b)
S R

P Q
5 cm
NOT TO
SCALE
D C

4 cm
A 8 cm B

The diagram shows a cuboid.


AB = 8 cm, BC = 4 cm and CR = 5 cm.

(i) Write down the number of planes of symmetry of this cuboid.

................................................ [1]

(ii) Calculate the angle between the diagonal AR and the plane BCRQ.

................................................ [4]

© UCLES 2018 0580/43/M/J/18 [Turn over


www.dynamicpapers.com
14

7 (a)

NOT TO
SCALE

1.5 cm

Water flows through a cylindrical pipe at a speed of 8 cm/s.


The radius of the circular cross-section is 1.5 cm and the pipe is always completely full of water.

Calculate the amount of water that flows through the pipe in 1 hour.
Give your answer in litres.

....................................... litres [4]

© UCLES 2018 0580/43/M/J/18


www.dynamicpapers.com
15

(b)
NOT TO
SCALE

12 cm y cm
x cm

6 cm

The diagram shows three solids.


The base radius of the cone is 6 cm and the slant height is 12 cm.
The radius of the sphere is x cm and the radius of the hemisphere is y cm.
The total surface area of each solid is the same.

(i) Show that the total surface area of the cone is 108r cm2.

[The curved surface area, A, of a cone with radius r and slant height l is A = rrl .]

[2]

(ii) Find the value of x and the value of y.

[The surface area, A, of a sphere with radius r is A = 4rr 2 .]

x = ...............................................

y = ............................................... [4]
© UCLES 2018 0580/43/M/J/18 [Turn over
www.dynamicpapers.com
16

M=c m N = ^1 2h
2 1 4
8 (a) P =f p
4 3 1

(i) For the following calculations, put a tick (ü) if it is possible or put a cross (û) if it is not possible.
There is no need to carry out any of the calculations.

Calculation ü or û

N+P
NP

M2

N2

MN

NM
[4]
1
(ii) Work out f p + P .
2

................................................ [1]

(iii) Work out PN.

................................................ [2]

(iv) Work out M -1 .

................................................ [2]
0 -1
(b) Describe fully the single transformation represented by the matrix f p.
1 0

.............................................................................................................................................................

............................................................................................................................................................. [3]
© UCLES 2018 0580/43/M/J/18
www.dynamicpapers.com
17

1
9 (a) Find the equation of the straight line that is perpendicular to the line y = x + 1 and passes through
2
the point (1, 3).

................................................ [3]

(b)
y

8
7
6
5
4
R
3
2
1
x
0 1 2 3 4 5 6 7 8 9 10 11 12

(i) Find the three inequalities that define the region R.

................................................

................................................

................................................ [4]

(ii) Find the point (x, y), with integer co-ordinates, inside the region R such that 3x + 5y = 35 .

( .................... , ....................) [2]

© UCLES 2018 0580/43/M/J/18 [Turn over


www.dynamicpapers.com
18

10 (a) f (x) = 2x - 3 g (x) = x 2 + 1

(i) Find gg(2).

................................................ [2]

(ii) Find g (x + 2) , giving your answer in its simplest form.

................................................ [2]

(iii) Find x when f (x) = 7 .

x = ............................................... [2]

(iv) Find f -1 (x) .

f -1 (x) = ............................................... [2]

© UCLES 2018 0580/43/M/J/18


www.dynamicpapers.com
19

(b) h (x) = x x , x 2 0

(i) Calculate h(0.3).


Give your answer correct to 2 decimal places.

................................................ [2]

(ii) Find x when h(x) = 256.

x = ............................................... [1]

© UCLES 2018 0580/43/M/J/18


www.dynamicpapers.com
20

BLANK PAGE

Permission to reproduce items where third-party owned material protected by copyright is included has been sought and cleared where possible. Every
reasonable effort has been made by the publisher (UCLES) to trace copyright holders, but if any items requiring clearance have unwittingly been included, the
publisher will be pleased to make amends at the earliest possible opportunity.

To avoid the issue of disclosure of answer-related information to candidates, all copyright acknowledgements are reproduced online in the Cambridge International
Examinations Copyright Acknowledgements Booklet. This is produced for each series of examinations and is freely available to download at www.cie.org.uk after
the live examination series.

Cambridge International Examinations is part of the Cambridge Assessment Group. Cambridge Assessment is the brand name of University of Cambridge Local
Examinations Syndicate (UCLES), which is itself a department of the University of Cambridge.

© UCLES 2018 0580/43/M/J/18


Cambridge International Examinations
Cambridge International General Certificate of Secondary Education
*3415255704*

MATHEMATICS 0580/41
Paper 4 (Extended) May/June 2014
2 hours 30 minutes
Candidates answer on the Question Paper.
Additional Materials: Electronic calculator Geometrical instruments
Tracing paper (optional)

READ THESE INSTRUCTIONS FIRST

Write your Centre number, candidate number and name on all the work you hand in.
Write in dark blue or black pen.
You may use an HB pencil for any diagrams or graphs.
Do not use staples, paper clips, glue or correction fluid.
DO NOT WRITE IN ANY BARCODES.

Answer all questions.


If working is needed for any question it must be shown below that question.
Electronic calculators should be used.
If the degree of accuracy is not specified in the question, and if the answer is not exact, give the answer to
three significant figures. Give answers in degrees to one decimal place.
For π, use either your calculator value or 3.142.

At the end of the examination, fasten all your work securely together.
The number of marks is given in brackets [ ] at the end of each question or part question.
The total of the marks for this paper is 130.

The syllabus is approved for use in England, Wales and Northern Ireland as a Cambridge International Level 1/Level 2 Certificate.

This document consists of 19 printed pages and 1 blank page.

IB14 06_0580_41/4RP
© UCLES 2014 [Turn over
2

A= f p C= e o D= f p
3 2 -2 2 0
1 B = (–2 5)
-1 1 5 0 2

(a) Work out, when possible, each of the following.


If it is not possible, write ‘not possible’ in the answer space.

(i) 2A

Answer(a)(i) [1]

(ii) B + C

Answer(a)(ii) [1]

(iii) AD

Answer(a)(iii) [2]

(iv) A–1, the inverse of A.

Answer(a)(iv) [2]

(b) Explain why it is not possible to work out CD.

Answer(b) ........................................................................................................................................... [1]

(c) Describe fully the single transformation represented by the matrix D.

Answer(c) ............................................................................................................................................

............................................................................................................................................................. [3]
__________________________________________________________________________________________

© UCLES 2014 0580/41/M/J/14


3

2 Ali leaves home at 10 00 to cycle to his grandmother’s house. He arrives at 13 00.


The distance-time graph represents his journey.

40

30
Distance from
home (km)

20

10

0
10 00 11 00 12 00 13 00 14 00 15 00 16 00 17 00
Time

(a) Calculate Ali’s speed between 10 00 and 11 30.


Give your answer in kilometres per hour.

Answer(a) ...................................... km/h [2]

(b) Show that Ali’s average speed for the whole journey to his grandmother’s house is 12 km/h.

Answer(b)

[2]

(c) Change 12 kilometres per hour into metres per minute.

Answer(c) ..................................... m/min [2]

(d) Ali stays for 45 minutes at his grandmother’s house and then returns home.
He arrives home at 16 42.

Complete the distance-time graph. [2]


__________________________________________________________________________________________

© UCLES 2014 0580/41/M/J/14 [Turn over


4

3 (a) The running costs for a papermill are $75 246.


This amount is divided in the ratio labour costs : materials = 5 : 1.

Calculate the labour costs.

Answer(a) $ ................................................ [2]

(b) In 2012 the company made a profit of $135 890.


In 2013 the profit was $150 675.

Calculate the percentage increase in the profit from 2012 to 2013.

Answer(b) ............................................ % [3]

(c) The profit of $135 890 in 2012 was an increase of 7% on the profit in 2011.

Calculate the profit in 2011.

Answer(c) $ ................................................ [3]

(d)
2 cm
NOT TO
SCALE

21 cm

30 cm

Paper is sold in cylindrical rolls.


There is a wooden cylinder of radius 2 cm and height 21 cm in the centre of each roll.
The outer radius of a roll of paper is 30 cm.

(i) Calculate the volume of paper in a roll.

Answer(d)(i) ......................................... cm3 [3]

© UCLES 2014 0580/41/M/J/14


5

(ii) The paper is cut into sheets which measure 21 cm by 29.7 cm.
The thickness of each sheet is 0.125 mm.

(a) Change 0.125 millimetres into centimetres.

Answer(d)(ii)(a) .......................................... cm [1]

(b) Work out how many whole sheets of paper can be cut from a roll.

Answer(d)(ii)(b) ................................................ [4]


__________________________________________________________________________________________

© UCLES 2014 0580/41/M/J/14 [Turn over


6

T B

11 9

x
6–x

In the Venn diagram, = {children in a nursery}

B = {children who received a book for their birthday}


T = {children who received a toy for their birthday}
P = {children who received a puzzle for their birthday}

x children received a book and a toy and a puzzle.


6 children received a toy and a puzzle.

(a) 4 children received a book and a toy.


5 children received a book and a puzzle.
7 children received a puzzle but not a book and not a toy.

Complete the Venn diagram above. [3]

(b) There are 40 children in the nursery.

Using the Venn diagram, write down and solve an equation in x.

Answer(b)

[3]

© UCLES 2014 0580/41/M/J/14


7

(c) Work out

(i) the probability that a child, chosen at random, received a book but not a toy and not a puzzle,

Answer(c)(i) ................................................ [1]

(ii) the number of children who received a book and a puzzle but not a toy,

Answer(c)(ii) ................................................ [1]

(iii) n(B),

Answer(c)(iii) ................................................ [1]

(iv) n(B ∪ P),

Answer(c)(iv) ................................................ [1]

(v) n(B ∪ T ∪ P)'.

Answer(c)(v) ................................................ [1]

(d)

T B

Shade the region B ∩ (T ∪ P)'. [1]


__________________________________________________________________________________________

© UCLES 2014 0580/41/M/J/14 [Turn over


8

5
S

North

Scale: 2 cm to 3 km

In the scale drawing, P is a port, L is a lighthouse and S is a ship.


The scale is 2 centimetres represents 3 kilometres.

(a) Measure the bearing of S from P.

Answer(a) ................................................ [1]

(b) Find the actual distance of S from L.

Answer(b) .......................................... km [2]

(c) The bearing of L from S is 160°.

Calculate the bearing of S from L.

Answer(c) ................................................ [1]

© UCLES 2014 0580/41/M/J/14


9

(d) Work out the scale of the map in the form 1 : n.

Answer(d) 1 : ................................................ [2]

(e) A boat B is

● equidistant from S and L


and
● equidistant from the lines PS and SL.

On the diagram, using a straight edge and compasses only, construct the position of B. [5]

(f) The lighthouse stands on an island of area 1.5 cm2 on the scale drawing.

Work out the actual area of the island.

Answer(f) ......................................... km2 [2]


__________________________________________________________________________________________

© UCLES 2014 0580/41/M/J/14 [Turn over


10

6 (a) A square spinner is biased.


The probabilities of obtaining the scores 1, 2, 3 and 4 when it is spun are given in the table.

Score 1 2 3 4

Probability 0.1 0.2 0.4 0.3

(i) Work out the probability that on one spin the score is 2 or 3.

Answer(a)(i) ................................................ [2]

(ii) In 5000 spins, how many times would you expect to score 4 with this spinner?

Answer(a)(ii) ................................................ [1]

(iii) Work out the probability of scoring 1 on the first spin and 4 on the second spin.

Answer(a)(iii) ................................................ [2]

(b) In a bag there are 7 red discs and 5 blue discs.


From the bag a disc is chosen at random and not replaced.
A second disc is then chosen at random.

Work out the probability that at least one of the discs is red.
Give your answer as a fraction.

Answer(b) ................................................ [3]


__________________________________________________________________________________________

© UCLES 2014 0580/41/M/J/14


11

7
y
4

3
A
2

x
–6 –5 –4 –3 –2 –1 0 1 2 3 4 5 6
–1

–2

–3

–4

–5

(a) On the grid,

(i) draw the image of shape A after a translation by the vector e o ,


-5
[2]
-4
(ii) draw the image of shape A after a rotation through 90° clockwise about the origin. [2]

(b) (i) On the grid, draw the image of shape A after the transformation represented by the matrix f p.
2 0
0 1

[3]

(ii) Describe fully the single transformation represented by the matrix f p.


2 0
0 1
Answer(b)(ii) ...............................................................................................................................

..................................................................................................................................................... [3]
__________________________________________________________________________________________

© UCLES 2014 0580/41/M/J/14 [Turn over


12

8 (a) Complete the table of values for y = x3 – 3x + 1 .

x –2.5 –2 –1.5 –1 –0.5 0 0.5 1 1.5 2 2.5

y –7.125 –1 3 1 –0.375 –1 –0.125 3 9.125

[2]

(b) Draw the graph of y = x3 – 3x + 1 for –2.5 Ğ x Ğ 2.5 .

y
10

x
–3 –2 –1 0 1 2 3
–1

–2

–3

–4

–5

–6

–7

–8
[4]

© UCLES 2014 0580/41/M/J/14


13

(c) By drawing a suitable tangent, estimate the gradient of the curve at the point where x = 2.

Answer(c) ................................................ [3]

(d) Use your graph to solve the equation x3 – 3x + 1 = 1 .

Answer(d) x = ..................... or x = ..................... or x = ..................... [2]

(e) Use your graph to complete the inequality in k for which the equation

x3 – 3x + 1 = k has three different solutions.

Answer(e) ........................ < k < ........................ [2]


__________________________________________________________________________________________

© UCLES 2014 0580/41/M/J/14 [Turn over


14

9
80

70

60

50
Cumulative
frequency
40

30

20

10

t
0
10 20 30 40 50
Time (minutes)

The times (t minutes) taken by 80 people to complete a charity swim were recorded.
The results are shown in the cumulative frequency diagram above.

(a) Find

(i) the median,

Answer(a)(i) ......................................... min [1]

(ii) the inter-quartile range,

Answer(a)(ii) ......................................... min [2]

© UCLES 2014 0580/41/M/J/14


15

(iii) the 70th percentile.

Answer(a)(iii) ......................................... min [2]

(b) The times taken by the 80 people are shown in this grouped frequency table.

Time (t minutes) 0 < t Ğ 20 20 < t Ğ 30 30 < t Ğ 45 45 < t Ğ 50

Frequency 12 21 33 14

(i) Calculate an estimate of the mean time.

Answer(b)(i) ......................................... min [4]

(ii) Draw a histogram to represent the grouped frequency table.

Frequency
density
2

t
0
10 20 30 40 50
Time (minutes)
[4]
__________________________________________________________________________________________

© UCLES 2014 0580/41/M/J/14 [Turn over


16

1
10 (a) f(x) = 2x – 3 g(x) = +2 h(x) = 3x
x +1
(i) Work out f(4).

Answer(a)(i) ................................................ [1]

(ii) Work out fh(–1).

Answer(a)(ii) ................................................ [2]

(iii) Find f –1(x), the inverse of f(x).

Answer(a)(iii) f –1(x) = ................................................ [2]

(iv) Find ff(x) in its simplest form.

Answer(a)(iv) ff(x) = ................................................ [2]

© UCLES 2014 0580/41/M/J/14


17

(v) Show that the equation f(x) = g(x) simplifies to 2x2 – 3x – 6 = 0 .

Answer(a)(v)

[3]

(vi) Solve the equation 2x2 – 3x – 6 = 0 .

Give your answers correct to 2 decimal places.


Show all your working.

Answer(a)(vi) x = ..................... or x = ..................... [4]


x2 - 3x + 2
(b) Simplify .
x2 + 3x - 10

Answer(b) ................................................ [4]


__________________________________________________________________________________________

© UCLES 2014 0580/41/M/J/14 [Turn over


18

=e o
-3
11 (a)
4
(i) P is the point (–2, 3).

Work out the co-ordinates of Q.

Answer(a)(i) (............. , .............) [1]

(ii) Work out  , the magnitude of .

Answer(a)(ii) ................................................ [2]

© UCLES 2014 0580/41/M/J/14


19

(b)
C

Y
NOT TO
A SCALE

N
a
B

b
O

OACB is a parallelogram.
= a and = b.
2
AN : NB = 2 : 3 and AY = 5 AC.

(i) Write each of the following in terms of a and/or b.


Give your answers in their simplest form.

(a)

Answer(b)(i)(a) = ................................................ [2]

(b)

Answer(b)(i)(b) = ................................................ [2]

(ii) Write down two conclusions you can make about the line segments NY and BC.

Answer(b)(ii) ...............................................................................................................................

..................................................................................................................................................... [2]
__________________________________________________________________________________________

© UCLES 2014 0580/41/M/J/14 [Turn over


20

BLANK PAGE

Permission to reproduce items where third-party owned material protected by copyright is included has been sought and cleared where possible. Every
reasonable effort has been made by the publisher (UCLES) to trace copyright holders, but if any items requiring clearance have unwittingly been included the
publisher will be pleased to make amends at the earliest possible opportunity.

Cambridge International Examinations is part of the Cambridge Assessment Group. Cambridge Assessment is the brand name of University of Cambridge Local
Examinations Syndicate (UCLES), which is itself a department of the University of Cambridge.

© UCLES 2014 0580/41/M/J/14


Cambridge International Examinations
Cambridge International General Certificate of Secondary Education
*0048847567*

MATHEMATICS 0580/42
Paper 4 (Extended) May/June 2014
2 hours 30 minutes
Candidates answer on the Question Paper.
Additional Materials: Electronic calculator Geometrical instruments
Tracing paper (optional)

READ THESE INSTRUCTIONS FIRST

Write your Centre number, candidate number and name on all the work you hand in.
Write in dark blue or black pen.
You may use an HB pencil for any diagrams or graphs.
Do not use staples, paper clips, glue or correction fluid.
DO NOT WRITE IN ANY BARCODES.

Answer all questions.


If working is needed for any question it must be shown below that question.
Electronic calculators should be used.
If the degree of accuracy is not specified in the question, and if the answer is not exact, give the answer to
three significant figures. Give answers in degrees to one decimal place.
For π, use either your calculator value or 3.142.

At the end of the examination, fasten all your work securely together.
The number of marks is given in brackets [ ] at the end of each question or part question.
The total of the marks for this paper is 130.

The syllabus is approved for use in England, Wales and Northern Ireland as a Cambridge International Level 1/Level 2 Certificate.

This document consists of 16 printed pages.

IB14 06_0580_42/2RP
© UCLES 2014 [Turn over
2

1 Jane and Kate share $240 in the ratio 5 : 7 .

(a) Show that Kate receives $140.

Answer(a)

[2]

(b) Jane and Kate each spend $20.

Find the new ratio Jane’s remaining money : Kate’s remaining money.
Give your answer in its simplest form.

Answer(b) ....................... : ....................... [2]

(c) Kate invests $120 for 5 years at 4% per year simple interest.

Calculate the total amount Kate has after 5 years.

Answer(c) $ ................................................ [3]

(d) Jane invests $80 for 3 years at 4% per year compound interest.

Calculate the total amount Jane has after 3 years.


Give your answer correct to the nearest cent.

Answer(d) $ ................................................ [3]

(e) An investment of $200 for 2 years at 4% per year compound interest is the same as an investment of
$200 for 2 years at r % per year simple interest.

Find the value of r.

Answer(e) r = ................................................ [3]


__________________________________________________________________________________________

© UCLES 2014 0580/42/M/J/14


3

1
2 f(x) = – 2x , x ≠ 0
x2
(a) Complete the table of values for f(x).

x –3 –2.5 –2 –1.5 –1 –0.5 0.4 0.5 1 1.5 2

f(x) 6.1 5.2 4.3 3.4 5 5.5 –2.6 –3.8


[3]

(b) On the grid, draw the graph of y = f(x) for –3 Y x Y – 0.5 and 0.4 Y x Y 2 .

y
7

x
–3 –2 –1 0 1 2
–1

–2

–3

–4
[5]

(c) Solve the equation f(x) = 2 .

Answer(c) x = ................................................ [1]

(d) Solve the equation f(x) = 2x + 3 .

Answer(d) x = ................................................ [3]

(e) (i) Draw the tangent to the graph of y = f(x) at the point where x = –1.5 . [1]

(ii) Use the tangent to estimate the gradient of the graph of y = f(x) where x = –1.5 .

Answer(e)(ii) ................................................ [2]


__________________________________________________________________________________________

© UCLES 2014 0580/42/M/J/14 [Turn over


4

3
C
90 m
D

NOT TO
80 m SCALE
95 m

49°
A
55°
B

The diagram shows a quadrilateral ABCD.


Angle BAD = 49° and angle ABD = 55°.
BD = 80 m, BC = 95 m and CD = 90 m.

(a) Use the sine rule to calculate the length of AD.

Answer(a) AD = ............................................ m [3]

(b) Use the cosine rule to calculate angle BCD.

Answer(b) Angle BCD = ................................................ [4]

© UCLES 2014 0580/42/M/J/14


5

(c) Calculate the area of the quadrilateral ABCD.

Answer(c) ........................................... m2 [3]

(d) The quadrilateral represents a field.


Corn seeds are sown across the whole field at a cost of $3250 per hectare.

Calculate the cost of the corn seeds used.


1 hectare = 10 000 m2

Answer(d) $ ................................................ [3]


__________________________________________________________________________________________

© UCLES 2014 0580/42/M/J/14 [Turn over


6

4
y
8

3
Q
2

x
–8 –7 –6 –5 –4 –3 –2 –1 0 1 2 3 4 5 6 7 8
–1

–2

–3

–4

–5

–6

–7

–8

(a) Draw the reflection of shape Q in the line x = –1 . [2]

(b) (i) Draw the enlargement of shape Q, centre (0, 0), scale factor –2 . [2]

(ii) Find the 2 × 2 matrix that represents an enlargement, centre (0, 0), scale factor –2 .

Answer(b)(ii) f p [2]

© UCLES 2014 0580/42/M/J/14


7

(c) (i) Draw the stretch of shape Q, factor 2, x-axis invariant. [2]

(ii) Find the 2 × 2 matrix that represents a stretch, factor 2, x-axis invariant.

Answer(c)(ii) f p [2]

(iii) Find the inverse of the matrix in part (c)(ii).

Answer(c)(iii) f p [2]

(iv) Describe fully the single transformation represented by the matrix in part (c)(iii).

Answer(c)(iv) ..............................................................................................................................

..................................................................................................................................................... [3]
__________________________________________________________________________________________

© UCLES 2014 0580/42/M/J/14 [Turn over


8

5
8 cm

12 cm NOT TO
SCALE

10 cm

4 cm

The diagram shows a cylinder with radius 8 cm and height 12 cm which is full of water.
A pipe connects the cylinder to a cone.
The cone has radius 4 cm and height 10 cm.

(a) (i) Calculate the volume of water in the cylinder.


Show that it rounds to 2410 cm3 correct to 3 significant figures.

Answer(a)(i)

[2]

(ii) Change 2410 cm3 into litres.

Answer(a)(ii) ....................................... litres [1]

© UCLES 2014 0580/42/M/J/14


9

(b) Water flows from the cylinder along the pipe into the cone at a rate of 2 cm3 per second.

Calculate the time taken to fill the empty cone.


Give your answer in minutes and seconds correct to the nearest second.
1
[The volume, V, of a cone with radius r and height h is V = 3 πr 2h.]

Answer(b) .................. min .................. s [4]

(c) Find the number of empty cones which can be filled completely from the full cylinder.

Answer(c) ................................................ [3]


__________________________________________________________________________________________

© UCLES 2014 0580/42/M/J/14 [Turn over


10

NOT TO
S
SCALE
21°
R
117°
T
y° Q

(a) The chords PR and SQ of the circle intersect at T.


Angle RST = 21° and angle STR = 117°.

(i) Find the values of x and y.

Answer(a)(i) x = ................................................

y = ................................................ [2]

(ii) SR = 8.23 cm, RT = 3.31 cm and PQ = 9.43 cm.

Calculate the length of TQ.

Answer(a)(ii) TQ = .......................................... cm [2]

© UCLES 2014 0580/42/M/J/14


11

(b) EFGH is a cyclic quadrilateral.


G H
EF is a diameter of the circle. NOT TO
KE is the tangent to the circle at E. SCALE
GH is parallel to FE and angle KEG = 115°.

F 115° E

Calculate angle GEH. K

Answer(b) Angle GEH = ................................................ [4]

(c) A, B, C and D are points on the circle centre O.


C
Angle AOB = 140° and angle OAC = 14°.
AD = DC.
D NOT TO
SCALE
O
14° 140°

A B

Calculate angle ACD.

Answer(c) Angle ACD = ................................................ [5]


__________________________________________________________________________________________

© UCLES 2014 0580/42/M/J/14 [Turn over


12

7 (a)
1.0

0.8

Frequency 0.6
density
0.4

0.2

m
0
10 20 30 40 50 60 70 80 90 100
Mass (grams)

The histogram shows some information about the masses (m grams) of 39 apples.

(i) Show that there are 12 apples in the interval 70 < m Y 100 .

Answer(a)(i)

[1]

(ii) Calculate an estimate of the mean mass of the 39 apples.

Answer(a)(ii) ............................................. g [5]

(b) The mean mass of 20 oranges is 70 g.


One orange is eaten.
The mean mass of the remaining oranges is 70.5 g.

Find the mass of the orange that was eaten.

Answer(b) ............................................. g [3]


__________________________________________________________________________________________

© UCLES 2014 0580/42/M/J/14


13

8 The distance a train travels on a journey is 600 km.

(a) Write down an expression, in terms of x, for the average speed of the train when

(i) the journey takes x hours,

Answer(a)(i) ....................................... km/h [1]

(ii) the journey takes (x + 1) hours.

Answer(a)(ii) ....................................... km/h [1]

(b) The difference between the average speeds in part(a)(i) and part(a)(ii) is 20 km/h.

(i) Show that x 2 + x – 30 = 0 .

Answer(b)(i)

[3]

(ii) Find the average speed of the train for the journey in part(a)(ii).
Show all your working.

Answer(b)(ii) ....................................... km/h [4]


__________________________________________________________________________________________

© UCLES 2014 0580/42/M/J/14 [Turn over


14

1
9 If the weather is fine the probability that Carlos is late arriving at school is 10 .
1
If the weather is not fine the probability that he is late arriving at school is 3 .
3
The probability that the weather is fine on any day is 4 .

(a) Complete the tree diagram to show this information.

Weather Arriving at school

1
10 Late

3 Fine
4 Not late
........

........ Late
........ Not fine
Not late
........
[3]

(b) In a school term of 60 days, find the number of days the weather is expected to be fine.

Answer(b) ................................................ [1]

(c) Find the probability that the weather is fine and Carlos is late arriving at school.

Answer(c) ................................................ [2]

(d) Find the probability that Carlos is not late arriving at school.

Answer(d) ................................................ [3]

(e) Find the probability that the weather is not fine on at least one day in a school week of 5 days.

Answer(e) ................................................ [2]


__________________________________________________________________________________________

© UCLES 2014 0580/42/M/J/14


15

1
10 f(x) = x , x ≠ 0 g(x) = 1 – x h(x) = x 2 + 1

(a) Find fg` 2 j .


1

Answer(a) ................................................ [2]

(b) Find g–1(x), the inverse of g(x).

Answer(b) g–1(x) = ................................................ [1]

(c) Find hg(x), giving your answer in its simplest form.

Answer(c) hg(x) = ................................................ [3]

(d) Find the value of x when g(x) = 7 .

Answer(d) x = ................................................ [1]

(e) Solve the equation h(x) = 3x.


Show your working and give your answers correct to 2 decimal places.

Answer(e) x = ......................... or x = ......................... [4]

(f) A function k(x) is its own inverse when k –1(x) = k(x).

For which of the functions f(x) , g(x) and h(x) is this true?

Answer(f) ................................................ [1]


__________________________________________________________________________________________

Question 11 is printed on the next page.

© UCLES 2014 0580/42/M/J/14 [Turn over


16

11 The total area of each of the following shapes is X.


The area of the shaded part of each shape is kX.

For each shape, find the value of k and write your answer below each diagram.

NOT TO NOT TO
SCALE J SCALE
NOT TO F
SCALE I
O 72°

K
G
A B C D
H
AB = BC = CD Angle JOK = 72° EF = FG and EI = IH

k = ..................................... k = ..................................... k = .....................................

A
NOT TO NOT TO
SCALE SCALE

O B

The shape is a regular hexagon. The diagram shows a sector of a circle centre O.
Angle AOB = 90°

k = ..................................... k = .....................................
[10]

Permission to reproduce items where third-party owned material protected by copyright is included has been sought and cleared where possible. Every
reasonable effort has been made by the publisher (UCLES) to trace copyright holders, but if any items requiring clearance have unwittingly been included the
publisher will be pleased to make amends at the earliest possible opportunity.

Cambridge International Examinations is part of the Cambridge Assessment Group. Cambridge Assessment is the brand name of University of Cambridge Local
Examinations Syndicate (UCLES), which is itself a department of the University of Cambridge.

© UCLES 2014 0580/42/M/J/14


Cambridge International Examinations
Cambridge International General Certificate of Secondary Education
*9468931136*

MATHEMATICS 0580/43
Paper 4 (Extended) May/June 2014
2 hours 30 minutes
Candidates answer on the Question Paper.
Additional Materials: Electronic calculator Geometrical instruments
Tracing paper (optional)

READ THESE INSTRUCTIONS FIRST

Write your Centre number, candidate number and name on all the work you hand in.
Write in dark blue or black pen.
You may use an HB pencil for any diagrams or graphs.
Do not use staples, paper clips, glue or correction fluid.
DO NOT WRITE IN ANY BARCODES.

Answer all questions.


If working is needed for any question it must be shown below that question.
Electronic calculators should be used.
If the degree of accuracy is not specified in the question, and if the answer is not exact, give the answer to
three significant figures. Give answers in degrees to one decimal place.
For π, use either your calculator value or 3.142.

At the end of the examination, fasten all your work securely together.
The number of marks is given in brackets [ ] at the end of each question or part question.
The total of the marks for this paper is 130.

The syllabus is approved for use in England, Wales and Northern Ireland as a Cambridge International Level 1/Level 2 Certificate.

This document consists of 16 printed pages.

IB14 06_0580_43/2RP
© UCLES 2014 [Turn over
2

1 In July, a supermarket sold 45 981 bottles of fruit juice.

(a) The cost of a bottle of fruit juice was $1.35 .

Calculate the amount received from the sale of the 45 981 bottles.
Give your answer correct to the nearest hundred dollars.

Answer(a) $ ................................................ [2]

(b) The number of bottles sold in July was 17% more than the number sold in January.

Calculate the number of bottles sold in January.

Answer(b) ................................................ [3]

(c) There were 3 different flavours of fruit juice.


The number of bottles sold in each flavour was in the ratio apple : orange : cherry = 3 : 4 : 2.
The total number of bottles sold was 45 981.

Calculate the number of bottles of orange juice sold.

Answer(c) ................................................ [2]

(d) One bottle contains 1.5 litres of fruit juice.

Calculate the number of 330 ml glasses that can be filled completely from one bottle.

Answer(d) ................................................ [3]


5
(e) 9
of the 45 981 bottles are recycled.

Calculate the number of bottles that are recycled.

Answer(e) ................................................ [2]


__________________________________________________________________________________________

© UCLES 2014 0580/43/M/J/14


3

2
4

Frequency
density
2

0
10 20 30 40 50 60
Amount ($x)

A survey asked 90 people how much money they gave to charity in one month.
The histogram shows the results of the survey.

(a) Complete the frequency table for the six columns in the histogram.

Amount ($x) 0 < x Y 10

Frequency 4

[5]

(b) Use your frequency table to calculate an estimate of the mean amount these 90 people gave to charity.

Answer(b) $ ................................................ [4]


__________________________________________________________________________________________

© UCLES 2014 0580/43/M/J/14 [Turn over


4

3 (a)
P

12 cm NOT TO
SCALE
X 17 cm

The diagram shows triangle PQR with PQ = 12 cm and PR = 17 cm.


The area of triangle PQR is 97 cm2 and angle QPR is acute.

(i) Calculate angle QPR.

Answer(a)(i) Angle QPR = ................................................ [3]

(ii) The midpoint of PQ is X.

Use the cosine rule to calculate the length of XR.

Answer(a)(ii) XR = .......................................... cm [4]

© UCLES 2014 0580/43/M/J/14


5

(b)
NOT TO
9.4 cm 42° a cm SCALE

37°

Calculate the value of a.

Answer(b) a = ................................................ [4]

(c) sin x = cos 40°, 0° Y x Y 180°

Find the two values of x.

Answer(c) x = .................. or x = .................. [2]


__________________________________________________________________________________________

© UCLES 2014 0580/43/M/J/14 [Turn over


6

1
4 The table shows some values for the function y = + x , x ≠ 0.
x2

x –3 –2 –1 –0.5 0.5 1 2 3 4

y –2.89 –1.75 3.5 2 2.25 4.06

(a) Complete the table of values. [3]


1
(b) On the grid, draw the graph of y = + x for –3 Y x Y – 0.5 and 0.5 Y x Y 4.
x2
y
5

x
–3 –2 –1 0 1 2 3 4

–1

–2

–3
[5]

© UCLES 2014 0580/43/M/J/14


7

1
(c) Use your graph to solve the equation +x–3=0.
x2

Answer(c) x = ..................... or x = ..................... or x = ..................... [3]

1
(d) Use your graph to solve the equation + x = 1 – x.
x2

Answer(d) x = ................................................ [3]

(e) By drawing a suitable tangent, find an estimate of the gradient of the curve at the point where x = 2.

Answer(e) ................................................ [3]


3 -1 .
(f) Using algebra, show that you can use the graph at y = 0 to find

Answer(f)

[3]
__________________________________________________________________________________________

© UCLES 2014 0580/43/M/J/14 [Turn over


8

5 (a)
y
5
A
4
3
2
B
1
x
0
1 2 3 4 5 6 7 8

(i) Write down the position vector of A.

Answer(a)(i) f p [1]

(ii) Find ì ì , the magnitude of .

Answer(a)(ii) ................................................ [2]

(b)
S
NOT TO
Q SCALE

O p P R

O is the origin, = p and = q.


OP is extended to R so that OP = PR.
OQ is extended to S so that OQ = QS.

(i) Write down in terms of p and q.

Answer(b)(i) = ................................................ [1]

(ii) PS and RQ intersect at M and RM = 2MQ.

Use vectors to find the ratio PM : PS, showing all your working.

Answer(b)(ii) PM : PS = ....................... : ....................... [4]


__________________________________________________________________________________________

© UCLES 2014 0580/43/M/J/14


9

6 In this question, give all your answers as fractions.

N A T I O N

The letters of the word NATION are printed on 6 cards.

(a) A card is chosen at random.

Write down the probability that

(i) it has the letter T printed on it,

Answer(a)(i) ................................................ [1]

(ii) it does not have the letter N printed on it,

Answer(a)(ii) ................................................ [1]

(iii) the letter printed on it has no lines of symmetry.

Answer(a)(iii) ................................................ [1]

(b) Lara chooses a card at random, replaces it, then chooses a card again.

Calculate the probability that only one of the cards she chooses has the letter N printed on it.

Answer(b) ................................................ [3]

(c) Jacob chooses a card at random and does not replace it.
He continues until he chooses a card with the letter N printed on it.

Find the probability that this happens when he chooses the 4th card.

Answer(c) ................................................ [3]


__________________________________________________________________________________________

© UCLES 2014 0580/43/M/J/14 [Turn over


10

7 (a)
E D
t° x°
NOT TO
SCALE
x° C

q° p° 32°
Y A B X

ABCDEF is a hexagon.
AB is parallel to ED and BC is parallel to FE.
YFE and YABX are straight lines.
Angle CBX = 32° and angle EFA = 90°.

Calculate the value of

(i) p,

Answer(a)(i) p = ................................................ [1]

(ii) q,

Answer(a)(ii) q = ................................................ [2]

(iii) t,

Answer(a)(iii) t = ................................................ [1]

(iv) x.

Answer(a)(iv) x = ................................................ [3]

© UCLES 2014 0580/43/M/J/14


11

(b)
R
x° Q

S

NOT TO
SCALE

63°
T P U

P, Q, R and S are points on a circle and PS = SQ.


PR is a diameter and TPU is the tangent to the circle at P.
Angle SPT = 63°.

Find the value of

(i) x,

Answer(b)(i) x = ................................................ [2]

(ii) y.

Answer(b)(ii) y = ................................................ [2]


__________________________________________________________________________________________

© UCLES 2014 0580/43/M/J/14 [Turn over


12

7 2x - 3
8 (a) (i) Show that the equation + = 1 can be simplified to 2x2 + 3x – 6 = 0 .
x+4 2
Answer(a)(i)

[3]

(ii) Solve the equation 2x2 + 3x – 6 = 0 .

Show all your working and give your answers correct to 2 decimal places.

Answer(a)(ii) x = ........................... or x = ........................... [4]

(b) The total surface area of a cone with radius x and slant height 3x is equal to the area of a circle with
radius r.

Show that r = 2x.


[The curved surface area, A, of a cone with radius r and slant height l is A = πrl.]

Answer(b)

[4]
__________________________________________________________________________________________

© UCLES 2014 0580/43/M/J/14


13
–x
9 f(x) = 4 – 3x g(x) = 3

(a) Find f(2x) in terms of x.

Answer(a) f(2x) = ................................................ [1]

(b) Find ff(x) in its simplest form.

Answer(b) ff(x) = ................................................ [2]

(c) Work out gg(–1).


Give your answer as a fraction.

Answer(c) ................................................ [3]

(d) Find f –1(x), the inverse of f(x).

Answer(d) f –1(x) = ................................................ [2]

(e) Solve the equation gf(x) = 1.

Answer(e) x = ................................................ [3]


__________________________________________________________________________________________

© UCLES 2014 0580/43/M/J/14 [Turn over


14

10 (a)

8 cm NOT TO
SCALE

r cm

The three sides of an equilateral triangle are tangents to a circle of radius r cm.
The sides of the triangle are 8 cm long.

Calculate the value of r.


Show that it rounds to 2.3, correct to 1 decimal place.

Answer(a)

[3]

(b)

8 cm
NOT TO
SCALE

12 cm

The diagram shows a box in the shape of a triangular prism of height 12 cm.
The cross section is an equilateral triangle of side 8 cm.

Calculate the volume of the box.

Answer(b) ......................................... cm3 [4]

© UCLES 2014 0580/43/M/J/14


15

(c) The box contains biscuits.


Each biscuit is a cylinder of radius 2.3 centimetres and height 4 millimetres.

Calculate

(i) the largest number of biscuits that can be placed in the box,

Answer(c)(i) ................................................ [3]

(ii) the volume of one biscuit in cubic centimetres,

Answer(c)(ii) ......................................... cm3 [2]

(iii) the percentage of the volume of the box not filled with biscuits.

Answer(c)(iii) ............................................ % [3]


__________________________________________________________________________________________

Question 11 is printed on the next page.

© UCLES 2014 0580/43/M/J/14 [Turn over


16

11

Diagram 1 Diagram 2 Diagram 3

The first three diagrams in a sequence are shown above.


Diagram 1 shows an equilateral triangle with sides of length 1 unit.
1
In Diagram 2, there are 4 triangles with sides of length 2 unit.
1
In Diagram 3, there are 16 triangles with sides of length 4 unit.

(a) Complete this table for Diagrams 4, 5, 6 and n.

Diagram 1 Diagram 2 Diagram 3 Diagram 4 Diagram 5 Diagram 6 Diagram n

1 1
Length of side 1 2 4

Length of side
20 2–1 2–2
as a power of 2
[6]

(b) (i) Complete this table for the number of the smallest triangles in Diagrams 4, 5 and 6.

Diagram 1 Diagram 2 Diagram 3 Diagram 4 Diagram 5 Diagram 6

Number of smallest
1 4 16
triangles
Number of smallest
20 22 24
triangles as a power of 2
[2]

(ii) Find the number of the smallest triangles in Diagram n, giving your answer as a power of 2.

Answer(b)(ii) ................................................ [1]

(c) Calculate the number of the smallest triangles in the diagram where the smallest triangles have sides of
1
length 128 unit.

Answer(c) ................................................ [2]

Permission to reproduce items where third-party owned material protected by copyright is included has been sought and cleared where possible. Every
reasonable effort has been made by the publisher (UCLES) to trace copyright holders, but if any items requiring clearance have unwittingly been included the
publisher will be pleased to make amends at the earliest possible opportunity.

Cambridge International Examinations is part of the Cambridge Assessment Group. Cambridge Assessment is the brand name of University of Cambridge Local
Examinations Syndicate (UCLES), which is itself a department of the University of Cambridge.

© UCLES 2014 0580/43/M/J/14


Cambridge International Examinations
Cambridge International General Certificate of Secondary Education
* 1 0 4 9 1 1 8 7 9 9 *

maThEmaTICs 0580/41
Paper 4 (Extended) may/June 2016
2 hours 30 minutes
Candidates answer on the Question Paper.
Additional Materials: Electronic calculator Geometrical instruments
Tracing paper (optional).

REaD ThEsE INsTRUCTIONs FIRsT

Write your Centre number, candidate number and name on all the work you hand in.
Write in dark blue or black pen.
You may use an HB pencil for any diagrams or graphs.
Do not use staples, paper clips, glue or correction fluid.
DO NOT WRITE IN ANY BARCODES.

Answer all questions.


If working is needed for any question it must be shown below that question.
Electronic calculators should be used.
If the degree of accuracy is not specified in the question, and if the answer is not exact, give the answer to
three significant figures. Give answers in degrees to one decimal place.
For r, use either your calculator value or 3.142.

At the end of the examination, fasten all your work securely together.
The number of marks is given in brackets [ ] at the end of each question or part question.
The total of the marks for this paper is 130.

The syllabus is approved for use in England, Wales and Northern Ireland as a Cambridge International Level 1/Level 2 Certificate.

This document consists of 19 printed pages and 1 blank page.

DC (CW/FD) 112697/3
© UCLES 2016 [Turn over
2

1 (a) Kristian and Stephanie share some money in the ratio 3 : 2.


Kristian receives $72.

(i) Work out how much Stephanie receives.

$ .................................................. [2]

(ii) Kristian spends 45% of his $72 on a computer game.

Calculate the price of the computer game.

$ .................................................. [1]

(iii) Kristian also buys a meal for $8.40 .

Calculate the fraction of the $72 Kristian has left after buying the computer game and the meal.
Give your answer in its lowest terms.

................................................... [2]

(iv) Stephanie buys a book in a sale for $19.20 .


This sale price is after a reduction of 20%.

Calculate the original price of the book.

$ .................................................. [3]
© UCLES 2016 0580/41/M/J/16
3

(b) Boris invests $550 at a rate of 2% per year simple interest.

Calculate the amount Boris has after 10 years.

$ .................................................. [3]

(c) Marlene invests $550 at a rate of 1.9% per year compound interest.

Calculate the amount Marlene has after 10 years.

$ .................................................. [2]

(d) Hans invests $550 at a rate of x% per year compound interest.


At the end of 10 years he has a total amount of $638.30, correct to the nearest cent.

Find the value of x.

x = .................................................. [3]
© UCLES 2016 0580/41/M/J/16 [Turn over
4

2 (a)
y

3
Q
2

x
–7 –6 –5 –4 –3 –2 –1 0 1 2 3 4 5 6 7 8 9 10
–1

–2
T
–3

–4

–5

–6

(i) Draw the image of triangle T after a translation by the vector c m .


5
[2]
-2

(ii) Draw the image of triangle T after a reflection in the line y = 1. [2]

(iii) Describe fully the single transformation that maps triangle T onto triangle Q.

......................................................................................................................................................

...................................................................................................................................................... [3]

© UCLES 2016 0580/41/M/J/16


5

(b) M = c m N=c m P=c m


1 2 4 3 1 3

3 4 1 k 0 6
(i) Work out M + P.

f p [1]

(ii) Work out PM.

f p [2]

(iii) M = N

Find the value of k.

k = .................................................. [3]

0 -1
(c) (i) Describe fully the single transformation represented by the matrix c m.
1 0

......................................................................................................................................................

...................................................................................................................................................... [3]

(ii) Find the matrix which represents a reflection in the line y = x.

f p [2]

© UCLES 2016 0580/41/M/J/16 [Turn over


6

3 (a) 200 students estimate the volume, V m3, of a classroom.


The cumulative frequency diagram shows their results.

200

180

160

140

120
Cumulative
frequency
100

80

60

40

20

0 V
50 100 150 200 250 300 350 400 450 500
Volume (m3)

Find

(i) the median,

............................................. m3 [1]

(ii) the lower quartile,

............................................. m3 [1]

(iii) the inter-quartile range,

............................................. m3 [1]

(iv) the number of students who estimate that the volume is greater than 300 m3.

................................................... [2]

© UCLES 2016 0580/41/M/J/16


7

(b) The 200 students also estimate the total area, A m2, of the windows in the classroom.
The results are shown in the table.

Area (A m2) 20 1 A G 60 60 1 A G 100 100 1 A G 150 150 1 A G 250


Frequency 32 64 80 24

(i) Calculate an estimate of the mean.


Show all your working.

............................................. m2 [4]

(ii) Complete the histogram to show the information in the table.

Frequency
density 1

A
0 50 100 150 200 250
Area (m2)
[4]

(iii) Two of the 200 students are chosen at random.

Find the probability that they both estimate that the area is greater than 100 m2.

................................................... [2]

© UCLES 2016 0580/41/M/J/16 [Turn over


8

4 (a) Calculate the volume of a metal sphere of radius 15 cm and show that it rounds to 14 140 cm3, correct
to 4 significant figures.
[The volume, V, of a sphere with radius r is V = 43 rr 3 .]

[2]

(b) (i) The sphere is placed inside an empty cylindrical tank of radius 25 cm and height 60 cm.
The tank is filled with water.

25 cm
NOT TO
SCALE
60 cm

Calculate the volume of water required to fill the tank.

........................................... cm3 [3]

(ii) The sphere is removed from the tank.

NOT TO
SCALE

Calculate the depth, d, of water in the tank.

d = ........................................... cm [2]
© UCLES 2016 0580/41/M/J/16
9

(c) The sphere is melted down and the metal is made into a solid cone of height 54 cm.

(i) Calculate the radius of the cone.


[The volume, V, of a cone with radius r and height h is V = 13 rr 2 h .]

............................................ cm [3]

(ii) Calculate the total surface area of the cone.


[The curved surface area, A, of a cone with radius r and slant height l is A = rrl .]

........................................... cm2 [4]

© UCLES 2016 0580/41/M/J/16 [Turn over


10

20
5 f (x) = + x, x ! 0
x
(a) Complete the table.

x -10 -8 -5 -2 -1.6 1.6 2 5 8 10

f(x) -12 -10.5 -9 -12 -14.1 14.1 12 12


[2]

(b) On the grid, draw the graph of y = f(x) for - 10 G x G - 1.6 and 1.6 G x G 10 .

18

16

14

12

10

x
–10 –8 –6 –4 –2 0 2 4 6 8 10
–2

–4

–6

–8

–10

–12

–14

–16

–18
[5]

© UCLES 2016 0580/41/M/J/16


11

(c) Using your graph, solve the equation f(x) = 11.

x = .................... or x = .................... [2]

(d) k is a prime number and f(x) = k has no solutions.

Find the possible values of k.

................................................... [2]

(e) The gradient of the graph of y = f(x) at the point (2, 12) is -4.

Write down the co-ordinates of the other point on the graph of y = f(x) where the gradient is -4.

(....................... , .......................) [1]

(f) (i) The equation f(x) = x2 can be written as x 3 + px 2 + q = 0 .

Show that p = -1 and q = -20.

[2]

(ii) On the grid opposite, draw the graph of y = x2 for - 4 G x G 4 . [2]

(iii) Using your graphs, solve the equation x 3 - x 2 - 20 = 0 .

x = .................................................. [1]

(iv)
y

NOT TO
SCALE

0 P x

The diagram shows a sketch of the graph of y = x 3 - x 2 - 20 .


P is the point (n, 0).

Write down the value of n.

n = .................................................. [1]
© UCLES 2016 0580/41/M/J/16 [Turn over
12

6 (a)

x cm NOT TO
SCALE

The perimeter of the rectangle is 80 cm.


The area of the rectangle is A cm2.

(i) Show that x 2 - 40x + A = 0 .

[3]

(ii) When A = 300, solve, by factorising, the equation x 2 - 40x + A = 0 .

x = ..................... or x = ....................[3]

(iii) When A = 200, solve, by using the quadratic formula, the equation x 2 - 40x + A = 0 .
Show all your working and give your answers correct to 2 decimal places.

x = ..................... or x = ....................[4]

© UCLES 2016 0580/41/M/J/16


13

(b) A car completes a 200 km journey with an average speed of x km/h.


The car completes the return journey of 200 km with an average speed of (x + 10) km/h.
2000
(i) Show that the difference between the time taken for each of the two journeys is hours.
x (x + 10)

[3]

(ii) Find the difference between the time taken for each of the two journeys when x = 80.
Give your answer in minutes and seconds.

.................... min .................... s [3]

© UCLES 2016 0580/41/M/J/16 [Turn over


14

7
R M Q

r NOT TO
SCALE

O p P

OPQR is a rectangle and O is the origin.


M is the midpoint of RQ and PT : TQ = 2 : 1.
OP = p and OR = r.

(a) Find, in terms of p and/or r, in its simplest form

(i) MQ,

MQ = .................................................. [1]

(ii) MT ,

MT = .................................................. [1]

(iii) OT .

OT = .................................................. [1]

(b) RQ and OT are extended to meet at U.

Find the position vector of U in terms of p and r.


Give your answer in its simplest form.

................................................... [2]

© UCLES 2016 0580/41/M/J/16


15

2k
(c) MT = c m and MT = 180 .
-k
Find the positive value of k.

k = .................................................. [3]

© UCLES 2016 0580/41/M/J/16 [Turn over


16

8 f(x) = 2x + 1 g(x) = x2 + 4 h(x) = 2x

(a) Solve the equation f(x) = g(1).

x = .................................................. [2]

(b) Find the value of fh(3).

................................................... [2]

(c) Find f -1(x).

f -1(x) = .................................................. [2]

(d) Find gf(x) in its simplest form.

................................................... [3]

© UCLES 2016 0580/41/M/J/16


17

(e) Solve the equation h-1(x) = 0.5 .

x = .................................................. [1]
1
(f) = 2 kx
h (x)
Write down the value of k.

k = .................................................. [1]

© UCLES 2016 0580/41/M/J/16 [Turn over


18

9
y

2 B

A x NOT TO
–4 0 4 SCALE

–2

x2 y
2
The diagram shows a curve with equation + = 1.
a2 b2
(a) A is the point (4, 0) and B is the point (0, 2).

(i) Find the equation of the straight line that passes through A and B.
Give your answer in the form y = mx + c.

y = .................................................. [3]

(ii) Show that a2 = 16 and b2 = 4.

[2]

© UCLES 2016 0580/41/M/J/16


19

(b)
y

2 P

NOT TO
x
–4 O 4 SCALE

Q
–2

x2 y
2
P (2, k) and Q (2, -k) are points on the curve + = 1.
16 4
(i) Find the value of k.

k = .................................................. [3]

(ii) Calculate angle POQ.

Angle POQ = .................................................. [3]


x2 y
2
(c) The area enclosed by a curve with equation + = 1 is rab .
a2 b2

x2 y
2
(i) Find the area enclosed by the curve + = 1.
16 4
Give your answer as a multiple of r.

................................................... [1]

(ii) A curve, mathematically similar to the one in the diagrams, intersects the x-axis at (12, 0) and
(-12, 0).

Work out the area enclosed by this curve, giving your answer as a multiple of r.

................................................... [2]
© UCLES 2016 0580/41/M/J/16
20

BLANK PAGE

Permission to reproduce items where third-party owned material protected by copyright is included has been sought and cleared where possible. Every reasonable
effort has been made by the publisher (UCLES) to trace copyright holders, but if any items requiring clearance have unwittingly been included, the publisher will
be pleased to make amends at the earliest possible opportunity.

To avoid the issue of disclosure of answer-related information to candidates, all copyright acknowledgements are reproduced online in the Cambridge International
Examinations Copyright Acknowledgements Booklet. This is produced for each series of examinations and is freely available to download at www.cie.org.uk after
the live examination series.

Cambridge International Examinations is part of the Cambridge Assessment Group. Cambridge Assessment is the brand name of University of Cambridge Local
Examinations Syndicate (UCLES), which is itself a department of the University of Cambridge.

© UCLES 2016 0580/41/M/J/16


Cambridge International Examinations
Cambridge International General Certificate of Secondary Education
* 4 4 9 0 9 8 7 7 9 8 *

MATHEMATICS 0580/42
Paper 4 (Extended) May/June 2016
2 hours 30 minutes
Candidates answer on the Question Paper.
Additional Materials: Electronic calculator Geometrical instruments
Tracing paper (optional)

READ THESE INSTRUCTIONS FIRST

Write your Centre number, candidate number and name on all the work you hand in.
Write in dark blue or black pen.
You may use an HB pencil for any diagrams or graphs.
Do not use staples, paper clips, glue or correction fluid.
DO NOT WRITE IN ANY BARCODES.

Answer all questions.


If working is needed for any question it must be shown below that question.
Electronic calculators should be used.
If the degree of accuracy is not specified in the question, and if the answer is not exact, give the answer to
three significant figures. Give answers in degrees to one decimal place.
For π, use either your calculator value or 3.142.

At the end of the examination, fasten all your work securely together.
The number of marks is given in brackets [ ] at the end of each question or part question.
The total of the marks for this paper is 130.

The syllabus is approved for use in England, Wales and Northern Ireland as a Cambridge International Level 1/Level 2 Certificate.

This document consists of 16 printed pages.

DC (NH/SW) 112696/3
© UCLES 2016 [Turn over
2

1 Mr Chan flies from London to Los Angeles, a distance of 8800 km.


The flight takes 11 hours and 10 minutes.

(a) (i) His plane leaves London at 09 35 local time.


The local time in Los Angeles is 8 hours behind the time in London.

Calculate the local time when the plane arrives in Los Angeles.

.................................................. [2]

(ii) Work out the average speed of the plane in km/h.

..........................................km/h [2]

(b) There are three types of tickets, economy, business and first class.
The price of these tickets is in the ratio economy : business : first class = 2 : 5 : 9.

(i) The price of a business ticket is $2350.

Calculate the price of a first class ticket.

$ .................................................. [2]

(ii) Work out the price of an economy ticket as a percentage of the price of a first class ticket.

...............................................% [1]

(c) The price of a business ticket for the same journey with another airline is $2240.

(i) The price of a first class ticket is 70% more than a business ticket.

Calculate the price of this first class ticket.

$ .................................................. [2]

© UCLES 2016 0580/42/M/J/16


3

(ii) The price of a business ticket is 180% more than an economy ticket.

Calculate the price of this economy ticket.

$ .................................................. [3]

(d) Mr Chan hires a car in Los Angeles.


The charges are shown below.

Car Hire

$28.00 per day plus $6.50 per day insurance.

$1.25 for every kilometre travelled after the first 800 km.
The first 800 km are included in the price.

Mr Chan hired the car for 12 days and paid $826.50 .

(i) Find the number of kilometres Mr Chan travelled in this car.

............................................. km [4]

(ii) The car used fuel at an average rate of 1 litre for every 10 km travelled.
Fuel costs $1.30 per litre.

Calculate the cost of the fuel used by the car during the 12 days.

$ .................................................. [2]

© UCLES 2016 0580/42/M/J/16 [Turn over


4

2 (a) Work out the value of x in each of the following.

(i) 3x = 243

x = ................................................. [1]

(ii) 16x = 4

x = ................................................. [1]

(iii) 8x = 32

x = ................................................. [2]
1
(iv) 27x =
9

x = ................................................. [2]

(b) Solve by factorisation.


y2 – 7y – 30 = 0

Show your working.

y = ................... or y = ................... [3]

© UCLES 2016 0580/42/M/J/16


5

3 (a)
y

3
A
2

x
–4 –3 –2 –1 0 1 2 3 4 5 6 7
–1

–2

–3

–4

–5

–6

On the grid, draw the image of

(i) shape A after a reflection in the line x = 1, [2]

(ii) shape A after an enlargement with scale factor –2, centre (0, 1), [2]

0 -1
(iii) shape A after the transformation represented by the matrix c m . [3]
1 0

3 0
(b) Describe fully the single transformation represented by the matrix c m.
0 3

..............................................................................................................................................................

.............................................................................................................................................................. [3]

© UCLES 2016 0580/42/M/J/16 [Turn over


6

1
4 f(x) = x2 – –4, x≠0
x

(a) (i) Complete the table.

x –3 –2 –1 –0.5 –0.1 0.2 0.5 1 2 3

f(x) 5.3 0.5 –1.8 6.0 –9.0 –5.8 –4 4.7


[2]

(ii) On the grid, draw the graph of y = f(x) for –3 G x G –0.1 and 0.2 G x G 3.

y
10

x
–3 –2 –1 0 1 2 3
–1

–2

–3

–4

–5

–6

–7

–8

–9

–10
[5]

© UCLES 2016 0580/42/M/J/16


7

(b) Use your graph to solve the equation f(x) = 0.

x = ................. or x = ................. or x = ................... [3]

(c) Find an integer k, for which f(x) = k has one solution.

k = ................................................. [1]

(d) (i) By drawing a suitable straight line, solve the equation f(x) + 2 = – 5x.

x = ................... or x = ................... [4]

(ii) f(x) + 2 = – 5x can be written as x3 + ax2 + bx – 1 = 0.

Find the value of a and the value of b.

a = .................................................

b = ................................................. [2]

© UCLES 2016 0580/42/M/J/16 [Turn over


8

5 Kiah plays a game.


The game involves throwing a coin onto a circular board.
Points are scored for where the coin lands on the board.

10

20

If the coin lands on part of a line or misses the board then 0 points are scored.
The table shows the probabilities of Kiah scoring points on the board with one throw.

Points scored 20 10 5 0

Probability x 0.2 0.3 0.45

(a) Find the value of x.

x = ................................................. [2]

(b) Kiah throws a coin fifty times.

Work out the expected number of times she scores 5 points.

.................................................. [1]

(c) Kiah throws a coin two times.

Calculate the probability that

(i) she scores either 5 or 0 with her first throw,

.................................................. [2]

(ii) she scores 0 with her first throw and 5 with her second throw,

.................................................. [2]

© UCLES 2016 0580/42/M/J/16


9

(iii) she scores a total of 15 points with her two throws.

.................................................. [3]

(d) Kiah throws a coin three times.

Calculate the probability that she scores a total of 10 points with her three throws.

.................................................. [5]

© UCLES 2016 0580/42/M/J/16 [Turn over


10

6 The diagram shows a cuboid.

F G

E H
30 cm NOT TO
SCALE

B C
35 cm
A 60 cm D

AD = 60 cm, CD = 35 cm and CG = 30 cm.

(a) Write down the number of planes of symmetry of this cuboid.

.................................................. [1]

(b) (i) Work out the surface area of the cuboid.

........................................... cm2 [3]

(ii) Write your answer to part (b)(i) in square metres.

............................................. m2 [1]

(c) Calculate

(i) the length AG,

AG = ............................................ cm [4]

© UCLES 2016 0580/42/M/J/16


11

(ii) the angle between AG and the base ABCD.

.................................................. [3]

(d) (i) Show that the volume of the cuboid is 63 000 cm3.

[1]

(ii) A cylinder of height 40 cm has the same volume as the cuboid.

Calculate the radius of the cylinder.

............................................. cm [3]

© UCLES 2016 0580/42/M/J/16 [Turn over


12

NOT TO
SCALE
210°
cm
88cm
O
A
B

8 cm

72°

The diagram shows a design for a logo made from a sector and two triangles.
The sector, centre O, has radius 8 cm and sector angle 210°.
AC = 8 cm and angle ACB = 72°.

(a) Show that angle OAB = 15°.

[2]

(b) Calculate the length of the straight line AB.

AB = ............................................ cm [4]

© UCLES 2016 0580/42/M/J/16


13

(c) Calculate angle ABC.

Angle ABC = ................................................. [3]

(d) Calculate the total area of the logo design.

............................................cm2 [6]

(e) The logo design is an enlargement with scale factor 4 of the actual logo.

Calculate the area of the actual logo.

............................................cm2 [2]

© UCLES 2016 0580/42/M/J/16 [Turn over


14

4
8 f(x) = 5x + 7 g(x) = , x≠3
x-3
(a) Find

(i) fg(1),

.................................................. [2]

(ii) gf(x),

.................................................. [2]

(iii) g–1(x),

g–1(x) = ................................................. [3]

(iv) f –1f(2).

.................................................. [1]

© UCLES 2016 0580/42/M/J/16


15

(b) f(x) = g(x)

(i) Show that 5x2 – 8x – 25 = 0.

[3]

(ii) Solve 5x2 – 8x – 25 = 0.


Show all your working and give your answers correct to 2 decimal places.

x = ................... or x = ................... [4]

Question 9 is printed on the next page.

© UCLES 2016 0580/42/M/J/16 [Turn over


16

9 A line joins the points A (–2, –5) and B (4, 13).

(a) Calculate the length AB.

AB = ................................................. [3]

(b) Find the equation of the line through A and B.


Give your answer in the form y = mx + c.

y = ................................................. [3]

(c) Another line is parallel to AB and passes through the point (0, –5).

Write down the equation of this line.

.................................................. [2]

(d) Find the equation of the perpendicular bisector of AB.

.................................................. [5]

Permission to reproduce items where third-party owned material protected by copyright is included has been sought and cleared where possible. Every
reasonable effort has been made by the publisher (UCLES) to trace copyright holders, but if any items requiring clearance have unwittingly been included, the
publisher will be pleased to make amends at the earliest possible opportunity.

To avoid the issue of disclosure of answer-related information to candidates, all copyright acknowledgements are reproduced online in the Cambridge International
Examinations Copyright Acknowledgements Booklet. This is produced for each series of examinations and is freely available to download at www.cie.org.uk after
the live examination series.

Cambridge International Examinations is part of the Cambridge Assessment Group. Cambridge Assessment is the brand name of University of Cambridge Local
Examinations Syndicate (UCLES), which is itself a department of the University of Cambridge.

© UCLES 2016 0580/42/M/J/16


Cambridge International Examinations
Cambridge International General Certificate of Secondary Education
* 5 1 4 7 8 1 6 0 5 1 *

maThEmaTICs 0580/43
Paper 4 (Extended) may/June 2016
2 hours 30 minutes
Candidates answer on the Question Paper.
Additional Materials: Electronic calculator Geometrical instruments
Tracing paper (optional).

REaD ThEsE INsTRUCTIONs FIRsT

Write your Centre number, candidate number and name on all the work you hand in.
Write in dark blue or black pen.
You may use an HB pencil for any diagrams or graphs.
Do not use staples, paper clips, glue or correction fluid.
DO NOT WRITE IN ANY BARCODES.

Answer all questions.


If working is needed for any question it must be shown below that question.
Electronic calculators should be used.
If the degree of accuracy is not specified in the question, and if the answer is not exact, give the answer to
three significant figures. Give answers in degrees to one decimal place.
For π, use either your calculator value or 3.142.

At the end of the examination, fasten all your work securely together.
The number of marks is given in brackets [ ] at the end of each question or part question.
The total of the marks for this paper is 130.

The syllabus is approved for use in England, Wales and Northern Ireland as a Cambridge International Level 1/Level 2 Certificate.

This document consists of 15 printed pages and 1 blank page.

DC (LEG/SG) 112704/2
© UCLES 2016 [Turn over
2

1 A football club sells tickets at different prices dependent on age group.

(a) (i) At one game, the club sold tickets in the ratio

under 18 : 18 to 60 : over 60 = 2 : 7 : 3.

There were 6100 tickets sold for people aged under 18.

Calculate the total number of tickets sold for the game.

................................................... [3]

(ii) Calculate the percentage of tickets sold for people aged under 18.

..............................................% [1]

(b) The table shows the football ticket prices for the different age groups.

Age Price
Under 18 $15

18 to 60 $35

Over 60 $18

At a different game there were 42 600 tickets sold.

• 14% were sold to people aged under 18


• 23 of the tickets were sold to people aged 18 to 60
• The remainder were sold to people aged over 60

Calculate the total amount the football club receives from ticket sales for this game.

$ .................................................. [5]

© UCLES 2016 0580/43/M/J/16


3

(c) In a sale, the football club shop reduced the price of the football shirts to $23.80 .
An error was made when working out this sale price.
The price was reduced by 30% instead of 20%.

Calculate the correct sale price for the football shirt.

$ ................................................... [5]

2 (a) Solve the inequality.


5x – 3 > 9

................................................... [2]

(b) Factorise completely.

(i) xy – 18 + 3y – 6x

................................................... [2]

(ii) 8x 2 - 72y 2

................................................... [3]

(c) Make r the subject of the formula.


1 - 2r
p+5 =
r

r = .................................................. [4]
© UCLES 2016 0580/43/M/J/16 [Turn over
4

3 The diagram shows the graph of y = f(x) for - 3.5 G x G 2.5 .

16

15

14

13

12

11

10

x
–3 –2 –1 0 1 2
–1

–2

–3

–4

© UCLES 2016 0580/43/M/J/16


5

(a) (i) Find f(– 2).

................................................... [1]

(ii) Solve the equation f(x) = 2.

x = ....................... or x = ....................... or x = .......................[3]

(iii) Two tangents, each with gradient 0, can be drawn to the graph of y = f(x).

Write down the equation of each tangent.

...................................................

................................................... [2]
2
(b) (i) Complete the table for g(x) = + 3 for - 3.5 G x G - 0.5 and 0.5 G x G 2.5 .
x

x –3.5 –3 –2 –1 –0.5 0.5 1 2 2.5

g(x) 2.4 2.3 1 7 5 3.8


[3]

(ii) On the grid opposite, draw the graph of y = g(x). [4]

(iii) Use your graph to solve the equation f(x) = g(x).

x = ....................... or x = .......................[2]

(c) Find gf(–2).

................................................... [2]

(d) Find g–1(5).

................................................... [1]

© UCLES 2016 0580/43/M/J/16 [Turn over


6

4 Coins are put into a machine to pay for parking cars.


The probability that the machine rejects a coin is 0.05 .

(a) Adhira puts 2 coins into the machine.

(i) Calculate the probability that the machine rejects both coins.

................................................... [2]

(ii) Calculate the probability that the machine accepts at least one coin.

................................................... [1]

(b) Raj puts 4 coins into the machine.

Calculate the probability that the machine rejects exactly one coin.

................................................... [3]

(c) The table shows the amount of money, $a, received for parking each day for 200 days.

Amount ($a) 200 1 a G 250 250 1 a G 300 300 1 a G 350 350 1 a G 400 400 1 a G 450 450 1 a G 500
Frequency 13 19 27 56 62 23

Calculate an estimate of the mean amount of money received each day.

$ ................................................... [4]
© UCLES 2016 0580/43/M/J/16
7

(d) The histogram shows the length of time that 200 cars were parked.

1.2

1.1

0.9

0.8

0.7
Frequency
density 0.6

0.5

0.4

0.3

0.2

0.1

0 50 100 150 200 250 300 350 400


Time in minutes

(i) Calculate the number of cars that were parked for 100 minutes or less.

................................................... [1]

(ii) Calculate the percentage of cars that were parked for more than 250 minutes.

...............................................% [2]

© UCLES 2016 0580/43/M/J/16 [Turn over


8

5
NOT TO
A SCALE

North
510 km

720 km 40°
C

A plane flies from A to C and then from C to B.


AC = 510 km and CB = 720 km.
The bearing of C from A is 135° and angle ACB = 40°.

(a) Find the bearing of

(i) B from C,

................................................... [2]

(ii) C from B.

................................................... [2]

(b) Calculate AB and show that it rounds to 464.7 km, correct to 1 decimal place.

[4]

(c) Calculate angle ABC.

Angle ABC = .................................................. [3]

© UCLES 2016 0580/43/M/J/16


9

6
y

6
U
4
V
2

x
–8 –6 –4 –2 0 2 4 6 8
–2
T
–4

–6

(a) (i) Draw the image of triangle T after a reflection in the line x = 0. [2]

(ii) Draw the image of triangle T after a rotation through 90° clockwise about (–2, –1). [2]

(iii) Describe fully the single transformation that maps triangle T onto triangle U.

......................................................................................................................................................

...................................................................................................................................................... [2]

(iv) Describe fully the single transformation that maps triangle T onto triangle V.

......................................................................................................................................................

...................................................................................................................................................... [3]

(b) (i) Find the matrix that represents the transformation in part (a)(i).

f p [2]

(ii) Describe fully the single transformation represented by the inverse of the matrix in part (b)(i).

......................................................................................................................................................

...................................................................................................................................................... [2]

© UCLES 2016 0580/43/M/J/16 [Turn over


10

7 Alfonso runs 10 km at an average speed of x km/h.


The next day he runs 12 km at an average speed of (x – 1) km/h.

The time taken for the 10 km run is 30 minutes less than the time taken for the 12 km run.

(a) (i) Write down an equation in x and show that it simplifies to x2 – 5x – 20 = 0.

[4]

(ii) Use the quadratic formula to solve the equation x2 – 5x – 20 = 0.


Show your working and give your answers correct to 2 decimal places.

x = ....................... or x = ....................... [4]

(iii) Find the time that Alfonso takes to complete the 12 km run.
Give your answer in hours and minutes correct to the nearest minute.

................ hours ................ minutes [2]

© UCLES 2016 0580/43/M/J/16


11

(b) A cheetah runs for 60 seconds.


The diagram shows the speed-time graph.

NOT TO
SCALE
Speed
25
(m/s)

0 10 55 60
Time (seconds)

(i) Work out the acceleration of the cheetah during the first 10 seconds.

........................................... m/s2 [1]

(ii) Calculate the distance travelled by the cheetah.

............................................... m [3]

© UCLES 2016 0580/43/M/J/16 [Turn over


12

A = f- 1 5p
2 0
B=c m C=c m
1 3 7
8 D = ^2 5h
-1 5 -4
3 -4

(a) Work out each of the following if the answer is possible.


If a calculation is not possible, write “not possible” in the answer space.

(i) BA

[1]

(ii) 2A

[1]

(iii) CD

[2]

(iv) DC

[2]

(v) B2

[2]

(b) Find B–1, the inverse of B.

f p [2]

© UCLES 2016 0580/43/M/J/16


13

A
NOT TO
SCALE
12 cm
O
145°

The diagram shows a sector, centre O, and radius 12 cm.

(a) Calculate the area of the sector.

........................................... cm2 [3]

(b) The sector is made into a cone by joining OA to OB.

Calculate the volume of the cone.


1
[The volume, V, of a cone with base radius r and height h is V = rr 2 h .]
3

........................................... cm3 [6]

© UCLES 2016 0580/43/M/J/16 [Turn over


14

10

Diagram 1 Diagram 2 Diagram 3

Each diagram is made from tiles in the shape of equilateral triangles and rhombuses.
The length of a side of each tile is 1 unit.

(a) Complete the table below for this sequence of diagrams.

Diagram 1 2 3 4 5

Number of equilateral triangle shaped tiles 2 3 4 5 6

Number of rhombus shaped tiles 1 3 6

Total number of tiles 3 6 10

Number of 1 unit lengths 8 15 24


[6]

(b) (i) The number of 1 unit lengths in Diagram n is n2 + 4n + p.

Find the value of p.

p = .................................................. [2]

(ii) Calculate the number of 1 unit lengths in Diagram 10.

................................................... [1]

© UCLES 2016 0580/43/M/J/16


15

(c) The total number of tiles in Diagram n is an2 + bn + 1.

Find the value of a and the value of b.

a = ..................................................

b = .................................................. [5]

(d) Part of the Louvre museum in Paris is in the shape of a square-based pyramid made from glass tiles.
Each of the triangular faces of the pyramid is represented by Diagram 17 in the sequence.

(i) Calculate the total number of glass tiles on one triangular face of this pyramid.

................................................... [2]

(ii) 11 tiles are removed from one of the triangular faces to create an entrance into the pyramid.

Calculate the total number of glass tiles used to construct this pyramid.

................................................... [1]

© UCLES 2016 0580/43/M/J/16


16

BLANK PAGE

Permission to reproduce items where third-party owned material protected by copyright is included has been sought and cleared where possible. Every reasonable
effort has been made by the publisher (UCLES) to trace copyright holders, but if any items requiring clearance have unwittingly been included, the publisher will
be pleased to make amends at the earliest possible opportunity.

To avoid the issue of disclosure of answer-related information to candidates, all copyright acknowledgements are reproduced online in the Cambridge International
Examinations Copyright Acknowledgements Booklet. This is produced for each series of examinations and is freely available to download at www.cie.org.uk after
the live examination series.

Cambridge International Examinations is part of the Cambridge Assessment Group. Cambridge Assessment is the brand name of University of Cambridge Local
Examinations Syndicate (UCLES), which is itself a department of the University of Cambridge.

© UCLES 2016 0580/43/M/J/16


Cambridge International Examinations
Cambridge International General Certificate of Secondary Education
* 1 9 4 7 6 4 6 8 2 3 *

MATHEMATICS 0580/41
Paper 4 (Extended) October/November 2016
2 hours 30 minutes
Candidates answer on the Question Paper.
Additional Materials: Electronic calculator Geometrical instruments
Tracing paper (optional)

READ THESE INSTRUCTIONS FIRST

Write your Centre number, candidate number and name on all the work you hand in.
Write in dark blue or black pen.
You may use an HB pencil for any diagrams or graphs.
Do not use staples, paper clips, glue or correction fluid.
DO NOT WRITE IN ANY BARCODES.

Answer all questions.


If working is needed for any question it must be shown below that question.
Electronic calculators should be used.
If the degree of accuracy is not specified in the question, and if the answer is not exact, give the answer to
three significant figures. Give answers in degrees to one decimal place.
For r, use either your calculator value or 3.142.

At the end of the examination, fasten all your work securely together.
The number of marks is given in brackets [ ] at the end of each question or part question.
The total of the marks for this paper is 130.

The syllabus is approved for use in England, Wales and Northern Ireland as a Cambridge International Level 1/Level 2 Certificate.

This document consists of 19 printed pages and 1 blank page.

DC (LEG/SG) 117804/2
© UCLES 2016 [Turn over
www.egyptigstudentroom.com
2

1 (a) (i) Divide $105 in the ratio 4 : 3.

$ ..................... and $ ..................... [2]

(ii) Increase $105 by 12%.

$ ................................................ [2]

(iii) In a sale the original price of a jacket is reduced by 16% to $105.

Calculate the original price of the jacket.

$ ................................................ [3]

(b) Jakob invests $500 at a rate of 2% per year compound interest.


Claudia invests $500 at a rate of 2.5% per year simple interest.

Calculate the difference between these two investments after 30 years.


Give your answer in dollars correct to the nearest cent.

$ ................................................ [6]
www.egyptigstudentroom.com
© UCLES 2016 0580/41/O/N/16
3

(c) Michel invests $P at a rate of 3.8% per year compound interest.


After 30 years the value of this investment is $1469.

Calculate the value of P.

P = ................................................ [3]

(d) The population of a city increases exponentially at a rate of x% every 5 years.


In 1960 the population was 60 100.
In 2015 the population was 120 150.

Calculate the value of x.

x = ................................................ [3]

www.egyptigstudentroom.com
© UCLES 2016 0580/41/O/N/16 [Turn over
4

2 (a) 200 students record the time, t minutes, for their journey from home to school.
The cumulative frequency diagram shows the results.

Cumulative
frequency

200

180

160

140

120

100

80

60

40

20

0 t
5 10 15 20 25 30 35 40
Time (minutes)

Find

(i) the median,

.......................................... min [1]

(ii) the lower quartile,

.......................................... min [1]

(iii) the inter-quartile range,

.......................................... min [1]

(iv) the 15th percentile,

.......................................... min [1]

(v) the number of students whose journey time was more than 30 minutes.

................................................. [2]

www.egyptigstudentroom.com
© UCLES 2016 0580/41/O/N/16
5

(b) The 200 students record the time, t minutes, for their journey from school to home.
The frequency table shows the results.

Time (t minutes) 0 1 t G 10 10 1 t G 15 15 1 t G 20 20 1 t G 30 30 1 t G 60
Frequency 48 48 60 26 18

(i) Calculate an estimate of the mean.

.......................................... min [4]

(ii) On the grid, complete the histogram to show the information in the frequency table.

12

11

10

8
Frequency
density 7

t
0 10 20 30 40 50 60
Time (minutes)

www.egyptigstudentroom.com [4]
© UCLES 2016 0580/41/O/N/16 [Turn over
6

3 (a)
NOT TO
SCALE

13 cm

25 cm

The diagram shows a solid made up of a cylinder and two hemispheres.


The radius of the cylinder and the hemispheres is 13 cm.
The length of the cylinder is 25 cm.

(i) One cubic centimetre of the solid has a mass of 2.3 g.

Calculate the mass of the solid.


Give your answer in kilograms.
4
[The volume, V, of a sphere with radius r is V = rr 3 .]
3

............................................ kg [4]

(ii) The surface of the solid is painted at a cost of $4.70 per square metre.

Calculate the cost of painting the solid.

[The surface area, A, of a sphere with radius r is A = 4rr 2 .]

www.egyptigstudentroom.com
$ ................................................. [4]
© UCLES 2016 0580/41/O/N/16
7

(b)

NOT TO
2x cm SCALE

x cm

The cone in the diagram has radius x cm and height 2x cm.


The volume of the cone is 500 cm3.

Find the value of x.


1
[The volume, V, of a cone with radius r and height h is V = rr 2 h .]
3

x = ................................................ [3]

(c) Two mathematically similar solids have volumes of 180 cm3 and 360 cm3.
The surface area of the smaller solid is 180 cm2.

Calculate the surface area of the larger solid.

www.egyptigstudentroom.com
..........................................cm 2 [3]
© UCLES 2016 0580/41/O/N/16 [Turn over
8

2
4 y = 1- , x!0
x2
(a) Complete the table.

x –5 –4 –3 –2 –1 –0.5 0.5 1 2 3 4 5

y 0.88 0.78 –7 –7 0.78 0.88


[3]
2
(b) On the grid, draw the graph of y = 1 - for - 5 G x G - 0.5 and 0.5 G x G 5 .
x2
y
2

x
–5 –4 –3 –2 –1 0 1 2 3 4 5
–1

–2

–3

–4

–5

–6

–7

–8
[5]

(c) (i) On the grid, draw the graph of y =- x - 1 for - 3 G x G 5 . [2]


2
(ii) Solve the equation 1 - =- x - 1.
x2

x = ................................................ [1]

www.egyptigstudentroom.com
© UCLES 2016 0580/41/O/N/16
9

2
(iii) The equation 1 - =- x - 1 can be written in the form x 3 + px 2 + q = 0 .
x2
Find the value of p and the value of q.

p = ................................................

q = ................................................ [3]
2
(d) The graph of y = 1 - cuts the positive x-axis at A.
x2
B is the point (0, – 2).

(i) Write down the co-ordinates of A.

( ...................... , ......................) [1]

(ii) On the grid, draw the straight line that passes through A and B. [1]

(iii) Complete the statement.

The straight line that passes through A and B is a ...................................................................

at the point ................................................... [2]

www.egyptigstudentroom.com
© UCLES 2016 0580/41/O/N/16 [Turn over
10

5
y

6
A
5

3
B
2

x
–8 –7 –6 –5 –4 –3 –2 –1 0 1 2 3 4 5 6 7 8
–1

–2

–3

–4

–5

–6

–7

–8
-4
(a) v = c m
-8
(i) Draw the image of triangle A after the translation by vector v. [2]

(ii) Calculate v .

................................................. [2]

www.egyptigstudentroom.com
© UCLES 2016 0580/41/O/N/16
11

(b) (i) Describe fully the single transformation that maps triangle A onto triangle B.

......................................................................................................................................................

...................................................................................................................................................... [3]

(ii) Find the matrix that represents the transformation that maps triangle A onto triangle B.

f p [2]

(iii) Calculate the determinant of the matrix in part (b)(ii).

................................................. [1]

www.egyptigstudentroom.com
© UCLES 2016 0580/41/O/N/16 [Turn over
12

6
D

North
170 m
NOT TO
SCALE
C
33° 180 m
A

220 m

The diagram shows five straight footpaths in a park.


AB = 220 m, AC = 180 m and AD = 170 m.
Angle ACB = 90° and angle DAC = 33°.

(a) Calculate BC.

BC = ............................................ m [3]

(b) Calculate CD.

www.egyptigstudentroom.com
CD = ............................................ m [4]
© UCLES 2016 0580/41/O/N/16
13

(c) Calculate the shortest distance from D to AC.

............................................. m [2]

(d) The bearing of D from A is 047°.

Calculate the bearing of B from A.

................................................. [3]

(e) Calculate the area of the quadrilateral ABCD.

www.egyptigstudentroom.com
............................................m 2 [3]
© UCLES 2016 0580/41/O/N/16 [Turn over
14

7 A train stops at station A and then at station B.


If the train is late at station A, the probability that it is late at station B is 0.9 .
If the train is not late at station A, the probability that it is late at station B is 0.2 .
The probability that the train is late at station A is 0.3 .

(a) Complete the tree diagram.

Station A Station B

late
0.9

late
0.3
not late
...............

late
...............

............... not late

not late
...............
[2]
(b) (i) Find the probability that the train is late at one or both of the stations.

................................................. [3]

(ii) This train makes 250 journeys.

Find the number of journeys that the train is expected to be late at one or both of the stations.

................................................. [1]

(c) The train continues to station C.


The probability that it is late at all 3 stations is 0.27 .

Describe briefly what this probability shows.

..............................................................................................................................................................

www.egyptigstudentroom.com
.............................................................................................................................................................. [1]
© UCLES 2016 0580/41/O/N/16
15

8 Apples cost x cents each and oranges cost (x + 2) cents each.


Dylan spends $3.23 on apples and $3.23 on oranges.
The total of the number of apples and the number of oranges Dylan buys is 36.

(a) Write an equation in x and show that it simplifies to 18x 2 - 287x - 323 = 0 .

[4]

(b) (i) Find the two prime factors of 323.

....................... , .......................[1]

(ii) Complete the statement.

18x 2 - 287x - 323 = (18x ......................)(x ......................) [2]

(iii) Solve the equation 18x 2 - 287x - 323 = 0 .

x = .......................... or x = ..........................[1]

(c) Find the largest number of apples Dylan can buy for $2.

................................................. [1]

www.egyptigstudentroom.com
© UCLES 2016 0580/41/O/N/16 [Turn over
16

9 f (x) = 2x + 1 g (x) = 3x - 2 h (x) = 3 x

(a) Find hf(2) – f h(1).

................................................. [3]

(b) Find gf(x), giving your answer in its simplest form.

................................................. [2]

(c) Solve the inequality f (x) 2 g (x) .

................................................. [2]
1
(d) Solve the equation h (x) = .
9

x = ................................................ [1]

(e) Find g -1 (x) .

g -1 (x) = ................................................ [2]

www.egyptigstudentroom.com
© UCLES 2016 0580/41/O/N/16
17

5
(f) Find + g (x) .
f (x)
Give your answer as a single fraction.

................................................. [3]

(g) Solve the equation f -1 (x) = 4 .

x = ................................................ [1]

www.egyptigstudentroom.com
© UCLES 2016 0580/41/O/N/16 [Turn over
18

10 (a)

r cm NOT TO
SCALE


r cm

The area of this sector is r 2 square centimetres.

Find the value of w.

w = ................................................ [3]

(b)

r cm NOT TO
SCALE


r cm

7rr
The perimeter of this sector is 2r + centimetres.
10
Find the value of x.

www.egyptigstudentroom.com
x = ................................................ [3]
© UCLES 2016 0580/41/O/N/16
19

(c)

q cm q cm NOT TO
SCALE

cm

The perimeter of the isosceles triangle is 2q + q 3 centimetres.

Find the value of y.

y = ................................................ [4]

www.egyptigstudentroom.com
© UCLES 2016 0580/41/O/N/16
20

blank page

Permission to reproduce items where third-party owned material protected by copyright is included has been sought and cleared where possible. Every
reasonable effort has been made by the publisher (UCLES) to trace copyright holders, but if any items requiring clearance have unwittingly been included, the
publisher will be pleased to make amends at the earliest possible opportunity.

To avoid the issue of disclosure of answer-related information to candidates, all copyright acknowledgements are reproduced online in the Cambridge International
Examinations Copyright Acknowledgements Booklet. This is produced for each series of examinations and is freely available to download at www.cie.org.uk after
the live examination series.

Cambridge International Examinations is part of the Cambridge Assessment Group. Cambridge Assessment is the brand name of University of Cambridge Local

www.egyptigstudentroom.com
Examinations Syndicate (UCLES), which is itself a department of the University of Cambridge.

© UCLES 2016 0580/41/O/N/16


Cambridge International Examinations
Cambridge International General Certificate of Secondary Education
* 2 0 9 6 7 0 5 3 6 5 *

MATHEMATICS 0580/42
Paper 4 (Extended) October/November 2016
2 hours 30 minutes
Candidates answer on the Question Paper.
Additional Materials: Electronic calculator Geometrical instruments
Tracing paper (optional)

READ THESE INSTRUCTIONS FIRST

Write your Centre number, candidate number and name on all the work you hand in.
Write in dark blue or black pen.
You may use an HB pencil for any diagrams or graphs.
Do not use staples, paper clips, glue or correction fluid.
DO NOT WRITE IN ANY BARCODES.

Answer all questions.


If working is needed for any question it must be shown below that question.
Electronic calculators should be used.
If the degree of accuracy is not specified in the question, and if the answer is not exact, give the answer to
three significant figures. Give answers in degrees to one decimal place.
For r, use either your calculator value or 3.142.

At the end of the examination, fasten all your work securely together.
The number of marks is given in brackets [ ] at the end of each question or part question.
The total of the marks for this paper is 130.

The syllabus is approved for use in England, Wales and Northern Ireland as a Cambridge International Level 1/Level 2 Certificate.

This document consists of 20 printed pages.

DC (LEG/SG) 118013/2
© UCLES 2016 [Turn over
www.egyptigstudentroom.com
2

1 (a) (i) Each year the value of a car decreases by 15% of its value at the beginning of that year.
Alberto buys a car for $18 000.

Calculate the value of Alberto’s car after 3 years.

$ ................................................ [2]

(ii) Belinda bought a car one year ago.


The value of this car has decreased by 15% to $14025.

Calculate how much Belinda paid for the car.

$ ................................................ [3]

(b) Chris invested some money at a rate of 5% per year compound interest.
After 2 years the value of this investment is $286.65 .

Calculate how much Chris invested.

www.egyptigstudentroom.com
$ ................................................ [2]
© UCLES 2016 0580/42/O/N/16
3

(c) Dani invested $200 and after 2 years the value of this investment is $224.72 .

Calculate the rate of interest per year when the interest is

(i) simple,

.............................................% [3]

(ii) compound.

.............................................% [3]

www.egyptigstudentroom.com
© UCLES 2016 0580/42/O/N/16 [Turn over
4

x3
2 (a) Complete the table of values for y = - x 2 + 1.
3

x –1.5 –1 –0.5 0 0.5 1 1.5 2 2.5 3


y –2.38 –0.33 0.71 0.79 0.33 –0.13 –0.33 –0.04
[2]
3
x
(b) Draw the graph of y = - x 2 + 1 for -1.5 G x G 3.
3
The first 3 points have been plotted for you.
y

x
–1 0 1 2 3

–1

–2

–3
[4]

www.egyptigstudentroom.com
© UCLES 2016 0580/42/O/N/16
5

(c) Using your graph, solve the equations.


x3
(i) - x2 + 1 = 0
3
 x = ...................... or x = ...................... or x = ......................[3]
3
x
(ii) - x2 + x + 1 = 0
3

x = ................................................ [2]
x3
(d) Two tangents to the graph of y = - x 2 + 1 can be drawn parallel to the x-axis.
3
(i) Write down the equation of each of these tangents.

.................................................

................................................. [2]

(ii) For 0 G x G 3, write down the smallest possible value of y.

y = ................................................ [1]

www.egyptigstudentroom.com
© UCLES 2016 0580/42/O/N/16 [Turn over
6

3
D

180 m
North
C NOT TO
85° SCALE

240 m

50°
B

The diagram shows a field, ABCD.


AD = 180 m and AC = 240 m.
Angle ABC = 50° and angle ACB = 85°.

(a) Use the sine rule to calculate AB.

AB = ............................................ m [3]

(b) The area of triangle ACD = 12 000 m2.

Show that angle CAD = 33.75°, correct to 2 decimal places.

www.egyptigstudentroom.com [3]
© UCLES 2016 0580/42/O/N/16
7

(c) Calculate BD.

BD = ............................................ m [5]

(d) The bearing of D from A is 030°.

Find the bearing of

(i) B from A,

................................................. [1]

(ii) A from B.

................................................. [2]

www.egyptigstudentroom.com
© UCLES 2016 0580/42/O/N/16 [Turn over
8

4 200 people run 10 km.


The table shows some information about the times, t minutes, taken to run the 10 km.

Time
30 1 t G 40 40 1 t G 45 45 1 t G 50 50 1 t G 55 55 1 t G 60 60 1 t G 80
(tminutes)
Frequency 8 22 95 55 14 6

(a) Howard takes 40 minutes to run the 10 km.

Calculate his average speed in kilometres per hour.

........................................ km/h [2]

(b) Calculate an estimate of the mean time.

.......................................... min [4]

(c) Complete the histogram to show the information in the table.

20

18

16

14

12
Frequency
density 10

0 t
30 40 50 60 70 80
Time (minutes)
www.egyptigstudentroom.com [4]
© UCLES 2016 0580/42/O/N/16
9

(d) (i) Use the frequency table opposite to complete the cumulative frequency table.

Time
t G 40 t G 45 t G 50 t G 55 t G 60 t G 80
(tminutes)
Cumulative
8 30 194 200
frequency
[1]

(ii) Draw a cumulative frequency diagram to show the information in the table above.

200

180

160

140

120
Cumulative
frequency 100

80

60

40

20

0 t
30 40 50 60 70 80
Time (minutes)
[3]

(iii) Use your diagram to find

(a) the median,

.......................................... min [1]

(b) the 90th percentile,

.......................................... min [1]

(c) the number of people who took more than 58 minutes to run the 10 km.

................................................. [2]
www.egyptigstudentroom.com
© UCLES 2016 0580/42/O/N/16 [Turn over
10

7
5 The probability that a plant will produce flowers is .
8
The flowers are either red or yellow.
3
If the plant produces flowers, the probability that the flowers are red is .
4
(a) (i) Complete the tree diagram by writing a probability beside each branch.

Red
............

Flowers

............
............ Yellow

............

No flowers
[2]

(ii) Calculate the probability that a plant, chosen at random, will produce red flowers.

................................................. [2]

www.egyptigstudentroom.com
© UCLES 2016 0580/42/O/N/16
11

(iii) Two plants are chosen at random.

Calculate the probability that both will produce red flowers.

................................................. [2]

(b) Alphonse buys 200 of these plants.

Calculate the number of plants that are expected to produce flowers.

................................................. [2]

(c) Gabriel has 1575 plants with red flowers.

Estimate the total number of plants that Gabriel has.

................................................. [2]

www.egyptigstudentroom.com
© UCLES 2016 0580/42/O/N/16 [Turn over
12

6 (a)

NOT TO
0.8 cm SCALE
0.8 cm
1.1 cm
1.5 cm

The diagram shows two sweets.


The cuboid has length 1.5 cm, width 1.1 cm and height 0.8 cm.
The cylinder has height 0.8 cm and the same volume as the cuboid.

(i) Calculate the volume of the cuboid.

..........................................cm3 [2]

(ii) Calculate the radius of the cylinder.

........................................... cm [2]

(iii) Calculate the difference between the surface areas of the two sweets.

..........................................cm2 [5]

www.egyptigstudentroom.com
© UCLES 2016 0580/42/O/N/16
13

(b) A bag of sweets contains x orange sweets and y lemon sweets.


Each orange sweet costs 2 cents and each lemon sweet costs 3 cents.

The cost of a bag of sweets is less than 24 cents.


There are at least 9 sweets in each bag.
There are at least 2 lemon sweets in each bag.

(i) One of the inequalities that shows this information is 2x + 3y 1 24 .

Write down the other two inequalities.

.................................................

................................................. [2]

(ii) On the grid, by shading the unwanted regions, show the region which satisfies the three
inequalities.

12
11
10
9
8
7
6
5
4
3
2
1
0 x
1 2 3 4 5 6 7 8 9 10 11 12
[4]

(iii) Find the lowest cost of a bag of sweets.


Write down the value of x and the value of y that give this cost.

Lowest cost = .................................... cents

x = .............................................

y = ............................................. [3]

www.egyptigstudentroom.com
© UCLES 2016 0580/42/O/N/16 [Turn over
14

7 (a) $1= 3.67 dirhams

Calculate the value, in dollars, of 200 dirhams.


Give your answer correct to 2 decimal places.

$ ................................................. [2]

(b) (i) Write as a single fraction, in its simplest form.


1000 1000
-
x x+1

................................................. [3]

(ii) One day in 2014, 1 euro was worth x rand.


One year later, 1 euro was worth (x + 1) rand.

Winston changed 1000 rand into euros in both years.


In 2014 he received 4.50 euros more than in 2015.

Write an equation in terms of x and show that it simplifies to

9x 2 + 9x - 2000 = 0 .

www.egyptigstudentroom.com [3]
© UCLES 2016 0580/42/O/N/16
15

(iii) Use the quadratic formula to solve the equation 9x 2 + 9x - 2000 = 0 .


Show all your working and give your answers correct to 2 decimal places.

x = ........................ or x = ........................ [4]

(iv) Calculate the number of euros Winston received in 2014.


Give your answer correct to 2 decimal places.

........................................euros [2]

www.egyptigstudentroom.com
© UCLES 2016 0580/42/O/N/16 [Turn over
16

8 (a) C


NOT TO
SCALE
O

D
B

A 80°

A, B, C and D lie on the circle, centre O.


DAE is a straight line.

Find the value of u and the value of v.

u = ................................................

v = ................................................ [2]

(b)
G

NOT TO
SCALE

O H

The diagram shows a circle, centre O, radius 8 cm.


  GH is a chord of length 10 cm.

Calculate the length of the perpendicular from O toGH.

www.egyptigstudentroom.com
........................................... cm [3]
© UCLES 2016 0580/42/O/N/16
17

(c) K,L,MandN lie on the circle. M


  KM and LN intersect at X.
  KL= 9.7 cm, KX= 4.8 cm,
  LX= 7.8 cm and NX= 2.5 cm. N NOT TO
2.5 cm
SCALE
Calculate MN. X
4.8 cm
K 7.8 cm

9.7 cm

MN = .......................................... cm [2]

(d) All lengths are in centimetres. S


R
  P, Q, R and S lie on the circle. x
  PR and QS intersect at Y. Y NOT TO
  PY = 2x and YS = x.
SCALE
5 ^ 2x
The area of triangle YRS = x x - 1h .
12
The area of triangle YQP = x ^x + 1h . Q
P
Find the value of x.

www.egyptigstudentroom.com
x = ................................................ [4]
© UCLES 2016 0580/42/O/N/16 [Turn over
18

3
9 (a) y= + 2 , x ! 0
x
(i) Find the value of y when x =- 6 .

y = ................................................ [1]

(ii) Find x in terms of y.

x = ................................................ [3]

(b) g (x) = 2 - x h (x) = 2 x

(i) Find g(5).

................................................. [1]

(ii) Find hhh(2).

................................................. [2]

www.egyptigstudentroom.com
© UCLES 2016 0580/42/O/N/16
19

(iii) Find x when g (x) = h (3) .

x = ................................................ [2]

(iv) Find x when g –1 (x) =- 1.

x = ................................................ [1]

Question 10 is printed on the next page.

www.egyptigstudentroom.com
© UCLES 2016 0580/42/O/N/16 [Turn over
20

10 The perimeter of each of the three shapes is 60 cm.

Find x in each part.

(a)
Rectangle NOT TO
x
SCALE

3x

x = .......................................... cm [2]

(b)

Triangle 3x NOT TO
SCALE

4x

x = .......................................... cm [3]

(c)

Sector x
NOT TO
O x SCALE

x = .......................................... cm [3]

Permission to reproduce items where third-party owned material protected by copyright is included has been sought and cleared where possible. Every
reasonable effort has been made by the publisher (UCLES) to trace copyright holders, but if any items requiring clearance have unwittingly been included, the
publisher will be pleased to make amends at the earliest possible opportunity.

To avoid the issue of disclosure of answer-related information to candidates, all copyright acknowledgements are reproduced online in the Cambridge International
Examinations Copyright Acknowledgements Booklet. This is produced for each series of examinations and is freely available to download at www.cie.org.uk after
the live examination series.

Cambridge International Examinations is part of the Cambridge Assessment Group. Cambridge Assessment is the brand name of University of Cambridge Local

www.egyptigstudentroom.com
Examinations Syndicate (UCLES), which is itself a department of the University of Cambridge.

© UCLES 2016 0580/42/O/N/16


Cambridge International Examinations
Cambridge International General Certificate of Secondary Education
* 8 6 5 0 2 7 9 9 6 7 *

MATHEMATICS 0580/43
Paper 4 (Extended) October/November 2016
2 hours 30 minutes
Candidates answer on the Question Paper.
Additional Materials: Electronic calculator Geometrical instruments
Tracing paper (optional)

READ THESE INSTRUCTIONS FIRST

Write your Centre number, candidate number and name on all the work you hand in.
Write in dark blue or black pen.
You may use an HB pencil for any diagrams or graphs.
Do not use staples, paper clips, glue or correction fluid.
DO NOT WRITE IN ANY BARCODES.

Answer all questions.


If working is needed for any question it must be shown below that question.
Electronic calculators should be used.
If the degree of accuracy is not specified in the question, and if the answer is not exact, give the answer to
three significant figures. Give answers in degrees to one decimal place.
For r, use either your calculator value or 3.142.

At the end of the examination, fasten all your work securely together.
The number of marks is given in brackets [ ] at the end of each question or part question.
The total of the marks for this paper is 130.

The syllabus is approved for use in England, Wales and Northern Ireland as a Cambridge International Level 1/Level 2 Certificate.

This document consists of 19 printed pages and 1 blank page.

DC (NH/JG) 118014/2
© UCLES 2016 [Turn over
www.egyptigstudentroom.com
2

1 (a) A jigsaw puzzle has edge pieces and inside pieces.


The ratio edge pieces : inside pieces = 3 : 22.

(i) There are 924 inside pieces.

Calculate the total number of pieces in the puzzle.

................................................... [2]

(ii) Find the percentage of the total number of pieces that are edge pieces.

...............................................% [1]

(iii) Anjum and Betty spent a total of 9 hours completing the puzzle.
The ratio Anjum’s time : Betty’s time = 7 : 5.

Work out how much time Anjum spent on the puzzle.

......................................... hours [2]

(b) The price of the puzzle was $15.99 in a sale.


This was 35% less than the original price.

Calculate the original price of the puzzle.

$ ................................................... [3]

www.egyptigstudentroom.com
© UCLES 2016 0580/43/O/N/16
3

(c) Betty takes a photograph of the completed puzzle.


The photograph and the completed puzzle are mathematically similar.

The area of the photograph is 875 cm2 and the area of the puzzle is 2835 cm2.
The length of the photograph is 35 cm.

Work out the length of the puzzle.

............................................. cm [3]

(d) (i) The area of another puzzle is 6610 cm2.

Change 6610 cm2 into m2.

..............................................m2 [1]

(ii) The cost price of this puzzle is $12.50 .


The selling price is $18.50 .

Calculate the percentage profit.

...............................................% [3]

www.egyptigstudentroom.com
© UCLES 2016 0580/43/O/N/16 [Turn over
4

2
2 (a) Complete the table for y = 3x + + 1, x ! 0 .
x2

x –3 –2 –1 –0.5 –0.3 0.3 0.5 1 2 3


y –7.8 0 7.5 22.3 24.1 6 7.5 10.2
[2]
2
(b) On the grid, draw the graph of y = 3x + + 1 for -3 G x G -0.3 and 0.3 G x G 3.
x2

y
25

20

15

10

x
–3 –2 –1 0 1 2 3

–5

–10
[5]

2
(c) Write down the value of the largest integer, k, so that the equation 3x + 2 + 1 = k has exactly one
solution. x

k = .................................................. [1]

www.egyptigstudentroom.com
© UCLES 2016 0580/43/O/N/16
5

2
(d) (i) By drawing a suitable straight line on the grid, solve 3x + + 1 = 15 - 3x .
x2

x = ..................... or x = ..................... or x = ..................... [4]

2
(ii) The equation 3x + 2 + 1 = 15 - 3x can be written in the form ax + bx + cx + 2 = 0 ,
3 2
x
where a, b and c are integers.

Find a, b and c.

a = ..................................................

b = ...................................................

c = ................................................... [3]

www.egyptigstudentroom.com
© UCLES 2016 0580/43/O/N/16 [Turn over
6

3 (a) Solve.
8x – 5 = 22 – 4x

x = .................................................. [2]

(b) Solve.
6x H 2x + 14

................................................... [2]

(c) Factorise.
x2 – 4x – 21

................................................... [2]

(d) Expand the brackets and simplify.


(3x – 2y)(4x + 3y)

................................................... [3]

www.egyptigstudentroom.com
© UCLES 2016 0580/43/O/N/16
7

4
y
8
B
7

4
A
3

x
–5 –4 –3 –2 –1 0 1 2 3 4 5 6 7 8 9 10
–1

–2

(a) Draw the image when triangle A is reflected in the line x = 1. [2]

-2
(b) Draw the image when triangle A is translated by the vector c m . [2]
3

(c) Draw the image when triangle A is enlarged by scale factor 2 with centre (4, 5). [2]

(d) Describe fully the single transformation that maps triangle A onto triangle B.

..............................................................................................................................................................

.............................................................................................................................................................. [3]

www.egyptigstudentroom.com
© UCLES 2016 0580/43/O/N/16 [Turn over
8

5 Sandra has a fair eight-sided spinner.


The numbers on the spinner are 3, 4, 4, 4, 5, 5, 6 and 8.
5

6 8
Sandra spins the spinner twice and records each number it lands on. 5

4 4
3
4
Find the probability that

(a) both numbers are 8,

................................................... [2]

(b) the two numbers are not both 8,

................................................... [1]

(c) one number is odd and one number is even,

................................................... [2]

www.egyptigstudentroom.com
© UCLES 2016 0580/43/O/N/16
9

(d) the total of the two numbers is at least 13,

................................................... [3]

(e) the second number is bigger than the first number.

................................................... [3]

www.egyptigstudentroom.com
© UCLES 2016 0580/43/O/N/16 [Turn over
10

6
D

C
NOT TO
SCALE

B 40 km
L

61.1 km
North
92.1 km

The diagram shows the position of a port, A, and a lighthouse, L.


The circle, centre L and radius 40 km, shows the region where the light from the lighthouse can be seen.
The straight line, ABCD, represents the course taken by a ship after leaving the port.
When the ship reaches position B it is due west of the lighthouse.

AL = 92.1 km, AB = 61.1 km and BL = 40 km.

(a) Use the cosine rule to show that angle ABL = 130.1°, correct to 1 decimal place.

[4]

www.egyptigstudentroom.com
© UCLES 2016 0580/43/O/N/16
11

(b) Calculate the bearing of the lighthouse, L, from the port, A.

................................................... [4]

(c) The ship sails at a speed of 28 km/h.

Calculate the length of time for which the light from the lighthouse can be seen from the ship.
Give your answer correct to the nearest minute.

.................... h .................... min [5]

www.egyptigstudentroom.com
© UCLES 2016 0580/43/O/N/16 [Turn over
12

7 (a) (i)
100

80

60
Cumulative
frequency

40

20

0
2 4 6 8 10 12 14 16 18 20
Price in $(thousands)

The cumulative frequency diagram shows information about the prices of 100 cars on Website A.
Use the information to complete this table.

Lower Upper Inter-quartile


Median
quartile quartile range
$ $7600 $ $
[2]

(ii) This table shows information about the prices of cars on Website B.

Lower Upper Inter-quartile


Median
quartile quartile range
$7600 $10 800 $13 600 $6000

Here are two statements comparing the distributions of the prices of cars on Website A and
Website B.

For each statement write True or False.


Give a reason for each answer, stating clearly which statistic you use to make your decision.

(a) The prices of cars on Website A are lower than the prices of cars on Website B.

.............................. because .................................................................................................

.............................................................................................................................................. [1]
www.egyptigstudentroom.com
© UCLES 2016 0580/43/O/N/16
13

(b) A greater percentage of cars have a price more than $13 600 on Website A compared to
Website B.

.............................. because .................................................................................................

.............................................................................................................................................. [1]

(b) The table shows the prices of cars on Website B.

Price ($P) Number of cars

0 1 P G 6 000 9

6 000 1 P G 8 000 29

8 000 1 P G 10 000 20

10 000 1 P G 12 000 14

12 000 1 P G 14 000 21

14 000 1 P G 22 000 27

Calculate an estimate of the mean price of the 120 cars.

$ ................................................... [4]

(c) The price of a car is $8760.


Bryan pays a deposit of 25% of this price and then 24 equal monthly payments.
After 24 months, he will have paid a total of $9948.

Calculate the cost of one monthly payment.

$ ................................................... [3]

www.egyptigstudentroom.com
© UCLES 2016 0580/43/O/N/16 [Turn over
14

24 cm
A B

r
O NOT TO
18 cm SCALE

The diagram shows the cross section of a cylinder, centre O, radius r, lying on its side.
The cylinder contains water to a depth of 18 cm.
The width, AB, of the surface of the water is 24 cm.

(a) Use an algebraic method to show that r = 13 cm.

[4]

(b) Show that angle AOB = 134.8°, correct to 1 decimal place.

[2]

(c) (i) Calculate the area of the major sector OAPB.

............................................cm2 [3]

www.egyptigstudentroom.com
© UCLES 2016 0580/43/O/N/16
15

(ii) Calculate the area of the shaded segment APB.

............................................cm2 [3]

(iii) The length of the cylinder is 40 cm.

Calculate the volume of water in the cylinder.

............................................cm3 [1]

(d) The cylinder is turned so that it stands on one of its circular ends.
In this position, the depth of the water is h.

Find h.

NOT TO
SCALE
h

h = ............................................ cm [2]

www.egyptigstudentroom.com
© UCLES 2016 0580/43/O/N/16 [Turn over
16

-2
(a) m = c m n =c m
3
9
2 3

(i) Work out 2m – 3n.

f p [2]

(ii) Calculate 2m - 3n .

................................................... [2]

(b) (i)
A

a NOT TO
M SCALE

O B
b

In the diagram, O is the origin, OA = a and OB = b.


The point M lies on AB such that AM : MB = 3 : 2.

Find, in terms of a and b, in its simplest form

(a) AB,

AB = .................................................. [1]

(b) AM ,

AM = .................................................. [1]

www.egyptigstudentroom.com
© UCLES 2016 0580/43/O/N/16
17

(c) the position vector of M.

................................................... [2]

(ii) OM is extended to the point C.


The position vector of C is a + kb.

Find the value of k.

k = .................................................. [1]

www.egyptigstudentroom.com
© UCLES 2016 0580/43/O/N/16 [Turn over
18

10 (a) Complete the table for the four sequences A, B, C and D.

Sequence Next term nth term


A 2 5 8 11
B 20 14 8 2
C 1 4 9 16
D 0 2 6 12

[10]
n ^3n + 1h
(b) The sum of the first n terms of a sequence is .
2
(i) When the sum of the first n terms is 155, show that 3n2 + n – 310 = 0.

[2]

(ii) Solve 3n2 + n – 310 = 0.

n = ..................... or n = ..................... [3]

(iii) Complete the statement.

The sum of the first ............... terms of this sequence is 155. [1]

www.egyptigstudentroom.com
© UCLES 2016 0580/43/O/N/16
19

11 Solve.
2 1 3
+ =
x + 3 12 2x - 1

x = ..................... or x = ..................... [7]

www.egyptigstudentroom.com
© UCLES 2016 0580/43/O/N/16
20

BLANK PAGE

Permission to reproduce items where third-party owned material protected by copyright is included has been sought and cleared where possible. Every
reasonable effort has been made by the publisher (UCLES) to trace copyright holders, but if any items requiring clearance have unwittingly been included, the
publisher will be pleased to make amends at the earliest possible opportunity.

To avoid the issue of disclosure of answer-related information to candidates, all copyright acknowledgements are reproduced online in the Cambridge International
Examinations Copyright Acknowledgements Booklet. This is produced for each series of examinations and is freely available to download at www.cie.org.uk after
the live examination series.

Cambridge International Examinations is part of the Cambridge Assessment Group. Cambridge Assessment is the brand name of University of Cambridge Local

www.egyptigstudentroom.com
Examinations Syndicate (UCLES), which is itself a department of the University of Cambridge.

© UCLES 2016 0580/43/O/N/16


International General Certificate of Secondary Education
CAMBRIDGE INTERNATIONAL EXAMINATIONS
MATHEMATICS 0580/4, 0581/4
PAPER 4
MAY/JUNE SESSION 2002
2 hours 30 minutes

Additional materials:
Answer paper
Electronic calculator
Geometrical instruments
Graph paper (1 sheet)
Mathematical tables (optional)
Tracing paper (optional)

TIME 2 hours 30 minutes

INSTRUCTIONS TO CANDIDATES
Write your name, Centre number and candidate number in the spaces provided on the answer paper/
answer booklet.
Answer all questions.
Write your answers and working on the separate answer paper provided.
All working must be clearly shown. It should be done on the same sheet as the rest of the answer.
Marks will be given for working which shows that you know how to solve the problem even if you get
the answer wrong.
If you use more than one sheet of paper, fasten the sheets together.

INFORMATION FOR CANDIDATES


The number of marks is given in brackets [ ] at the end of each question or part question.
The total of the marks for this paper is 130.
Electronic calculators should be used.
If the degree of accuracy is not specified in the question, and if the answer is not exact, give the
answer to three significant figures. Give answers in degrees to one decimal place.
For π, use either your calculator value or 3.142.

This question paper consists of 8 printed pages.


SB (SC/DG) S07107/4 UNIVERSITY of CAMBRIDGE
© CIE 2002 Local Examinations Syndicate [Turn over
2

1 (a) One day Amit works from 08 00 until 17 00.


The time he spends on filing, computing, writing and having lunch is in the ratio

Filing: Computing: Writing: Lunch = 2: 5 : 4 : 1.

Calculate the time he spends

(i) writing, [1]


(ii) having lunch, giving this answer in minutes. [1]

(b) The amount earned by Amit, Bernard and Chris is in the ratio 2 : 5 : 3.
Bernard earns $855 per week.
Calculate how much

(i) Amit earns each week, [1]


(ii) Chris earns each week. [1]

(c) After 52 weeks Bernard has saved $2964.


What fraction of his earnings has he saved?
Give your answer in its lowest terms. [2]

(d) Chris saves $3500 this year. This is 40% more than he saved last year.
Calculate how much he saved last year. [3]

2 B

North
A
NOT TO
6m

SCALE
14

88 m

O
C

OABC is a field.
A is 88 metres due North of O.
B is 146 metres from O on a bearing of 040°.
C is equidistant from A and from B. The bearing of C from O is 098°.

(a) Using a scale of 1 centimetre to represent 10 metres, make an accurate scale drawing of the field
OABC, by

(i) constructing the triangle OAB, [3]


(ii) drawing the locus of points equidistant from A and from B, [2]
(iii) completing the scale diagram of OABC. [2]

0580/4, 0581/4 Jun02


3

(b) Use your scale drawing to write down

(i) the distance OC correct to the nearest metre, [1]


(ii) the size of angle OAB correct to the nearest degree. [1]

(c) Find the bearing of A from B. [2]

(d) A donkey in the field is not more than 40 metres from C and is closer to B than to A.
Shade the area where the donkey could be and label it D. [3]

(e) A horse in the field is not more than 20 metres from the side AB and is closer to A than to B.
Shade the area where the horse could be and label it H. [3]

3 Paula and Tarek take part in a quiz.


The probability that Paula thinks she knows the answer to any question is 0.6.
If Paula thinks she knows, the probability that she is correct is 0.9.
Otherwise she guesses and the probability that she is correct is 0.2.

(a) Copy and complete the tree diagram.

0.9 correct answer


Paula thinks
0.6 she knows
wrong answer
.......
0.2 correct answer
Paula
....... guesses
wrong answer
....... [3]

(b) Find the probability that Paula

(i) thinks she knows the answer and is correct, [1]


(ii) gets the correct answer. [2]

(c) The probability that Tarek thinks he knows the answer to any question is 0.55.
If Tarek thinks he knows, he is always correct.
Otherwise he guesses and the probability that he is correct is 0.2.

(i) Draw a tree diagram for Tarek. Write all the probabilities on your diagram. [3]
(ii) Find the probability that Tarek gets the correct answer. [2]

(d) There are 100 questions in the quiz.


Estimate the number of correct answers given by

(i) Paula, [1]


(ii) Tarek. [1]

0580/4, 0581/4 Jun02 [Turn over


4

NOT TO W
SCALE B

c° C d° D


42° a°
G A X

A sphere, centre C, rests on horizontal ground at A and touches a vertical wall at D.


A straight plank of wood, GBW, touches the sphere at B, rests on the ground at G and against the wall at W.
The wall and the ground meet at X.
Angle WGX = 42°.

(a) Find the values of a, b, c, d and e marked on the diagram. [5]

(b) Write down one word which completes the following sentence.

‘Angle CGA is 21° because triangle GBC and triangle GAC are …………………’. [1]

(c) The radius of the sphere is 54 cm.

(i) Calculate the distance GA. Show all your working. [3]
(ii) Show that GX = 195 cm correct to the nearest centimetre. [1]
(iii) Calculate the length of the plank GW. [3]
(iv) Find the distance BW. [1]

5 Answer the whole of this question on a sheet of graph paper.

Dimitra stands by a river and watches a fish.


The distance (d metres) of the fish from Dimitra after t minutes is given by
48
d = (t + 1) 2 + ____ – 20.
(t + 1)

Some values for d and t are given in the table below.

t 0 0.5 1 1.5 2 2.5 3 3.5 4 5 6 7


d p 14.3 8 5.5 5 6 8 10.9 14.6 q 35.9 r

(a) Find the values of p, q and r. [3]

0580/4, 0581/4 Jun02


5

(b) Using a scale of 2 cm to represent 1 minute on the horizontal t-axis and 2 cm to represent 10 metres on
48
the vertical d-axis, draw the graph of d = (t + 1)2 + ____ – 20 for 0 ⭐ t ⭐ 7. [6]
(t + 1)

(c) Mark and label F the point on your graph when the fish is 12 metres from Dimitra and swimming
away from her. Write down the value of t at this point, correct to one decimal place. [2]

(d) For how many minutes is the fish less than 10 metres from Dimitra? [2]

(e) By drawing a suitable line on your grid, calculate the speed of the fish when t = 2.5. [4]

6 A'

xc
m
xc

m
A x cm x cm B
P Q

12 cm

D' B'

D C

C'

An equilateral 16-sided figure APA′QB …… is formed when the square ABCD is rotated 45° clockwise
about its centre to position A′B′C′D′.
AB = 12 cm and AP = x cm.

(a) (i) Use triangle PA′Q to explain why 2x2 = (12 – 2x)2. [3]
(ii) Show that this simplifies to x2 – 24x + 72 = 0. [3]
(iii) Solve x2 – 24x + 72 = 0. Give your answers correct to 2 decimal places. [4]

(b) (i) Calculate the perimeter of the 16-sided figure. [2]


(ii) Calculate the area of the 16-sided figure. [3]

0580/4, 0581/4 Jun02 [Turn over


6

7 y

5
P Q
4

A 2 B
H C
1

–5 –4 –3 –2 –1 0 1 2 3 4 5 x
–1
G D
F –2 E

–3

–4
J
–5

(a) Describe fully a single transformation which maps both

(i) A onto C and B onto D, [2]


(ii) A onto D and B onto C, [2]
(iii) A onto P and B onto Q. [3]

(b) Describe fully a single transformation which maps triangle 0AB onto triangle JFE. [2]

冢–– 01 –10冣 .
(c) The matrix M is

(i) Describe the transformation which M represents. [2]


(ii) Write down the co-ordinates of P after transformation by matrix M. [2]

(d) (i) Write down the matrix R which represents a rotation by 90° anticlockwise about 0. [2]
(ii) Write down the letter representing the new position of F after the transformation RM(F). [2]

0580/4, 0581/4 Jun02


7

8 (a) A sector of a circle, radius 6 cm, has an angle of 20°.

NOT TO
20° SCALE
Calculate 6 cm

(i) the area of the sector, [2]


(ii) the arc length of the sector. [2]

(b)
NOT TO
20° SCALE
6 cm

5 cm

A whole cheese is a cylinder, radius 6 cm and height 5 cm.


The diagram shows a slice of this cheese with sector angle 20°.

Calculate

(i) the volume of the slice of cheese, [2]


(ii) the total surface area of the slice of cheese. [4]

(c) The radius, r, and height, h, of cylindrical cheeses vary but the volume remains constant.

(i) Which one of the following statements A, B, C or D is true?


A: h is proportional to r.
B: h is proportional to r2.
C: h is inversely proportional to r.
D: h is inversely proportional to r2. [2]
(ii) What happens to the height h of the cylindrical cheese when the volume remains constant but the
radius is doubled? [2]

0580/4, 0581/4 Jun02 [Turn over


8

9 (a) The number of people living in six houses is

3, 8, 4, x, y and z.

The median is 7W .
The mode is 8.
The mean is 7.

Find a value for each of x, y and z. [5]

(b) The grouped frequency table below shows the amount ($A) spent on travel by a number of students.

Cost of travel ($A) 0 < A ⭐ 10 10 < A ⭐ 20 20 < A ⭐ 40

Frequency 15 m n

(i) Write down an estimate for the total amount in terms of m and n. [2]
(ii) The calculated estimate of the mean amount is $13 exactly.
Write down an equation containing m and n.
Show that it simplifies to 2m + 17n = 120. [3]
(iii) A student drew a histogram to represent this data.
The area of the rectangle representing the 0 < A ⭐ 10 group was equal to the sum of the areas of
the other two rectangles.
Explain why m + n = 15. [1]
(iv) Find the values of m and n by solving the simultaneous equations

2m + 17n = 120,
m + n = 15. [3]

0580/4, 0581/4 Jun02


CAMBRIDGE INTERNATIONAL EXAMINATIONS
International General Certificate of Secondary Education
MATHEMATICS 0580/04
0581/04
Paper 4
May/June 2003

2 hours 30 minutes
Additional Materials: Answer Booklet/Paper
Electronic calculator
Geometric instruments
Graph paper (2 sheets)
Mathematical tables (optional)
Tracing paper (optional)

READ THESE INSTRUCTIONS FIRST

Write your answers and working on the separate Answer Booklet/Paper provided.
Write your Centre number, candidate number and name on all the work you hand in.
Write in dark blue or black pen on both sides of the paper.
You may use a soft pencil for any diagrams or graphs.
Do not use staples, paper clips, highlighters, glue or correction fluid.

Answer all questions.


At the end of the examination, fasten all your work securely together.
The number of marks is given in brackets [ ] at the end of each question or part question.

All working must be clearly shown. It should be done on the same sheet as the rest of the answer.
Marks will be given for working which shows that you know how to solve the problem even if you get the
answer wrong.
The total of the marks for this paper is 130.
Electronic calculators should be used.
If the degree of accuracy is not specified in the question and if the answer is not exact, give the answer to
three significant figures. Give answers in degrees to one decimal place.
For π, use either your calculator value or 3.142.

This document consists of 8 printed pages.


MCSA-UCB216-S34078/4
© CIE 2003 [Turn over
2

1 Tickets for the theatre cost either $10 or $16.

(a) Calculate the total cost of 197 tickets at $10 each and 95 tickets at $16 each. [1]

(b) On Monday, 157 tickets at $10 and n tickets at $16 were sold. The total cost was $4018.
Calculate the value of n. [2]

(c) On Tuesday, 319 tickets were sold altogether. The total cost was $3784.
Using x for the number of $10 tickets sold and y for the number of $16 tickets sold, write down
two equations in x and y.

Solve your equations to find the number of $10 tickets and the number of $16 tickets sold. [5]

(d) On Wednesday, the cost of a $16 ticket was reduced by 15%. Calculate this new reduced cost.
[2]

(e) The $10 ticket costs 25% more than it did last year. Calculate the cost last year. [2]

2 C
NOT TO
120 m SCALE
B
m

60 m
55

77 m

x° 45° y°
D
A

In quadrilateral ABCD, AB # 77 m, BC # 120 m, CD # 60 m and diagonal AC # 55 m.


Angle CAD # 45°, angle BAC # x° and angle ADC # y°.

(a) Calculate the value of x. [4]

(b) Calculate the value of y. [4]

(c) The bearing of D from A is 090°.


Find the bearing of
(i) A from C, [2]
(ii) B from A. [2]

0580/4, 0581/4 Jun/03


3

3 There are 2 sets of road signals on the direct 12 kilometre route from Acity to Beetown.
The signals say either “GO” or “STOP”.
The probabilities that the signals are “GO” when a car arrives are shown in the tree diagram.

(a) Copy and complete the tree diagram for a car driver travelling along this route.

1st signal 2nd signal

0.65 GO

0.4 GO
STOP
.......
0.45 GO
....... STOP
STOP
.......
[3]

(b) Find the probability that a car driver


(i) finds both signals are “GO”, [2]
(ii) finds exactly one of the two signals is “GO”, [3]
(iii) does not find two “STOP” signals. [2]

(c) With no stops, Damon completes the 12 kilometre journey at an average speed
of 40 kilometres per hour.
(i) Find the time taken in minutes for this journey. [1]
(ii) When Damon has to stop at a signal it adds 3 minutes to this journey time.

Calculate his average speed, in kilometres per hour, if he stops at both road signals. [2]

(d) Elsa takes a different route from Acity to Beetown.


This route is 15 kilometres and there are no road signals.
Elsa’s average speed for this journey is 40 kilometres per hour.
Find
(i) the time taken in minutes for this journey, [1]
(ii) the probability that Damon takes more time than this on his 12 kilometre journey. [2]

0580/4, 0581/4 Jun/03 [Turn over


4

4 Answer the whole of this question on a sheet of graph paper.

x 04 03 02 01 0 1 2 3 4
f(x) 08 4.5 8 5.5 0 05.5 08 04.5 8

(a) Using a scale of 2 cm to represent 1 unit on the x-axis and 2 cm to represent 4 units on the
y-axis, draw axes for 04 ≤ x ≤ 4 and 08 ≤ y ≤ 8.
Draw the curve y # f(x) using the table of values given above. [5]

(b) Use your graph to solve the equation f(x) # 0. [2]

(c) On the same grid, draw y # g(x) for 04 ≤ x ≤ 4, where g(x) # x ! 1. [2]

(d) Write down the value of


(i) g(1),
(ii) fg(1),
(iii) g01(4),
(iv) the positive solution of f(x) # g(x). [4]

(e) Draw the tangent to y # f(x) at x # 3. Use it to calculate an estimate of the gradient of the curve
at this point. [3]

5 (a) Calculate the area of an equilateral triangle with sides 10 cm. [2]

(b) Calculate the radius of a circle with circumference 10 cm. [2]

(c)

10 cm 10 cm
Diagram 1 Diagram 2 Diagram 3

The diagrams represent the nets of 3 solids. Each straight line is 10 cm long. Each circle has
circumference 10 cm. The arc length in Diagram 3 is 10 cm.

(i) Name the solid whose net is Diagram 1. Calculate its surface area. [3]
(ii) Name the solid whose net is Diagram 2. Calculate its volume. [4]
(iii) Name the solid whose net is Diagram 3. Calculate its perpendicular height. [4]

0580/4, 0581/4 Jun/03


5

(x + 1) cm

(x + 4) cm

NOT TO
SCALE
2x cm

A rectangular-based open box has external dimensions of 2x cm, (x ! 4) cm and (x ! 1) cm.

(a) (i) Write down the volume of a cuboid with these dimensions. [1]
(ii) Expand and simplify your answer. [1]

(b) The box is made from wood 1 cm thick.

(i) Write down the internal dimensions of the box in terms of x. [3]
(ii) Find the volume of the inside of the box and show that the volume of the wood
is 8x2 ! 12x cubic centimetres. [3]

(c) The volume of the wood is 1980 cm3.

(i) Show that 2x2 ! 3x 0 495 # 0 and solve this equation. [5]
(ii) Write down the external dimensions of the box. [2]

0580/4, 0581/4 Jun/03 [Turn over


6

7 Q

C D
P R

B E
X
b

O F S
a A

A star is made up of a regular hexagon, centre X, surrounded by 6 equilateral triangles.


{O|A # a and {O|B # b.

(a) Write the following vectors in terms of a andor b, giving your answers in their simplest form.
(i) {O|S, [1]
(ii) {A|B, [1]
(iii) {C|D, [1]
(iv) {O|R, [2]
(v) {C|F. [2]

(b) When | a | # 5, write down the value of


(i) | b |, [1]
(ii) | a 0 b |. [1]

(c) Describe fully a single transformation which maps


(i) triangle OBA onto triangle OQS, [2]
(ii) triangle OBA onto triangle RDE, with O mapped onto R and B mapped onto D. [2]

(d) (i) How many lines of symmetry does the star have? [1]
(ii) When triangle OQS is rotated clockwise about X, it lies on triangle PRT, with O on P.
Write down the angle of rotation. [1]

0580/4, 0581/4 Jun/03


7

8 Answer the whole of this question on a sheet of graph paper.

In a survey, 200 shoppers were asked how much they had just spent in a supermarket.
The results are shown in the table.

Amount($x) 0 ` x ≤ 20 20 ` x ≤ 40 40 ` x ≤ 60 60 ` x ≤ 80 80 ` x ≤ 100 100 ` x ≤ 140


Number of shoppers 10 32 48 54 36 20

(a) (i) Write down the modal class. [1]


(ii) Calculate an estimate of the mean amount, giving your answer correct to 2 decimal
places. [4]

(b) (i) Make a cumulative frequency table for these 200 shoppers. [2]
(ii) Using a scale of 2 cm to represent $20 on the horizontal axis and 2 cm to represent
20 shoppers on the vertical axis, draw a cumulative frequency diagram for this data. [4]

(c) Use your cumulative frequency diagram to find


(i) the median amount, [1]
(ii) the upper quartile, [1]
(iii) the interquartile range, [1]
(iv) how many shoppers spent at least $75. [2]

Question 9 is on the next page

0580/4, 0581/4 Jun/03 [Turn over


8

9 (a)
4 1
1
1
3
1

1 3 1 3
Diagram 1 Diagram 2 Diagram 3 Diagram 4

Diagram 1 shows a triangle with its base divided in the ratio 1 : 3.

Diagram 2 shows a parallelogram with its base divided in the ratio 1 : 3.

Diagram 3 shows a kite with a diagonal divided in the ratio 1 : 3.

Diagram 4 shows two congruent triangles and a trapezium each of height 1 unit.

For each of the four diagrams, write down the percentage of the total area which is shaded. [7]

(b)
1
2
80° 80°
O
O

Diagram 5 Diagram 6 Diagram 7

Diagram 5 shows a semicircle, centre O.

Diagram 6 shows two circles with radii 1 unit and 5 units.

Diagram 7 shows two sectors, centre O, with radii 2 units and 3 units.

For each of diagrams 5, 6 and 7, write down the fraction of the total area which is shaded. [6]

0580/4, 0581/4 Jun/03


UNIVERSITY OF CAMBRIDGE INTERNATIONAL EXAMINATIONS
International General Certificate of Secondary Education

MATHEMATICS 0580/04
0581/04
Paper 4 (Extended)
May/June 2004
Additional Materials: Answer Booklet/Paper 2 hours 30 minutes
Electronic calculator
Geometrical instruments
Graph paper (2 sheets)
Mathematical tables (optional)
Tracing paper (optional)

READ THESE INSTRUCTIONS FIRST

Write your answers and working on the separate Answer Booklet/Paper provided.
Write your name, Centre number and candidate number on all the work you hand in.
Write in dark blue or black pen on both sides of the paper.
You may use a soft pencil for any diagrams or graphs.
Do not use staples, paper clips, highlighters, glue or correction fluid.

Answer all questions.


At the end of the examination, fasten all your work securely together.
The number of marks is given in brackets [ ] at the end of each question or part question.

All working must be clearly shown. It should be done on the same sheet as the rest of the answer.
Marks will be given for working which shows that you know how to solve the problem even if you get the
answer wrong.
The total of the marks for this paper is 130.
Electronic calculators should be used.
If the degree of accuracy is not specified in the question, and if the answer is not exact, give the answer to
three significant figures.
Answers in degrees should be given to one decimal place.
For p use either your calculator value or 3.142.

This document consists of 9 printed pages and 3 blank pages.


IB04 06_0580_04/4RP
Ó UCLES 2004 [Turn over
2

1 Fatima and Mohammed each buys a bike.

(a) Fatima buys a city-bike which has a price of $120.


She pays 60 % of this price and then pays $10 per month for 6 months.

(i) How much does Fatima pay altogether? [2]

(ii) Work out your answer to part (a)(i) as a percentage of the original price of $120. [2]

(b) Mohammed pays $159.10 for a mountain-bike in a sale.


The original price had been reduced by 14 %.
Calculate the original price of the mountain-bike. [2]

(c) Mohammed’s height is 169 cm and Fatima’s height is 156 cm.


The frame sizes of their bikes are in the same ratio as their heights.
The frame size of Mohammed’s bike is 52 cm.
Calculate the frame size of Fatima’s bike. [2]

(d) Fatima and Mohammed are members of a school team which takes part in a bike ride for charity.

(i) Fatima and Mohammed ride a total distance of 36 km.


The ratio distance Fatima rides : distance Mohammed rides is 11 : 9.
Work out the distance Fatima rides. [2]

2
(ii) The distance of 36 km is only 23
of the total distance the team rides.

Calculate this total distance. [2]

Ó UCLES 2004 0580/4, 0581/4 Jun/04


3

2 Answer all of this question on a sheet of graph paper.

2
(a) f ( x ) = x - x - 3 .

x -3 -2 -1 0 1 2 3 4
f(x) p 3 -1 -3 q -1 3 r

(i) Find the values of p, q and r. [3]

(ii) Draw the graph of y = f ( x ) for - 3 x 4.


Use a scale of 1 cm to represent 1 unit on each axis. [4]

(iii) By drawing a suitable line, estimate the gradient of the graph at the point where x = - 1 . [3]

3
x
(b) g ( x ) = 6 - .
3

x -2 -1 0 1 2 3
g(x) 8.67 u v 5.67 3.33 -3

(i) Find the values of u and v. [2]

(ii) On the same grid as part (a) (ii) draw the graph of y = g ( x ) for –2 x 3. [4]

(c) (i) Show that the equation f(x) = g(x) simplifies to x3 + 3x2 – 3x - 27 = 0. [1]

(ii) Use your graph to write down a solution of the equation x3 + 3x2 – 3x - 27 = 0. [1]

Ó UCLES 2004 0580/4, 0581/4 Jun/04 [Turn over


4

3 The depth, d centimetres, of a river was recorded each day during a period of one year (365 days).
The results are shown by the cumulative frequency curve.

400

300

cumulative
frequency 200

100

0
0 10 20 30 40 50 60 70

depth, d (cm)

(a) Use the cumulative frequency curve to find

(i) the median depth, [1]

(ii) the inter-quartile range, [2]

(iii) the depth at the 40th percentile, [2]

(iv) the number of days when the depth of the river was at least 25 cm. [2]

(b)

d 0<d 10 10<d 20 20<d 30 30<d 40 40<d 50 50<d 60 60<d 70


Number of days 17 41 62 98 85 p q

(i) Show that p = 47 and q = 15. [2]

(ii) Use the information in the table and the values of p and q to calculate an estimate of the mean
depth of the river. [4]

Ó UCLES 2004 0580/4, 0581/4 Jun/04


5

(c) The following information comes from the table in part (b).

d 0<d 20 20<d 40 40<d 70


Number of days 58 160 147

A histogram was drawn to show this information.


The height of the column for the interval 20 < d 40 was 8 cm.
Calculate the height of each of the other two columns.
[Do not draw the histogram.] [3]

4
C

NOT TO
SCALE

D
11.1 cm

37o
E
A

9.5 cm
70o
B

ABCD is a cyclic quadrilateral.


AB = 9.5 cm, BC = 11.1 cm, angle ABC = 70o and angle CAD = 37o.

(a) Calculate the length of AC. [4]

(b) Explain why angle ADC = 110o. [1]

(c) Calculate the length of AD. [4]

(d) A point E lies on the circle such that triangle ACE is isosceles, with EA = EC.

(i) Write down the size of angle AEC. [1]

(ii) Calculate the area of triangle ACE. [3]

Ó UCLES 2004 0580/4, 0581/4 Jun/04 [Turn over


6

5 Maria walks 10 kilometres to a waterfall at an average speed of x kilometres per hour.

(a) Write down, in terms of x, the time taken in hours. [1]

(b) Maria returns from the waterfall but this time she walks the 10 kilometres at an average speed of
(x + 1) kilometres per hour. The time of the return journey is 30 minutes less than the time of the first
journey.
Write down an equation in x and show that it simplifies to x2 + x – 20 = 0. [4]

(c) Solve the equation x2 + x – 20 = 0. [2]

(d) Find the time Maria takes to walk to the waterfall. [2]

NOT TO
SCALE
13 cm

7 cm

The diagram shows a solid made up of a hemisphere and a cone.


The base radius of the cone and the radius of the hemisphere are each 7 cm.
The height of the cone is 13 cm.

(a) (i) Calculate the total volume of the solid.


2 3
[The volume of a hemisphere of radius r is given by V = 3
πr .]

[The volume of a cone of radius r and height h is given by V = 13 πr h .]


2
[2]

(ii) The solid is made of wood and 1 cm3 of this wood has a mass of 0.94 g.
Calculate the mass of the solid, in kilograms, correct to 1 decimal place. [3]

(b) Calculate the curved surface area of the cone.


[The curved surface area of a cone of radius r and sloping edge l is given by A = πrl .] [3]

(c) The cost of covering all the solid with gold plate is $411.58.
Calculate the cost of this gold plate per square centimetre.
2
[The curved surface area of a hemisphere is given by A = 2 πr .] [5]

Ó UCLES 2004 0580/4, 0581/4 Jun/04


7

7 (a) There are 30 students in a class.


20 study Physics, 15 study Chemistry and 3 study neither Physics nor Chemistry.

P C

(i) Copy and complete the Venn diagram to show this information. [2]

(ii) Find the number of students who study both Physics and Chemistry. [1]

(iii) A student is chosen at random. Find the probability that the student studies Physics but not
Chemistry. [2]

(iv) A student who studies Physics is chosen at random. Find the probability that this student
does not study Chemistry. [2]

(b)

A B

Bag A contains 6 white beads and 3 black beads.


Bag B contains 6 white beads and 4 black beads.
One bead is chosen at random from each bag.
Find the probability that

(i) both beads are black, [2]

(ii) at least one of the two beads is white. [2]

The beads are not replaced.


A second bead is chosen at random from each bag.
Find the probability that

(iii) all four beads are white, [3]

(iv) the beads are not all the same colour. [3]

Ó UCLES 2004 0580/4, 0581/4 Jun/04 [Turn over


8

8
y

5
F
4
G
3
B
2
A
1

x
_ _ _ _ _ 0
5 4 3 2 1 1 2 3 4 5
_
1
_
2
D
C _
3
_
4
E
_
5

(a) Describe fully the single transformation which maps

(i) shape A onto shape B, [2]

(ii) shape B onto shape C, [2]

(iii) shape A onto shape D, [2]

(iv) shape B onto shape E, [2]

(v) shape B onto shape F, [2]

(vi) shape A onto shape G. [2]

æ0 - 1ö
(b) A transformation is represented by the matrix ç ÷.
è1 0ø
Which shape above is the image of shape A after this transformation? [2]

(c) Find the 2 by 2 matrix representing the transformation which maps

(i) shape B onto shape D, [2]

(ii) shape A onto shape G. [2]

Ó UCLES 2004 0580/4, 0581/4 Jun/04


9

9 Answer all of this question on a sheet of graph paper.

A shop buys x pencils and y pens.


Pencils cost 15 cents each and pens cost 25 cents each.

(a) There is a maximum of $20 to spend.


Show that 3 x + 5 y 400. [1]

(b) The number of pens must not be greater than the number of pencils.
Write down an inequality, in terms of x and y, to show this information. [2]

(c) There must be at least 35 pens.


Write down an inequality to show this information. [1]

(d) (i) Using a scale of 1 cm to represent 10 units on each axis, draw an x-axis for 0 x 150
and a y-axis for 0 y 100. [1]

(ii) Draw three lines on your graph to show the inequalities in parts (a), (b) and (c).
Shade the unwanted regions. [5]

(e) When 70 pencils are bought, what is the largest possible number of pens? [1]

(f) The profit on each pencil is 5 cents and the profit on each pen is 7 cents.
Find the largest possible profit. [3]

© UCLES 2004 0580/4, 0581/4 Jun/04


10

BLANK PAGE

0580/4, 0581/4 Jun/04


11

BLANK PAGE

0580/4, 0581/4 Jun/04


12

BLANK PAGE

University of Cambridge International Examinations is part of the University of Cambridge Local Examinations Syndicate (UCLES), which is itself a department of the
University of Cambridge.

0580/4, 0581/4 Jun/04


UNIVERSITY OF CAMBRIDGE INTERNATIONAL EXAMINATIONS
International General Certificate of Secondary Education

MATHEMATICS 0580/04
0581/04
Paper 4 (Extended)
May/June 2005

Additional Materials: Answer Booklet/Paper 2 hours 30 minutes


Electronic calculator
Geometrical instruments
Graph paper (3 sheets)
Mathematical tables (optional)
Tracing paper (optional)

READ THESE INSTRUCTIONS FIRST

Write your answers and working on the separate Answer Booklet/Paper provided.
Write your name, Centre number and candidate number on all the work you hand in.
Write in dark blue or black pen on both sides of the paper.
You may use a soft pencil for any diagrams or graphs.
Do not use staples, paper clips, highlighters, glue or correction fluid.

Answer all questions.


At the end of the examination, fasten all your work securely together.
The number of marks is given in brackets [ ] at the end of each question or part question.

All working must be clearly shown. It should be done on the same sheet as the rest of the answer.
Marks will be given for working which shows that you know how to solve the problem even if you get the
answer wrong.
The total of the marks for this paper is 130.
Electronic calculators should be used.
If the degree of accuracy is not specified in the question, and if the answer is not exact, give the answer to
three significant figures.
Give answers in degrees to one decimal place.
For π use either your calculator value or 3.142.

This document consists of 8 printed pages.


IB05 06_0580_04/6RP
 UCLES 2005 [Turn over
2

1 Hassan sells fruit and vegetables at the market.

(a) The mass of fruit and vegetables he sells is in the ratio


fruit : vegetables = 5 : 7.
Hassan sells 1.33 tonnes of vegetables.
How many kilograms of fruit does he sell? [3]

(b) The amount of money Hassan receives from selling fruit and vegetables is in the ratio
fruit : vegetables = 9 : 8.
Hassan receives a total of $765 from selling fruit and vegetables.
Calculate how much Hassan receives from selling fruit. [2]

(c) Calculate the average price of Hassan’s fruit, in dollars per kilogram. [2]

(d) (i) Hassan sells oranges for $0.35 per kilogram.


He reduces this price by 40%.
Calculate the new price per kilogram. [2]

(ii) The price of $0.35 per kilogram of oranges is an increase of 25% on the previous day’s price.
Calculate the previous day’s price. [2]

2 Answer the whole of this question on a new page.

D 9 cm C

NOT TO
7 cm SCALE

A 12 cm B

The diagram shows a trapezium ABCD.


AB = 12 cm, DC = 9 cm and the perpendicular distance between these parallel sides is 7 cm.
AD = BC.

(a) Approximately halfway down your page, draw a line AB of length 12 cm. [1]

(b) Using a straight edge and compasses only, construct the perpendicular bisector of AB. [2]

(c) Complete an accurate drawing of the trapezium ABCD. [2]

(d) Measure angle ABC, giving your answer correct to the nearest degree. [1]

(e) Use trigonometry to calculate angle ABC.


Show all your working and give your answer correct to 1 decimal place. [2]

(f) On your diagram,


(i) draw the locus of points inside the trapezium which are 5 cm from D, [1]
(ii) using a straight edge and compasses only, construct the locus of points equidistant from DA
and from DC, [2]
(iii) shade the region inside the trapezium containing points which are less than 5 cm from D and
nearer to DA than to DC. [1]

© UCLES 2005 0580/04, 0581/04 Jun 05


3

3
y

6
P
5
X S
4

2
R
1

x
–6 –5 –4 –3 –2 –1 0 1 2 3 4 5 6
–1
Q
–2

–3

–4

–5

–6

(a) Describe fully the single transformation which maps


(i) triangle X onto triangle P, [2]
(ii) triangle X onto triangle Q, [2]
(iii) triangle X onto triangle R, [3]
(iv) triangle X onto triangle S. [3]

(b) Find the 2 by 2 matrix which represents the transformation that maps
(i) triangle X onto triangle Q, [2]
(ii) triangle X onto triangle S. [2]

© UCLES 2005 0580/04, 0581/04 Jun 05 [Turn over


4

4 Answer the whole of this question on a sheet of graph paper.

The table gives values of f(x) = 2x, for – 2 x 4.

x -2 -1 0 1 2 3 4

f(x) p 0.5 q 2 4 r 16

(a) Find the values of p, q and r. [3]

(b) Using a scale of 2 cm to 1 unit on the x-axis and 1 cm to 1 unit on the y-axis, draw the graph of
y = f(x) for – 2 x 4. [5]

(c) Use your graph to solve the equation 2x = 7. [1]

(d) What value does f(x) approach as x decreases? [1]

(e) By drawing a tangent, estimate the gradient of the graph of y = f(x) when x = 1.5. [3]

(f) On the same grid draw the graph of y = 2x + 1 for 0 x 4. [2]

(g) Use your graph to find the non-integer solution of 2x = 2x + 1. [2]

5 C

D B

d NOT TO
c
SCALE
E A

O
OABCDE is a regular hexagon.
With O as origin the position vector of C is c and the position vector of D is d.
(a) Find, in terms of c and d,
(i) , [1]
(ii) , [2]
(iii) the position vector of B. [2]
(b) The sides of the hexagon are each of length 8 cm.
Calculate
(i) the size of angle ABC, [1]
(ii) the area of triangle ABC, [2]
(iii) the length of the straight line AC, [3]
(iv) the area of the hexagon. [3]

© UCLES 2005 0580/04, 0581/04 Jun 05


5

6
NOT TO SCALE
l
0.7 cm h

16.5 cm 1.5 cm

The diagram shows a pencil of length 18 cm.


It is made from a cylinder and a cone.
The cylinder has diameter 0.7 cm and length 16.5 cm.
The cone has diameter 0.7 cm and length 1.5 cm.

(a) Calculate the volume of the pencil.


1
[The volume, V, of a cone of radius r and height h is given by V =
3
πr 2h.] [3]

(b)

NOT TO
SCALE

18 cm
x cm

w cm

Twelve of these pencils just fit into a rectangular box of length 18 cm, width w cm and height x cm.
The pencils are in 2 rows of 6 as shown in the diagram.

(i) Write down the values of w and x. [2]

(ii) Calculate the volume of the box. [2]

(iii) Calculate the percentage of the volume of the box occupied by the pencils. [2]

(c) Showing all your working, calculate

(i) the slant height, l, of the cone, [2]

(ii) the total surface area of one pencil, giving your answer correct to 3 significant figures.
[The curved surface area, A, of a cone of radius r and slant height l is given by A = πrl .] [6]

© UCLES 2005 0580/04, 0581/04 Jun 05 [Turn over


6

7 The speeds (v kilometres/hour) of 150 cars passing a 50 km/h speed limit sign are recorded.
A cumulative frequency curve to show the results is drawn below.

150

140

130

120

110

100

90

Cumulative 80
frequency
70

60

50

40

30

20

10

0 30 35 40 45 50 55 60
Speed (v kilometres / hour)

(a) Use the graph to find

(i) the median speed, [1]

(ii) the inter-quartile range of the speeds, [2]

(iii) the number of cars travelling with speeds of more than 50 km/h. [2]

© UCLES 2005 0580/04, 0581/04 Jun 05


7

(b) A frequency table showing the speeds of the cars is

Speed (v km/h) 30<v 35 35<v 40 40<v 45 45<v 50 50<v 55 55<v 60

Frequency 10 17 33 42 n 16

(i) Find the value of n. [1]

(ii) Calculate an estimate of the mean speed. [4]

(c) Answer this part of this question on a sheet of graph paper.

Another frequency table for the same speeds is

Speed (v km/h) 30<v 40 40<v 55 55<v 60

Frequency 27 107 16

Draw an accurate histogram to show this information.


Use 2 cm to represent 5 units on the speed axis and 1 cm to represent 1 unit on the frequency density
axis (so that 1 cm2 represents 2.5 cars). [5]

8 f(x) = x2 – 4x + 3 and g(x) = 2x – 1.

(a) Solve f(x) = 0. [2]

(b) Find g-1(x). [2]

(c) Solve f(x) = g(x), giving your answers correct to 2 decimal places. [5]

(d) Find the value of gf(–2). [2]

(e) Find fg(x). Simplify your answer. [3]

© UCLES 2005 0580/04, 0581/04 Jun 05 [Turn over


8

9 Answer the whole of this question on a sheet of graph paper.

A taxi company has “SUPER” taxis and “MINI” taxis.


One morning a group of 45 people needs taxis.
For this group the taxi company uses x “SUPER” taxis and y “MINI” taxis.
A “SUPER” taxi can carry 5 passengers and a “MINI” taxi can carry 3 passengers.
So 5x + 3y 45.

(a) The taxi company has 12 taxis.


Write down another inequality in x and y to show this information. [1]

(b) The taxi company always uses at least 4 “MINI” taxis.


Write down an inequality in y to show this information. [1]

(c) Draw x and y axes from 0 to 15 using 1 cm to represent 1 unit on each axis. [1]

(d) Draw three lines on your graph to show the inequality 5x + 3y 45 and the inequalities from parts
(a) and (b).
Shade the unwanted regions. [6]

(e) The cost to the taxi company of using a “SUPER” taxi is $20 and the cost of using a “MINI” taxi is
$10.
The taxi company wants to find the cheapest way of providing “SUPER” and “MINI” taxis for this
group of people.
Find the two ways in which this can be done. [3]

(f) The taxi company decides to use 11 taxis for this group.
(i) The taxi company charges $30 for the use of each “SUPER” taxi and $16 for the use of each
“MINI” taxi.
Find the two possible total charges. [3]

(ii) Find the largest possible profit the company can make, using 11 taxis. [1]

Permission to reproduce items where third-party owned material protected by copyright is included has been sought and cleared where possible. Every
reasonable effort has been made by the publisher (UCLES) to trace copyright holders, but if any items requiring clearance have unwittingly been included, the
publisher will be pleased to make amends at the earliest possible opportunity.

University of Cambridge International Examinations is part of the University of Cambridge Local Examinations Syndicate (UCLES), which is itself a department
of the University of Cambridge.
© UCLES 2005 0580/04, 0581/04 Jun 05
UNIVERSITY OF CAMBRIDGE INTERNATIONAL EXAMINATIONS
International General Certificate of Secondary Education

MATHEMATICS 0580/04
0581/04
Paper 4 (Extended)
May/June 2006

Additional Materials: Answer Booklet/Paper 2 hours 30 minutes


Electronic calculator
Geometrical instruments
Graph paper (2 sheets)
Mathematical tables (optional)
Tracing paper (optional)

READ THESE INSTRUCTIONS FIRST

Write your answers and working on the separate Answer Booklet/Paper provided.
Write your name, Centre number and candidate number on all the work you hand in.
Write in dark blue or black pen on both sides of the paper.
You may use a soft pencil for any diagrams or graphs.
Do not use staples, paper clips, highlighters, glue or correction fluid.

Answer all questions.


At the end of the examination, fasten all your work securely together.
The number of marks is given in brackets [ ] at the end of each question or part question.

All working must be clearly shown. It should be done on the same sheet as the rest of the answer.
Marks will be given for working which shows that you know how to solve the problem even if you get the
answer wrong.
The total of the marks for this paper is 130.
Electronic calculators should be used.
If the degree of accuracy is not specified in the question, and if the answer is not exact, give the answer to
three significant figures.
Give answers in degrees to one decimal place.
For π use either your calculator value or 3.142.

This document consists of 7 printed pages and 1 blank page.


IB06 06_0580_04/7RP
 UCLES 2006 [Turn over
2

1 (a) A train completed a journey of 850 kilometres with an average speed of 80 kilometres per hour.
Calculate, giving exact answers, the time taken for this journey in

(i) hours, [2]

(ii) hours, minutes and seconds. [1]

(b) Another train took 10 hours 48 minutes to complete the same 850 km journey.

(i) It departed at 19 20.


At what time, on the next day, did this train complete the journey? [1]

(ii) Calculate the average speed, in kilometres per hour, for the journey. [2]

(c)

25

C D
20

Speed 15 BB
(metres per second)
10

5 A

O 1 2 3 4 5 6 7 8 9 10
Time (seconds)

The solid line OABCD on the grid shows the first 10 seconds of a car journey.

(i) Describe briefly what happens to the speed of the car between B and C. [1]

(ii) Describe briefly what happens to the acceleration of the car between B and C. [1]

(iii) Calculate the acceleration between A and B. [2]

(iv) Using the broken straight line OC, estimate the total distance travelled by the car in the
whole 10 seconds. [3]

(v) Explain briefly why, in this case, using the broken line makes the answer to part (iv) a good
estimate of the distance travelled. [1]

(vi) Calculate the average speed of the car during the 10 seconds.
Give your answer in kilometres per hour. [2]

© UCLES 2006 0580/04, 0581/04 Jun 2006


3

2
NOT TO B C
SCALE
B
C
12 cm
18 cm
A O
12 cm
A

40 cm E 22 cm D
E D

Diagram 1 Diagram 2

Diagram 1 shows a closed box. The box is a prism of length 40 cm.


The cross-section of the box is shown in Diagram 2, with all the right-angles marked.
AB is an arc of a circle, centre O, radius 12 cm.
ED = 22 cm and DC = 18 cm.

Calculate

(a) the perimeter of the cross-section, [3]


(b) the area of the cross-section, [3]
(c) the volume of the box, [1]
(d) the total surface area of the box. [4]

3 Answer the whole of this question on a sheet of graph paper.

(a) Find the values of k, m and n in each of the following equations, where a > 0.

(i) a0 = k, [1]

1
(ii) am = , [1]
a

(iii) an = a3 . [1]

(b) The table shows some values of the function f(x) = 2x.

x −2 −1 −0.5 0 0.5 1 1.5 2 3


f(x) r 0.5 0.71 s 1.41 2 2.83 4 t

(i) Write down the values of r, s and t. [3]


(ii) Using a scale of 2 cm to represent 1 unit on each axis, draw an x-axis from −2 to 3 and a
y-axis from 0 to 10. [1]
(iii) On your grid, draw the graph of y = f(x) for −2 x 3. [4]

(c) The function g is given by g(x) = 6 – 2x.

(i) On the same grid as part (b), draw the graph of y = g(x) for – 2 x 3. [2]
(ii) Use your graphs to solve the equation 2x = 6 – 2x. [1]
(iii) Write down the value of x for which 2x < 6 – 2x for x ∈ {positive integers}. [1]

© UCLES 2006 0580/04, 0581/04 Jun 2006 [Turn over


4

North

A B
O

NOT TO
SCALE

The diagram shows a plan for a new city.


It is to be built inside a circle of radius 5 km.
The areas where homes can be built are shaded on the diagram.
The homes must be at least 2 km from the centre of the city, O.
The homes must also be at least 0.5 km from two main roads CD and AB, which are in North-South and
West-East directions.

(a) Using 1 cm to represent 1 km, make an accurate scale drawing showing the areas for the homes.
(You do not need to shade these areas.) [4]

(b) The town hall, T, will be built so that it is equidistant from the roads OA and OC.
It will be 1 km from O and West of CD.

(i) On your scale drawing, using a straight edge and compasses only, draw the locus of points, inside
the town, which are equidistant from OA and OC. [2]

(ii) Mark and label the point T. [1]

(c) The police station, P, will be built so that it is equidistant from T and B.
It will be 3 km from O and North of AB.
Showing all your construction lines, find and label the point P. [3]

(d) What will be the actual straight line distance between the town hall and the police station? [1]

© UCLES 2006 0580/04, 0581/04 Jun 2006


5

5 The length, y, of a solid is inversely proportional to the square of its height, x.

(a) Write down a general equation for x and y.


Show that when x = 5 and y = 4.8 the equation becomes x 2 y = 120 . [2]

(b) Find y when x = 2. [1]

(c) Find x when y = 10. [2]

(d) Find x when y = x. [2]

(e) Describe exactly what happens to y when x is doubled. [2]

(f) Describe exactly what happens to x when y is decreased by 36%. [2]

(g) Make x the subject of the formula x 2 y = 120 . [2]


6

13 cm NOT TO
SCALE

D C

E 6 cm

A 8 cm B

The diagram shows a pyramid on a horizontal rectangular base ABCD.


The diagonals of ABCD meet at E.
P is vertically above E.
AB = 8 cm, BC = 6 cm and PC = 13 cm.

(a) Calculate PE, the height of the pyramid. [3]

(b) Calculate the volume of the pyramid.


1
[The volume of a pyramid is given by 3
× area of base × height.] [2]

(c) Calculate angle PCA. [2]

(d) M is the mid-point of AD and N is the mid-point of BC.


Calculate angle MPN. [3]

(e) (i) Calculate angle PBC. [2]

(ii) K lies on PB so that BK = 4 cm.


Calculate the length of KC. [3]

© UCLES 2006 0580/04, 0581/04 Jun 2006 [Turn over


6

 3
7 Transformation T is translation by the vector   .
 2
Transformation M is reflection in the line y = x.

(a) The point A has co-ordinates (2, 1).

Find the co-ordinates of

(i) T(A), [1]

(ii) MT(A). [2]

(b) Find the 2 by 2 matrix M, which represents the transformation M. [2]

(c) Show that, for any value of k, the point Q (k – 2, k – 3) maps onto a point on the line y = x following
the transformation TM(Q). [3]

(d) Find M-1, the inverse of the matrix M. [2]

 0 3  0 4
(e) N is the matrix such that N +   = .
1 0 0 0

(i) Write down the matrix N. [2]

(ii) Describe completely the single transformation represented by N. [3]

8 (a)

2x + 4

x+2 x NOT TO
SCALE

x2 – 40

The diagram shows a trapezium.


Two of its angles are 90o.
The lengths of the sides are given in terms of x.
The perimeter is 62 units.

(i) Write down a quadratic equation in x to show this information. Simplify your equation. [2]

(ii) Solve your quadratic equation. [2]

(iii) Write down the only possible value of x. [1]

(iv) Calculate the area of the trapezium. [2]

© UCLES 2006 0580/04, 0581/04 Jun 2006


7

(b)

2y – 1
y NOT TO
SCALE

y+2

The diagram shows a right-angled triangle.


The lengths of the sides are given in terms of y.

(i) Show that 2y2 – 8y – 3 = 0. [3]


(ii) Solve the equation 2y2 – 8y – 3 = 0, giving your answers to 2 decimal places. [4]
(iii) Calculate the area of the triangle. [2]

9 (a) The numbers 0, 1, 1, 1, 2, k, m, 6, 9, 9 are in order (k ≠ m).


Their median is 2.5 and their mean is 3.6.

(i) Write down the mode. [1]


(ii) Find the value of k. [1]
(iii) Find the value of m. [2]
(iv) Maria chooses a number at random from the list.
The probability of choosing this number is 1 . Which number does she choose? [1]
5

(b) 100 students are given a question to answer.


The time taken (t seconds) by each student is recorded and the results are shown in the table.

t 0<t 20 20<t 30 30<t 35 35<t 40 40<t 50 50<t 60 60<t 80


Frequency 10 10 15 28 22 7 8

(i) Calculate an estimate of the mean time taken. [4]

(ii) Two students are picked at random.


What is the probability that they both took more than 50 seconds?
Give your answer as a fraction in its lowest terms. [3]

Answer part (c) on a sheet of graph paper.

(c) The data in part (b) is re-grouped to give the following table.

t 0<t 30 30<t 60 60<t 80


Frequency p q 8

(i) Write down the values of p and q. [2]


(ii) Draw an accurate histogram to show these results.
Use a scale of 1 cm to represent 5 seconds on the horizontal time axis.
Use a scale of 1 cm to 0.2 units of frequency density (so that 1 cm2 on your histogram represents
1 student). [4]

© UCLES 2006 0580/04, 0581/04 Jun 2006


8

BLANK PAGE

Permission to reproduce items where third-party owned material protected by copyright is included has been sought and cleared where possible. Every reasonable effort has
been made by the publisher (UCLES) to trace copyright holders, but if any items requiring clearance have unwittingly been included, the publisher will be pleased to make
amends at the earliest possible opportunity.

University of Cambridge International Examinations is part of the University of Cambridge Local Examinations Syndicate (UCLES), which is itself a department of the
University of Cambridge.

0580/04, 0581/04 Jun 2006


UNIVERSITY OF CAMBRIDGE INTERNATIONAL EXAMINATIONS
International General Certificate of Secondary Education

MATHEMATICS 0580/04, 0581/04


Paper 4 (Extended) May/June 2007
2 hours 30 minutes
Additional Materials: Answer Booklet/Paper Graph paper (2 sheets)
*5128615949*

Electronic calculator Mathematical tables (optional)


Geometrical instruments Tracing paper (optional)

READ THESE INSTRUCTIONS FIRST

If you have been given an Answer Booklet, follow the instructions on the front cover of the Booklet.
Write your Centre number, candidate number and name on all the work you hand in.
Write in dark blue or black pen.
You may use a pencil for any diagrams or graphs.
Do not use staples, paper clips, highlighters, glue or correction fluid.

Answer all questions.


All working must be clearly shown. It should be done on the same sheet as the rest of the answer.
Marks will be given for working which shows that you know how to solve the problem even if you get the
answer wrong.
Electronic calculators should be used.
If the degree of accuracy is not specified in the question, and if the answer is not exact, give the answer to
three significant figures.
Give answers in degrees to one decimal place.
For π use either your calculator value or 3.142.

At the end of the examination, fasten all your work securely together.
The number of marks is given in brackets [ ] at the end of each question or part question.
The total of the marks for this paper is 130.

This document consists of 11 printed pages and 1 blank page.

IB07 06_0580_04/6RP
© UCLES 2007 [Turn over
2

DO NOT WRITE YOUR ANSWERS ON THIS QUESTION PAPER.

WRITE ALL YOUR WORKING AND ANSWERS ON THE SEPARATE ANSWER BOOK OR
PAPER PROVIDED.

1 (a) The scale of a map is 1:20 000 000.

On the map, the distance between Cairo and Addis Ababa is 12 cm.

(i) Calculate the distance, in kilometres, between Cairo and Addis Ababa. [2]

(ii) On the map the area of a desert region is 13 square centimetres.

Calculate the actual area of this desert region, in square kilometres. [2]

(b) (i) The actual distance between Cairo and Khartoum is 1580 km.

On a different map this distance is represented by 31.6 cm.

Calculate, in the form 1 : n, the scale of this map. [2]

(ii) A plane flies the 1580 km from Cairo to Khartoum.

It departs from Cairo at 11 55 and arrives in Khartoum at 14 03.

Calculate the average speed of the plane, in kilometres per hour. [4]

© UCLES 2007 0580/04/M/J/07


3

2 Answer the whole of this question on a sheet of graph paper.

(a) Draw and label x and y axes from −6 to 6, using a scale of 1 cm to 1 unit. [1]

(b) Draw triangle ABC with A (2,1), B (3,3) and C (5,1). [1]

(c) Draw the reflection of triangle ABC in the line y = x. Label this A1B1C1. [2]

(d) Rotate triangle A1B1C1 about (0,0) through 90° anti-clockwise. Label this A2B2C2. [2]

(e) Describe fully the single transformation which maps triangle ABC onto triangle A2B2C2. [2]

 1 0
(f) A transformation is represented by the matrix  .
−1 1 

(i) Draw the image of triangle ABC under this transformation. Label this A3B3C3. [3]

 1 0
(ii) Describe fully the single transformation represented by the matrix  . [2]
−1 1 

(iii) Find the matrix which represents the transformation that maps triangle A3B3C3
onto triangle ABC. [2]

© UCLES 2007 0580/04/M/J/07 [Turn over


4

3 (a)

A
B
55°

40.3 cm NOT TO
26.8cm
26.8 cm
SCALE
92°

X 20.1 cm
cm

A, B, C and D lie on a circle.

AC and BD intersect at X.

Angle ABX = 55° and angle AXB = 92°.

BX = 26.8 cm, AX = 40.3 cm and XC = 20.1 cm.

(i) Calculate the area of triangle AXB.


You must show your working. [2]

(ii) Calculate the length of AB.


You must show your working. [3]

(iii) Write down the size of angle ACD. Give a reason for your answer. [2]

(iv) Find the size of angle BDC. [1]

(v) Write down the geometrical word which completes the statement

“Triangle AXB is to triangle DXC.” [1]

(vi) Calculate the length of XD.


You must show your working. [2]

© UCLES 2007 0580/04/M/J/07


5

(b)

P Q

) cm
+2

yc
(y

m
X NOT TO
SCALE
) cm

(y
–1

+
(2y

1)
cm
R S

In the diagram PQ is parallel to RS.

PS and QR intersect at X.

PX = y cm, QX = (y + 2) cm, RX = (2y – 1) cm and SX = (y + 1) cm.

(i) Show that y2 – 4y – 2 = 0. [3]

(ii) Solve the equation y2 – 4y – 2 = 0.

Show all your working and give your answers correct to two decimal places. [4]

(iii) Write down the length of RX. [1]

© UCLES 2007 0580/04/M/J/07 [Turn over


6

4
y

15

10

x
–2 0 2 4 6 8 10

–5

–10

The diagram shows the accurate graph of y = f(x).

(a) Use the graph to find

(i) f(0), [1]

(ii) f(8). [1]

(b) Use the graph to solve

(i) f(x) = 0, [2]

(ii) f(x) = 5. [1]

(c) k is an integer for which the equation f(x) = k has exactly two solutions.

Use the graph to find the two values of k. [2]

(d) Write down the range of values of x for which the graph of y = f(x) has a negative gradient. [2]

(e) The equation f(x) + x – 1 = 0 can be solved by drawing a line on the grid.

(i) Write down the equation of this line. [1]

(ii) How many solutions are there for f(x) + x – 1 = 0? [1]

© UCLES 2007 0580/04/M/J/07


7

5
B E

25 cm
M

O C
14 cm

NOT TO
SCALE

OBCD is a rhombus with sides of 25 cm. The length of the diagonal OC is 14 cm.

(a) Show, by calculation, that the length of the diagonal BD is 48 cm. [3]

(b) Calculate, correct to the nearest degree,

(i) angle BCD, [2]

(ii) angle OBC. [1]

(c) DB = 2p and OC = 2q.


Find, in terms of p and q,

(i) OB , [1]

(ii) OD. [1]

(d) BE is parallel to OC and DCE is a straight line.


Find, in its simplest form, OE in terms of p and q. [2]

(e) M is the mid-point of CE.


Find, in its simplest form, OM in terms of p and q. [2]

7
(f) O is the origin of a co-ordinate grid. OC lies along the x-axis and q =   .
0
 

(DB is vertical and DB = 48.)


Write down as column vectors

(i) p, [1]

(ii) BC . [2]

(g) Write down the value of │DE │. [1]

© UCLES 2007 0580/04/M/J/07 [Turn over


8

6 Answer the whole of this question on a sheet of graph paper.

Kristina asked 200 people how much water they drink in one day.

The table shows her results.

Amount of water (x litres) Number of people

0<x 0.5 8

0.5 < x 1 27

1<x 1.5 45

1.5 < x 2 50

2<x 2.5 39

2.5 < x 3 21

3<x 3.5 7

3.5 < x 4 3

(a) Write down the modal interval. [1]

(b) Calculate an estimate of the mean. [4]

(c) Make a cumulative frequency table for this data. [2]

(d) Using a scale of 4 cm to 1 litre of water on the horizontal axis and 1 cm to 10 people on the
vertical axis, draw the cumulative frequency graph. [5]

(e) Use your cumulative frequency graph to find

(i) the median, [1]

(ii) the 40th percentile, [1]

(iii) the number of people who drink at least 2.6 litres of water. [2]

(f) A doctor recommends that a person drinks at least 1.8 litres of water each day.
What percentage of these 200 people do not drink enough water? [2]

© UCLES 2007 0580/04/M/J/07


9

NOT TO
SCALE

0.8 m

0.3 m

1.2 m

The diagram shows water in a channel.

This channel has a rectangular cross-section, 1.2 metres by 0.8 metres.

(a) When the depth of water is 0.3 metres, the water flows along the channel at 3 metres/minute.

Calculate the number of cubic metres which flow along the channel in one hour. [3]

(b) When the depth of water in the channel increases to 0.8 metres, the water flows at
15 metres/minute.

Calculate the percentage increase in the number of cubic metres which flow along the channel in one
hour.
[4]

(c) The water comes from a cylindrical tank.

When 2 cubic metres of water leave the tank, the level of water in the tank goes down by
1.3 millimetres.

Calculate the radius of the tank, in metres, correct to one decimal place. [4]

(d) When the channel is empty, its interior surface is repaired.

This costs $0.12 per square metre. The total cost is $50.40.

Calculate the length, in metres, of the channel. [4]

© UCLES 2007 0580/04/M/J/07 [Turn over


10

8 A packet of sweets contains chocolates and toffees.

(a) There are x chocolates which have a total mass of 105 grams.

Write down, in terms of x, the mean mass of a chocolate. [1]

(b) There are x + 4 toffees which have a total mass of 105 grams.

Write down, in terms of x, the mean mass of a toffee. [1]

(c) The difference between the two mean masses in parts (a) and (b) is 0.8 grams.

Write down an equation in x and show that it simplifies to x2 + 4x – 525 = 0. [4]

(d) (i) Factorise x2 + 4x – 525. [2]

(ii) Write down the solutions of x2 + 4x – 525 = 0. [1]

(e) Write down the total number of sweets in the packet. [1]

(f) Find the mean mass of a sweet in the packet. [2]

© UCLES 2007 0580/04/M/J/07


11

Diagram 1 Diagram 2 Diagram 3

The first three diagrams in a sequence are shown above.

The diagrams are made up of dots and lines. Each line is one centimetre long.

(a) Make a sketch of the next diagram in the sequence. [1]

(b) The table below shows some information about the diagrams.

Diagram 1 2 3 4 --------- n

Area 1 4 9 16 --------- x

Number of dots 4 9 16 p --------- y

Number of one centimetre lines 4 12 24 q --------- z

(i) Write down the values of p and q. [2]

(ii) Write down each of x, y and z in terms of n. [4]

(c) The total number of one centimetre lines in the first n diagrams is given by the expression

2 n3 + fn2 + gn.
3

(i) Use n = 1 in this expression to show that f + g = 10 . [1]


3

(ii) Use n = 2 in this expression to show that 4f + 2g = 32 . [2]


3

(iii) Find the values of f and g. [3]

(iv) Find the total number of one centimetre lines in the first 10 diagrams. [1]

© UCLES 2007 0580/04/M/J/07


12

BLANK PAGE

Permission to reproduce items where third-party owned material protected by copyright is included has been sought and cleared where possible. Every
reasonable effort has been made by the publisher (UCLES) to trace copyright holders, but if any items requiring clearance have unwittingly been included, the
publisher will be pleased to make amends at the earliest possible opportunity.

University of Cambridge International Examinations is part of the Cambridge Assessment Group. Cambridge Assessment is the brand name of University of
Cambridge Local Examinations Syndicate (UCLES), which is itself a department of the University of Cambridge.

0580/04/M/J/07
UNIVERSITY OF CAMBRIDGE INTERNATIONAL EXAMINATIONS
International General Certificate of Secondary Education

MATHEMATICS 0580/04, 0581/04


Paper 4 (Extended) May/June 2008
2 hours 30 minutes
Additional Materials: Answer Booklet/Paper Electronic calculator
Geometrical instruments Graph paper (1 sheet)
*2879260860*

Mathematical tables (optional) Tracing paper (optional)

READ THESE INSTRUCTIONS FIRST

If you have been given an Answer Booklet, follow the instructions on the front cover of the Booklet.
Write your Centre number, candidate number and name on all the work you hand in.
Write in dark blue or black pen.
You may use a soft pencil for any diagrams or graphs.
Do not use staples, paper clips, highlighters, glue or correction fluid.

Answer all questions.


All working must be clearly shown. It should be done on the same sheet as the rest of the answer.
Marks will be given for working which shows that you know how to solve the problem even if you get the
answer wrong.
Electronic calculators should be used.
If the degree of accuracy is not specified in the question, and if the answer is not exact, give the answer to
three significant figures.
Give answers in degrees to one decimal place.
For π use either your calculator value or 3.142.

At the end of the examination, fasten all your work securely together.
The number of marks is given in brackets [ ] at the end of each question or part question.
The total of the marks for this paper is 130.

This document consists of 13 printed pages and 3 blank pages.

IB08 06_0580_04_RP
© UCLES 2008 [Turn over
2

DO NOT WRITE YOUR ANSWERS ON THIS QUESTION PAPER.

WRITE ALL YOUR WORKING AND ANSWERS ON THE SEPARATE ANSWER BOOK OR PAPER
PROVIDED.

1 Vreni took part in a charity walk.


She walked a distance of 20 kilometres.

(a) She raised money at a rate of $12.50 for each kilometre.

(i) How much money did she raise by walking the 20 kilometres? [1]

(ii) The money she raised in part (a)(i) was 5 of the total money raised.
52
Work out the total money raised. [2]

(iii) In the previous year the total money raised was $2450.
Calculate the percentage increase on the previous year’s total. [2]

(b) Part of the 20 kilometres was on a road and the rest was on a footpath.
The ratio road distance : footpath distance was 3:2.

(i) Work out the road distance. [2]

(ii) Vreni walked along the road at 3 km / h and along the footpath at 2.5 km / h.
How long, in hours and minutes, did Vreni take to walk the 20 kilometres? [2]

(iii) Work out Vreni’s average speed. [1]

(iv) Vreni started at 08 55. At what time did she finish? [1]

(c) On a map, the distance of 20 kilometres was represented by a length of 80 centimetres.


The scale of the map was 1 : n.
Calculate the value of n. [2]

2 (a) (i) Factorise x2 − x − 20. [2]

(ii) Solve the equation x2 − x − 20 = 0. [1]

(b) Solve the equation 3x2 − 2x − 2 = 0.


Show all your working and give your answers correct to 2 decimal places. [4]

(c) y = m2 − 4n2.

(i) Factorise m2 − 4n2. [1]

(ii) Find the value of y when m = 4.4 and n = 2.8. [1]

(iii) m = 2x + 3 and n = x − 1.
Find y in terms of x, in its simplest form. [2]

(iv) Make n the subject of the formula y = m2 − 4n2. [3]

© UCLES 2008 0580/04/M/J/08


3

(d) (i) m4 − 16n4 can be written as (m2 − kn2)(m2 + kn2).


Write down the value of k. [1]

(ii) Factorise completely m4n − 16n5. [2]

3 (a)

Bag A Bag B

Nadia must choose a ball from Bag A or from Bag B.


The probability that she chooses Bag A is 2 .
3
Bag A contains 5 white and 3 black balls.
Bag B contains 6 white and 2 black balls.

The tree diagram below shows some of this information.

5
8 white ball

2
Bag A
3 q black ball

r white ball
p
Bag B
s black ball

(i) Find the values of p, q, r and s. [3]

(ii) Find the probability that Nadia chooses Bag A and then a white ball. [2]

(iii) Find the probability that Nadia chooses a white ball. [2]

(b) Another bag contains 7 green balls and 3 yellow balls.


Sani takes three balls out of the bag, without replacement.

(i) Find the probability that all three balls he chooses are yellow. [2]

(ii) Find the probability that at least one of the three balls he chooses is green. [1]

© UCLES 2008 0580/04/M/J/2008 [Turn over


4

200

180

160

140

120

Cumulative
frequency 100
( h)

80

60

40

20

0
0 10 20 30 40 50 60 70 80
Number of hours worked (h)

200 people record the number of hours they work in a week.


The cumulative frequency graph shows this information.

© UCLES 2008 0580/04/M/J/08


5

(a) Use the graph to find

(i) the median, [1]

(ii) the upper quartile, [1]

(iii) the inter-quartile range, [1]

(iv) the number of people who work more than 60 hours in a week. [2]

(b) Omar uses the graph to make the following frequency table.

Hours
0IhY10 10IhY20 20IhY30 30IhY40 40IhY50 50IhY60 60IhY70 70IhY80
worked (h)

Frequency 12 34 36 30 38 30 p q

(i) Use the graph to find the values of p and q. [2]

(ii) Calculate an estimate of the mean number of hours worked in a week. [4]

(c) Shalini uses the graph to make a different frequency table.

Hours worked (h) 0IhY30 30IhY40 40IhY50 50IhY80

Frequency 82 30 38 50

When she draws a histogram, the height of the column for the interval 30IhY40 is 9 cm.

Calculate the height of each of the other three columns. [4]

© UCLES 2008 0580/04/M/J/2008 [Turn over


6

5
E F Q

NOT TO
SCALE

D G

C H

B P
12 cm A

A circle, centre O, touches all the sides of the regular octagon ABCDEFGH shaded in the diagram.

The sides of the octagon are of length 12 cm.

BA and GH are extended to meet at P. HG and EF are extended to meet at Q.

(a) (i) Show that angle BAH is 135°. [2]

(ii) Show that angle APH is 90°. [1]

(b) Calculate

(i) the length of PH, [2]

(ii) the length of PQ, [2]

(iii) the area of triangle APH, [2]

(iv) the area of the octagon. [3]

(c) Calculate

(i) the radius of the circle, [2]

(ii) the area of the circle as a percentage of the area of the octagon. [3]

© UCLES 2008 0580/04/M/J/08


7

y C
5
B
4

1
D
A x
–4 –3 –2 –1 O 1 2 3 4
–1

The pentagon OABCD is shown on the grid above.

(a) Write as column vectors

(i) , [1]

(ii) BC . [1]

(b) Describe fully the single transformation which maps the side BC onto the side OD. [2]

(c) The shaded area inside the pentagon is defined by 5 inequalities.

1
One of these inequalities is y Y x + 4.
2

Find the other 4 inequalities. [5]

© UCLES 2008 0580/04/M/J/08 [Turn over


8

7 (a)

C
NOT TO
SCALE
40°
D

T

O
130°



A B

A, B, C and D lie on a circle, centre O.


SCT is the tangent at C and is parallel to OB.
Angle AOB = 130°, and angle BCT = 40°.
Angle OBC = x°, angle OBA = y° and angle ADC = z°.

(i) Write down the geometrical word which completes the following statement.

“ABCD is a quadrilateral.” [1]

(ii) Find the values of x, y and z. [3]

(iii) Write down the value of angle OCT. [1]

(iv) Find the value of the reflex angle AOC. [1]

© UCLES 2008 0580/04/M/J/08


9

(b)

Q
7 cm
NOT TO P
SCALE
X

S R
10 cm

P, Q, R and S lie on a circle.

PQ = 7 cm and SR = 10 cm.

PR and QS intersect at X.

The area of triangle SRX = 20 cm2.

(i) Write down the geometrical word which completes the following statement.

“Triangle PQX is to triangle SRX.” [1]

(ii) Calculate the area of triangle PQX. [2]

(iii) Calculate the length of the perpendicular height from X to RS. [2]

© UCLES 2008 0580/04/M/J/08 [Turn over


10

8 Answer the whole of this question on a sheet of graph paper.


Use one side for your working and one side for your graphs.

Alaric invests $100 at 4% per year compound interest.

(a) How many dollars will Alaric have after 2 years? [2]

(b) After x years, Alaric will have y dollars.


He knows a formula to calculate y.
The formula is y = 100 × 1.04x

x (Years) 0 10 20 30 40

y (Dollars) 100 p 219 q 480

Use this formula to calculate the values of p and q in the table. [2]

(c) Using a scale of 2 cm to represent 5 years on the x-axis and 2 cm to represent $50 on the y-axis, draw
an x-axis for 0 Y x Y 40 and a y-axis for 0 Y y Y 500.

Plot the five points in the table and draw a smooth curve through them. [5]

(d) Use your graph to estimate

(i) how many dollars Alaric will have after 25 years, [1]

(ii) how many years, to the nearest year, it takes for Alaric to have $200. [1]

(e) Beatrice invests $100 at 7% per year simple interest.

(i) Show that after 20 years Beatrice has $240. [2]

(ii) How many dollars will Beatrice have after 40 years? [1]

(iii) On the same grid, draw a graph to show how the $100 which Beatrice invests will increase
during the 40 years. [2]

(f) Alaric first has more than Beatrice after n years.


Use your graphs to find the value of n. [1]

© UCLES 2008 0580/04/M/J/08


11

S R Q

NOT TO
SCALE L

O P

OPQR is a parallelogram.
O is the origin.
OP = p and OR = r.
M is the mid-point of PQ and L is on OR such that OL : LR = 2 : 1.
The line PL is extended to the point S.

(a) Find, in terms of p and r, in their simplest forms,

(i) OQ, [1]

(ii) PR , [1]

(iii) PL , [1]

(iv) the position vector of M. [1]

3
(b) PLS is a straight line and PS = PL.
2
Find, in terms of p and / or r, in their simplest forms,

(i) PS , [1]

(ii) QS . [2]

(c) What can you say about the points Q, R and S? [1]

© UCLES 2008 0580/04/M/J/08 [Turn over


12

10
1 2 3 4 5 6

7 8 9 10 11 12

13 14 15 16 17 18

19 20 21 22 23 24

25 26 27 28 29 30

31 32 33 34 35 36

x b c
A 3 by 3 square can be chosen from the 6 by 6 grid above.
d e f

g h i

8 9 10
(a) One of these squares is .
14 15 16

20 21 22

In this square, x = 8, c = 10, g = 20 and i = 22.

For this square, calculate the value of

(i) (i − x) − (g − c), [1]

(ii) cg − xi. [1]

(b)
x b c

d e f

g h i

(i) c = x + 2. Write down g and i in terms of x. [2]

(ii) Use your answers to part(b)(i) to show that (i − x) − (g − c) is constant. [1]

(iii) Use your answers to part(b)(i) to show that cg − xi is constant. [2]

© UCLES 2008 0580/04/M/J/08


13

(c) The 6 by 6 grid is replaced by a 5 by 5 grid as shown.

1 2 3 4 5

6 7 8 9 10

11 12 13 14 15

16 17 18 19 20

21 22 23 24 25

x b c
A 3 by 3 square can be chosen from the 5 by 5 grid.
d e f

g h i

For any 3 by 3 square chosen from this 5 by 5 grid, calculate the value of

(i) (i − x) − (g − c), [1]

(ii) cg − xi. [1]

(d) A 3 by 3 square is chosen from an n by n grid.

(i) Write down the value of (i − x) − (g − c). [1]

(ii) Find g and i in terms of x and n. [2]

(iii) Find cg − xi in its simplest form. [1]

© UCLES 2008 0580/04/M/J/08


14

BLANK PAGE

0580/04/M/J/08
15

BLANK PAGE

0580/04/M/J/08
16

BLANK PAGE

Permission to reproduce items where third-party owned material protected by copyright is included has been sought and cleared where possible. Every
reasonable effort has been made by the publisher (UCLES) to trace copyright holders, but if any items requiring clearance have unwittingly been included, the
publisher will be pleased to make amends at the earliest possible opportunity.

University of Cambridge International Examinations is part of the Cambridge Assessment Group. Cambridge Assessment is the brand name of University of
Cambridge Local Examinations Syndicate (UCLES), which is itself a department of the University of Cambridge.

0580/04/M/J/08
UNIVERSITY OF CAMBRIDGE INTERNATIONAL EXAMINATIONS
International General Certificate of Secondary Education
*8086281837*

MATHEMATICS 0580/04, 0581/04


Paper 4 (Extended) May/June 2009
2 hours 30 minutes
Candidates answer on the question paper.
Additional Materials: Electronic calculator Geometrical instruments
Mathematical tables (optional) Tracing paper (optional)

READ THESE INSTRUCTIONS FIRST

Write your Centre number, candidate number and name on all the work you hand in.
Write in dark blue or black pen.
You may use a soft pencil for any diagrams or graphs.
Do not use staples, paper clips, highlighters, glue or correction fluid.
DO NOT WRITE IN ANY BARCODES.

Answer all questions.


If working is needed for any question it must be shown below that question.
Electronic calculators should be used.
If the degree of accuracy is not specified in the question, and if the answer is not exact, give the answer to
three significant figures.
Give answers in degrees to one decimal place.
For π use either your calculator value or 3.142.

At the end of the examination, fasten all your work securely together.
The number of marks is given in brackets [ ] at the end of each question or part question.
The total of the marks for this paper is 130.

For Examiner's Use

This document consists of 19 printed pages and 1 blank page.

IB09 06_0580_04/5RP
© UCLES 2009 [Turn over
2

1 Marcus receives $800 from his grandmother. For


Examiner's
Use
(a) He decides to spend $150 and to divide the remaining $650 in the ratio
savings : holiday = 9 : 4.
Calculate the amount of his savings.

Answer(a) $ [2]

(b) (i) He uses 80% of the $150 to buy some clothes.

Calculate the cost of the clothes.

Answer(b)(i) $ [2]

(ii) The money remaining from the $150 is 37 12 % of the cost of a day trip to Cairo.

Calculate the cost of the trip.

Answer(b)(ii) $ [2]

(c) (i) Marcus invests $400 of his savings for 2 years at 5 % per year compound interest.

Calculate the amount he has at the end of the 2 years.

Answer(c)(i) $ [2]

(ii) Marcus’s sister also invests $400, at r % per year simple interest.
At the end of 2 years she has exactly the same amount as Marcus.

Calculate the value of r.

Answer(c)(ii) r = [3]

© UCLES 2009 0580/04/M/J/09


3

2 A normal die, numbered 1 to 6, is rolled 50 times. For


Examiner's
Use

1
2 4
The results are shown in the frequency table.

Score 1 2 3 4 5 6

Frequency 15 10 7 5 6 7

(a) Write down the modal score.

Answer(a) [1]

(b) Find the median score.

Answer(b) [1]

(c) Calculate the mean score.

Answer(c) [2]

(d) The die is then rolled another 10 times.


The mean score for the 60 rolls is 2.95.
Calculate the mean score for the extra 10 rolls.

Answer(d) [3]

© UCLES 2009 0580/04/M/J/09 [Turn over


4

3 For
Examiner's
P Use

NOT TO
SCALE

10 cm
14 cm

In triangle PQR, angle QPR is acute, PQ = 10 cm and PR = 14 cm.

(a) The area of triangle PQR is 48 cm2.

Calculate angle QPR and show that it rounds to 43.3°, correct to 1 decimal place.
You must show all your working.

Answer (a)

[3]

(b) Calculate the length of the side QR.

Answer(b) QR = cm [4]

© UCLES 2009 0580/04/M/J/09


5

4 For
Examiner's
Use
North
NOT TO
North SCALE
A

126°
B
North

250 m
23°
P

The diagram shows three straight horizontal roads in a town, connecting points P, A and B.

PB =250 m, angle APB = 23° and angle BAP = 126°.

(a) Calculate the length of the road AB.

Answer(a) AB = m [3]

(b) The bearing of A from P is 303°.

Find the bearing of

(i) B from P,

Answer(b)(i) [1]

(ii) A from B.

Answer(b)(ii) [2]

© UCLES 2009 0580/04/M/J/09 [Turn over


6

x_2 For
5 (a) The table shows some values for the equation y = for – 4 Y x Y=–0.5 and 0.5 Y x Y 4. Examiner's
2 x Use

x –4 –3 –2 –1.5 –1 –0.5 0.5 1 1.5 2 3 4

y –1.5 –0.83 0 0.58 –3.75 –0.58 0 0.83 1.5

(i) Write the missing values of y in the empty spaces. [3]

x_2
(ii) On the grid, draw the graph of y = for – 4 Y x Y=–0.5 and 0.5 Y x Y 4.
2 x

y
4

x
–4 –3 –2 –1 0 1 2 3 4

–1

–2

–3

–4

[5]

© UCLES 2009 0580/04/M/J/09


7

x_2 For
(b) Use your graph to solve the equation = 1. Examiner's
2 x
Use

Answer(b) x = or x = [2]

(c) (i) By drawing a tangent, work out the gradient of the graph where x = 2.

Answer(c)(i) [3]

(ii) Write down the gradient of the graph where x = –2.

Answer(c)(ii) [1]

(d) (i) On the grid, draw the line y = – x for – 4 Y x Y4. [1]

x_2 _
(ii) Use your graphs to solve the equation = x.
2 x

Answer(d)(ii) x = or x = [2]

(e) Write down the equation of a straight line which passes through the origin and does not
x 2
intersect the graph of y = _ .
2 x

Answer(e) [2]

© UCLES 2009 0580/04/M/J/09 [Turn over


8

6 (a) For
Examiner's
B Use

NOT TO
SCALE

(x + 1) cm

A (x + 6) cm D (x + 2) cm C

In triangle ABC, the line BD is perpendicular to AC.

AD = (x + 6) cm, DC = (x + 2) cm and the height BD = (x + 1) cm.

The area of triangle ABC is 40 cm2.

(i) Show that x2 + 5x – 36 = 0.

Answer (a)(i)

[3]

(ii) Solve the equation x2 + 5x – 36 = 0.

Answer(a)(ii) x = or x = [2]

(iii) Calculate the length of BC.

Answer(a)(iii) BC = cm [2]

© UCLES 2009 0580/04/M/J/09


9

(b) Amira takes 9 hours 25 minutes to complete a long walk. For


113 Examiner's
(i) Show that the time of 9 hours 25 minutes can be written as hours. Use
12

Answer (b)(i)

[1]

(ii) She walks (3y + 2) kilometres at 3 km/h and then a further (y + 4) kilometres at 2 km/h.

9 y + 16
Show that the total time taken is hours.
6
Answer(b)(ii)

[2]

9 y + 16 113
(iii) Solve the equation = .
6 12

Answer(b)(iii) y = [2]

(iv) Calculate Amira’s average speed, in kilometres per hour, for the whole walk.

Answer(b)(iv) km/h [3]

© UCLES 2009 0580/04/M/J/09 [Turn over


10

7 For
Examiner's
NOT TO Use

SCALE
x cm

250 cm x cm

A solid metal bar is in the shape of a cuboid of length of 250 cm.


The cross-section is a square of side x cm.
The volume of the cuboid is 4840 cm3.

(a) Show that x = 4.4.

Answer (a)

[2]

(b) The mass of 1 cm3 of the metal is 8.8 grams.


Calculate the mass of the whole metal bar in kilograms.

Answer(b) kg [2]

(c) A box, in the shape of a cuboid measures 250 cm by 88 cm by h cm.


120 of the metal bars fit exactly in the box.
Calculate the value of h.

Answer(c) h = [2]

© UCLES 2009 0580/04/M/J/09


11

(d) One metal bar, of volume 4840 cm3, is melted down to make 4200 identical small spheres. For
Examiner's
Use
All the metal is used.

(i) Calculate the radius of each sphere. Show that your answer rounds to 0.65 cm, correct to
2 decimal places.
[The volume, V, of a sphere, radius r, is given by V = 4 πr 3 .]
3
Answer(d)(i)

[4]

(ii) Calculate the surface area of each sphere, using 0.65 cm for the radius.
[The surface area, A, of a sphere, radius r, is given by A = 4πr 2 .]

Answer(d)(ii) cm2 [1]

(iii) Calculate the total surface area of all 4200 spheres as a percentage of the surface area of the
metal bar.

Answer(d)(iii) % [4]

© UCLES 2009 0580/04/M/J/09 [Turn over


12

8 For
Examiner's
First Second Third Use

Calculator Calculator Calculator

p F F
F
p q NF NF

F F
q
NF
NF NF
F = faulty
NF = not faulty

The tree diagram shows a testing procedure on calculators, taken from a large batch.

1
Each time a calculator is chosen at random, the probability that it is faulty (F) is .
20

(a) Write down the values of p and q.

Answer(a) p = and q = [1]

(b) Two calculators are chosen at random.

Calculate the probability that

(i) both are faulty,

Answer(b)(i) [2]

(ii) exactly one is faulty.

Answer(b)(ii) [2]

© UCLES 2009 0580/04/M/J/09


13

(c) If exactly one out of two calculators tested is faulty, then a third calculator is chosen at random. For
Examiner's
Use
Calculate the probability that exactly one of the first two calculators is faulty and the third one
is faulty.

Answer(c) [2]

(d) The whole batch of calculators is rejected


either if the first two chosen are both faulty
or if a third one needs to be chosen and it is faulty.

Calculate the probability that the whole batch is rejected.

Answer(d) [2]

(e) In one month, 1000 batches of calculators are tested in this way.

How many batches are expected to be rejected?

Answer(e) [1]

© UCLES 2009 0580/04/M/J/09 [Turn over


14

9 The heights of 100 students are measured. For


Examiner's
Use
The results have been used to draw this cumulative frequency diagram.

Cumulative
frequency

100

90

80

70

60

50

40

30

20

10

0
0 150 155 160 165 170 175 180 185 190
Height (cm)

© UCLES 2009 0580/04/M/J/09


15

(a) Find For


Examiner's
Use
(i) the median height,

Answer(a)(i) cm [1]

(ii) the lower quartile,

Answer(a)(ii) cm [1]

(iii) the inter-quartile range,

Answer(a)(iii) cm [1]

(iv) the number of students with a height greater than 177 cm.

Answer(a)(iv) [2]

(b) The frequency table shows the information about the 100 students who were measured.

Height (h cm) 150 < h Y=160 160 < h Y=170 170 < h Y=180 180 < h Y=190

Frequency 47 18

(i) Use the cumulative frequency diagram to complete the table above. [1]

(ii) Calculate an estimate of the mean height of the 100 students.

Answer(b)(ii) cm [4]

© UCLES 2009 0580/04/M/J/09 [Turn over


16

10 f(x) = 2x – 1 g(x) = x2 + 1 h(x) = 2x For


Examiner's
Use

(a) Find the value of

( )
(i) f _ 1 ,
2

Answer(a)(i) [1]
_
(ii) g ( 5 ) ,

Answer(a)(ii) [1]

(iii) h (_3) .

Answer(a)(iii) [1]

(b) Find the inverse function f –1(x).

Answer(b) f –1(x) = [2]

(c) g(x) = z.
Find x in terms of z.

Answer(c) x = [2]

(d) Find gf(x), in its simplest form.

Answer(d) gf(x) = [2]

© UCLES 2009 0580/04/M/J/09


17

(e) h(x) = 512. For


Find the value of x. Examiner's
Use

Answer(e) x = [1]

(f) Solve the equation 2f(x) + g(x) = 0, giving your answers correct to 2 decimal places.

Answer(f) x = or x = [5]

(g) Sketch the graph of

(i) y = f(x),

(ii) y = g(x).

y y

x x
O O

(i) y = f(x) (ii) y = g(x)


[3]

© UCLES 2009 0580/04/M/J/09 [Turn over


18

11 For
Examiner's
Use

Diagram 1 Diagram 2 Diagram 3 Diagram 4

The first four terms in a sequence are 1, 3, 6 and 10.


They are shown by the number of dots in the four diagrams above.

(a) Write down the next four terms in the sequence.

Answer(a) , , , [2]

(b) (i) The sum of the two consecutive terms 3 and 6 is 9.


The sum of the two consecutive terms 6 and 10 is 16.

Complete the following statements using different pairs of terms.

The sum of the two consecutive terms and is .

The sum of the two consecutive terms and is . [1]

(ii) What special name is given to these sums?

Answer(b)(ii) [1]

n( n +1)
(c) (i) The formula for the nth term in the sequence 1, 3, 6, 10… is ,
k
where k is an integer.

Find the value of k.

Answer(c)(i) k = [1]

© UCLES 2009 0580/04/M/J/09


19

(ii) Test your formula when n = 4, showing your working. For


Examiner's
Use
Answer (c)(ii)

[1]

(iii) Find the value of the 180th term in the sequence.

Answer(c)(iii) [1]

(d) (i) Show clearly that the sum of the nth and the (n + 1)th terms is (n + 1)2.

Answer (d)(i)

[3]

(ii) Find the values of the two consecutive terms which have a sum of 3481.

Answer(d)(ii) and [2]

© UCLES 2009 0580/04/M/J/09


20

BLANK PAGE

Permission to reproduce items where third-party owned material protected by copyright is included has been sought and cleared where possible. Every
reasonable effort has been made by the publisher (UCLES) to trace copyright holders, but if any items requiring clearance have unwittingly been included, the
publisher will be pleased to make amends at the earliest possible opportunity.

University of Cambridge International Examinations is part of the Cambridge Assessment Group. Cambridge Assessment is the brand name of University of
Cambridge Local Examinations Syndicate (UCLES), which is itself a department of the University of Cambridge.

0580/04/M/J/09
w
w
w
.X
tr
me
eP
ap
UNIVERSITY OF CAMBRIDGE INTERNATIONAL EXAMINATIONS

er
s
International General Certificate of Secondary Education

.c
om
*5070593749*

MATHEMATICS 0580/41
Paper 4 (Extended) May/June 2010
2 hours 30 minutes
Candidates answer on the Question Paper.
Additional Materials: Electronic calculator Geometrical instruments
Mathematical tables (optional) Tracing paper (optional)

READ THESE INSTRUCTIONS FIRST

Write your Centre number, candidate number and name on all the work you hand in.
Write in dark blue or black pen.
You may use a pencil for any diagrams or graphs.
Do not use staples, paper clips, highlighters, glue or correction fluid.
DO NOT WRITE IN ANY BARCODES.

Answer all questions.


If working is needed for any question it must be shown below that question.
Electronic calculators should be used.
If the degree of accuracy is not specified in the question, and if the answer is not exact, give the answer to
three significant figures. Give answers in degrees to one decimal place.
For π use either your calculator value or 3.142.

At the end of the examination, fasten all your work securely together.
The number of marks is given in brackets [ ] at the end of each question or part question.
The total of the marks for this paper is 130.

This document consists of 19 printed pages and 1 blank page.

IB10 06_0580_41/2RP
© UCLES 2010 [Turn over
2

For
1 A school has 220 boys and 280 girls. Examiner's
Use

(a) Find the ratio of boys to girls, in its simplest form.

Answer(a) : [1]

(b) The ratio of students to teachers is 10 : 1.


Find the number of teachers.

Answer(b) [2]

(c) There are 21 students on the school’s committee.


The ratio of boys to girls is 3 : 4.
Find the number of girls on the committee.

Answer(c) [2]

(d) The committee organises a disco and sells tickets.


35% of the school’s students each buy a ticket. Each ticket costs $1.60.
Calculate the total amount received from selling the tickets.

Answer(d) $ [3]

(e) The cost of running the disco is $264.


This is an increase of 10% on the cost of running last year’s disco.
Calculate the cost of running last year’s disco.

Answer(e) $ [2]

© UCLES 2010 0580/41/M/J/10


3

For
2 40 students are asked about the number of people in their families. Examiner's
Use

The table shows the results.

Number of people in family 2 3 4 5 6 7

Frequency 1 1 17 12 6 3

(a) Find

(i) the mode,

Answer(a)(i) [1]

(ii) the median,

Answer(a)(ii) [1]

(iii) the mean.

Answer(a)(iii) [3]

(b) Another n students are asked about the number of people in their families.

The mean for these n students is 3.

Find, in terms of n, an expression for the mean number for all (40 + n) students.

Answer(b) [2]

© UCLES 2010 0580/41/M/J/10 [Turn over


4

For
3 Examiner's
y Use

7
T
6

x
–8 –7 –6 –5 –4 –3 –2 –1 0 1 2 3 4 5 6 7 8
–1

–2

–3
P
–4

–5

–6
Q
–7

–8

1
(a) On the grid, draw the enlargement of the triangle T, centre (0, 0), scale factor 2 . [2]

© UCLES 2010 0580/41/M/J/10


5

For
 −1 0 Examiner's
(b) The matrix   represents a transformation. Use
 0 1 

 − 1 0  8 8 2 
(i) Calculate the matrix product    .
 0 1  4 8 8 

Answer(b)(i) [2]

(ii) On the grid, draw the image of the triangle T under this transformation. [2]

(iii) Describe fully this single transformation.

Answer(b)(iii) [2]

(c) Describe fully the single transformation which maps

(i) triangle T onto triangle P,

Answer(c)(i) [2]

(ii) triangle T onto triangle Q.

Answer(c)(ii) [3]

(d) Find the 2 by 2 matrix which represents the transformation in part (c)(ii).

 
Answer(d)   [2]
 
 

© UCLES 2010 0580/41/M/J/10 [Turn over


6

For
4 Examiner's
Use

A B

Box A contains 3 black balls and 1 white ball.


Box B contains 3 black balls and 2 white balls.

(a) A ball can be chosen at random from either box.


Complete the following statement.

There is a greater probability of choosing a white ball from Box .

Explain your answer.

Answer(a) [1]

(b) Abdul chooses a box and then chooses a ball from this box at random.

2
The probability that he chooses box A is 3 .

(i) Complete the tree diagram by writing the four probabilities in the empty spaces.

BOX COLOUR
1
white
4

2 A
3
black

white

black

[4]

© UCLES 2010 0580/41/M/J/10


7

For
(ii) Find the probability that Abdul chooses box A and a black ball. Examiner's
Use

Answer(b)(ii) [2]

(iii) Find the probability that Abdul chooses a black ball.

Answer(b)(iii) [2]

(c) Tatiana chooses a box and then chooses two balls from this box at
random (without replacement).

2
The probability that she chooses box A is 3 .

Find the probability that Tatiana chooses two white balls.

Answer(c) [2]

© UCLES 2010 0580/41/M/J/10 [Turn over


8

For
5 Examiner's
D Use

30° C

NOT TO
SCALE
24 cm

40°
40°
A 26 cm B

ABCD is a quadrilateral and BD is a diagonal.


AB = 26 cm, BD = 24 cm, angle ABD = 40°, angle CBD = 40° and angle CDB = 30°.

(a) Calculate the area of triangle ABD.

Answer(a) cm2 [2]

(b) Calculate the length of AD.

Answer(b) cm [4]

(c) Calculate the length of BC.

Answer(c) cm [4]

(d) Calculate the shortest distance from the point C to the line BD.

Answer(d) cm [2]

© UCLES 2010 0580/41/M/J/10


9

For
6 The masses of 60 potatoes are measured. Examiner's
Use
The table shows the results.

Mass (m grams) 10 I m Y 20 20 I m Y 40 40 I m Y 50

Frequency 10 30 20

(a) Calculate an estimate of the mean.

Answer(a) g [4]

(b) On the grid, draw an accurate histogram to show the information in the table.

Frequency
density

m
0 10 20 30 40 50
Mass (grams)
[3]

© UCLES 2010 0580/41/M/J/10 [Turn over


10

For
7 (a) Calculate the volume of a cylinder of radius 31 centimetres and length 15 metres. Examiner's
Use
Give your answer in cubic metres.

Answer(a) m3 [3]

(b) A tree trunk has a circular cross-section of radius 31 cm and length 15 m.


One cubic metre of the wood has a mass of 800 kg.
Calculate the mass of the tree trunk, giving your answer in tonnes.

Answer(b) tonnes [2]

(c)

NOT TO
SCALE plastic
sheet

D
C

The diagram shows a pile of 10 tree trunks.


Each tree trunk has a circular cross-section of radius 31 cm and length 15 m.
A plastic sheet is wrapped around the pile.

C is the centre of one of the circles.


CE and CD are perpendicular to the straight edges, as shown.

© UCLES 2010 0580/41/M/J/10


11

For
(i) Show that angle ECD = 120°. Examiner's
Use

Answer(c)(i)

[2]

(ii) Calculate the length of the arc DE, giving your answer in metres.

Answer(c)(ii) m [2]

(iii) The edge of the plastic sheet forms the perimeter of the cross-section of the pile.
The perimeter consists of three straight lines and three arcs.
Calculate this perimeter, giving your answer in metres.

Answer(c)(iii) m [3]

(iv) The plastic sheet does not cover the two ends of the pile.
Calculate the area of the plastic sheet.

Answer(c)(iv) m2 [1]

© UCLES 2010 0580/41/M/J/10 [Turn over


12

For
8 (a) f(x) = 2x Examiner's
Use

Complete the table.

x –2 –1 0 1 2 3 4

y = f(x) 0.5 1 2 4
[3]

(b) g(x) = x(4 – x)

Complete the table.

x –1 0 1 2 3 4

y = g(x) 0 3 3 0
[2]

© UCLES 2010 0580/41/M/J/10


13

For
(c) On the grid, draw the graphs of Examiner's
Use

(i) y = f(x) for −2 Y x Y 4, [3]

(ii) y = g(x) for −1 Y x Y 4. [3]

y
16

14

12

10

x
–2 –1 0 1 2 3 4
–2

–4

–6

(d) Use your graphs to solve the following equations.

(i) f(x) = 10

Answer(d)(i) x = [1]

(ii) f(x) = g(x)

Answer(d)(ii) x = or x = [2]

(iii) f -1(x) = 1.7

Answer(d)(iii) x = [1]

© UCLES 2010 0580/41/M/J/10 [Turn over


14

For
9 (a) Solve the following equations. Examiner's
Use

5 3
(i) =
w w +1

Answer(a)(i) w = [2]

(ii) ( y + 1) 2 = 4

Answer(a)(ii) y = or y = [2]

x +1 x −2
(iii) − = 2
3 5

Answer(a)(iii) x = [3]

(b) (i) Factorise u2 −9u −10.

Answer(b)(i) [2]

(ii) Solve the equation u2 −9u −10 = 0.

Answer(b)(ii) u = or u = [1]

© UCLES 2010 0580/41/M/J/10


15

For
(c) Examiner's
Use

NOT TO
SCALE

x+1 x

x
x+2

The area of the triangle is equal to the area of the square.


All lengths are in centimetres.

(i) Show that x2 −3x −2 = 0.

Answer(c)(i)

[3]

(ii) Solve the equation x2 – 3x – 2 = 0, giving your answers correct to 2 decimal places.
Show all your working.

Answer(c)(ii) x = or x = [4]

(iii) Calculate the area of one of the shapes.

Answer(c)(iii) cm2 [1]

© UCLES 2010 0580/41/M/J/10 [Turn over


16

For
10 A company has a vehicle parking area of 1200 m2 with space for x cars and y trucks. Examiner's
Use

Each car requires 20 m2 of space and each truck requires 100 m2 of space.

(a) Show that x + 5y Y 60.

Answer(a)

[1]

(b) There must also be space for

(i) at least 40 vehicles,

(ii) at least 2 trucks.

Write down two more inequalities to show this information.

Answer(b)(i) [1]

Answer(b)(ii) [1]

(c) One line has been drawn for you.


On the grid, show the three inequalities by drawing the other two lines and shading the
unwanted regions.

y
40

30

20

10

x
0 10 20 30 40 50 60
[4]

© UCLES 2010 0580/41/M/J/10


17

For
(d) Use your graph to find the largest possible number of trucks. Examiner's
Use

Answer(d) [1]

(e) The company charges $5 for parking each car and $10 for parking each truck.
Find the number of cars and the number of trucks which give the company the greatest possible
income.

Calculate this income.

Answer(e) Number of cars =

Number of trucks =

Greatest possible income = $ [3]

© UCLES 2010 0580/41/M/J/10 [Turn over


18

For
11 Examiner's
Use

Diagram 1 Diagram 2 Diagram 3 Diagram 4


1 white dot 4 white dots 9 white dots 16 white dots
5 black dots 7 black dots 9 black dots 11 black dots
6 lines 14 lines 26 lines 42 lines

The four diagrams above are the first four of a pattern.

(a) Diagram 5 has been started below.


Complete this diagram and write down the information about the numbers of dots and lines.

Diagram 5

............... white dots

............... black dots

............... lines
[4]

© UCLES 2010 0580/41/M/J/10


19

For
(b) Complete the information about the number of dots and lines in Diagram 8. Examiner's
Use

Answer(b) white dots

black dots

lines [3]

(c) Complete the information about the number of dots in Diagram n.


Give your answers in terms of n.

Answer(c) white dots

black dots [2]

(d) The number of lines in diagram n is k(n2 + n +1).

Find

(i) the value of k,

Answer(d)(i) k = [1]

(ii) the number of lines in Diagram 100.

Answer(d)(ii) [1]

© UCLES 2010 0580/41/M/J/10


20

BLANK PAGE

Permission to reproduce items where third-party owned material protected by copyright is included has been sought and cleared where possible. Every
reasonable effort has been made by the publisher (UCLES) to trace copyright holders, but if any items requiring clearance have unwittingly been included, the
publisher will be pleased to make amends at the earliest possible opportunity.

University of Cambridge International Examinations is part of the Cambridge Assessment Group. Cambridge Assessment is the brand name of University of
Cambridge Local Examinations Syndicate (UCLES), which is itself a department of the University of Cambridge.

0580/41/M/J/10
w
w
w
.X
tr
me
eP
ap
UNIVERSITY OF CAMBRIDGE INTERNATIONAL EXAMINATIONS

er
s
International General Certificate of Secondary Education

.c
om
*7685405294*

MATHEMATICS 0580/42
Paper 4 (Extended) May/June 2010
2 hours 30 minutes
Candidates answer on the Question Paper.
Additional Materials: Electronic calculator Geometrical instruments
Mathematical tables (optional) Tracing paper (optional)

READ THESE INSTRUCTIONS FIRST

Write your Centre number, candidate number and name on all the work you hand in.
Write in dark blue or black pen.
You may use a pencil for any diagrams or graphs.
Do not use staples, paper clips, highlighters, glue or correction fluid.
DO NOT WRITE IN ANY BARCODES.

Answer all questions.


If working is needed for any question it must be clearly shown below that question.
Electronic calculators should be used.
If the degree of accuracy is not specified in the question, and if the answer is not exact, give the answer to
three significant figures. Give answers in degrees to one decimal place.
For π use either your calculator value or 3.142.

At the end of the examination, fasten all your work securely together.
The number of marks is given in brackets [ ] at the end of each question or part question.
The total of the marks for this paper is 130.

This document consists of 19 printed pages and 1 blank page.

IB10 06_0580_42/2RP
© UCLES 2010 [Turn over
Name: ____________________ 2 Date: _____________________

Class: _____________________
For
1 Alberto and Maria share $240 in the ratio 3 : 5. Examiner's
Use

(a) Show that Alberto receives $90 and Maria receives $150.

Answer(a)

[1]

(b) (i) Alberto invests his $90 for 2 years at r % per year simple interest.
At the end of 2 years the amount of money he has is $99.
Calculate the value of r.

Answer(b)(i) r = [2]

(ii) The $99 is 60% of the cost of a holiday.


Calculate the cost of the holiday.

Answer(b)(ii) $ [2]

(c) Maria invests her $150 for 2 years at 4% per year compound interest.
Calculate the exact amount Maria has at the end of 2 years.

Answer(c) $ [2]

(d) Maria continues to invest her money at 4% per year compound interest.
After 20 years she has $328.67.

(i) Calculate exactly how much more this is than $150 invested for 20 years at 4% per year
simple interest.

Answer(d)(i) $ [3]

(ii) Calculate $328.67 as a percentage of $150.

Answer(d)(ii) % [2]

© UCLES 2010 0580/42/M/J/10


3

For

2
3 6
(a) p =   and q =   .
Examiner's
Use
 2 3  

(i) Find, as a single column vector, p + 2q.


 
 
 
 
Answer(a)(i)   [2]

(ii) Calculate the value of | p + 2q |.

Answer(a)(ii) [2]

(b) C

NOT TO
SCALE
M

O L V

In the diagram, CM = MV and OL = 2LV.


O is the origin. = c and = v.

Find, in terms of c and v, in their simplest forms

(i) ,

Answer(b)(i) [2]

(ii) the position vector of M,

Answer(b)(ii) [2]

(iii) .

Answer(b)(iii) [2]

© UCLES 2010 0580/42/M/J/10 [Turn over


4

For
3 Examiner's
Use

1
1
10

2
1

The diagram shows a spinner with six numbered sections.


Some of the sections are shaded.
Each time the spinner is spun it stops on one of the six sections.
It is equally likely that it stops on any one of the sections.

(a) The spinner is spun once.

Find the probability that it stops on

(i) a shaded section,

Answer(a)(i) [1]

(ii) a section numbered 1,

Answer(a)(ii) [1]

(iii) a shaded section numbered 1,

Answer(a)(iii) [1]

(iv) a shaded section or a section numbered 1.

Answer(a)(iv) [1]

© UCLES 2010 0580/42/M/J/10


5

For
(b) The spinner is now spun twice. Examiner's
Use

Find the probability that the total of the two numbers is

(i) 20,

Answer(b)(i) [2]

(ii) 11.

Answer(b)(ii) [2]

(c) (i) The spinner stops on a shaded section.

Find the probability that this section is numbered 2.

Answer(c)(i) [1]

(ii) The spinner stops on a section numbered 2.

Find the probability that this section is shaded.

Answer(c)(ii) [1]

(d) The spinner is now spun until it stops on a section numbered 2.


16
The probability that this happens on the nth spin is .
243

Find the value of n.

Answer(d) n = [2]

© UCLES 2010 0580/42/M/J/10 [Turn over


6

For
4 Examiner's
y Use

5
V
4

2
T
1

x
–4 –3 –2 –1 0 1 2 3 4 5 6 7 8 9
–1

–2

–3

–4

–5
U
–6

(a) On the grid, draw

−7
(i) the translation of triangle T by the vector   , [2]
 3

(ii) the rotation of triangle T about (0, 0), through 90° clockwise. [2]

(b) Describe fully the single transformation that maps

(i) triangle T onto triangle U,

Answer(b)(i) [2]

(ii) triangle T onto triangle V.

Answer(b)(ii) [3]

© UCLES 2010 0580/42/M/J/10


7

For
(c) Find the 2 by 2 matrix which represents the transformation that maps Examiner's
Use

(i) triangle T onto triangle U,

 
 
 
 
 
Answer(c)(i)   [2]

(ii) triangle T onto triangle V,

 
 
 
 
 
Answer(c)(ii)   [2]

(iii) triangle V onto triangle T.

 
 
 
 
 
Answer(c)(iii)   [1]

© UCLES 2010 0580/42/M/J/10 [Turn over


8

For
5 Examiner's
North Use

A NOT TO
SCALE

180 km

115 km

90 km T
H 30°

70°
R

The diagram shows some straight line distances between Auckland (A), Hamilton (H), Tauranga (T)
and Rotorua (R).
AT = 180 km, AH = 115 km and HT = 90 km.

(a) Calculate angle HAT.


Show that this rounds to 25.0°, correct to 3 significant figures.

Answer(a)

[4]

(b) The bearing of H from A is 150°.

Find the bearing of

(i) T from A,

Answer(b)(i) [1]

(ii) A from T.

Answer(b)(ii) [1]

© UCLES 2010 0580/42/M/J/10


9

For
(c) Calculate how far T is east of A. Examiner's
Use

Answer(c) km [3]

(d) Angle THR = 30° and angle HRT = 70°.

Calculate the distance TR.

Answer(d) km [3]

(e) On a map the distance representing HT is 4.5cm.

The scale of the map is 1 : n.

Calculate the value of n.

Answer(e) n = [2]

© UCLES 2010 0580/42/M/J/10 [Turn over


10

For
6 A spherical ball has a radius of 2.4 cm. Examiner's
Use

(a) Show that the volume of the ball is 57.9 cm3, correct to 3 significant figures.

4 3
[The volume V of a sphere of radius r is V = πr . ]
3

Answer(a)

[2]

(b)

NOT TO
SCALE

Six spherical balls of radius 2.4 cm fit exactly into a closed box.
The box is a cuboid.

Find

(i) the length, width and height of the box,

Answer(b)(i) cm, cm, cm [3]

(ii) the volume of the box,

Answer(b)(ii) cm3 [1]

(iii) the volume of the box not occupied by the balls,

Answer(b)(iii) cm3 [1]

(iv) the surface area of the box.

Answer(b)(iv) cm2 [2]

© UCLES 2010 0580/42/M/J/10


11

For
(c) Examiner's
Use

NOT TO
SCALE

The six balls can also fit exactly into a closed cylindrical container, as shown in the diagram.

Find

(i) the volume of the cylindrical container,

Answer(c)(i) cm3 [3]

(ii) the volume of the cylindrical container not occupied by the balls,

Answer(c)(ii) cm3 [1]

(iii) the surface area of the cylindrical container.

Answer(c)(iii) cm2 [3]

© UCLES 2010 0580/42/M/J/10 [Turn over


12

For
7 200 students were asked how many hours they exercise each week. Examiner's
Use

The table shows the results.

Time (t hours) 0ItY5 5ItY10 10ItY15 15ItY20 20ItY25 25ItY30 30ItY35 35ItY40

Number of
12 15 23 30 40 35 25 20
students

(a) Calculate an estimate of the mean.

Answer(a) h [4]

(b) Use the information in the table above to complete the cumulative frequency table.

Time (t hours) t Y=5 t Y=10 t Y=15 t Y=20 t Y=25 t Y=30 t Y=35 t Y=40

Cumulative frequency 12 27 50 80 120 200

[1]

© UCLES 2010 0580/42/M/J/10


13

For
Examiner's
Use
200

180

160

140
Cumulative frequency

120

100

80

60

40

20

t
0
5 10 15 20 25 30 35 40
Time (t hours)

(c) On the grid, draw a cumulative frequency diagram to show the information in the table in
part (b). [4]

(d) On your cumulative frequency diagram show how to find the lower quartile. [1]

(e) Use your cumulative frequency diagram to find

(i) the median,

Answer(e)(i) [1]

(ii) the inter-quartile range,

Answer(e)(ii) [1]

(iii) the 64th percentile,

Answer(e)(iii) [1]

(iv) the number of students who exercise for more than 17 hours.

Answer(e)(iv) [2]

© UCLES 2010 0580/42/M/J/10 [Turn over


14

For
8 (a) y is 5 less than the square of the sum of p and q. Examiner's
Use

Write down a formula for y in terms of p and q.

Answer(a) y = [2]

(b) The cost of a magazine is $x and the cost of a newspaper is $(x – 3).

The total cost of 6 magazines and 9 newspapers is $51.

Write down and solve an equation in x to find the cost of a magazine.

Answer(b) $ [4]

© UCLES 2010 0580/42/M/J/10


15

For
(c) Bus tickets cost $3 for an adult and $2 for a child. Examiner's
Use

There are a adults and c children on a bus.

The total number of people on the bus is 52.

The total cost of the 52 tickets is $139.

Find the number of adults and the number of children on the bus.

Answer(c) Number of adults =

Number of children = [5]

© UCLES 2010 0580/42/M/J/10 [Turn over


16

For
9 (a) Examiner's
9 cm Use
A B
x° y°

NOT TO
SCALE
X

3 cm

C 6 cm D E

The lines AB and CDE are parallel.


AD and CB intersect at X.
AB = 9 cm, CD = 6 cm and DX = 3 cm.

(i) Complete the following statement.

Triangle ABX is to triangle DCX. [1]

(ii) Calculate the length of AX.

Answer(a)(ii) AX = cm [2]

(iii) The area of triangle DCX is 6 cm2.

Calculate the area of triangle ABX.

Answer(a)(iii) cm2 [2]

(iv) Angle BAX = x° and angle ABX = y°.

Find angle AXB and angle XDE in terms of x and/or y.

Answer(a)(iv) Angle AXB =

Angle XDE = [2]

© UCLES 2010 0580/42/M/J/10


17

For
(b) R Examiner's
Use

35°

NOT TO
SCALE
Q

42°
P S

P, Q, R and S lie on a circle, centre O.


Angle OPS = 42° and angle PRQ = 35°.

Calculate

(i) angle POS,


Answer(b)(i) Angle POS = [1]

(ii) angle PRS,


Answer(b)(ii) Angle PRS = [1]

(iii) angle SPQ,


Answer(b)(iii) Angle SPQ = [1]

(iv) angle PSQ.


Answer(b)(iv) Angle PSQ = [1]

(c) The interior angle of a regular polygon is 8 times as large as the exterior angle.

Calculate the number of sides of the polygon.

Answer(c) [3]

© UCLES 2010 0580/42/M/J/10 [Turn over


18

For
10 Examiner's
Use

The diagrams show some polygons and their diagonals.

(a) Complete the table.

Number of sides Name of polygon Total number of diagonals

3 triangle 0

4 quadrilateral 2

5 5

6 hexagon 9

7 heptagon 14

[3]

(b) Write down the total number of diagonals in

(i) a decagon (a 10-sided polygon),

Answer(b)(i) [1]

(ii) a 12-sided polygon.

Answer(b)(ii) [1]

© UCLES 2010 0580/42/M/J/10


19

For
1 Examiner's
(c) A polygon with n sides has a total of n ( n − q ) diagonals, where p and q are integers. Use
p

(i) Find the values of p and q.

Answer(c)(i) p =

q= [3]

(ii) Find the total number of diagonals in a polygon with 100 sides.

Answer(c)(ii) [1]

(iii) Find the number of sides of a polygon which has a total of 170 diagonals.

Answer(c)(iii) [2]

(d) A polygon with n + 1 sides has 30 more diagonals than a polygon with n sides.

Find n.

Answer(d) n = [1]

© UCLES 2010 0580/42/M/J/10


20

BLANK PAGE

Permission to reproduce items where third-party owned material protected by copyright is included has been sought and cleared where possible. Every
reasonable effort has been made by the publisher (UCLES) to trace copyright holders, but if any items requiring clearance have unwittingly been included, the
publisher will be pleased to make amends at the earliest possible opportunity.

University of Cambridge International Examinations is part of the Cambridge Assessment Group. Cambridge Assessment is the brand name of University of
Cambridge Local Examinations Syndicate (UCLES), which is itself a department of the University of Cambridge.

0580/42/M/J/10
w
w
w
.X
tr
me
eP
ap
UNIVERSITY OF CAMBRIDGE INTERNATIONAL EXAMINATIONS

er
s
International General Certificate of Secondary Education

.c
om
*4306934856*

MATHEMATICS 0580/43
Paper 4 (Extended) May/June 2010
2 hours 30 minutes
Candidates answer on the Question Paper.
Additional Materials: Electronic calculator Geometrical instruments
Mathematical tables (optional) Tracing paper (optional)

READ THESE INSTRUCTIONS FIRST

Write your Centre number, candidate number and name on all the work you hand in.
Write in dark blue or black pen.
You may use a pencil for any diagrams or graphs.
Do not use staples, paper clips, highlighters, glue or correction fluid.
DO NOT WRITE IN ANY BARCODES.

Answer all questions.


If working is needed for any question it must be shown below that question.
Electronic calculators should be used.
If the degree of accuracy is not specified in the question, and if the answer is not exact, give the answer to
three significant figures. Give answers in degrees to one decimal place.
For π use either your calculator value or 3.142.

At the end of the examination, fasten all your work securely together.
The number of marks is given in brackets [ ] at the end of each question or part question.
The total of the marks for this paper is 130.

This document consists of 19 printed pages and 1 blank page.

IB10 06_0580_43/3RP
© UCLES 2010 [Turn over
2

For
1 Daniella is 8 years old and Edward is 12 years old. Examiner's
Use

(a) Their parents give them some money in the ratio of their ages.

(i) Write the ratio Daniella’s age : Edward’s age in its simplest form.

Answer(a)(i) : [1]

(ii) Daniella receives $30.


Show that Edward receives $45.

Answer(a)(ii)

[1]

(iii) What percentage of the total amount of money given by their parents does Edward receive?

Answer(a)(iii) % [2]

(b) Daniella invests her $30 at 3% per year, compound interest.


Calculate the amount Daniella has after 2 years.
Give your answer correct to 2 decimal places.

Answer(b) $ [3]

(c) Edward also invests $30.


He invests this money at a rate of r % per year, simple interest.
After 5 years he has a total amount of $32.25.
Calculate the value of r.

Answer(c) r = [2]

© UCLES 2010 0580/43/M/J/10


3

For
2 Examiner's
C Use

B 8 cm NOT TO
5 cm
SCALE
3 cm
D
A 11 cm

In the quadrilateral ABCD, AB = 3 cm, AD = 11 cm and DC = 8 cm.


The diagonal AC = 5 cm and angle BAC = 90°.

Calculate

(a) the length of BC,

Answer(a) BC = cm [2]

(b) angle ACD,

Answer(b) Angle ACD = [4]

(c) the area of the quadrilateral ABCD.

Answer(c) cm2 [3]

© UCLES 2010 0580/43/M/J/10 [Turn over


4

For
3 Examiner's
Use

1
2 4

4 1

1 3

2 4
1

The diagram shows a circular board, divided into 10 numbered sectors.

When the arrow is spun it is equally likely to stop in any sector.

(a) Complete the table below which shows the probability of the arrow stopping at each number.

Number 1 2 3 4

Probability 0.2 0.3


[1]

(b) The arrow is spun once.

Find

(i) the most likely number,

Answer(b)(i) [1]

(ii) the probability of a number less than 4.

Answer(b)(ii) [1]

© UCLES 2010 0580/43/M/J/10


5

For
(c) The arrow is spun twice. Examiner's
Use

Find the probability that

(i) both numbers are 2,

Answer(c)(i) [1]

(ii) the first number is 3 and the second number is 4,

Answer(c)(ii) [2]

(iii) the two numbers add up to 4.

Answer(c)(iii) [3]

(d) The arrow is spun several times until it stops at a number 4.

Find the probability that this happens on the third spin.

Answer(d) [2]

© UCLES 2010 0580/43/M/J/10 [Turn over


6

For
4 Examiner's
y Use

12

11

10

5
P
4

3
Q T
2

0 x
1 2 3 4 5 6 7 8 9 10 11 12 13 14 15 16
–1

–2

–3

–4

(a) Draw the reflection of triangle T in the line y = 6.

Label the image A. [2]

(b) Draw the translation of triangle T by the vector   .


−4
6
Label the image B. [2]

© UCLES 2010 0580/43/M/J/10


7

For
(c) Describe fully the single transformation which maps triangle B onto triangle T. Examiner's
Use

Answer(c) [2]

(d) (i) Describe fully the single transformation which maps triangle T onto triangle P.

Answer(d)(i) [3]

(ii) Complete the following statement.

Area of triangle P = × Area of triangle T [1]

(e) (i) Describe fully the single transformation which maps triangle T onto triangle Q.

Answer(e)(i) [3]

(ii) Find the 2 by 2 matrix which represents the transformation mapping triangle T onto
triangle Q.

 
 
 
 
Answer(e)(ii)   [2]

© UCLES 2010 0580/43/M/J/10 [Turn over


8

For
5 (a) Examiner's
A Use

3 cm

Q NOT TO
SCALE
4 cm

B
3.6 cm
C

The diagram shows two triangles ACB and APQ.

Angle PAQ = angle BAC and angle AQP = angle ABC.

AB = 4 cm, BC = 3.6 cm and AQ = 3 cm.

(i) Complete the following statement.

Triangle ACB is to triangle APQ. [1]

(ii) Calculate the length of PQ.

Answer(a)(ii) PQ = cm [2]

(iii) The area of triangle ACB is 5.6 cm2.

Calculate the area of triangle APQ.

Answer(a)(iii) cm2 [2]

© UCLES 2010 0580/43/M/J/10


9

For
(b) H Examiner's
Use
S
R

NOT TO
SCALE

61°
M T N

R, H, S, T and U lie on a circle, centre O.


HT is a diameter and MN is a tangent to the circle at T.
Angle RTM = 61°.

Find

(i) angle RTH,


Answer(b)(i) Angle RTH = [1]

(ii) angle RHT,


Answer(b)(ii) Angle RHT = [1]

(iii) angle RST,


Answer(b)(iii) Angle RST = [1]

(iv) angle RUT.


Answer(b)(iv) Angle RUT = [1]

(c) ABCDEF is a hexagon.


The interior angle B is 4° greater than interior angle A.
The interior angle C is 4° greater than interior angle B, and so on, with each of the next interior
angles 4° greater than the previous one.

(i) By how many degrees is interior angle F greater than interior angle A?

Answer(c)(i) [1]

(ii) Calculate interior angle A.

Answer(c)(ii) [3]

© UCLES 2010 0580/43/M/J/10 [Turn over


10

For
1 Examiner's
6 (a) Complete the table of values for y = x+ . Use
x

x –4 –3 –2 –1 –0.5 0.5 1 2 3 4
y –4.3 –3.3 –2.5 2.5 3.3 4.3

[2]

(b) y

x
–4 –3 –2 –1 0 1 2 3 4

–1

–2

–3

–4

1
On the grid, draw the graph of y = x+ for −4 Y x Y −0.5 and 0.5 Y x Y 4.
x
Six of the ten points have been plotted for you. [3]

© UCLES 2010 0580/43/M/J/10


11

For
1 Examiner's
(c) There are three integer values of k for which the equation x + =k has no solutions. Use
x
Write down these three values of k.

Answer(c) k = or k = or k = [2]

1
(d) Write down the ranges of x for which the gradient of the graph of y =x + is positive.
x

Answer(d) [2]

1
(e) To solve the equation x + = 2x + 1 , a straight line can be drawn on the grid.
x

(i) Draw this line on the grid for −2.5 Y x Y 1.5. [2]

(ii) On the grid, show how you would find the solutions. [1]

1
(iii) Show how the equation x + = 2x + 1 can be rearranged into the form x2 + bx + c = 0
x
and find the values of b and c.

Answer(e)(iii) b =

c= [3]

© UCLES 2010 0580/43/M/J/10 [Turn over


12

For
7 (a) The table shows how many books were borrowed by the 126 members of a library group in a Examiner's
month. Use

Number of books 11 12 13 14 15 16

Number of members
35 28 22 18 14 9
(frequency)

Find the mode, the median and the mean for the number of books borrowed.

Answer(a) mode =

median =

mean = [6]

(b) The 126 members record the number of hours they read in one week.

The histogram shows the results.

Frequency
density

15

10

0 h
5 8 10 12 16 20
Time (hours)

© UCLES 2010 0580/43/M/J/10


13

For
(i) Use the information from the histogram to complete the frequency table. Examiner's
Use

Number of
0IhY5 5IhY8 8 I h Y 10 10 I h Y 12 12 I h Y 16 16 I h Y 20
hours (h)

Frequency 20 24 10

[3]

(ii) Use the information in this table to calculate an estimate of the mean number of hours.
Show your working.

Answer(b)(ii) hours [4]

© UCLES 2010 0580/43/M/J/10 [Turn over


14

For
8 Examiner's
Use
NOT TO
SCALE

3 cm

6 cm

10 cm

A solid metal cuboid measures 10 cm by 6 cm by 3 cm.

(a) Show that 16 of these solid metal cuboids will fit exactly into a box which has internal
measurements 40 cm by 12 cm by 6 cm.

Answer(a)

[2]

(b) Calculate the volume of one metal cuboid.

Answer(b) cm3 [1]

(c) One cubic centimetre of the metal has a mass of 8 grams.


The box has a mass of 600 grams.

Calculate the total mass of the 16 cuboids and the box in

(i) grams,

Answer(c)(i) g [2]

(ii) kilograms.

Answer(c)(ii) kg [1]

© UCLES 2010 0580/43/M/J/10


15

For
(d) (i) Calculate the surface area of one of the solid metal cuboids. Examiner's
Use

Answer(d)(i) cm2 [2]

(ii) The surface of each cuboid is painted. The cost of the paint is $25 per square metre.

Calculate the cost of painting all 16 cuboids.

Answer(d)(ii) $ [3]

(e) One of the solid metal cuboids is melted down.


Some of the metal is used to make 200 identical solid spheres of radius 0.5 cm.

Calculate the volume of metal from this cuboid which is not used.

4
[The volume, V, of a sphere of radius r is V = π r 3.]
3

Answer(e) cm3 [3]

(f) 50 cm3 of metal is used to make 20 identical solid spheres of radius r.

Calculate the radius r.

Answer(f) r = cm [3]

© UCLES 2010 0580/43/M/J/10 [Turn over


16

For
9 (a) The cost of a bottle of water is $w. Examiner's
Use

The cost of a bottle of juice is $j.

The total cost of 8 bottles of water and 2 bottles of juice is $12.

The total cost of 12 bottles of water and 18 bottles of juice is $45.

Find the cost of a bottle of water and the cost of a bottle of juice.

Answer(a) Cost of a bottle of water = $

Cost of a bottle of juice = $ [5]

(b) Roshni cycles 2 kilometres at y km/h and then runs 4 kilometres at (y – 4) km/h.
The whole journey takes 40 minutes.

(i) Write an equation in y and show that it simplifies to y2 − 13y + 12 = 0.

Answer(b)(i)

[4]

© UCLES 2010 0580/43/M/J/10


17

For
(ii) Factorise y2 − 13y + 12. Examiner's
Use

Answer(b)(ii) [2]

(iii) Solve the equation y2 − 13y + 12 = 0.

Answer(b)(iii) y = or y = [1]

(iv) Work out Roshni’s running speed.

Answer(b)(iv) km/h [1]

(c) Solve the equation

u2 − u – 4 = 0.

Show all your working and give your answers correct to 2 decimal places.

Answer(c) u = or u = [4]

© UCLES 2010 0580/43/M/J/10 [Turn over


18

For
10 Examiner's
Use

Diagram 1 Diagram 2 Diagram 3 Diagram 4

The diagrams show squares and dots on a grid.

Some dots are on the sides of each square and other dots are inside each square.

The area of the square (shaded) in Diagram 1 is 1 unit2.

(a) Complete Diagram 4 by marking all the dots. [1]

(b) Complete the columns in the table below for Diagrams 4, 5 and n.

Diagram 1 2 3 4 5 ------- n

Number of units of area 1 4 9 -------

Number of dots inside the


1 5 13 ------- (n − 1)2 + n2
square
Number of dots on the sides
4 8 12 -------
of the square

Total number of dots 5 13 25 -------

[7]

© UCLES 2010 0580/43/M/J/10


19

For
(c) For Diagram 200, find the number of dots Examiner's
Use

(i) inside the square,

Answer(c)(i) [1]

(ii) on the sides of the square.

Answer(c)(ii) [1]

(d) Which diagram has 265 dots inside the square?

Answer(d) [1]

© UCLES 2010 0580/43/M/J/10


20

BLANK PAGE

Permission to reproduce items where third-party owned material protected by copyright is included has been sought and cleared where possible. Every
reasonable effort has been made by the publisher (UCLES) to trace copyright holders, but if any items requiring clearance have unwittingly been included, the
publisher will be pleased to make amends at the earliest possible opportunity.

University of Cambridge International Examinations is part of the Cambridge Assessment Group. Cambridge Assessment is the brand name of University of
Cambridge Local Examinations Syndicate (UCLES), which is itself a department of the University of Cambridge.

© UCLES 2010 0580/43/M/J/10


UNIVERSITY OF CAMBRIDGE INTERNATIONAL EXAMINATIONS
International General Certificate of Secondary Education
*4338708562*

MATHEMATICS 0580/41
Paper 4 (Extended) May/June 2011
2 hours 30 minutes
Candidates answer on the Question Paper.
Additional Materials: Electronic calculator Geometrical instruments
Mathematical tables (optional) Tracing paper (optional)

READ THESE INSTRUCTIONS FIRST

Write your Centre number, candidate number and name on all the work you hand in.
Write in dark blue or black pen.
You may use a pencil for any diagrams or graphs.
Do not use staples, paper clips, highlighters, glue or correction fluid.
DO NOT WRITE IN ANY BARCODES.

Answer all questions.


If working is needed for any question it must be shown below that question.
Electronic calculators should be used.
If the degree of accuracy is not specified in the question, and if the answer is not exact, give the answer to
three significant figures. Give answers in degrees to one decimal place.
For π use either your calculator value or 3.142.

At the end of the examination, fasten all your work securely together.
The number of marks is given in brackets [ ] at the end of each question or part question.
The total of the marks for this paper is 130.

This document consists of 16 printed pages.

IB11 06_0580_41/6RP
© UCLES 2011 [Turn over
2

1 A school has a sponsored swim in summer and a sponsored walk in winter. For
In 2010, the school raised a total of $1380. Examiner's
Use
The ratio of the money raised in summer : winter = 62 : 53.

(a) (i) Show clearly that $744 was raised by the swim in summer.

Answer (a)(i)

[1]

(ii) Alesha’s swim raised $54.10. Write this as a percentage of $744.

Answer(a)(ii) % [1]

(iii) Bryan’s swim raised $31.50.


He received 75 cents for each length of the pool which he swam.

Calculate the number of lengths Bryan swam.

Answer(a)(iii) [2]

(b) The route for the sponsored walk in winter is triangular.

North

B 110°
NOT TO
SCALE

(i) Senior students start at A, walk North to B, then walk on a bearing 110° to C.
They then return to A.
AB = BC.

Calculate the bearing of A from C.

Answer(b)(i) [3]

© UCLES 2011 0580/41/M/J/11


3

(ii) For
North Examiner's
Use

B 110°
NOT TO
SCALE
110°

4 km C

AB = BC = 6 km.
Junior students follow a similar path but they only walk 4 km North from A, then 4 km on a
bearing 110° before returning to A.

Senior students walk a total of 18.9 km.

Calculate the distance walked by junior students.

Answer(b)(ii) km [3]

(c) The total amount, $1380, raised in 2010 was 8% less than the total amount raised in 2009.

Calculate the total amount raised in 2009.

Answer(c) $ [3]

© UCLES 2011 0580/41/M/J/11 [Turn over


4

2 In this question give all your answers as fractions. For


Examiner's
Use
3
The probability that it rains on Monday is .
5
4
If it rains on Monday, the probability that it rains on Tuesday is .
7
5
If it does not rain on Monday, the probability that it rains on Tuesday is .
7
(a) Complete the tree diagram.

Monday Tuesday

Rain
Rain
No rain

Rain
No rain
No rain
[3]
(b) Find the probability that it rains

(i) on both days,

Answer(b)(i) [2]

(ii) on Monday but not on Tuesday,

Answer(b)(ii) [2]

(iii) on only one of the two days.

Answer(b)(iii) [2]

(c) If it does not rain on Monday and it does not rain on Tuesday, the probability that it does not
1
rain on Wednesday is .
4
Calculate the probability that it rains on at least one of the three days.

Answer(c) [3]

© UCLES 2011 0580/41/M/J/11


5

3 (a) p varies inversely as (m + 1). For


Examiner's
Use
When p = 4, m = 8.

Find the value of p when m = 11.

Answer(a) p = [3]

(b) (i) Factorise x2 – 25.

Answer(b)(i) [1]

2 x 2 + 11x + 5
(ii) Simplify .
x 2 − 25

Answer(b)(ii) [3]

(c) Solve the inequality 5(x – 4) I 3(12 – x).

Answer(c) [3]

© UCLES 2011 0580/41/M/J/11 [Turn over


6

4 (a) For
Examiner's
Use
H 12 cm
G

NOT TO
6 cm SCALE
14 cm

The diagram shows triangle FGH, with FG = 14 cm, GH = 12 cm and FH = 6 cm.

(i) Calculate the size of angle HFG.

Answer(a)(i) Angle HFG = [4]

(ii) Calculate the area of triangle FGH.

Answer(a)(ii) cm2 [2]

© UCLES 2011 0580/41/M/J/11


7

(b) For
Q Examiner's
18 cm Use

NOT TO
SCALE
12 cm

117°

The diagram shows triangle PQR, with RP = 12 cm, RQ = 18 cm and angle RPQ = 117°.

Calculate the size of angle RQP.

Answer(b) Angle RQP = [3]

© UCLES 2011 0580/41/M/J/11 [Turn over


8

5 For
y Examiner's
Use
4
A B
3

x
–5 –4 –3 –2 –1 0 1 2 3 4 5 6 7
–1
C
–2

–3

–4

–5

–6

(a) On the grid above, draw the image of

 − 3
(i) shape A after translation by the vector  , [2]
 − 2

(ii) shape A after reflection in the line x = −1 . [2]

(b) Describe fully the single transformation which maps

(i) shape A onto shape B,

Answer(b)(i) [3]

(ii) shape A onto shape C.

Answer(b)(ii) [3]

(c) Find the matrix representing the transformation which maps shape A onto shape B.

 
Answer(c)   [2]
 

 −1 0
(d) Describe fully the single transformation represented by the matrix   .
 0 −1 

Answer(d) [3]

© UCLES 2011 0580/41/M/J/11


9

6 For
F Examiner's
Use
NOT TO
SCALE
C
D E
14 cm
36 cm
A B
19 cm

In the diagram, ABCDEF is a prism of length 36 cm.


The cross-section ABC is a right-angled triangle.
AB = 19 cm and AC = 14 cm.

Calculate

(a) the length BC,

Answer(a) BC = cm [2]

(b) the total surface area of the prism,

Answer(b) cm2 [4]

(c) the volume of the prism,

Answer(c) cm3 [2]

(d) the length CE,

Answer(d) CE = cm [2]

(e) the angle between the line CE and the base ABED.

Answer(e) [3]

© UCLES 2011 0580/41/M/J/11 [Turn over


10

4 For
7 (a) Complete the table of values for the equation y = , x ≠ 0. Examiner's
x2 Use

x O4 O3 O2 O1 O0.6 0.6 1 2 3 4

y 0.25 0.44 11.11 4.00 0.44


[3]

4
(b) On the grid, draw the graph of y = for O4 Y x Y O0.6 and 0.6 Y x Y 4 .
x2

y
12

11

10

x
–4 –3 –2 –1 0 1 2 3 4
–1

–2

[5]

© UCLES 2011 0580/41/M/J/11


11

4 For
(c) Use your graph to solve the equation =6. Examiner's
x2 Use

Answer(c)x = or x = [2]

(d) By drawing a suitable tangent, estimate the gradient of the graph where x = 1.5.

Answer(d) [3]

4
(e) (i) The equation O x + 2 = 0 can be solved by finding the intersection of the graph
x2
4
of y = and a straight line.
x2
Write down the equation of this straight line.

Answer(e)(i) [1]

(ii) On the grid, draw the straight line from your answer to part (e)(i). [2]

4
(iii) Use your graphs to solve the equation O x + 2 = 0.
x2

Answer(e)(iii) x = [1]

© UCLES 2011 0580/41/M/J/11 [Turn over


12

8 The table below shows the marks scored by a group of students in a test. For
Examiner's
Use
Mark 11 12 13 14 15 16 17 18

Frequency 10 8 16 11 7 8 6 9

(a) Find the mean, median and mode.

Answer(a) mean =

median =

mode = [6]

(b) The table below shows the time (t minutes) taken by the students to complete the test.

Time (t) 0 I=t Y=10 10 I=t Y=20 20 I=t Y=30 30 I=t Y=40 40 I=t Y=50 50 I=t Y=60

Frequency 2 19 16 14 15 9

(i) Cara rearranges this information into a new table.

Complete her table.

Time (t) 0 I=t Y=20 20 I=t Y=40 40 I=t Y=50 50 I=t Y=60

Frequency 9
[2]

(ii) Cara wants to draw a histogram to show the information in part (b)(i).

Complete the table below to show the interval widths and the frequency densities.

0 I=t Y=20 20 I=t Y=40 40 I=t Y=50 50 I=t Y=60


Interval
10
width
Frequency
0.9
density
[3]

© UCLES 2011 0580/41/M/J/11


13

(c) Some of the students were asked how much time they spent revising for the test. For
Examiner's
Use
10 students revised for 2.5 hours, 12 students revised for 3 hours and n students revised for
4 hours.

The mean time that these students spent revising was 3.1 hours.

Find n.

Show all your working.

Answer(c) n = [4]

© UCLES 2011 0580/41/M/J/11 [Turn over


14

9 Peter wants to plant x plum trees and y apple trees. For


Examiner's
Use
He wants at least 3 plum trees and at least 2 apple trees.

(a) Write down one inequality in x and one inequality in y to represent these conditions.

Answer(a) , [2]

(b) There is space on his land for no more than 9 trees.

Write down an inequality in x and y to represent this condition.

Answer(b) [1]

(c) Plum trees cost $6 and apple trees cost $14.

Peter wants to spend no more than $84.

Write down an inequality in x and y, and show that it simplifies to 3x + 7y Y 42.

Answer(c)

[1]

© UCLES 2011 0580/41/M/J/11


15

(d) On the grid, draw four lines to show the four inequalities and shade the unwanted regions. For
Examiner's
y Use

12

11

10

x
0
1 2 3 4 5 6 7 8 9 10 11 12 13 14 15

[7]

(e) Calculate the smallest cost when Peter buys a total of 9 trees.

Answer(e) $ [2]

Question 10 is printed on the next page.

© UCLES 2011 0580/41/M/J/11 [Turn over


16

10 The first and the nth terms of sequences A, B and C are shown in the table below. For
Examiner's
Use
(a) Complete the table for each sequence.

1st term 2nd term 3rd term 4th term 5th term nth term

Sequence A 1 n3

Sequence B 4 4n

Sequence C 4 (n + 1)2
[5]
(b) Find

(i) the 8th term of sequence A,

Answer(b)(i) [1]

(ii) the 12th term of sequence C.

Answer(b)(ii) [1]

(c) (i) Which term in sequence A is equal to 15 625?

Answer(c)(i) [1]

(ii) Which term in sequence C is equal to 10 000?

Answer(c)(ii) [1]

(d) The first four terms of sequences D and E are shown in the table below.

Use the results from part (a) to find the 5th and the nth terms of the sequences D and E.

1st term 2nd term 3rd term 4th term 5th term nth term

Sequence D 5 16 39 80

Sequence E 0 1 4 9
[4]

Permission to reproduce items where third-party owned material protected by copyright is included has been sought and cleared where possible. Every
reasonable effort has been made by the publisher (UCLES) to trace copyright holders, but if any items requiring clearance have unwittingly been included, the
publisher will be pleased to make amends at the earliest possible opportunity.

University of Cambridge International Examinations is part of the Cambridge Assessment Group. Cambridge Assessment is the brand name of University of
Cambridge Local Examinations Syndicate (UCLES), which is itself a department of the University of Cambridge.

© UCLES 2011 0580/41/M/J/11


UNIVERSITY OF CAMBRIDGE INTERNATIONAL EXAMINATIONS
International General Certificate of Secondary Education
*9387816354*

MATHEMATICS 0580/42
Paper 4 (Extended) May/June 2011
2 hours 30 minutes
Candidates answer on the Question Paper.
Additional Materials: Electronic calculator Geometrical instruments
Mathematical tables (optional) Tracing paper (optional)

READ THESE INSTRUCTIONS FIRST

Write your Centre number, candidate number and name on all the work you hand in.
Write in dark blue or black pen.
You may use a pencil for any diagrams or graphs.
Do not use staples, paper clips, highlighters, glue or correction fluid.
DO NOT WRITE IN ANY BARCODES.

Answer all questions.


If working is needed for any question it must be shown below that question.
Electronic calculators should be used.
If the degree of accuracy is not specified in the question, and if the answer is not exact, give the answer to
three significant figures. Give answers in degrees to one decimal place.
For π use either your calculator value or 3.142.

At the end of the examination, fasten all your work securely together.
The number of marks is given in brackets [ ] at the end of each question or part question.
The total of the marks for this paper is 130.

This document consists of 15 printed pages and 1 blank page.

IB11 06_0580_42/6RP
© UCLES 2011 [Turn over
2

1 (a) Work out the following. For


Examiner's
Use
1
(i)
0.2 2

Answer(a)(i) [1]

(ii) 5.12 + 4 × 7.3 2

Answer(a)(ii) [1]

1 2

2 3
(iii) 25 × 1000

Answer(a)(iii) [2]

(b) Mia invests $7500 at 3.5% per year simple interest.


Calculate the total amount she has after 5 years.

Answer(b) $ [3]

(c) Written as the product of prime factors 48 = 24 × 3.

(i) Write 60 as the product of prime factors.

Answer(c)(i) [2]

(ii) Work out the highest common factor (HCF) of 48 and 60.

Answer(c)(ii) [2]

(iii) Work out the lowest common multiple (LCM) of 48 and 60.

Answer(c)(iii) [2]

© UCLES 2011 0580/42/M/J/11


3

2 For
B C Examiner's
Use
NOT TO
A 1.7 m SCALE
D

F
G
1.5 m
E 2m H

The diagram shows a box ABCDEFGH in the shape of a cuboid measuring 2 m by 1.5 m by 1.7 m.

(a) Calculate the length of the diagonal EC.

Answer(a) EC = m [4]

(b) Calculate the angle between EC and the base EFGH.

Answer(b) [3]

(c) (i) A rod has length 2.9 m, correct to 1 decimal place.

What is the upper bound for the length of the rod?

Answer(c)(i) m [1]

(ii) Will the rod fit completely in the box?

Give a reason for your answer.

Answer(c)(ii) [1]

© UCLES 2011 0580/42/M/J/11 [Turn over


4

3 (a) For
North Examiner's
Use

North

The scale drawing shows the positions of two towns A and C on a map.
On the map, 1 centimetre represents 20 kilometres.

(i) Find the distance in kilometres from town A to town C.

Answer(a)(i) km [2]

(ii) Measure and write down the bearing of town C from town A.

Answer(a)(ii) [1]

(iii) Town B is 140 km from town C on a bearing of 150°.

Mark accurately the position of town B on the scale drawing. [2]

(iv) Find the bearing of town C from town B.

Answer(a)(iv) [1]

(v) A lake on the map has an area of 0.15 cm2.

Work out the actual area of the lake.

Answer(a)(v) km2 [2]

© UCLES 2011 0580/42/M/J/11


5

(b) A plane leaves town C at 11 57 and flies 1500 km to another town, landing at 14 12. For
Examiner's
Use
Calculate the average speed of the plane.

Answer(b) km/h [3]

(c)
Q

NOT TO
SCALE
1125 km
790 km

P
1450 km R

The diagram shows the distances between three towns P, Q and R.

Calculate angle PQR.

Answer(c)Angle PQR = [4]

© UCLES 2011 0580/42/M/J/11 [Turn over


6

3 For
4 (a) Complete the table of values for the function y = x2 O , x ≠ 0. Examiner's
x Use

x O3 O2 O1 O0.5 O0.25 0.25 0.5 1 2 3

y 10 5.5 6.3 12.1 O11.9 2.5 8


[3]

3
(b) Draw the graph of y = x2 O for O3 Y x Y O0.25 and 0.25 Y x Y 3.
x

y
14

12

10

x
–3 –2 –1 0 1 2 3
–2

–4

–6

–8

–10

–12

–14
[5]

© UCLES 2011 0580/42/M/J/11


7

3 For
(c) Use your graph to solve x2 O = 7. Examiner's
x Use

Answer(c) x = or x = or x = [3]

(d) Draw the tangent to the curve where x = O2.


Use the tangent to calculate an estimate of the gradient of the curve where x = O2.

Answer(d) [3]

© UCLES 2011 0580/42/M/J/11 [Turn over


8

5 (a) Solve 9 I 3n + 6 Y 21 for integer values of n. For


Examiner's
Use

Answer(a) [3]

(b) Factorise completely.

(i) 2x2 + 10xy

Answer(b)(i) [2]

(ii) 3a2 O 12b2

Answer(b)(ii) [3]

(c)

NOT TO
SCALE
x cm

(x + 17) cm

The area of this triangle is 84 cm2.

(i) Show that x2 + 17x O 168 = 0.

Answer (c)(i)

[2]

(ii) Factorise x2 + 17x O 168.

Answer(c)(ii) [2]

(iii) Solve x2 + 17x O 168 = 0.

Answer(c)(iii) x = or x = [1]

© UCLES 2011 0580/42/M/J/11


9

(d) Solve For


15 − x Examiner's
= 3 − 2 x. Use
2

Answer(d) x = [3]

(e) Solve 2x2 O 5x O 6 = 0.

Show all your working and give your answers correct to 2 decimal places.

Answer(e) x = or x = [4]

© UCLES 2011 0580/42/M/J/11 [Turn over


10

6 For
Examiner's
Time
0 I t Y 20 20 I t Y 35 35 I t Y 45 45 I t Y 55 55 I t Y 70 70 I t Y 80 Use
(t mins)

Frequency 6 15 19 37 53 20

The table shows the times taken, in minutes, by 150 students to complete their homework on one day.

(a) (i) In which interval is the median time?

Answer(a)(i) [1]

(ii) Using the mid-interval values 10, 27.5, ……..calculate an estimate of the mean time.

Answer(a)(ii) min [3]

(b) (i) Complete the table of cumulative frequencies.

Time
t Y 20 t Y 35 t Y 45 t Y 55 t Y 70 t Y 80
(t mins)
Cumulative
6 21
frequency
[2]

(ii) On the grid, label the horizontal axis from 0 to 80, using the scale 1 cm represents 5 minutes
and the vertical axis from 0 to 150, using the scale 1 cm represents 10 students.

Draw a cumulative frequency diagram to show this information. [5]

© UCLES 2011 0580/42/M/J/11


11

For
Examiner's
Use

(c) Use your graph to estimate

(i) the median time, Answer(c)(i) min [1]

(ii) the inter-quartile range,


Answer(c)(ii) min [2]

(iii) the number of students whose time was in the range 50 I t Y 60,

Answer(c)(iii) [1]

(iv) the probability, as a fraction, that a student, chosen at random, took longer than 50 minutes,

Answer(c)(iv) [2]

(v) the probability, as a fraction, that two students, chosen at random, both took longer than 50
minutes.

Answer(c)(v) [2]

© UCLES 2011 0580/42/M/J/11 [Turn over


12

7 (a) For
V Examiner's
V Use
B C
NOT TO
SCALE
B C A D
9.5 cm
A D 2.5 cm
2.5 cm F 2.5 cm E
F E F E

A solid pyramid has a regular hexagon of side 2.5 cm as its base.


Each sloping face is an isosceles triangle with base 2.5 cm and height 9.5 cm.

Calculate the total surface area of the pyramid.

Answer(a) cm2 [4]

(b)
O

55°
NOT TO
15 cm
SCALE

A B

A sector OAB has an angle of 55° and a radius of 15 cm.

Calculate the area of the sector and show that it rounds to 108 cm2, correct to 3 significant figures.

Answer (b)

[3]

© UCLES 2011 0580/42/M/J/11


13

(c) For
Examiner's
Use

15 cm NOT TO
SCALE

The sector radii OA and OB in part (b) are joined to form a cone.

(i) Calculate the base radius of the cone.


[The curved surface area, A, of a cone with radius r and slant height l is A = πrl.]

Answer(c)(i) cm [2]

(ii) Calculate the perpendicular height of the cone.

Answer(c)(ii) cm [3]

(d)

7.5 cm
NOT TO
SCALE

A solid cone has the same dimensions as the cone in part (c).
A small cone with slant height 7.5 cm is removed by cutting parallel to the base.

Calculate the volume of the remaining solid.

1
[The volume, V, of a cone with radius r and height h is V = πr2h.]
3

Answer(d) cm3 [3]

© UCLES 2011 0580/42/M/J/11 [Turn over


14

8 (a) For
Examiner's
Use
A

Draw the enlargement of triangle P with centre A and scale factor 2. [2]

(b)
y

Q R

x
0
(i) Describe fully the single transformation which maps shape Q onto shape R.

Answer(b)(i) [3]

(ii) Find the matrix which represents this transformation.


 
Answer(b)(ii)   [2]
 

(c)
y

S
T

x
0
Describe fully the single transformation which maps shape S onto shape T.

Answer(c) [3]

© UCLES 2011 0580/42/M/J/11


15

9 (a) (i) Work out the first 3 terms of the sequence whose nth term is n(n + 2). For
Examiner's
Use
Answer(a)(i) , , [2]

(ii) Which term in this sequence is equal to 168?

Answer(a)(ii) [3]

(b) Find a formula for the nth term of the following sequences.

(i) 5 8 11 14 17 ……

Answer(b)(i) [2]

(ii) 1 2 4 8 16 ……

Answer(b)(ii) [2]

(c)

Diagram 1 Diagram 2 Diagram 3

A sequence of diagrams is formed by drawing equilateral triangles each of side one centimetre.
Diagram 1 has 3 one centimetre lines.
Diagram 2 has 9 one centimetre lines.

The formula for the total number of one centimetre lines needed to draw all of the first n
diagrams is
an3 + bn2 + n.
Find the values of a and b.

Answer(c) a =

b= [6]

© UCLES 2011 0580/42/M/J/11


16

BLANK PAGE

Permission to reproduce items where third-party owned material protected by copyright is included has been sought and cleared where possible. Every
reasonable effort has been made by the publisher (UCLES) to trace copyright holders, but if any items requiring clearance have unwittingly been included, the
publisher will be pleased to make amends at the earliest possible opportunity.

University of Cambridge International Examinations is part of the Cambridge Assessment Group. Cambridge Assessment is the brand name of University of
Cambridge Local Examinations Syndicate (UCLES), which is itself a department of the University of Cambridge.

© UCLES 2011 0580/42/M/J/11


UNIVERSITY OF CAMBRIDGE INTERNATIONAL EXAMINATIONS
International General Certificate of Secondary Education
*8044643715*

MATHEMATICS 0580/43
Paper 4 (Extended) May/June 2011
2 hours 30 minutes
Candidates answer on the Question Paper.
Additional Materials: Electronic calculator Geometrical instruments
Mathematical tables (optional) Tracing paper (optional)

READ THESE INSTRUCTIONS FIRST

Write your Centre number, candidate number and name on all the work you hand in.
Write in dark blue or black pen.
You may use a pencil for any diagrams or graphs.
Do not use staples, paper clips, highlighters, glue or correction fluid.
DO NOT WRITE IN ANY BARCODES.

Answer all questions.


If working is needed for any question it must be shown below that question.
Electronic calculators should be used.
If the degree of accuracy is not specified in the question, and if the answer is not exact, give the answer to
three significant figures. Give answers in degrees to one decimal place.
For π use either your calculator value or 3.142.

At the end of the examination, fasten all your work securely together.
The number of marks is given in brackets [ ] at the end of each question or part question.
The total of the marks for this paper is 130.

This document consists of 19 printed pages and 1 blank page.

IB11 06_0580_43/4RP
© UCLES 2011 [Turn over
2

1 Lucy works in a clothes shop. For


Examiner's
Use
(a) In one week she earned $277.20.
1
(i) She spent of this on food.
8
Calculate how much she spent on food.

Answer(a)(i) $ [1]

(ii) She paid 15% of the $277.20 in taxes.


Calculate how much she paid in taxes.

Answer(a)(ii) $ [2]

(iii) The $277.20 was 5% more than Lucy earned in the previous week.
Calculate how much Lucy earned in the previous week.

Answer(a)(iii) $ [3]

(b) The shop sells clothes for men, women and children.

(i) In one day Lucy sold clothes with a total value of $2200 in the ratio

men : women : children = 2 : 5 : 4.

Calculate the value of the women’s clothes she sold.

Answer(b)(i) $ [2]

44
(ii) The $2200 was of the total value of the clothes sold in the shop on this day.
73
Calculate the total value of the clothes sold in the shop on this day.

Answer(b)(ii) $ [2]

© UCLES 2011 0580/43/M/J/11


3

2 For
y Examiner's
Use
6

5 X

x
–6 –5 –4 –3 –2 –1 0 1 2 3 4 5 6
–1

–2

–3

–4

–5

–6
(a) (i) Draw the reflection of shape X in the x-axis. Label the image Y. [2]

(ii) Draw the rotation of shape Y, 90° clockwise about (0, 0). Label the image Z. [2]

(iii) Describe fully the single transformation that maps shape Z onto shape X.

Answer(a)(iii) [2]
1
(b) (i) Draw the enlargement of shape X, centre (0, 0), scale factor . [2]
2
1
(ii) Find the matrix which represents an enlargement, centre (0, 0), scale factor .
2

 
Answer(b)(ii)   [2]
 
(c) (i) Draw the shear of shape X with the x-axis invariant and shear factor –1. [2]

(ii) Find the matrix which represents a shear with the x-axis invariant and shear factor –1.

 
Answer(c)(ii)   [2]
 

© UCLES 2011 0580/43/M/J/11 [Turn over


4

3 For
Examiner's
NOT TO Use
SCALE

x cm

2x cm

(x + 5) cm

The diagram shows a square of side (x + 5) cm and a rectangle which measures 2x cm by x cm.

The area of the square is 1 cm2 more than the area of the rectangle.

(a) Show that x2 – 10x – 24 = 0 .

Answer(a)

[3]

© UCLES 2011 0580/43/M/J/11


5

(b) Find the value of x. For


Examiner's
Use

Answer(b) x = [3]

(c) Calculate the acute angle between the diagonals of the rectangle.

Answer(c) [3]

© UCLES 2011 0580/43/M/J/11 [Turn over


6

4 For
A Examiner's
Use

8 cm NOT TO
6 cm SCALE

B C
9 cm

The circle, centre O, passes through the points A, B and C.

In the triangle ABC, AB = 8 cm, BC = 9 cm and CA = 6 cm.

(a) Calculate angle BAC and show that it rounds to 78.6°, correct to 1 decimal place.

Answer(a)

[4]

(b) M is the midpoint of BC.

(i) Find angle BOM.

Answer(b)(i) Angle BOM = [1]

© UCLES 2011 0580/43/M/J/11


7

(ii) Calculate the radius of the circle and show that it rounds to 4.59 cm, correct to 3 significant For
figures. Examiner's
Use

Answer(b)(ii)

[3]

(c) Calculate the area of the triangle ABC as a percentage of the area of the circle.

Answer(c) % [4]

© UCLES 2011 0580/43/M/J/11 [Turn over


8

1 For
5 (a) Complete the table of values for the function f(x), where f(x) = x2 + 2
, x ≠ 0. Examiner's
x Use

x O3 O2.5 O2 O1.5 O1 O0.5 0.5 1 1.5 2 2.5 3


f(x) 6.41 2.69 4.25 4.25 2.69 6.41
[3]

(b) On the grid, draw the graph of y = f(x) for O3 Y x Y O0.5 and 0.5 Y x Y 3 .

y
10

x
–3 –2 –1 0 1 2 3
–1

–2
[5]

© UCLES 2011 0580/43/M/J/11


9

(c) (i) Write down the equation of the line of symmetry of the graph. For
Examiner's
Use

Answer(c)(i) [1]

(ii) Draw the tangent to the graph of y = f(x) where x = O1.5.


Use the tangent to estimate the gradient of the graph of y = f(x) where x = O1.5.

Answer(c)(ii) [3]

1
(iii) Use your graph to solve the equation x2 + = 3.
x2

Answer(c)(iii) x = or x = or x = or x = [2]

1
(iv) Draw a suitable line on the grid and use your graphs to solve the equation x2 + = 2x.
x2

Answer(c)(iv) x = or x = [3]

© UCLES 2011 0580/43/M/J/11 [Turn over


10

6 For
Examiner's
200 Use

180

160

140

120
Cumulative
100
frequency

80

60

40

20

m
0
1 2 3 4 5 6 7 8 9 10
Mass (kilograms)

The masses of 200 parcels are recorded.

The results are shown in the cumulative frequency diagram above.

(a) Find

(i) the median,

Answer(a)(i) kg [1]

(ii) the lower quartile,

Answer(a)(ii) kg [1]

(iii) the inter-quartile range,

Answer(a)(iii) kg [1]

(iv) the number of parcels with a mass greater than 3.5 kg.

Answer(a)(iv) [2]

© UCLES 2011 0580/43/M/J/11


11

(b) (i) Use the information from the cumulative frequency diagram to complete the grouped For
frequency table. Examiner's
Use

Mass (m) kg 0ImY4 4ImY6 6ImY7 7 I m Y 10

Frequency 36 50
[2]
(ii) Use the grouped frequency table to calculate an estimate of the mean.

Answer(b)(ii) kg [4]

(iii) Complete the frequency density table and use it to complete the histogram.

Mass (m) kg 0ImY4 4ImY6 6ImY7 7 I m Y 10

Frequency
9 16.7
density

40

35

30

25
Frequency
density 20

15

10

m
0
1 2 3 4 5 6 7 8 9 10
Mass (kilograms)
[4]

© UCLES 2011 0580/43/M/J/11 [Turn over


12

7 Katrina puts some plants in her garden. For


7 Examiner's
The probability that a plant will produce a flower is . Use
10
If there is a flower, it can only be red, yellow or orange.
2 1
When there is a flower, the probability it is red is and the probability it is yellow is .
3 4

(a) Draw a tree diagram to show all this information.

Label the diagram and write the probabilities on each branch.

Answer(a)

[5]

(b) A plant is chosen at random.

Find the probability that it will not produce a yellow flower.

Answer(b) [3]

(c) If Katrina puts 120 plants in her garden, how many orange flowers would she expect?

Answer(c) [2]

© UCLES 2011 0580/43/M/J/11


13

8 For
Examiner's
Use
D

A B

(a) Draw accurately the locus of points, inside the quadrilateral ABCD, which are 6 cm from the
point D. [1]

(b) Using a straight edge and compasses only, construct

(i) the perpendicular bisector of AB, [2]

(ii) the locus of points, inside the quadrilateral, which are equidistant from AB and from BC. [2]

(c) The point Q is equidistant from A and from B and equidistant from AB and from BC.

(i) Label the point Q on the diagram. [1]

(ii) Measure the distance of Q from the line AB.

Answer(c)(ii) cm [1]

(d) On the diagram, shade the region inside the quadrilateral which is

• less than 6 cm from D


and
• nearer to A than to B
and
• nearer to AB than to BC. [1]

© UCLES 2011 0580/43/M/J/11 [Turn over


14

9 f(x) = 3x + 1 g(x) = (x + 2)2 For


Examiner's
Use
(a) Find the values of

(i) gf(2),

Answer(a)(i) [2]

(ii) ff(0.5).

Answer(a)(ii) [2]

(b) Find f –1(x), the inverse of f(x).

Answer(b) [2]

(c) Find fg(x).

Give your answer in its simplest form.

Answer(c) [2]

© UCLES 2011 0580/43/M/J/11


15

(d) Solve the equation x2 + f(x) = 0. For


Examiner's
Use
Show all your working and give your answers correct to 2 decimal places.

Answer(d) x = or x = [4]

© UCLES 2011 0580/43/M/J/11 [Turn over


Name: __________________________________________

Class: ___________________________________________
16 ------
Date: ___________________________________________
10 (a) For
D L C Examiner's
NOT TO Use
SCALE
N

M
q

A p B

ABCD is a parallelogram.
L is the midpoint of DC, M is the midpoint of BC and N is the midpoint of LM.
= p and = q.

(i) Find the following in terms of p and q, in their simplest form.

(a)

Answer(a)(i)(a) = [1]

(b)

Answer(a)(i)(b) = [2]

(c)

Answer(a)(i)(c) = [2]

(ii) Explain why your answer for shows that the point N lies on the line AC.

Answer(a)(ii) [1]

© UCLES 2011 0580/43/M/J/11


17

(b) For
F G Examiner's
Use
2x° (x + 15)°

NOT TO
H J SCALE

75°

EFG is a triangle.
HJ is parallel to FG.
Angle FEG = 75°.
Angle EFG = 2x° and angle FGE = (x + 15)°.

(i) Find the value of x.

Answer(b)(i) x = [2]

(ii) Find angle HJG.

Answer(b)(ii) Angle HJG = [1]

© UCLES 2011 0580/43/M/J/11 [Turn over


18

11 (a) (i) The first three positive integers 1, 2 and 3 have a sum of 6. For
Examiner's
Use
Write down the sum of the first 4 positive integers.

Answer(a)(i) [1]

n( n + 1)
(ii) The formula for the sum of the first n integers is .
2

Show the formula is correct when n = 3.

Answer(a)(ii)

[1]

(iii) Find the sum of the first 120 positive integers.

Answer(a)(iii) [1]

(iv) Find the sum of the integers

121 + 122 + 123 + 124 + …………………………… + 199 + 200.

Answer(a)(iv) [2]

(v) Find the sum of the even numbers

2+4+6+ ………………………… + 800.

Answer(a)(v) [2]

© UCLES 2011 0580/43/M/J/11


19

(b) (i) Complete the following statements about the sums of cubes and the sums of integers. For
Examiner's
13 = 1 1=1 Use

13 + 23 = 9 1+2=3

13 + 23 + 33 = 1+2+3=

13 + 23 + 33 + 43 = 1+2+3+4= [2]

(ii) The sum of the first 14 integers is 105.

Find the sum of the first 14 cubes.

Answer(b)(ii) [1]

(iii) Use the formula in part(a)(ii) to write down a formula for the sum of the first n cubes.

Answer(b)(iii) [1]

(iv) Find the sum of the first 60 cubes.

Answer(b)(iv) [1]

(v) Find n when the sum of the first n cubes is 278 784.

Answer(b)(v) n = [2]

© UCLES 2011 0580/43/M/J/11


20

BLANK PAGE

Permission to reproduce items where third-party owned material protected by copyright is included has been sought and cleared where possible. Every
reasonable effort has been made by the publisher (UCLES) to trace copyright holders, but if any items requiring clearance have unwittingly been included, the
publisher will be pleased to make amends at the earliest possible opportunity.

University of Cambridge International Examinations is part of the Cambridge Assessment Group. Cambridge Assessment is the brand name of University of
Cambridge Local Examinations Syndicate (UCLES), which is itself a department of the University of Cambridge.

© UCLES 2011 0580/43/M/J/11


UNIVERSITY OF CAMBRIDGE INTERNATIONAL EXAMINATIONS
International General Certificate of Secondary Education
*6574307018*

MATHEMATICS 0580/41
Paper 4 (Extended) May/June 2012
2 hours 30 minutes
Candidates answer on the Question Paper.
Additional Materials: Electronic calculator Geometrical instruments
Mathematical tables (optional) Tracing paper (optional)

READ THESE INSTRUCTIONS FIRST

Write your Centre number, candidate number and name on all the work you hand in.
Write in dark blue or black pen.
You may use a pencil for any diagrams or graphs.
Do not use staples, paper clips, highlighters, glue or correction fluid.
DO NOT WRITE IN ANY BARCODES.

Answer all questions.


If working is needed for any question it must be shown below that question.
Electronic calculators should be used.
If the degree of accuracy is not specified in the question, and if the answer is not exact, give the answer to
three significant figures. Give answers in degrees to one decimal place.
For π use either your calculator value or 3.142.

At the end of the examination, fasten all your work securely together.
The number of marks is given in brackets [ ] at the end of each question or part question.
The total of the marks for this paper is 130.

This document consists of 16 printed pages.

IB12 06_0580_41/3RP
© UCLES 2012 [Turn over
2

1 Anna, Bobby and Carl receive a sum of money. For


They share it in the ratio 12 : 7 : 8 . Examiner's
Use
Anna receives $504.

(a) Calculate the total amount.

Answer(a) $ [3]

(b) (i) Anna uses 7% of her $504 to pay a bill.


Calculate how much she has left.

Answer(b)(i) $ [3]

(ii) She buys a coat in a sale for $64.68.


This was 23% less than the original price.
Calculate the original price of the coat.

Answer(b)(ii) $ [3]

(c) Bobby uses $250 of his share to open a bank account.


This account pays compound interest at a rate of 1.6% per year.
Calculate the amount in the bank account after 3 years.
Give your answer correct to 2 decimal places.

Answer(c) $ [3]

(d) Carl buys a computer for $288 and sells it for $324.
Calculate his percentage profit.

Answer(d) % [3]

© UCLES 2012 0580/41/M/J/12


3

2 For
North Examiner's
Use

K
108° NOT TO
SCALE
4 km
9 km

L
Three buoys K, L and M show the course of a boat race.
MK = 4 km, KL = 9 km and angle MKL = 108°.

(a) Calculate the distance ML.

Answer(a) ML = km [4]

(b) The bearing of L from K is 125°.

(i) Calculate how far L is south of K.

Answer(b)(i) km [3]

(ii) Find the three figure bearing of K from M.

Answer(b)(ii) [2]

© UCLES 2012 0580/41/M/J/12 [Turn over


4

3 The table shows some values for the equation y = x3 O 2x for −2 Y x Y 2 . For
Examiner's
Use
x –2 −1.5 −1 −0.6 −0.3 0 0.3 0.6 1 1.5 2

y –4 –0.38 0.57 –0.57 0.38 4

(a) Complete the table of values. [3]

(b) On the grid below, draw the graph of y = x3 O 2x for −2 Y x Y 2 .


The first two points have been plotted for you.

y
4

x
–2 –1 0 1 2

–1

–2

–3

–4
[4]

© UCLES 2012 0580/41/M/J/12


5

For
(c) (i) On the grid, draw the line y = 0.8 for −2 Y x Y 2 . [1] Examiner's
Use

(ii) Use your graph to solve the equation x3 – 2x = 0.8 .

Answer(c)(ii) x = or x = or x = [3]

(d) By drawing a suitable tangent, work out an estimate for the gradient of the graph of y = x3 O 2x
where x = −1.5.

You must show your working.

Answer(d) [3]

© UCLES 2012 0580/41/M/J/12 [Turn over


6

4 For
D Examiner's
Use

E NOT TO
O SCALE

10 cm

7 cm 77°
B

A, B, C and D lie on a circle, centre O .


AB = 7 cm, BC = 10 cm and angle ABD = 77°.
AOC is a diameter of the circle.

(a) Find angle ABC.

Answer(a) Angle ABC = [1]

(b) Calculate angle ACB and show that it rounds to 35° correct to the nearest degree.

Answer(b)

[2]

(c) Explain why angle ADB = angle ACB.

Answer(c) [1]

© UCLES 2012 0580/41/M/J/12


7

(d) (i) Calculate the length of AD. For


Examiner's
Use

Answer(d)(i) AD = cm [3]

(ii) Calculate the area of triangle ABD.

Answer(d)(ii) cm2 [2]

(e) The area of triangle AED = 12.3 cm2, correct to 3 significant figures.

Use similar triangles to calculate the area of triangle BEC.

Answer(e) cm2 [3]

© UCLES 2012 0580/41/M/J/12 [Turn over


8

5 Felix asked 80 motorists how many hours their journey took that day. For
He used the results to draw a cumulative frequency diagram. Examiner's
Use

Cumulative
frequency
80

70

60

50

40

30

20

10

t
0
1 2 3 4 5 6 7 8
Time (hours)
(a) Find

(i) the median,


Answer(a)(i) h [1]
(ii) the upper quartile,
Answer(a)(ii) h [1]
(iii) the inter-quartile range.

Answer(a)(iii) h [1]

© UCLES 2012 0580/41/M/J/12


9

(b) Find the number of motorists whose journey took more than 5 hours but no more than 7 hours. For
Examiner's
Use
Answer(b) [1]

(c) The frequency table shows some of the information about the 80 journeys.

Time in hours (t) 0ItY2 2ItY3 3ItY4 4ItY5 5ItY6 6ItY8

Frequency 20 25 18

(i) Use the cumulative frequency diagram to complete the table above. [2]

(ii) Calculate an estimate of the mean number of hours the 80 journeys took.

Answer(c)(ii) h [4]

(d) On the grid, draw a histogram to represent the information in your table in part (c).

[5]

© UCLES 2012 0580/41/M/J/12 [Turn over


10

6 (a) A parallelogram has base (2x O 1) metres and height (4x O7) metres. For
The area of the parallelogram is 1 m2. Examiner's
Use

(i) Show that 4x2 O 9x + 3 = 0 .

Answer (a)(i)

[3]

(ii) Solve the equation 4x2 O 9x + 3 = 0 .

Show all your working and give your answers correct to 2 decimal places.

Answer(a)(ii) x = or x = [4]

(iii) Calculate the height of the parallelogram.

Answer(a)(iii) m [1]

© UCLES 2012 0580/41/M/J/12


11

(b) (i) Factorise x2 O 16. For


Examiner's
Use

Answer(b)(i) [1]

2x + 3 x + 40
(ii) Solve the equation + = 2.
x−4 x 2 − 16

Answer(b)(ii) x = [4]

© UCLES 2012 0580/41/M/J/12 [Turn over


12

7 For
y Examiner's
Use
10

4
P
3

x
–5 –4 –3 –2 –1 0 1 2 3 4 5 6 7 8 9
–1
R
–2

–3
Q
–4

–5

–6

(a) Describe fully

(i) the single transformation which maps triangle P onto triangle Q,

Answer(a)(i) [3]

(ii) the single transformation which maps triangle Q onto triangle R,

Answer(a)(ii) [3]

(iii) the single transformation which maps triangle R onto triangle P.

Answer(a)(iii) [3]

© UCLES 2012 0580/41/M/J/12


13

(b) On the grid, draw the image of For


Examiner's
Use
 − 4
(i) triangle P after translation by  , [2]
 − 5

(ii) triangle P after reflection in the line x = −1 . [2]

(c) (i) On the grid, draw the image of triangle P after a stretch, scale factor 2 and the y-axis as the
invariant line. [2]

(ii) Find the matrix which represents this stretch.

 
 
Answer(c)(ii)   [2]
 
 

© UCLES 2012 0580/41/M/J/12 [Turn over


14

8 = {1, 2, 3, 4, 5, 6, 7, 8, 9} For
Examiner's
Use
E = {x : x is an even number}

F = {2, 5, 7}

G = {x : x2 O 13x + 36 = 0}

(a) List the elements of set E.

Answer(a) E = { } [1]

(b) Write down n(F ).

Answer(b) n(F ) = [1]

(c) (i) Factorise x2 O 13x + 36.

Answer(c)(i) [2]

(ii) Using your answer to part (c)(i), solve x2 O 13x + 36 = 0 to find the two elements of G.

Answer(c)(ii) x = or x = [1]

(d) Write all the elements of in their correct place in the Venn diagram.

E
F

[2]

(e) Use set notation to complete the following statements.

(i) F ∩ G = [1]

(ii) 7 E [1]

(iii) n(E F) = 6 [1]

© UCLES 2012 0580/41/M/J/12


15

9 f(x) = 3x + 5 g(x) = 7 O 2x h(x) = x2 O 8 For


Examiner's
Use
(a) Find

(i) f(3),

Answer(a)(i) [1]

(ii) g(x O 3) in terms of x in its simplest form,

Answer(a)(ii) [2]

(iii) h(5x) in terms of x in its simplest form.

Answer(a)(iii) [1]

(b) Find the inverse function g –1(x).

Answer(b) g –1(x) = [2]

(c) Find hf(x) in the form ax2 + bx + c .

Answer(c) hf(x) = [3]

(d) Solve the equation ff(x) = 83.

Answer(d) x = [3]

(e) Solve the inequality 2f(x) I g(x).

Answer(e) [3]

Question 10 is printed on the next page.

© UCLES 2012 0580/41/M/J/12 [Turn over


16

10 For
Examiner's
Use

NOT TO
24 cm SCALE

9 cm

A solid metal cone has base radius 9 cm and vertical height 24 cm.

(a) Calculate the volume of the cone.


1
[The volume, V, of a cone with radius r and height h is V = πr2h.]
3

Answer(a) cm3 [2]


(b)

16 cm NOT TO
SCALE
9 cm

A cone of height 8 cm is removed by cutting parallel to the base, leaving the solid shown above.
Show that the volume of this solid rounds to 1960 cm3, correct to 3 significant figures.

Answer (b)

[4]

(c) The 1960 cm3 of metal in the solid in part (b) is melted and made into 5 identical cylinders,
each of length 15 cm.
Show that the radius of each cylinder rounds to 2.9 cm, correct to 1 decimal place.

Answer (c)

[4]

Permission to reproduce items where third-party owned material protected by copyright is included has been sought and cleared where possible. Every
reasonable effort has been made by the publisher (UCLES) to trace copyright holders, but if any items requiring clearance have unwittingly been included, the
publisher will be pleased to make amends at the earliest possible opportunity.

University of Cambridge International Examinations is part of the Cambridge Assessment Group. Cambridge Assessment is the brand name of University of
Cambridge Local Examinations Syndicate (UCLES), which is itself a department of the University of Cambridge.

© UCLES 2012 0580/41/M/J/12


UNIVERSITY OF CAMBRIDGE INTERNATIONAL EXAMINATIONS
International General Certificate of Secondary Education
*9494404256*

MATHEMATICS 0580/42
Paper 4 (Extended) May/June 2012
2 hours 30 minutes
Candidates answer on the Question Paper.
Additional Materials: Electronic calculator Geometrical instruments
Mathematical tables (optional) Tracing paper (optional)

READ THESE INSTRUCTIONS FIRST

Write your Centre number, candidate number and name on all the work you hand in.
Write in dark blue or black pen.
You may use a pencil for any diagrams or graphs.
Do not use staples, paper clips, highlighters, glue or correction fluid.
DO NOT WRITE IN ANY BARCODES.

Answer all questions.


If working is needed for any question it must be shown below that question.
Electronic calculators should be used.
If the degree of accuracy is not specified in the question, and if the answer is not exact, give the answer to
three significant figures. Give answers in degrees to one decimal place.
For π use either your calculator value or 3.142.

At the end of the examination, fasten all your work securely together.
The number of marks is given in brackets [ ] at the end of each question or part question.
The total of the marks for this paper is 130.

This document consists of 16 printed pages.

IB12 06_0580_42/6RP
© UCLES 2012 [Turn over
2

1 For
Examiner's
Mathematics mark 30 50 35 25 5 39 48 40 10 15 Use

English mark 26 39 35 28 9 37 45 33 16 12

The table shows the test marks in Mathematics and English for 10 students.

(a) (i) On the grid, complete the scatter diagram to show the Mathematics and English marks for
the 10 students. The first four points have been plotted for you.

50

40

30
English
mark
20

10

0
5 10 15 20 25 30 35 40 45 50
Mathematics mark
[2]
(ii) What type of correlation does your scatter diagram show?

Answer(a)(ii) [1]

(iii) Draw a line of best fit on the grid. [1]

(iv) Ann missed the English test but scored 22 marks in the Mathematics test.
Use your line of best fit to estimate a possible English mark for Ann.

Answer(a)(iv) [1]

(b) Show that the mean English mark for the 10 students is 28.

Answer(b)

[2]

(c) Two new students do the English test. They both score the same mark.
The mean English mark for the 12 students is 31.
Calculate the English mark for the new students.

Answer(c) [3]

© UCLES 2012 0580/42/M/J/12


3

2 (a) In a sale, Jen buys a laptop for $351.55. For


This price is 21% less than the price before the sale. Examiner's
Use

Calculate the price before the sale.

Answer(a) $ [3]

(b) Alex invests $4000 at a rate of 8% per year simple interest for 2 years.
Bob invests $4000 at a rate of 7.5% per year compound interest for 2 years.

Who receives more interest and by how much?

Answer(b) receives $ more interest. [6]

© UCLES 2012 0580/42/M/J/12 [Turn over


4

3 Pablo plants x lemon trees and y orange trees. For


Examiner's
Use
(a) (i) He plants at least 4 lemon trees.

Write down an inequality in x to show this information.

Answer(a)(i) [1]

(ii) Pablo plants at least 9 orange trees.

Write down an inequality in y to show this information.

Answer(a)(ii) [1]

(iii) The greatest possible number of trees he can plant is 20.

Write down an inequality in x and y to show this information.

Answer(a)(iii) [1]

(b) Lemon trees cost $5 each and orange trees cost $10 each.

The maximum Pablo can spend is $170.

Write down an inequality in x and y and show that it simplifies to x + 2y Y 34.

Answer (b)

[1]

(c) (i) On the grid opposite, draw four lines to show the four inequalities and shade the unwanted
region.

© UCLES 2012 0580/42/M/J/12


5

y For
Examiner's
24 Use

22

20

18

16

14

12

10

x
0
2 4 6 8 10 12 14 16 18 20 22 24 26 28 30 32 34

[7]

(ii) Calculate the smallest cost when Pablo buys a total of 20 trees.

Answer(c)(ii) $ [2]

© UCLES 2012 0580/42/M/J/12 [Turn over


6

4 (a) For
B Examiner's
Use

A C NOT TO
42° SCALE

D
O

A, B, C, D, E and F are points on the circumference of a circle centre O .


AE is a diameter of the circle.
BC is parallel to AE and angle CAE = 42°.

Giving a reason for each answer, find

(i) angle BCA,

Answer(a)(i) Angle BCA =

Reason [2]

(ii) angle ACE,

Answer(a)(ii) Angle ACE =

Reason [2]

(iii) angle CFE,

Answer(a)(iii) Angle CFE =

Reason [2]

(iv) angle CDE.

Answer(a)(iv) Angle CDE =

Reason [2]

© UCLES 2012 0580/42/M/J/12


7

(b) For
Examiner's
P Use
NOT TO
5 cm
SCALE

O
12 cm
Q

In the diagram, O is the centre of the circle and PQ is a tangent to the circle at P.
OP = 5 cm and OQ = 12 cm.

Calculate PQ.

Answer(b) PQ = cm [3]
(c)
C

B D E

NOT TO
SCALE

G F

In the diagram, ABCD and DEFG are squares.

(i) In the triangles CDG and ADE, explain with a reason which sides and/or angles are equal.

Answer (c)(i)

[3]

(ii) Complete the following statement.

Triangle CDG is to triangle ADE. [1]

© UCLES 2012 0580/42/M/J/12 [Turn over


8

5 (a) In Portugal, Miguel buys a book about planets. For


The book costs €34.95. Examiner's
Use
In England the same book costs £27.50.
The exchange rate is £1 = €1.17.

Calculate the difference in pounds (£) between the cost of the book in Portugal and England.

Answer(a) £ [2]

(b) In the book, the distance between two planets is given as 4.07 × 1012 kilometres.
The speed of light is 1.1 × 109 kilometres per hour.

Calculate the time taken for light to travel from one of these planets to the other.
Give your answer in days and hours.

Answer(b) days hours [3]

(c) In one of the pictures in the book, a rectangle is drawn.


The rectangle has length 9.3 cm and width 5.6 cm, both correct to one decimal place.

(i) What is the lower bound for the length?

Answer(c)(i) cm [1]

(ii) Work out the lower and upper bounds for the area of the rectangle.

Answer(c)(ii) Lower bound = cm2

Upper bound = cm2 [2]

© UCLES 2012 0580/42/M/J/12


9

6 (a) For
Examiner's
114° Use

2x°
NOT TO
SCALE

(x – 10)°

Find the value of x.

Answer(a) x = [3]

(b) (i) Write the four missing terms in the table for sequences A, B, C and D.

Term 1 2 3 4 5 n

Sequence A –4 2 5 8 3n – 7

Sequence B 1 4 9 16 25

Sequence C 5 10 15 20 25

Sequence D 6 14 24 36 50

[4]
(ii) Which term in sequence D is equal to 500?

Answer(b)(ii) [2]

x 2 − 16
(c) Simplify .
2
2x + 7x − 4

Answer(c) [4]

© UCLES 2012 0580/42/M/J/12 [Turn over


Name: __________________________

Class: __________________________ 10

Date: ______________
 
=  3 .
For
7 (a) P is the point (2, 5) and Examiner's
 − 2 Use

Write down the co-ordinates of Q.

Answer(a) ( , ) [1]

(b)
D

C B NOT TO
E
SCALE

c M

O 3a A

O is the origin and OABC is a parallelogram.


M is the midpoint of AB.
1
= c, = 3a and CE = CB.
3

OED is a straight line with OE : ED = 2 : 1 .

Find in terms of a and c, in their simplest forms

(i) ,

Answer(b)(i) = [1]

(ii) the position vector of M,

Answer(b)(ii) [2]

(iii) ,
Answer(b)(iii) = [1]

(iv) .

Answer(b)(iv) = [2]

(c) Write down two facts about the lines CD and OB.

Answer (c)

[2]

© UCLES 2012 0580/42/M/J/12


11

8 In all parts of this question give your answer as a fraction in its lowest terms. For
Examiner's
Use
1
(a) (i) The probability that it will rain today is .
3

What is the probability that it will not rain today?

Answer(a)(i) [1]

2
(ii) If it rains today, the probability that it will rain tomorrow is .
5

1
If it does not rain today, the probability that it will rain tomorrow is .
6

Complete the tree diagram.

Today Tomorrow

Rain
Rain
No rain

Rain
No rain
No rain

[2]

(b) Find the probability that it will rain on at least one of these two days.

Answer(b) [3]

(c) Find the probability that it will rain on only one of these two days.

Answer(c) [3]

© UCLES 2012 0580/42/M/J/12 [Turn over


12

9 For
F Examiner's
Use

E
Scale 1 : 10 000

The diagram is a scale drawing of a park EFGH. The scale is 1 : 10 000.

A statue is to be placed in the park so that it is

• nearer to G than to H

• nearer to HG than to FG

• more than 550 metres from F.

Construct accurately the boundaries of the region R in which the statue can be placed.

Leave in all your construction arcs and shade the region R. [7]

© UCLES 2012 0580/42/M/J/12


13

10 (a) Simplify For


Examiner's
(i) (2x2y3)3, Use

Answer(a)(i) [2]
_ 1

 27 3
(ii)   .
 6
x 

Answer(a)(ii) [3]

(b) Multiply out and simplify.


(3x – 2y)(2x + 5y)

Answer(b) [3]

(c) Make h the subject of

(i) V = πr3 + 2πr2h,

Answer(c)(i) h = [2]

(ii) V = 3h .

Answer(c)(ii) h = [2]

(d) Write as a single fraction in its simplest form.

x 5x 7x
+ –
2 3 4

Answer(d) [2]

© UCLES 2012 0580/42/M/J/12 [Turn over


14

11 (a) Calculate the area of a circle with radius 12 cm. For


Examiner's
Use

Answer(a) cm2 [2]

(b)

22°

7 cm
12 cm
NOT TO
SCALE

A circular cake has radius 12 cm and height 7 cm.


The uniform cross-section of a slice of the cake is a sector with angle 22°.
The top and the curved surface of the slice, shaded in the diagram, are covered with chocolate.

Calculate the area of the slice which is covered with chocolate.

Answer(b) cm2 [5]

© UCLES 2012 0580/42/M/J/12


15

(c) For
A Examiner's
Use
D

NOT TO
31 cm SCALE
100°
50° 50°
B 22 cm C

The frame of a child’s bicycle is made from metal rods.


ABC is an isosceles triangle with base 22 cm and base angles 50°.
Angle ACD = 100° and CD = 31 cm.

Calculate the length AD.

Answer(c) AD = cm [6]

Question 12 is printed on the next page.

© UCLES 2012 0580/42/M/J/12 [Turn over


16

12 (a) The cost of 1 kg of tomatoes is $x and the cost of 1 kg of onions is $y. For
Examiner's
Use
Ian pays a total of $10.70 for 10 kg of tomatoes and 4 kg of onions.

Jao pays a total of $10.10 for 8 kg of tomatoes and 6 kg of onions.

Write down simultaneous equations and solve them to find x and y.

Answer(a) x =

y= [6]

(b) Solve 2x2– 5x – 8 = 0 .

Give your answers correct to 2 decimal places.


Show all your working.

Answer(b) x = or x = [4]

Permission to reproduce items where third-party owned material protected by copyright is included has been sought and cleared where possible. Every
reasonable effort has been made by the publisher (UCLES) to trace copyright holders, but if any items requiring clearance have unwittingly been included, the
publisher will be pleased to make amends at the earliest possible opportunity.

University of Cambridge International Examinations is part of the Cambridge Assessment Group. Cambridge Assessment is the brand name of University of
Cambridge Local Examinations Syndicate (UCLES), which is itself a department of the University of Cambridge.

© UCLES 2012 0580/42/M/J/12


UNIVERSITY OF CAMBRIDGE INTERNATIONAL EXAMINATIONS
International General Certificate of Secondary Education
*8301609714*

MATHEMATICS 0580/43
Paper 4 (Extended) May/June 2012
2 hours 30 minutes
Candidates answer on the Question Paper.
Additional Materials: Electronic calculator Geometrical instruments
Mathematical tables (optional) Tracing paper (optional)

READ THESE INSTRUCTIONS FIRST

Write your Centre number, candidate number and name on all the work you hand in.
Write in dark blue or black pen.
You may use a pencil for any diagrams or graphs.
Do not use staples, paper clips, highlighters, glue or correction fluid.
DO NOT WRITE IN ANY BARCODES.

Answer all questions.


If working is needed for any question it must be shown below that question.
Electronic calculators should be used.
If the degree of accuracy is not specified in the question, and if the answer is not exact, give the answer to
three significant figures. Give answers in degrees to one decimal place.
For π use either your calculator value or 3.142.

At the end of the examination, fasten all your work securely together.
The number of marks is given in brackets [ ] at the end of each question or part question.
The total of the marks for this paper is 130.

This document consists of 19 printed pages and 1 blank page.

IB12 06_0580_43/5RP
© UCLES 2012 [Turn over
2

1 A train travels from Paris to Milan. For


Examiner's
Use
(a) The train departs from Paris at 20 28 and the journey takes 9 hours 10 minutes.

(i) Find the time the train arrives in Milan.

Answer(a)(i) [1]

(ii) The distance between Paris and Milan is 850 km.

Calculate the average speed of the train.

Answer(a)(ii) km/h [2]

(b) The total number of passengers on the train is 640.

(i) 160 passengers have tickets which cost $255 each.


330 passengers have tickets which cost $190 each.
150 passengers have tickets which cost $180 each.

Calculate the mean cost of a ticket.

Answer(b)(i) $ [3]

© UCLES 2012 0580/43/M/J/12


3

(ii) There are men, women and children on the train in the ratio For
Examiner's
Use
men : women : children = 4:3:1.

Show that the number of women on the train is 240.

Answer(b)(ii)

[2]

(iii) 240 is an increase of 60% on the number of women on the train the previous day.

Calculate the number of women on the train the previous day.

Answer(b)(iii) [3]

(c) The length of the train is 210 m.


It passes through a station of length 340 m, at a speed of 180 km/h.

Calculate the number of seconds the train takes to pass completely through the station.

Answer(c) s [3]

© UCLES 2012 0580/43/M/J/12 [Turn over


4

2 For
North Examiner's
Use
D
95°
10 km
40°
NOT TO
A 12 km SCALE
30° C

17 km

The diagram shows straight roads connecting the towns A, B, C and D.

AB = 17 km, AC = 12 km and CD = 10 km.

Angle BAC = 30° and angle ADC = 95°.

(a) Calculate angle CAD.

Answer(a) Angle CAD = [3]

(b) Calculate the distance BC.

Answer(b) BC = km [4]

© UCLES 2012 0580/43/M/J/12


5

(c) The bearing of D from A is 040°. For


Examiner's
Use
Find the bearing of

(i) B from A,

Answer(c)(i) [1]

(ii) A from B.

Answer(c)(ii) [1]

(d) Angle ACB is obtuse.

Calculate angle BCD.

Answer(d) Angle BCD = [4]

© UCLES 2012 0580/43/M/J/12 [Turn over


6

3 For
y Examiner's
Use
11

10

4
Q
3

2
P
1

x
–3 –2 –1 0 1 2 3 4 5 6 7 8 9 10 11 12

 5
(a) Draw the translation of triangle P by   . [2]
 3

(b) Draw the reflection of triangle P in the line x = 6 . [2]

(c) (i) Describe fully the single transformation that maps triangle P onto triangle Q.
Answer(c)(i) [3]

(ii) Find the 2 by 2 matrix which represents the transformation in part(c)(i).

 
 
Answer(c)(ii)   [2]
 
 
(d) (i) Draw the stretch of triangle P with scale factor 3 and the x-axis as the invariant line. [2]

(ii) Find the 2 by 2 matrix which represents a stretch, scale factor 3 and x-axis invariant.

 
 
Answer(d)(ii)   [2]
 
 

© UCLES 2012 0580/43/M/J/12


7

4 (a) In a football league a team is given 3 points for a win, 1 point for a draw and 0 points for a loss. For
Examiner's
Use
The table shows the 20 results for Athletico Cambridge.

Points 3 1 0

Frequency 10 3 7

(i) Find the median and the mode.

Answer(a)(i) Median =

Mode = [3]

(ii) Thomas wants to draw a pie chart using the information in the table.

Calculate the angle of the sector which shows the number of times Athletico Cambridge
were given 1 point.

Answer(a)(ii) [2]

(b) Athletico Cambridge has 20 players.

The table shows information about the heights (h centimetres) of the players.

Height (h cm) 170 I h Y 180 180 I h Y 190 190 I h Y 200

Frequency 5 12 3

Calculate an estimate of the mean height of the players.

Answer(b) cm [4]

© UCLES 2012 0580/43/M/J/12 [Turn over


8

5 For
Examiner's
Use

NOT TO
SCALE

3 cm 3 cm
6 cm

8 cm

The diagram shows two solid spheres of radius 3 cm lying on the base of a cylinder of radius 8 cm.

Liquid is poured into the cylinder until the spheres are just covered.

4
[The volume, V, of a sphere with radius r is V = πr3.]
3

(a) Calculate the volume of liquid in the cylinder in

(i) cm3,

Answer(a)(i) cm3 [4]

(ii) litres.

Answer(a)(ii) litres [1]

© UCLES 2012 0580/43/M/J/12


9

(b) One cubic centimetre of the liquid has a mass of 1.22 grams. For
Examiner's
Use
Calculate the mass of the liquid in the cylinder.

Give your answer in kilograms.

Answer(b) kg [2]

(c) The spheres are removed from the cylinder.

Calculate the new height of the liquid in the cylinder.

Answer(c) cm [2]

© UCLES 2012 0580/43/M/J/12 [Turn over


10

6 For
Examiner's
H C Use

30 150 20

40

= {240 passengers who arrive on a flight in Cyprus}

H = {passengers who are on holiday}

C = {passengers who hire a car}

(a) Write down the number of passengers who

(i) are on holiday,

Answer(a)(i) [1]

(ii) hire a car but are not on holiday.

Answer(a)(ii) [1]

(b) Find the value of n(H ∪ CV ).

Answer(b) [1]

(c) One of the 240 passengers is chosen at random.

Write down the probability that this passenger

(i) hires a car,

Answer(c)(i) [1]

(ii) is on holiday and hires a car.

Answer(c)(ii) [1]

© UCLES 2012 0580/43/M/J/12


11

(d) Give your answers to this part correct to 4 decimal places. For
Examiner's
Use
Two of the 240 passengers are chosen at random.

Find the probability that

(i) they are both on holiday,

Answer(d)(i) [2]

(ii) exactly one of the two passengers is on holiday.

Answer(d)(ii) [3]

(e) Give your answer to this part correct to 4 decimal places.

Two passengers are chosen at random from those on holiday.

Find the probability that they both hire a car.

Answer(e) [3]

© UCLES 2012 0580/43/M/J/12 [Turn over


12

7 f(x) = 2x For
Examiner's
Use
(a) Complete the table.

x 0 0.5 1 1.5 2 2.5 3 3.5 4

f(x) 1.4 2 2.8 4 5.7 8

[3]

(b) Draw the graph of y = f(x) for 0 Y x Y 4 .

y
16

15

14

13

12

11

10

x
0 0.5 1 1.5 2 2.5 3 3.5 4

[4]

© UCLES 2012 0580/43/M/J/12


13

(c) Use your graph to solve the equation 2x = 5 . For


Examiner's
Use

Answer(c) x = [1]

(d) Draw a suitable straight line and use it to solve the equation 2x = 3x.

Answer(d) x = or x = [3]

(e) Draw a suitable tangent and use it to find the co-ordinates of the point on the graph of y = f(x)
where the gradient of the graph is 3.

Answer(e) ( , ) [3]

© UCLES 2012 0580/43/M/J/12 [Turn over


14

8 (a) For
B Examiner's
Use

u° A
Y
C 68° NOT TO
88°
w° SCALE
O


D E

A, B, C, D and E lie on the circle, centre O.


CA and BD intersect at Y.
Angle DCA = 88° and angle CYD = 68°.
Angle BAC = u°, angle AED = v° and reflex angle AOD = w°.

Calculate the values of u, v and w.


Answer(a) u =

v=

w= [4]

(b)
S R

X NOT TO
SCALE
Q

P, Q, R and S lie on the circle. PR and QS intersect at X.


The area of triangle RSX = 1.2 cm2 and PX = 3 SX.

Calculate the area of triangle PQX.

Answer(b) cm2 [2]

© UCLES 2012 0580/43/M/J/12


15

(c) For
F Examiner's
Use

G 4x° J

2x° NOT TO
SCALE
O
H
I

GI is a diameter of the circle.


FGH is a tangent to the circle at G.
J and K also lie on the circle.
Angle JGI = x°, angle FGJ = 4x° and angle KGI = 2x°.

Find

(i) the value of x,

Answer(c)(i) x = [2]

(ii) the size of angle JKG,

Answer(c)(ii) Angle JKG = [2]

(iii) the size of angle GJK.

Answer(c)(iii) Angle GJK = [1]

© UCLES 2012 0580/43/M/J/12 [Turn over


16

1 For
9 f(x) = 1 – 2x g(x) = , x≠0 h(x) = x3 + 1 Examiner's
x Use

(a) Find the value of

(i) gf(2),

Answer(a)(i) [2]

(ii) h(–2).

Answer(a)(ii) [1]

(b) Find fg(x).


Write your answer as a single fraction.

Answer(b) fg(x) = [2]

(c) Find h –1(x) , the inverse of h(x).

Answer(c) h –1(x) = [2]

© UCLES 2012 0580/43/M/J/12


17

(d) Write down which of these sketches shows the graph of each of y = f(x), y = g(x) and y = h(x). For
Examiner's
Use
y y y

0 x 0 x 0 x

Graph A Graph B Graph C

y y y

0 x 0 x 0 x

Graph D Graph E Graph F

Answer(d) y = f(x) Graph

y = g(x) Graph

y = h(x) Graph [3]

(e) k(x) = x5 O=3

Solve the equation k –1 (x) = 2.

Answer(e) x = [2]

© UCLES 2012 0580/43/M/J/12 [Turn over


18

10 (a) Rice costs $x per kilogram. For


Potatoes cost $(x + 1) per kilogram. Examiner's
Use
The total cost of 12 kg of rice and 7 kg of potatoes is $31.70 .

Find the cost of 1 kg of rice.

Answer(a) $ [3]

(b) The cost of a small bottle of juice is $y.


The cost of a large bottle of juice is $(y + 1).
When Catriona spends $36 on small bottles only, she receives 25 more bottles than when she
spends $36 on large bottles only.

(i) Show that 25y2 + 25y O 36 = 0 .

Answer(b)(i)

[3]

(ii) Factorise 25y2 + 25y O 36 .

Answer(b)(ii) [2]

(iii) Solve the equation 25y2 + 25y O 36 = 0 .

Answer(b)(iii) y = or y = [1]

(iv) Find the total cost of 1 small bottle of juice and 1 large bottle of juice.

Answer(b)(iv) $ [1]

© UCLES 2012 0580/43/M/J/12


19

11 For
Examiner's
Use

Diagram 1 Diagram 2 Diagram 3

The diagrams show a sequence of dots and circles.


Each diagram has one dot at the centre and 8 dots on each circle.
The radius of the first circle is 1 unit.
The radius of each new circle is 1 unit greater than the radius of the previous circle.

(a) Complete the table for diagrams 4 and 5.

Diagram 1 2 3 4 5

Number of dots 9 17 25

Area of the largest circle π 4π 9π

Total length of the circumferences of the circles 2π 6π 12π

[4]
(b) (i) Write down, in terms of n, the number of dots in diagram n.

Answer(b)(i) [2]

(ii) Find n, when the number of dots in diagram n is 1097.

Answer(b)(ii) n = [2]

(c) Write down, in terms of n and π, the area of the largest circle in

(i) diagram n,
Answer(c)(i) [1]

(ii) diagram 3n.


Answer(c)(ii) [1]

(d) Find, in terms of n and π, the total length of the circumferences of the circles in diagram n.

Answer(d) [2]

© UCLES 2012 0580/43/M/J/12


20

BLANK PAGE

Permission to reproduce items where third-party owned material protected by copyright is included has been sought and cleared where possible. Every
reasonable effort has been made by the publisher (UCLES) to trace copyright holders, but if any items requiring clearance have unwittingly been included, the
publisher will be pleased to make amends at the earliest possible opportunity.

University of Cambridge International Examinations is part of the Cambridge Assessment Group. Cambridge Assessment is the brand name of University of
Cambridge Local Examinations Syndicate (UCLES), which is itself a department of the University of Cambridge.

© UCLES 2012 0580/43/M/J/12


UNIVERSITY OF CAMBRIDGE INTERNATIONAL EXAMINATIONS
International General Certificate of Secondary Education
*1191404320*

MATHEMATICS 0580/41
Paper 4 (Extended) May/June 2013
2 hours 30 minutes
Candidates answer on the Question Paper.
Additional Materials: Electronic calculator Geometrical instruments
Tracing paper (optional)

READ THESE INSTRUCTIONS FIRST

Write your Centre number, candidate number and name on all the work you hand in.
Write in dark blue or black pen.
You may use a pencil for any diagrams or graphs.
Do not use staples, paper clips, highlighters, glue or correction fluid.
DO NOT WRITE IN ANY BARCODES.

Answer all questions.


If working is needed for any question it must be shown below that question.
Electronic calculators should be used.
If the degree of accuracy is not specified in the question, and if the answer is not exact, give the answer to
three significant figures. Give answers in degrees to one decimal place.
For π, use either your calculator value or 3.142.

At the end of the examination, fasten all your work securely together.
The number of marks is given in brackets [ ] at the end of each question or part question.
The total of the marks for this paper is 130.

This document consists of 19 printed pages and 1 blank page.

IB13 06_0580_41/3RP
© UCLES 2013 [Turn over
2
For
1 (a) One day, Maria took 27 minutes to walk 1.8 km to school. Examiner′s
She left home at 07 48. Use

(i) Write down the time Maria arrived at school.

Answer(a)(i) ............................................... [1]

(ii) Show that Maria’s average walking speed was 4 km/h.

Answer(a)(ii)

[2]

(b) Another day, Maria cycled the 1.8 km to school at an average speed of 15 km/h.

(i) Calculate the percentage increase that 15 km/h is on Maria’s walking speed of 4 km/h.

Answer(b)(i) ........................................... % [3]

(ii) Calculate the percentage decrease that Maria’s cycling time is on her walking time of
27 minutes.

Answer(b)(ii) ........................................... % [3]

© UCLES 2013 0580/41/M/J/13


3
For
(iii) After school, Maria cycled to her friend’s home. Examiner′s
This took 9 minutes, which was 36% of the time Maria takes to walk to her friend’s home. Use

Calculate the time Maria takes to walk to her friend’s home.

Answer(b)(iii) ........................................ min [2]


_____________________________________________________________________________________

© UCLES 2013 0580/41/M/J/13 [Turn over


4
For
2 Examiner′s
2 x
f(x) = 3 – x – x g(x) = 3 Use

(a) Complete the tables of values for f(x) and g(x).

x –1.5 –1 –0.5 0 0.5 1 1.5

f(x) 2.25 3 3.25 2.25 1 –0.75

x –1.5 –1 –0.5 0 0.5 1 1.5

g(x) 0.19 0.58 1.73 3 5.20


[3]

(b) On the grid, draw the graphs of y = f(x) and y = g(x) for –1.5 Ğ x Ğ 1.5 .

x
–1.5 –1 –0.5 0 0.5 1 1.5

–1

–2
[6]

© UCLES 2013 0580/41/M/J/13


5
For
(c) For –1.5 Ğ x Ğ 1.5, use your graphs to solve Examiner′s
Use
(i) f(x) = 0,

Answer(c)(i) x = ............................................... [1]

(ii) g(x) = 4,

Answer(c)(ii) x = ............................................... [1]

(iii) f(x) = g(x).

Answer(c)(iii) x = ............................................... [1]

(d) By drawing a suitable tangent, find an estimate of the gradient of the graph of y = f(x) when x = 0.5.

Answer(d) ............................................... [3]


_____________________________________________________________________________________

© UCLES 2013 0580/41/M/J/13 [Turn over


6
For
3 200 students estimate the mass (m grams) of a coin. Examiner′s
The cumulative frequency diagram shows the results. Use

200

180

160

140

120
Cumulative
frequency

100

80

60

40

20

m
0
1 2 3 4 5 6 7 8 9 10
Mass (grams)

© UCLES 2013 0580/41/M/J/13


7
For
(a) Find Examiner′s
Use
(i) the median,

Answer(a)(i) ............................................ g [1]

(ii) the upper quartile,

Answer(a)(ii) ............................................ g [1]

(iii) the 80th percentile,

Answer(a)(iii) ............................................ g [1]

(iv) the number of students whose estimate is 7 g or less.

Answer(a)(iv) ............................................... [1]

(b) (i) Use the cumulative frequency diagram to complete the frequency table.

Mass (m grams) 0<mĞ2 2<mĞ4 4<mĞ6 6<mĞ8 8 < m Ğ 10

Frequency 40 2
[2]

(ii) A student is chosen at random.


The probability that the student estimates that the mass is greater than M grams is 0.3.

Find the value of M.

Answer(b)(ii) M = ............................................... [2]


_____________________________________________________________________________________

© UCLES 2013 0580/41/M/J/13 [Turn over


8
For
4 Examiner′s
y Use

9
8
7
6
5
4
3
2
Q
1
x
–8 –7 –6 –5 –4 –3 –2 –1 0 1 2 3 4 5 6 7 8
–1
–2
–3
R
–4
–5
–6

(a) Describe fully the single transformation that maps shape Q onto shape R.

Answer(a) ................................................................................................................................. [3]

(b) (i) Draw the image when shape Q is translated by the vector e o .
5
[2]
4
(ii) Draw the image when shape Q is reflected in the line x = 2. [2]

(iii) Draw the image when shape Q is stretched, factor 3, x-axis invariant. [2]

(iv) Find the 2 × 2 matrix that represents a stretch of factor 3, x-axis invariant.

Answer(b)(iv) e o [2]

(c) Describe fully the single transformation represented by the matrix e o.


0 1
1 0

Answer(c) .................................................................................................................................. [2]


_____________________________________________________________________________________

© UCLES 2013 0580/41/M/J/13


9
For
5 Examiner′s
Use
Height (h cm) 150 < h Ğ 160 160 < h Ğ 165 165 < h Ğ 180 180 < h Ğ 190

Frequency 5 9 18 10

The table shows information about the heights of a group of 42 students.

(a) Using mid-interval values, calculate an estimate of the mean height of the students.
Show your working.

Answer(a) ......................................... cm [3]

(b) Write down the interval which contains the lower quartile.

Answer(b) ............................................... [1]

(c) Complete the histogram to show the information in the table.


One column has already been drawn for you.

Frequency
density

0
150 155 160 165 170 175 180 185 190
Height (cm)
[4]
_____________________________________________________________________________________

© UCLES 2013 0580/41/M/J/13 [Turn over


10
For
6 Examiner′s
A
B Use
30° 52°
E
15.7 cm NOT TO
SCALE
16.5 cm

23.4 cm

In the diagram, BCD is a straight line and ABDE is a quadrilateral.


Angle BAC = 90°, angle ABC = 30° and angle CAE = 52°.
AC = 15.7 cm, CE = 16.5 cm and CD = 23.4 cm.

(a) Calculate BC.

Answer(a) BC = ......................................... cm [3]

(b) Use the sine rule to calculate angle AEC.


Show that it rounds to 48.57°, correct to 2 decimal places.

Answer(b)

[3]

© UCLES 2013 0580/41/M/J/13


11
For
(c) (i) Show that angle ECD = 40.6°, correct to 1 decimal place. Examiner′s
Use
Answer(c)(i)

[2]

(ii) Calculate DE.

Answer(c)(ii) DE = ......................................... cm [4]

(d) Calculate the area of the quadrilateral ABDE.

Answer(d) ........................................ cm2 [4]


_____________________________________________________________________________________

© UCLES 2013 0580/41/M/J/13 [Turn over


12
For
7 (a) Examiner′s
A Use

NOT TO
SCALE
(2x + 3) cm
(x + 2) cm

B C

In triangle ABC, AB = (x + 2) cm and AC = (2x + 3) cm.


9
sin ACB =
16
Find the length of BC.

Answer(a) BC = ......................................... cm [6]

(b) A bag contains 7 white beads and 5 red beads.

(i) The mass of a red bead is 2.5 grams more than the mass of a white bead.
The total mass of all the 12 beads is 114.5 grams.

Find the mass of a white bead and the mass of a red bead.

Answer(b)(i) White ............................................ g

Red ............................................ g [5]

© UCLES 2013 0580/41/M/J/13


13
For
(ii) Two beads are taken out of the bag at random, without replacement. Examiner′s
Use
Find the probability that

(a) they are both white,

Answer(b)(ii)(a) ............................................... [2]

(b) one is white and one is red.

Answer(b)(ii)(b) ............................................... [3]


_____________________________________________________________________________________

© UCLES 2013 0580/41/M/J/13 [Turn over


14
For
8 (a) Examiner′s
D Use

84°

NOT TO
SCALE
E x° x° C

110° 110°
A B

In the pentagon ABCDE, angle EAB = angle ABC = 110° and angle CDE = 84°.
Angle BCD = angle DEA = x°.

(i) Calculate the value of x.

Answer(a)(i) x = ........................... [2]

(ii) BC = CD.
Calculate angle CBD.

Answer(a)(ii) Angle CBD = ........................... [1]

(iii) This pentagon also has one line of symmetry.


Calculate angle ADB.

Answer(a)(iii) Angle ADB = ........................... [1]

(b) A, B and C lie on a circle centre O.


Angle AOC = 3y° and angle ABC = (4y + 4)°.

Find the value of y. NOT TO


SCALE
O
3y°

C
A
(4y + 4)°

Answer(b) y = ........................... [4]

© UCLES 2013 0580/41/M/J/13


15
For
(c) Examiner′s
S Use

NOT TO
SCALE

Q
78°
P

In the cyclic quadrilateral PQRS, angle SPQ = 78°.

(i) Write down the geometrical reason why angle QRS = 102°.

Answer(c)(i) ...................................................................................................................... [1]

(ii) Angle PRQ : Angle PRS = 1 : 2.

Calculate angle PQS.

Answer(c)(ii) Angle PQS = ........................... [3]

(d)

NOT TO
7.2 cm 2 SCALE
5 cm2

l cm 6.9 cm

The diagram shows two similar figures.


The areas of the figures are 5 cm2 and 7.2 cm2.
The lengths of the bases are l cm and 6.9 cm.

Calculate the value of l.

Answer(d) l = ........................... [3]


_____________________________________________________________________________________

© UCLES 2013 0580/41/M/J/13 [Turn over


16
For
9 f(x) = x2 + x – 3 g(x) = 2x + 7 h(x) = 2x Examiner′s
Use
(a) Solve the equation f(x) = 0.
Show all your working and give your answers correct to 2 decimal places.

Answer(a) x = ........................ or x = ........................ [4]

(b) fg(x) = px2 + qx + r

Find the values of p, q and r.

Answer(b) p = .....................................

q = .....................................

r = ..................................... [3]

© UCLES 2013 0580/41/M/J/13


17
For
(c) Find g –1(x). Examiner′s
Use

Answer(c) g –1(x) = ............................................... [2]

(d) Find x when h(x) = 0.25.

Answer(d) x = ............................................... [1]

(e) Find hhh(3).


Give your answer in standard form, correct to 4 significant figures.

Answer(e) ............................................... [4]


_____________________________________________________________________________________

© UCLES 2013 0580/41/M/J/13 [Turn over


18
For
10 Examiner′s
Use

Star 1 Star 2 Star 3

The diagrams show a sequence of stars made of lines and dots.

(a) Complete the table for Star 5, Star 7 and Star n.

Star 1 Star 2 Star 3 Star 4 Star 5 Star 7 Star n

Number of lines 10 20 30 40

Number of dots 11 21 31 41
[4]

(b) The sums of the number of dots in two consecutive stars are shown in the table.

Star 1 and Star 2 Star 2 and Star 3 Star 3 and Star 4

32 52 72

Find the sum of the number of dots in

(i) Star 10 and Star 11,

Answer(b)(i) ............................................... [1]

(ii) Star n and Star (n + 1),

Answer(b)(ii) ............................................... [1]

(iii) Star (n + 7) and Star (n + 8).

Answer(b)(iii) ............................................... [1]

© UCLES 2013 0580/41/M/J/13


19
For
(c) The total number of dots in the first n stars is given by the expression 5n2 + 6n . Examiner′s
Use
(i) Show that this expression is correct when n = 3.

Answer(c)(i)

[2]

(ii) Find the total number of dots in the first 10 stars.

Answer(c)(ii) ............................................... [1]

(d) The total number of dots in the first n stars is 5n2 + 6n .


The number of dots in the (n + 1)th star is 10(n + 1) + 1.

Add these two expressions to show that the total number of dots in the first (n + 1) stars is

5(n + 1)2 + 6(n + 1) .

You must show each step of your working.

Answer(d)

[4]
_____________________________________________________________________________________

© UCLES 2013 0580/41/M/J/13 [Turn over


20

BLANK PAGE

Permission to reproduce items where third-party owned material protected by copyright is included has been sought and cleared where possible. Every
reasonable effort has been made by the publisher (UCLES) to trace copyright holders, but if any items requiring clearance have unwittingly been included the
publisher will be pleased to make amends at the earliest possible opportunity.

University of Cambridge International Examinations is part of the Cambridge Assessment Group. Cambridge Assessment is the brand name of University of
Cambridge Local Examinations Syndicate (UCLES), which is itself a department of the University of Cambridge.

© UCLES 2013 0580/41/M/J/13


UNIVERSITY OF CAMBRIDGE INTERNATIONAL EXAMINATIONS
International General Certificate of Secondary Education
*1277865729*

MATHEMATICS 0580/42
Paper 4 (Extended) May/June 2013
2 hours 30 minutes
Candidates answer on the Question Paper.
Additional Materials: Electronic calculator Geometrical instruments
Tracing paper (optional)

READ THESE INSTRUCTIONS FIRST

Write your Centre number, candidate number and name on all the work you hand in.
Write in dark blue or black pen.
You may use a pencil for any diagrams or graphs.
Do not use staples, paper clips, highlighters, glue or correction fluid.
DO NOT WRITE IN ANY BARCODES.

Answer all questions.


If working is needed for any question it must be shown below that question.
Electronic calculators should be used.
If the degree of accuracy is not specified in the question, and if the answer is not exact, give the answer to
three significant figures. Give answers in degrees to one decimal place.
For π, use either your calculator value or 3.142.

At the end of the examination, fasten all your work securely together.
The number of marks is given in brackets [ ] at the end of each question or part question.
The total of the marks for this paper is 130.

This document consists of 19 printed pages and 1 blank page.

IB13 06_0580_42/3RP
© UCLES 2013 [Turn over
2
For
1 A tennis club has 560 members. Examiner′s
Use
(a) The ratio men : women : children = 5 : 6 : 3.

(i) Show that the club has 240 women members.

Answer(a)(i)

[2]

(ii) How many members are children?

Answer(a)(ii) ............................................... [1]

5
(b) of the 240 women members play in a tournament.
8
How many women members do not play in the tournament?

Answer(b) ............................................... [2]

(c) The annual membership fee in 2013 is $198 for each adult and $75 for each child.

(i) Calculate the total amount the 560 members pay in 2013.

Answer(c)(i) $ ............................................... [2]

(ii) The adult fee of $198 in 2013 is 5.6% more than the fee in 2012.

Calculate the adult fee in 2012.

Answer(c)(ii) $ ............................................... [3]

© UCLES 2013 0580/42/M/J/13


3
For
(d) The club buys 36 tennis balls for $9.50 and sells them to members for $0.75 each. Examiner′s
Use
Calculate the percentage profit the club makes.

Answer(d) ........................................... % [3]

(e) A tennis court is a rectangle with length 23.7 m and width 10.9 m, each correct to 1 decimal place.

Calculate the upper and lower bounds of the perimeter of the court.

Answer(e) Upper bound ........................................... m

Lower bound ........................................... m [3]

_____________________________________________________________________________________

© UCLES 2013 0580/42/M/J/13 [Turn over


4
For
2 (a) Examiner′s
y Use

3
Q
2

x
–7 –6 –5 –4 –3 –2 –1 0 1 2 3 4 5 6 7 8
–1

–2

–3

–4

–5
P
–6

–7

–8

(i) Describe fully the single transformation which maps shape P onto shape Q.

Answer(a)(i) ...................................................................................................................... [2]

(ii) On the grid above, draw the image of shape P after reflection in the line y = –1. [2]

(iii) On the grid above, draw the image of shape P under the transformation represented by the
matrix e0 -1 o . [3]
1 0

© UCLES 2013 0580/42/M/J/13


5
For
(b) Examiner′s
y Use

10

3
M L
2

x
–4 –3 –2 –1 0 1 2 3 4 5 6 7 8 9 10 11 12
–1

–2

–3

–4

–5

(i) Describe fully the single transformation which maps shape M onto shape L.

Answer(b)(i) ...................................................................................................................... [3]

(ii) On the grid above, draw the image of shape M after enlargement by scale factor 2,
centre (5, 0). [2]
_____________________________________________________________________________________

© UCLES 2013 0580/42/M/J/13 [Turn over


6
For
3 The table shows some values for the function y = 11x – 2x2 – 12 for 1 Ğ x Ğ 4.5. Examiner′s
Use

x 1 1.5 2 2.5 3 3.5 4 4.5

y –3 2 3 3

(a) Complete the table of values. [3]

(b) On the grid below, draw the graph of y = 11x – 2x2 – 12 for 1 Ğ x Ğ 4.5.

y
4

0 x
0.5 1 1.5 2 2.5 3 3.5 4 4.5

–1

–2

–3
[4]

© UCLES 2013 0580/42/M/J/13


7
For
(c) By drawing a suitable line, use your graph to solve the equation 11x – 2x2 = 11. Examiner′s
Use

Answer(c) x = ......................... or x = ......................... [2]

(d) The line y = mx + 2 is a tangent to the curve y = 11x – 2x2 – 12 at the point P.

By drawing this tangent,

(i) find the co-ordinates of the point P,

Answer(d)(i) (.................... , ....................) [2]

(ii) work out the value of m.

Answer(d)(ii) m = ............................................... [2]


_____________________________________________________________________________________

© UCLES 2013 0580/42/M/J/13 [Turn over


8
For
4 Examiner′s
Use
A
NOT TO
O SCALE
M

B
N

T C

A, B and C lie on the circle centre O, radius 8.5 cm.


AB = BC = 10.7 cm.
OM is perpendicular to AB and ON is perpendicular to BC.

(a) Calculate the area of the circle.

Answer(a) ........................................ cm2 [2]

(b) Write down the length of MB.

Answer(b) ......................................... cm [1]

(c) Calculate angle MOB and show that it rounds to 39° correct to the nearest degree.

Answer(c)

[2]

(d) Using angle MOB = 39°, calculate the length of the major arc AC.

Answer(d) ......................................... cm [3]

(e) The tangents to the circle at A and at C meet at T.

Explain clearly why triangle ATB is congruent to triangle CTB.

Answer(e)

[3]
_____________________________________________________________________________________

© UCLES 2013 0580/42/M/J/13


9
For
5 Paul buys a number of large sacks of fertiliser costing $x each. Examiner′s
Use
He spends $27.

(a) Write down, in terms of x, an expression for the number of large sacks which Paul buys.

Answer(a) ............................................... [1]

(b) Rula buys a number of small sacks of fertiliser.


Each small sack costs $2 less than a large sack.
Rula spends $25.

Write down, in terms of x, an expression for the number of small sacks which Rula buys.

Answer(b) ............................................... [1]

(c) Rula buys 4 more sacks than Paul.


Write down an equation in x and show that it simplifies to 2x2 – 3x – 27 = 0.

Answer(c)

[4]

(d) Solve 2x2 – 3x – 27 = 0.

Answer(d) x = ......................... or x = ......................... [3]

(e) Calculate the number of sacks which Paul buys.

Answer(e) ............................................... [1]


_____________________________________________________________________________________

© UCLES 2013 0580/42/M/J/13 [Turn over


10
For
6 (a) Examiner′s
L Use
15 cm
N

12 cm

NOT TO
21 cm SCALE

The diagram shows triangle LMN with LM = 12 cm, LN = 15 cm and MN = 21 cm.

(i) Calculate angle LMN.


Show that this rounds to 44.4°, correct to 1 decimal place.

Answer(a)(i)

[4]

(ii) Calculate the area of triangle LMN.

Answer(a)(ii) ........................................ cm2 [2]

© UCLES 2013 0580/42/M/J/13


11
For
(b) Examiner′s
Q Use
6.4 cm
82°
NOT TO
P 43° SCALE

The diagram shows triangle PQR with PQ = 6.4 cm, angle PQR = 82° and angle QPR = 43°.

Calculate the length of PR.

Answer(b) PR = ......................................... cm [4]


_____________________________________________________________________________________

© UCLES 2013 0580/42/M/J/13 [Turn over


12
For

A=e o C=e o D=e o


5 2 4 2 9 Examiner′s
7 B = (6 – 4) Use
7 1 3 -1 -3
(a) Calculate the result of each of the following, if possible.

If a calculation is not possible, write “not possible” in the answer space.

(i) 3A

Answer(a)(i) [1]

(ii) AC

Answer(a)(ii) [1]

(iii) BA

Answer(a)(iii) [2]

(iv) C + D

Answer(a)(iv) [1]

(v) D2

Answer(a)(v) [2]

(b) Calculate C–1, the inverse of C.

Answer(b) [2]
_____________________________________________________________________________________

© UCLES 2013 0580/42/M/J/13


13
For
8 In this question, give all your answers as fractions. Examiner′s
5 Use
When Ivan goes to school in winter, the probability that he wears a hat is .
8
2
If he wears a hat, the probability that he wears a scarf is .
3
1
If he does not wear a hat, the probability that he wears a scarf is .
6
(a) Complete the tree diagram.

........ Scarf
Hat
........
No scarf
........

........ Scarf
........ No hat
No scarf
........
[3]
(b) Find the probability that Ivan

(i) does not wear a hat and does not wear a scarf,

Answer(b)(i) ............................................... [2]

(ii) wears a hat but does not wear a scarf,

Answer(b)(ii) ............................................... [2]

(iii) wears a hat or a scarf but not both.

Answer(b)(iii) ............................................... [2]

7
(c) If Ivan wears a hat and a scarf, the probability that he wears gloves is .
10
Calculate the probability that Ivan does not wear all three of hat, scarf and gloves.

Answer(c) ............................................... [3]


_____________________________________________________________________________________

© UCLES 2013 0580/42/M/J/13 [Turn over


14
For
9 (a) Examiner′s
Use
NOT TO
SCALE

12 cm
4 cm

The diagram shows a prism of length 12 cm.


The cross section is a regular hexagon of side 4 cm.

Calculate the total surface area of the prism.

Answer(a) ........................................ cm2 [4]

(b) Water flows through a cylindrical pipe of radius 0.74 cm.


It fills a 12 litre bucket in 4 minutes.

(i) Calculate the speed of the water through the pipe in centimetres per minute.

Answer(b)(i) .................................. cm/min [4]

© UCLES 2013 0580/42/M/J/13


15
For
(ii) When the 12 litre bucket is emptied into a circular pool, the water level rises by 5 millimetres. Examiner′s
Use
Calculate the radius of the pool correct to the nearest centimetre.

Answer(b)(ii) ......................................... cm [5]


_____________________________________________________________________________________

© UCLES 2013 0580/42/M/J/13 [Turn over


16
For
10 (a) Write as a single fraction Examiner′s
Use
5 2x
(i) – ,
4 5

Answer(a)(i) ............................................... [2]

4 2x - 1 .
(ii) +
x+3 3

Answer(a)(ii) ............................................... [3]

(b) Solve the simultaneous equations.


9x – 2y = 12
3x + 4y = –10

Answer(b) x = ...............................................

y = ............................................... [3]

© UCLES 2013 0580/42/M/J/13


17
For
7x + 21 .
(c) Simplify Examiner′s
2x2 + 9x + 9 Use

Answer(c) ............................................... [4]


_____________________________________________________________________________________

© UCLES 2013 0580/42/M/J/13 [Turn over


18
For
11 Sidney draws the triangle OP1 P2. O Examiner′s
OP1 = 3 cm and P1 P2 = 1 cm. Use
NOT TO
Angle OP1 P2 = 90°.
SCALE
3 cm

P1 P2
1 cm

(a) Show that OP2 = 10 cm.

Answer(a)

[1]

(b) Sidney now draws the lines P2 P3 and OP3. O


Triangle OP2 P3 is mathematically similar
NOT TO
to triangle OP1 P2.
SCALE
3 cm

P3
P1
1 cm P2

a
(i) Write down the length of P2 P3 in the form where a and b are integers.
b

Answer(b)(i) P2 P3 = ......................................... cm [1]


c
(ii) Calculate the length of OP3 giving your answer in the form where c and d are integers.
d

Answer(b)(ii) OP3 = ......................................... cm [2]

(c) Sidney continues to add P5


mathematically similar triangles
to his drawing. O
NOT TO
Find the length of OP5. P4
SCALE
3 cm
P3
P1
1 cm P2

Answer(c) OP5 = ......................................... cm [2]

© UCLES 2013 0580/42/M/J/13


19
For
(d) (i) Show that angle P1OP2 = 18.4°, correct to 1 decimal place. Examiner′s
Use
Answer(d)(i)

[2]

(ii) Write down the size of angle P2OP3.

Answer(d)(ii) Angle P2OP3 = ............................................... [1]

(iii) The last triangle Sidney can draw without covering his first triangle is triangle OP(n–1) Pn.

P5

NOT TO
P4
SCALE

P3

P1 P2
P(n–1)

Pn

Calculate the value of n.

Answer(d)(iii) n = ............................................... [3]


_____________________________________________________________________________________

© UCLES 2013 0580/42/M/J/13 [Turn over


20

BLANK PAGE

Permission to reproduce items where third-party owned material protected by copyright is included has been sought and cleared where possible. Every
reasonable effort has been made by the publisher (UCLES) to trace copyright holders, but if any items requiring clearance have unwittingly been included the
publisher will be pleased to make amends at the earliest possible opportunity.

University of Cambridge International Examinations is part of the Cambridge Assessment Group. Cambridge Assessment is the brand name of University of
Cambridge Local Examinations Syndicate (UCLES), which is itself a department of the University of Cambridge.

© UCLES 2013 0580/42/M/J/13


UNIVERSITY OF CAMBRIDGE INTERNATIONAL EXAMINATIONS
International General Certificate of Secondary Education
*9687546924*

MATHEMATICS 0580/43
Paper 4 (Extended) May/June 2013
2 hours 30 minutes
Candidates answer on the Question Paper.
Additional Materials: Electronic calculator Geometrical instruments
Tracing paper (optional)

READ THESE INSTRUCTIONS FIRST

Write your Centre number, candidate number and name on all the work you hand in.
Write in dark blue or black pen.
You may use a pencil for any diagrams or graphs.
Do not use staples, paper clips, highlighters, glue or correction fluid.
DO NOT WRITE IN ANY BARCODES.

Answer all questions.


If working is needed for any question it must be shown below that question.
Electronic calculators should be used.
If the degree of accuracy is not specified in the question, and if the answer is not exact, give the answer to
three significant figures. Give answers in degrees to one decimal place.
For π, use either your calculator value or 3.142.

At the end of the examination, fasten all your work securely together.
The number of marks is given in brackets [ ] at the end of each question or part question.
The total of the marks for this paper is 130.

This document consists of 19 printed pages and 1 blank page.

IB13 06_0580_43/5RP
© UCLES 2013 [Turn over
2
For
1 (a) Ali and Ben receive a sum of money. Examiner′s
They share it in the ratio 5 : 1. Use
Ali receives $2345.

Calculate the total amount.

Answer(a) $ ............................................... [2]

(b) Ali uses 11% of his $2345 to buy a television.

Calculate the cost of the television.

Answer(b) $ ............................................... [2]

(c) A different television costs $330.

(i) Ben buys one in a sale when this cost is reduced by 15%.

How much does Ben pay?

Answer(c)(i) $ ............................................... [2]

(ii) $330 is 12% less than the cost last year.

Calculate the cost last year.

Answer(c)(ii) $ ............................................... [3]

© UCLES 2013 0580/43/M/J/13


3
For
(d) Ali invests $1500 of his share in a bank account. Examiner′s
The account pays compound interest at a rate of 2.3% per year. Use

Calculate the total amount in the account at the end of 3 years.

Answer(d) $ ............................................... [3]

(e) Ali also buys a computer for $325.


He later sells this computer for $250.

Calculate Ali’s percentage loss.

Answer(e) ........................................... % [3]


_____________________________________________________________________________________

© UCLES 2013 0580/43/M/J/13 [Turn over


4
For
2 (a) In this question show all your construction arcs and use only a ruler and compasses to draw Examiner′s
the boundaries of your region. Use

This scale drawing shows the positions of four towns, P, Q, R and S, on a map where 1 cm represents
10 km.

North

Scale: 1 cm to 10 km

A nature reserve lies in the quadrilateral PQRS.


The boundaries of the nature reserve are:

● equidistant from Q and from R


● equidistant from PS and from PQ
● 60 km from R
● along QR.

(i) Shade the region which represents the nature reserve. [7]

(ii) Measure the bearing of S from P.

Answer(a)(ii) ............................................... [1]

© UCLES 2013 0580/43/M/J/13


5
For
(b) A circular lake in the nature reserve has a radius of 45 m. Examiner′s
Use
(i) Calculate the area of the lake.

Answer(b)(i) .......................................... m2 [2]

(ii)

NOT TO
SCALE

A fence is placed along part of the circumference of the lake.


This arc subtends an angle of 210° at the centre of the circle.

Calculate the length of the fence.

Answer(b)(ii) ........................................... m [2]


_____________________________________________________________________________________

© UCLES 2013 0580/43/M/J/13 [Turn over


6
For
3 (a) Luk wants to buy x goats and y sheep. Examiner′s
Use
(i) He wants to buy at least 5 goats.

Write down an inequality in x to represent this condition.

Answer(a)(i) ............................................... [1]

(ii) He wants to buy at least 11 sheep.

Write down an inequality in y to represent this condition.

Answer(a)(ii) ............................................... [1]

(iii) He wants to buy at least 20 animals.

Write down an inequality in x and y to represent this condition.

Answer(a)(iii) ............................................... [1]

(b) Goats cost $4 and sheep cost $8.


The maximum Luk can spend is $160.

Write down an inequality in x and y and show that it simplifies to x + 2y Y 40 .

Answer(b)

[1]

© UCLES 2013 0580/43/M/J/13


7
For
(c) (i) On the grid below, draw four lines to show the four inequalities and shade the unwanted Examiner′s
regions. Use

y
40

35

30

25

20

15

10

0 x
5 10 15 20 25 30 35 40
[7]

(ii) Work out the maximum number of animals that Luk can buy.

Answer(c)(ii) ............................................... [2]


_____________________________________________________________________________________

© UCLES 2013 0580/43/M/J/13 [Turn over


8
For
4 Examiner′s
I Use

NOT TO
SCALE

H
J
F

7 cm 40 cm

E 22 cm G

EFGHIJ is a solid metal prism of length 40 cm.


The cross section EFG is a right-angled triangle.
EF = 7 cm and EG = 22 cm.

(a) Calculate the volume of the prism.

Answer(a) ........................................ cm3 [2]

(b) Calculate the length FJ.

Answer(b) FJ = ......................................... cm [4]

© UCLES 2013 0580/43/M/J/13


9
For
(c) Calculate the angle between FJ and the base EGJH of the prism. Examiner′s
Use

Answer(c) ............................................... [3]

(d) The prism is melted and made into spheres.


Each sphere has a radius 1.5 cm.

Work out the greatest number of spheres that can be made.


4
[The volume, V, of a sphere with radius r is V = πr3.]
3

Answer(d) ............................................... [3]

(e) (i) A right-angled triangle is the cross section of another prism.


This triangle has height 4.5 cm and base 11.0 cm.
Both measurements are correct to 1 decimal place.

Calculate the upper bound for the area of this triangle.

Answer(e)(i) ........................................ cm2 [2]

(ii) Write your answer to part (e)(i) correct to 4 significant figures.

Answer(e)(ii) ........................................ cm2 [1]


_____________________________________________________________________________________

© UCLES 2013 0580/43/M/J/13 [Turn over


10
For
5 (a) Complete this table of values for the function f(x) = 1x – x2, x ¸ 0. Examiner′s
Use

x –3 –2 –1 –0.5 –0.2 0.2 0.5 1 2 3

f(x) –9.33 –4.5 –2 –2.25 4.96 –3.5 –8.67


[3]

(b) Draw the graph of f(x) = 1x – x2 for –3 Y x Y –0.2 and 0.2 Y x Y 3.

y
5

x
–3 –2 –1 0 1 2 3
–1

–2

–3

–4

–5

–6

–7

–8

–9

–10
[5]

© UCLES 2013 0580/43/M/J/13


11
For
(c) Use your graph to solve f(x) = –3. Examiner′s
Use

Answer(c) x = ................ or x = ................ or x = ................ [3]

(d) By drawing a suitable line on your graph, solve the equation f(x) = 2x – 2.

Answer(d) x = ................ or x = ................ or x = ................ [3]

(e) By drawing a suitable tangent, work out an estimate of the gradient of the curve at the point where
x = –2.

You must show your working.

Answer(e) ............................................... [3]


_____________________________________________________________________________________

© UCLES 2013 0580/43/M/J/13 [Turn over


12
For
6 In a box there are 7 red cards and 3 blue cards. Examiner′s
A card is drawn at random from the box and is not replaced. Use
A second card is then drawn at random from the box.

(a) Complete this tree diagram.

First card Second card

........ Red
7 Red
10 Blue
........

........ Red
........ Blue
Blue
........
[3]

(b) Work out the probability that the two cards are of different colours.
Give your answer as a fraction.

Answer(b) ............................................... [3]


_____________________________________________________________________________________

© UCLES 2013 0580/43/M/J/13


13
For
7 Examiner′s
y Use

10

2
A B
1

x
0
1 2 3 4 5 6 7 8

(a) (i) Draw the image of shape A after a stretch, factor 3, x-axis invariant. [2]

(ii) Write down the matrix representing a stretch, factor 3, x-axis invariant.

Answer(a)(ii) e o [2]

(b) (i) Describe fully the single transformation which maps shape A onto shape B.

Answer(b)(i) ...................................................................................................................... [3]

(ii) Write down the matrix representing the transformation which maps shape A onto shape B.

Answer(b)(ii) e o [2]
_____________________________________________________________________________________

© UCLES 2013 0580/43/M/J/13 [Turn over


14
For
8 (a) Examiner′s
Use

27°

C NOT TO
A SCALE
O

A, B, C, D and E are points on the circle centre O.


Angle ABD = 27°.

Find

(i) angle ACD,

Answer(a)(i) Angle ACD = ............................................... [1]

(ii) angle AOD,

Answer(a)(ii) Angle AOD = ............................................... [1]

(iii) angle AED.

Answer(a)(iii) Angle AED = ............................................... [1]

(b)
M

L
67° NOT TO
100° SCALE
45 cm
32 cm

The diagram shows quadrilateral KLMN.


KL = 45 cm, LN = 32 cm, angle KLN = 100° and angle NLM = 67°.

© UCLES 2013 0580/43/M/J/13


15
For
(i) Calculate the length KN. Examiner′s
Use

Answer(b)(i) KN = ......................................... cm [4]

(ii) The area of triangle LMN is 324 cm2.

Calculate the length LM.

Answer(b)(ii) LM = ......................................... cm [3]

(iii) Another triangle XYZ is mathematically similar to triangle LMN.

L Y
X NOT TO
SCALE

Z
N

XZ = 16 cm and the area of triangle LMN is 324 cm2.

Calculate the area of triangle XYZ.

Answer(b)(iii) ........................................ cm2 [2]


_____________________________________________________________________________________

© UCLES 2013 0580/43/M/J/13 [Turn over


16
For
9 Sam asked 80 people how many minutes their journey to work took on one day. Examiner′s
The cumulative frequency diagram shows the times taken (m minutes). Use

80

70

60

50

Cumulative
frequency 40

30

20

10

0 m
10 20 30 40 50
Time (minutes)

(a) Find

(i) the median,


Answer(a)(i) ........................................ min [1]

(ii) the lower quartile,


Answer(a)(ii) ........................................ min [1]

(iii) the inter-quartile range.


Answer(a)(iii) ........................................ min [1]

© UCLES 2013 0580/43/M/J/13


17
For
(b) One of the 80 people is chosen at random. Examiner′s
Use
Find the probability that their journey to work took more than 35 minutes.
Give your answer as a fraction.

Answer(b) ............................................... [2]

(c) Use the cumulative frequency diagram to complete this frequency table.

Time (m minutes) 0 < m Y 10 10 < m Y 15 15 < m Y 30 30 < m Y 40 40 < m Y 50

Frequency 30 12 18
[2]

(d) Using mid-interval values, calculate an estimate of the mean journey time for the 80 people.

Answer(d) ........................................ min [3]

(e) Use the table in part (c) to complete the histogram to show the times taken by the 80 people.
One column has already been completed for you.

Frequency
2
density

m
0
10 20 30 40 50
Time (minutes)
[5]
_____________________________________________________________________________________

© UCLES 2013 0580/43/M/J/13 [Turn over


18
For
10 (a) (i) Solve 2(3x – 7) = 13. Examiner′s
Use

Answer(a)(i) x = ............................................... [3]

(ii) Solve by factorising x2 – 7x + 6 = 0.

Answer(a)(ii) x = ................. or x = ................. [3]

3x - 2 x 2
(iii) Solve + + = 4.
5 10

Answer(a)(iii) x = ............................................... [4]

© UCLES 2013 0580/43/M/J/13


19
For
(b) 12 =1 Examiner′s
Use
12 + 22 =5

12 + 22 + 32 = 14

12 + 22 + 32 + 42 = 30

n
12 + 22 + 32 + 42 + ..................... + n2 = an3 + bn2 +
6
Work out the values of a and b.

Answer(b) a = ...............................................

b = ............................................... [6]
_____________________________________________________________________________________

© UCLES 2013 0580/43/M/J/13 [Turn over


20

BLANK PAGE

Permission to reproduce items where third-party owned material protected by copyright is included has been sought and cleared where possible. Every
reasonable effort has been made by the publisher (UCLES) to trace copyright holders, but if any items requiring clearance have unwittingly been included the
publisher will be pleased to make amends at the earliest possible opportunity.

University of Cambridge International Examinations is part of the Cambridge Assessment Group. Cambridge Assessment is the brand name of University of
Cambridge Local Examinations Syndicate (UCLES), which is itself a department of the University of Cambridge.

© UCLES 2013 0580/43/M/J/13


Cambridge International Examinations
Cambridge International General Certificate of Secondary Education
*3415255704*

MATHEMATICS 0580/41
Paper 4 (Extended) May/June 2014
2 hours 30 minutes
Candidates answer on the Question Paper.
Additional Materials: Electronic calculator Geometrical instruments
Tracing paper (optional)

READ THESE INSTRUCTIONS FIRST

Write your Centre number, candidate number and name on all the work you hand in.
Write in dark blue or black pen.
You may use an HB pencil for any diagrams or graphs.
Do not use staples, paper clips, glue or correction fluid.
DO NOT WRITE IN ANY BARCODES.

Answer all questions.


If working is needed for any question it must be shown below that question.
Electronic calculators should be used.
If the degree of accuracy is not specified in the question, and if the answer is not exact, give the answer to
three significant figures. Give answers in degrees to one decimal place.
For π, use either your calculator value or 3.142.

At the end of the examination, fasten all your work securely together.
The number of marks is given in brackets [ ] at the end of each question or part question.
The total of the marks for this paper is 130.

The syllabus is approved for use in England, Wales and Northern Ireland as a Cambridge International Level 1/Level 2 Certificate.

This document consists of 19 printed pages and 1 blank page.

IB14 06_0580_41/4RP
© UCLES 2014 [Turn over
2

A= f p C= e o D= f p
3 2 -2 2 0
1 B = (–2 5)
-1 1 5 0 2

(a) Work out, when possible, each of the following.


If it is not possible, write ‘not possible’ in the answer space.

(i) 2A

Answer(a)(i) [1]

(ii) B + C

Answer(a)(ii) [1]

(iii) AD

Answer(a)(iii) [2]

(iv) A–1, the inverse of A.

Answer(a)(iv) [2]

(b) Explain why it is not possible to work out CD.

Answer(b) ........................................................................................................................................... [1]

(c) Describe fully the single transformation represented by the matrix D.

Answer(c) ............................................................................................................................................

............................................................................................................................................................. [3]
__________________________________________________________________________________________

© UCLES 2014 0580/41/M/J/14


3

2 Ali leaves home at 10 00 to cycle to his grandmother’s house. He arrives at 13 00.


The distance-time graph represents his journey.

40

30
Distance from
home (km)

20

10

0
10 00 11 00 12 00 13 00 14 00 15 00 16 00 17 00
Time

(a) Calculate Ali’s speed between 10 00 and 11 30.


Give your answer in kilometres per hour.

Answer(a) ...................................... km/h [2]

(b) Show that Ali’s average speed for the whole journey to his grandmother’s house is 12 km/h.

Answer(b)

[2]

(c) Change 12 kilometres per hour into metres per minute.

Answer(c) ..................................... m/min [2]

(d) Ali stays for 45 minutes at his grandmother’s house and then returns home.
He arrives home at 16 42.

Complete the distance-time graph. [2]


__________________________________________________________________________________________

© UCLES 2014 0580/41/M/J/14 [Turn over


4

3 (a) The running costs for a papermill are $75 246.


This amount is divided in the ratio labour costs : materials = 5 : 1.

Calculate the labour costs.

Answer(a) $ ................................................ [2]

(b) In 2012 the company made a profit of $135 890.


In 2013 the profit was $150 675.

Calculate the percentage increase in the profit from 2012 to 2013.

Answer(b) ............................................ % [3]

(c) The profit of $135 890 in 2012 was an increase of 7% on the profit in 2011.

Calculate the profit in 2011.

Answer(c) $ ................................................ [3]

(d)
2 cm
NOT TO
SCALE

21 cm

30 cm

Paper is sold in cylindrical rolls.


There is a wooden cylinder of radius 2 cm and height 21 cm in the centre of each roll.
The outer radius of a roll of paper is 30 cm.

(i) Calculate the volume of paper in a roll.

Answer(d)(i) ......................................... cm3 [3]

© UCLES 2014 0580/41/M/J/14


5

(ii) The paper is cut into sheets which measure 21 cm by 29.7 cm.
The thickness of each sheet is 0.125 mm.

(a) Change 0.125 millimetres into centimetres.

Answer(d)(ii)(a) .......................................... cm [1]

(b) Work out how many whole sheets of paper can be cut from a roll.

Answer(d)(ii)(b) ................................................ [4]


__________________________________________________________________________________________

© UCLES 2014 0580/41/M/J/14 [Turn over


6

T B

11 9

x
6–x

In the Venn diagram, = {children in a nursery}

B = {children who received a book for their birthday}


T = {children who received a toy for their birthday}
P = {children who received a puzzle for their birthday}

x children received a book and a toy and a puzzle.


6 children received a toy and a puzzle.

(a) 4 children received a book and a toy.


5 children received a book and a puzzle.
7 children received a puzzle but not a book and not a toy.

Complete the Venn diagram above. [3]

(b) There are 40 children in the nursery.

Using the Venn diagram, write down and solve an equation in x.

Answer(b)

[3]

© UCLES 2014 0580/41/M/J/14


7

(c) Work out

(i) the probability that a child, chosen at random, received a book but not a toy and not a puzzle,

Answer(c)(i) ................................................ [1]

(ii) the number of children who received a book and a puzzle but not a toy,

Answer(c)(ii) ................................................ [1]

(iii) n(B),

Answer(c)(iii) ................................................ [1]

(iv) n(B ∪ P),

Answer(c)(iv) ................................................ [1]

(v) n(B ∪ T ∪ P)'.

Answer(c)(v) ................................................ [1]

(d)

T B

Shade the region B ∩ (T ∪ P)'. [1]


__________________________________________________________________________________________

© UCLES 2014 0580/41/M/J/14 [Turn over


8

5
S

North

Scale: 2 cm to 3 km

In the scale drawing, P is a port, L is a lighthouse and S is a ship.


The scale is 2 centimetres represents 3 kilometres.

(a) Measure the bearing of S from P.

Answer(a) ................................................ [1]

(b) Find the actual distance of S from L.

Answer(b) .......................................... km [2]

(c) The bearing of L from S is 160°.

Calculate the bearing of S from L.

Answer(c) ................................................ [1]

© UCLES 2014 0580/41/M/J/14


9

(d) Work out the scale of the map in the form 1 : n.

Answer(d) 1 : ................................................ [2]

(e) A boat B is

● equidistant from S and L


and
● equidistant from the lines PS and SL.

On the diagram, using a straight edge and compasses only, construct the position of B. [5]

(f) The lighthouse stands on an island of area 1.5 cm2 on the scale drawing.

Work out the actual area of the island.

Answer(f) ......................................... km2 [2]


__________________________________________________________________________________________

© UCLES 2014 0580/41/M/J/14 [Turn over


10

6 (a) A square spinner is biased.


The probabilities of obtaining the scores 1, 2, 3 and 4 when it is spun are given in the table.

Score 1 2 3 4

Probability 0.1 0.2 0.4 0.3

(i) Work out the probability that on one spin the score is 2 or 3.

Answer(a)(i) ................................................ [2]

(ii) In 5000 spins, how many times would you expect to score 4 with this spinner?

Answer(a)(ii) ................................................ [1]

(iii) Work out the probability of scoring 1 on the first spin and 4 on the second spin.

Answer(a)(iii) ................................................ [2]

(b) In a bag there are 7 red discs and 5 blue discs.


From the bag a disc is chosen at random and not replaced.
A second disc is then chosen at random.

Work out the probability that at least one of the discs is red.
Give your answer as a fraction.

Answer(b) ................................................ [3]


__________________________________________________________________________________________

© UCLES 2014 0580/41/M/J/14


11

7
y
4

3
A
2

x
–6 –5 –4 –3 –2 –1 0 1 2 3 4 5 6
–1

–2

–3

–4

–5

(a) On the grid,

(i) draw the image of shape A after a translation by the vector e o ,


-5
[2]
-4
(ii) draw the image of shape A after a rotation through 90° clockwise about the origin. [2]

(b) (i) On the grid, draw the image of shape A after the transformation represented by the matrix f p.
2 0
0 1

[3]

(ii) Describe fully the single transformation represented by the matrix f p.


2 0
0 1
Answer(b)(ii) ...............................................................................................................................

..................................................................................................................................................... [3]
__________________________________________________________________________________________

© UCLES 2014 0580/41/M/J/14 [Turn over


12

8 (a) Complete the table of values for y = x3 – 3x + 1 .

x –2.5 –2 –1.5 –1 –0.5 0 0.5 1 1.5 2 2.5

y –7.125 –1 3 1 –0.375 –1 –0.125 3 9.125

[2]

(b) Draw the graph of y = x3 – 3x + 1 for –2.5 Ğ x Ğ 2.5 .

y
10

x
–3 –2 –1 0 1 2 3
–1

–2

–3

–4

–5

–6

–7

–8
[4]

© UCLES 2014 0580/41/M/J/14


13

(c) By drawing a suitable tangent, estimate the gradient of the curve at the point where x = 2.

Answer(c) ................................................ [3]

(d) Use your graph to solve the equation x3 – 3x + 1 = 1 .

Answer(d) x = ..................... or x = ..................... or x = ..................... [2]

(e) Use your graph to complete the inequality in k for which the equation

x3 – 3x + 1 = k has three different solutions.

Answer(e) ........................ < k < ........................ [2]


__________________________________________________________________________________________

© UCLES 2014 0580/41/M/J/14 [Turn over


14

9
80

70

60

50
Cumulative
frequency
40

30

20

10

t
0
10 20 30 40 50
Time (minutes)

The times (t minutes) taken by 80 people to complete a charity swim were recorded.
The results are shown in the cumulative frequency diagram above.

(a) Find

(i) the median,

Answer(a)(i) ......................................... min [1]

(ii) the inter-quartile range,

Answer(a)(ii) ......................................... min [2]

© UCLES 2014 0580/41/M/J/14


15

(iii) the 70th percentile.

Answer(a)(iii) ......................................... min [2]

(b) The times taken by the 80 people are shown in this grouped frequency table.

Time (t minutes) 0 < t Ğ 20 20 < t Ğ 30 30 < t Ğ 45 45 < t Ğ 50

Frequency 12 21 33 14

(i) Calculate an estimate of the mean time.

Answer(b)(i) ......................................... min [4]

(ii) Draw a histogram to represent the grouped frequency table.

Frequency
density
2

t
0
10 20 30 40 50
Time (minutes)
[4]
__________________________________________________________________________________________

© UCLES 2014 0580/41/M/J/14 [Turn over


16

1
10 (a) f(x) = 2x – 3 g(x) = +2 h(x) = 3x
x +1
(i) Work out f(4).

Answer(a)(i) ................................................ [1]

(ii) Work out fh(–1).

Answer(a)(ii) ................................................ [2]

(iii) Find f –1(x), the inverse of f(x).

Answer(a)(iii) f –1(x) = ................................................ [2]

(iv) Find ff(x) in its simplest form.

Answer(a)(iv) ff(x) = ................................................ [2]

© UCLES 2014 0580/41/M/J/14


17

(v) Show that the equation f(x) = g(x) simplifies to 2x2 – 3x – 6 = 0 .

Answer(a)(v)

[3]

(vi) Solve the equation 2x2 – 3x – 6 = 0 .

Give your answers correct to 2 decimal places.


Show all your working.

Answer(a)(vi) x = ..................... or x = ..................... [4]


x2 - 3x + 2
(b) Simplify .
x2 + 3x - 10

Answer(b) ................................................ [4]


__________________________________________________________________________________________

© UCLES 2014 0580/41/M/J/14 [Turn over


18

=e o
-3
11 (a)
4
(i) P is the point (–2, 3).

Work out the co-ordinates of Q.

Answer(a)(i) (............. , .............) [1]

(ii) Work out  , the magnitude of .

Answer(a)(ii) ................................................ [2]

© UCLES 2014 0580/41/M/J/14


19

(b)
C

Y
NOT TO
A SCALE

N
a
B

b
O

OACB is a parallelogram.
= a and = b.
2
AN : NB = 2 : 3 and AY = 5 AC.

(i) Write each of the following in terms of a and/or b.


Give your answers in their simplest form.

(a)

Answer(b)(i)(a) = ................................................ [2]

(b)

Answer(b)(i)(b) = ................................................ [2]

(ii) Write down two conclusions you can make about the line segments NY and BC.

Answer(b)(ii) ...............................................................................................................................

..................................................................................................................................................... [2]
__________________________________________________________________________________________

© UCLES 2014 0580/41/M/J/14 [Turn over


20

BLANK PAGE

Permission to reproduce items where third-party owned material protected by copyright is included has been sought and cleared where possible. Every
reasonable effort has been made by the publisher (UCLES) to trace copyright holders, but if any items requiring clearance have unwittingly been included the
publisher will be pleased to make amends at the earliest possible opportunity.

Cambridge International Examinations is part of the Cambridge Assessment Group. Cambridge Assessment is the brand name of University of Cambridge Local
Examinations Syndicate (UCLES), which is itself a department of the University of Cambridge.

© UCLES 2014 0580/41/M/J/14


Cambridge International Examinations
Cambridge International General Certificate of Secondary Education
*0048847567*

MATHEMATICS 0580/42
Paper 4 (Extended) May/June 2014
2 hours 30 minutes
Candidates answer on the Question Paper.
Additional Materials: Electronic calculator Geometrical instruments
Tracing paper (optional)

READ THESE INSTRUCTIONS FIRST

Write your Centre number, candidate number and name on all the work you hand in.
Write in dark blue or black pen.
You may use an HB pencil for any diagrams or graphs.
Do not use staples, paper clips, glue or correction fluid.
DO NOT WRITE IN ANY BARCODES.

Answer all questions.


If working is needed for any question it must be shown below that question.
Electronic calculators should be used.
If the degree of accuracy is not specified in the question, and if the answer is not exact, give the answer to
three significant figures. Give answers in degrees to one decimal place.
For π, use either your calculator value or 3.142.

At the end of the examination, fasten all your work securely together.
The number of marks is given in brackets [ ] at the end of each question or part question.
The total of the marks for this paper is 130.

The syllabus is approved for use in England, Wales and Northern Ireland as a Cambridge International Level 1/Level 2 Certificate.

This document consists of 16 printed pages.

IB14 06_0580_42/2RP
© UCLES 2014 [Turn over
2

1 Jane and Kate share $240 in the ratio 5 : 7 .

(a) Show that Kate receives $140.

Answer(a)

[2]

(b) Jane and Kate each spend $20.

Find the new ratio Jane’s remaining money : Kate’s remaining money.
Give your answer in its simplest form.

Answer(b) ....................... : ....................... [2]

(c) Kate invests $120 for 5 years at 4% per year simple interest.

Calculate the total amount Kate has after 5 years.

Answer(c) $ ................................................ [3]

(d) Jane invests $80 for 3 years at 4% per year compound interest.

Calculate the total amount Jane has after 3 years.


Give your answer correct to the nearest cent.

Answer(d) $ ................................................ [3]

(e) An investment of $200 for 2 years at 4% per year compound interest is the same as an investment of
$200 for 2 years at r % per year simple interest.

Find the value of r.

Answer(e) r = ................................................ [3]


__________________________________________________________________________________________

© UCLES 2014 0580/42/M/J/14


3

1
2 f(x) = – 2x , x ≠ 0
x2
(a) Complete the table of values for f(x).

x –3 –2.5 –2 –1.5 –1 –0.5 0.4 0.5 1 1.5 2

f(x) 6.1 5.2 4.3 3.4 5 5.5 –2.6 –3.8


[3]

(b) On the grid, draw the graph of y = f(x) for –3 Y x Y – 0.5 and 0.4 Y x Y 2 .

y
7

x
–3 –2 –1 0 1 2
–1

–2

–3

–4
[5]

(c) Solve the equation f(x) = 2 .

Answer(c) x = ................................................ [1]

(d) Solve the equation f(x) = 2x + 3 .

Answer(d) x = ................................................ [3]

(e) (i) Draw the tangent to the graph of y = f(x) at the point where x = –1.5 . [1]

(ii) Use the tangent to estimate the gradient of the graph of y = f(x) where x = –1.5 .

Answer(e)(ii) ................................................ [2]


__________________________________________________________________________________________

© UCLES 2014 0580/42/M/J/14 [Turn over


4

3
C
90 m
D

NOT TO
80 m SCALE
95 m

49°
A
55°
B

The diagram shows a quadrilateral ABCD.


Angle BAD = 49° and angle ABD = 55°.
BD = 80 m, BC = 95 m and CD = 90 m.

(a) Use the sine rule to calculate the length of AD.

Answer(a) AD = ............................................ m [3]

(b) Use the cosine rule to calculate angle BCD.

Answer(b) Angle BCD = ................................................ [4]

© UCLES 2014 0580/42/M/J/14


5

(c) Calculate the area of the quadrilateral ABCD.

Answer(c) ........................................... m2 [3]

(d) The quadrilateral represents a field.


Corn seeds are sown across the whole field at a cost of $3250 per hectare.

Calculate the cost of the corn seeds used.


1 hectare = 10 000 m2

Answer(d) $ ................................................ [3]


__________________________________________________________________________________________

© UCLES 2014 0580/42/M/J/14 [Turn over


6

4
y
8

3
Q
2

x
–8 –7 –6 –5 –4 –3 –2 –1 0 1 2 3 4 5 6 7 8
–1

–2

–3

–4

–5

–6

–7

–8

(a) Draw the reflection of shape Q in the line x = –1 . [2]

(b) (i) Draw the enlargement of shape Q, centre (0, 0), scale factor –2 . [2]

(ii) Find the 2 × 2 matrix that represents an enlargement, centre (0, 0), scale factor –2 .

Answer(b)(ii) f p [2]

© UCLES 2014 0580/42/M/J/14


7

(c) (i) Draw the stretch of shape Q, factor 2, x-axis invariant. [2]

(ii) Find the 2 × 2 matrix that represents a stretch, factor 2, x-axis invariant.

Answer(c)(ii) f p [2]

(iii) Find the inverse of the matrix in part (c)(ii).

Answer(c)(iii) f p [2]

(iv) Describe fully the single transformation represented by the matrix in part (c)(iii).

Answer(c)(iv) ..............................................................................................................................

..................................................................................................................................................... [3]
__________________________________________________________________________________________

© UCLES 2014 0580/42/M/J/14 [Turn over


8

5
8 cm

12 cm NOT TO
SCALE

10 cm

4 cm

The diagram shows a cylinder with radius 8 cm and height 12 cm which is full of water.
A pipe connects the cylinder to a cone.
The cone has radius 4 cm and height 10 cm.

(a) (i) Calculate the volume of water in the cylinder.


Show that it rounds to 2410 cm3 correct to 3 significant figures.

Answer(a)(i)

[2]

(ii) Change 2410 cm3 into litres.

Answer(a)(ii) ....................................... litres [1]

© UCLES 2014 0580/42/M/J/14


9

(b) Water flows from the cylinder along the pipe into the cone at a rate of 2 cm3 per second.

Calculate the time taken to fill the empty cone.


Give your answer in minutes and seconds correct to the nearest second.
1
[The volume, V, of a cone with radius r and height h is V = 3 πr 2h.]

Answer(b) .................. min .................. s [4]

(c) Find the number of empty cones which can be filled completely from the full cylinder.

Answer(c) ................................................ [3]


__________________________________________________________________________________________

© UCLES 2014 0580/42/M/J/14 [Turn over


10

NOT TO
S
SCALE
21°
R
117°
T
y° Q

(a) The chords PR and SQ of the circle intersect at T.


Angle RST = 21° and angle STR = 117°.

(i) Find the values of x and y.

Answer(a)(i) x = ................................................

y = ................................................ [2]

(ii) SR = 8.23 cm, RT = 3.31 cm and PQ = 9.43 cm.

Calculate the length of TQ.

Answer(a)(ii) TQ = .......................................... cm [2]

© UCLES 2014 0580/42/M/J/14


11

(b) EFGH is a cyclic quadrilateral.


G H
EF is a diameter of the circle. NOT TO
KE is the tangent to the circle at E. SCALE
GH is parallel to FE and angle KEG = 115°.

F 115° E

Calculate angle GEH. K

Answer(b) Angle GEH = ................................................ [4]

(c) A, B, C and D are points on the circle centre O.


C
Angle AOB = 140° and angle OAC = 14°.
AD = DC.
D NOT TO
SCALE
O
14° 140°

A B

Calculate angle ACD.

Answer(c) Angle ACD = ................................................ [5]


__________________________________________________________________________________________

© UCLES 2014 0580/42/M/J/14 [Turn over


12

7 (a)
1.0

0.8

Frequency 0.6
density
0.4

0.2

m
0
10 20 30 40 50 60 70 80 90 100
Mass (grams)

The histogram shows some information about the masses (m grams) of 39 apples.

(i) Show that there are 12 apples in the interval 70 < m Y 100 .

Answer(a)(i)

[1]

(ii) Calculate an estimate of the mean mass of the 39 apples.

Answer(a)(ii) ............................................. g [5]

(b) The mean mass of 20 oranges is 70 g.


One orange is eaten.
The mean mass of the remaining oranges is 70.5 g.

Find the mass of the orange that was eaten.

Answer(b) ............................................. g [3]


__________________________________________________________________________________________

© UCLES 2014 0580/42/M/J/14


13

8 The distance a train travels on a journey is 600 km.

(a) Write down an expression, in terms of x, for the average speed of the train when

(i) the journey takes x hours,

Answer(a)(i) ....................................... km/h [1]

(ii) the journey takes (x + 1) hours.

Answer(a)(ii) ....................................... km/h [1]

(b) The difference between the average speeds in part(a)(i) and part(a)(ii) is 20 km/h.

(i) Show that x 2 + x – 30 = 0 .

Answer(b)(i)

[3]

(ii) Find the average speed of the train for the journey in part(a)(ii).
Show all your working.

Answer(b)(ii) ....................................... km/h [4]


__________________________________________________________________________________________

© UCLES 2014 0580/42/M/J/14 [Turn over


14

1
9 If the weather is fine the probability that Carlos is late arriving at school is 10 .
1
If the weather is not fine the probability that he is late arriving at school is 3 .
3
The probability that the weather is fine on any day is 4 .

(a) Complete the tree diagram to show this information.

Weather Arriving at school

1
10 Late

3 Fine
4 Not late
........

........ Late
........ Not fine
Not late
........
[3]

(b) In a school term of 60 days, find the number of days the weather is expected to be fine.

Answer(b) ................................................ [1]

(c) Find the probability that the weather is fine and Carlos is late arriving at school.

Answer(c) ................................................ [2]

(d) Find the probability that Carlos is not late arriving at school.

Answer(d) ................................................ [3]

(e) Find the probability that the weather is not fine on at least one day in a school week of 5 days.

Answer(e) ................................................ [2]


__________________________________________________________________________________________

© UCLES 2014 0580/42/M/J/14


15

1
10 f(x) = x , x ≠ 0 g(x) = 1 – x h(x) = x 2 + 1

(a) Find fg` 2 j .


1

Answer(a) ................................................ [2]

(b) Find g–1(x), the inverse of g(x).

Answer(b) g–1(x) = ................................................ [1]

(c) Find hg(x), giving your answer in its simplest form.

Answer(c) hg(x) = ................................................ [3]

(d) Find the value of x when g(x) = 7 .

Answer(d) x = ................................................ [1]

(e) Solve the equation h(x) = 3x.


Show your working and give your answers correct to 2 decimal places.

Answer(e) x = ......................... or x = ......................... [4]

(f) A function k(x) is its own inverse when k –1(x) = k(x).

For which of the functions f(x) , g(x) and h(x) is this true?

Answer(f) ................................................ [1]


__________________________________________________________________________________________

Question 11 is printed on the next page.

© UCLES 2014 0580/42/M/J/14 [Turn over


16

11 The total area of each of the following shapes is X.


The area of the shaded part of each shape is kX.

For each shape, find the value of k and write your answer below each diagram.

NOT TO NOT TO
SCALE J SCALE
NOT TO F
SCALE I
O 72°

K
G
A B C D
H
AB = BC = CD Angle JOK = 72° EF = FG and EI = IH

k = ..................................... k = ..................................... k = .....................................

A
NOT TO NOT TO
SCALE SCALE

O B

The shape is a regular hexagon. The diagram shows a sector of a circle centre O.
Angle AOB = 90°

k = ..................................... k = .....................................
[10]

Permission to reproduce items where third-party owned material protected by copyright is included has been sought and cleared where possible. Every
reasonable effort has been made by the publisher (UCLES) to trace copyright holders, but if any items requiring clearance have unwittingly been included the
publisher will be pleased to make amends at the earliest possible opportunity.

Cambridge International Examinations is part of the Cambridge Assessment Group. Cambridge Assessment is the brand name of University of Cambridge Local
Examinations Syndicate (UCLES), which is itself a department of the University of Cambridge.

© UCLES 2014 0580/42/M/J/14


Cambridge International Examinations
Cambridge International General Certificate of Secondary Education
*9468931136*

MATHEMATICS 0580/43
Paper 4 (Extended) May/June 2014
2 hours 30 minutes
Candidates answer on the Question Paper.
Additional Materials: Electronic calculator Geometrical instruments
Tracing paper (optional)

READ THESE INSTRUCTIONS FIRST

Write your Centre number, candidate number and name on all the work you hand in.
Write in dark blue or black pen.
You may use an HB pencil for any diagrams or graphs.
Do not use staples, paper clips, glue or correction fluid.
DO NOT WRITE IN ANY BARCODES.

Answer all questions.


If working is needed for any question it must be shown below that question.
Electronic calculators should be used.
If the degree of accuracy is not specified in the question, and if the answer is not exact, give the answer to
three significant figures. Give answers in degrees to one decimal place.
For π, use either your calculator value or 3.142.

At the end of the examination, fasten all your work securely together.
The number of marks is given in brackets [ ] at the end of each question or part question.
The total of the marks for this paper is 130.

The syllabus is approved for use in England, Wales and Northern Ireland as a Cambridge International Level 1/Level 2 Certificate.

This document consists of 16 printed pages.

IB14 06_0580_43/2RP
© UCLES 2014 [Turn over
2

1 In July, a supermarket sold 45 981 bottles of fruit juice.

(a) The cost of a bottle of fruit juice was $1.35 .

Calculate the amount received from the sale of the 45 981 bottles.
Give your answer correct to the nearest hundred dollars.

Answer(a) $ ................................................ [2]

(b) The number of bottles sold in July was 17% more than the number sold in January.

Calculate the number of bottles sold in January.

Answer(b) ................................................ [3]

(c) There were 3 different flavours of fruit juice.


The number of bottles sold in each flavour was in the ratio apple : orange : cherry = 3 : 4 : 2.
The total number of bottles sold was 45 981.

Calculate the number of bottles of orange juice sold.

Answer(c) ................................................ [2]

(d) One bottle contains 1.5 litres of fruit juice.

Calculate the number of 330 ml glasses that can be filled completely from one bottle.

Answer(d) ................................................ [3]


5
(e) 9
of the 45 981 bottles are recycled.

Calculate the number of bottles that are recycled.

Answer(e) ................................................ [2]


__________________________________________________________________________________________

© UCLES 2014 0580/43/M/J/14


3

2
4

Frequency
density
2

0
10 20 30 40 50 60
Amount ($x)

A survey asked 90 people how much money they gave to charity in one month.
The histogram shows the results of the survey.

(a) Complete the frequency table for the six columns in the histogram.

Amount ($x) 0 < x Y 10

Frequency 4

[5]

(b) Use your frequency table to calculate an estimate of the mean amount these 90 people gave to charity.

Answer(b) $ ................................................ [4]


__________________________________________________________________________________________

© UCLES 2014 0580/43/M/J/14 [Turn over


4

3 (a)
P

12 cm NOT TO
SCALE
X 17 cm

The diagram shows triangle PQR with PQ = 12 cm and PR = 17 cm.


The area of triangle PQR is 97 cm2 and angle QPR is acute.

(i) Calculate angle QPR.

Answer(a)(i) Angle QPR = ................................................ [3]

(ii) The midpoint of PQ is X.

Use the cosine rule to calculate the length of XR.

Answer(a)(ii) XR = .......................................... cm [4]

© UCLES 2014 0580/43/M/J/14


5

(b)
NOT TO
9.4 cm 42° a cm SCALE

37°

Calculate the value of a.

Answer(b) a = ................................................ [4]

(c) sin x = cos 40°, 0° Y x Y 180°

Find the two values of x.

Answer(c) x = .................. or x = .................. [2]


__________________________________________________________________________________________

© UCLES 2014 0580/43/M/J/14 [Turn over


6

1
4 The table shows some values for the function y = + x , x ≠ 0.
x2

x –3 –2 –1 –0.5 0.5 1 2 3 4

y –2.89 –1.75 3.5 2 2.25 4.06

(a) Complete the table of values. [3]


1
(b) On the grid, draw the graph of y = + x for –3 Y x Y – 0.5 and 0.5 Y x Y 4.
x2
y
5

x
–3 –2 –1 0 1 2 3 4

–1

–2

–3
[5]

© UCLES 2014 0580/43/M/J/14


7

1
(c) Use your graph to solve the equation +x–3=0.
x2

Answer(c) x = ..................... or x = ..................... or x = ..................... [3]

1
(d) Use your graph to solve the equation + x = 1 – x.
x2

Answer(d) x = ................................................ [3]

(e) By drawing a suitable tangent, find an estimate of the gradient of the curve at the point where x = 2.

Answer(e) ................................................ [3]


3 -1 .
(f) Using algebra, show that you can use the graph at y = 0 to find

Answer(f)

[3]
__________________________________________________________________________________________

© UCLES 2014 0580/43/M/J/14 [Turn over


8

5 (a)
y
5
A
4
3
2
B
1
x
0
1 2 3 4 5 6 7 8

(i) Write down the position vector of A.

Answer(a)(i) f p [1]

(ii) Find ì ì , the magnitude of .

Answer(a)(ii) ................................................ [2]

(b)
S
NOT TO
Q SCALE

O p P R

O is the origin, = p and = q.


OP is extended to R so that OP = PR.
OQ is extended to S so that OQ = QS.

(i) Write down in terms of p and q.

Answer(b)(i) = ................................................ [1]

(ii) PS and RQ intersect at M and RM = 2MQ.

Use vectors to find the ratio PM : PS, showing all your working.

Answer(b)(ii) PM : PS = ....................... : ....................... [4]


__________________________________________________________________________________________

© UCLES 2014 0580/43/M/J/14


9

6 In this question, give all your answers as fractions.

N A T I O N

The letters of the word NATION are printed on 6 cards.

(a) A card is chosen at random.

Write down the probability that

(i) it has the letter T printed on it,

Answer(a)(i) ................................................ [1]

(ii) it does not have the letter N printed on it,

Answer(a)(ii) ................................................ [1]

(iii) the letter printed on it has no lines of symmetry.

Answer(a)(iii) ................................................ [1]

(b) Lara chooses a card at random, replaces it, then chooses a card again.

Calculate the probability that only one of the cards she chooses has the letter N printed on it.

Answer(b) ................................................ [3]

(c) Jacob chooses a card at random and does not replace it.
He continues until he chooses a card with the letter N printed on it.

Find the probability that this happens when he chooses the 4th card.

Answer(c) ................................................ [3]


__________________________________________________________________________________________

© UCLES 2014 0580/43/M/J/14 [Turn over


10

7 (a)
E D
t° x°
NOT TO
SCALE
x° C

q° p° 32°
Y A B X

ABCDEF is a hexagon.
AB is parallel to ED and BC is parallel to FE.
YFE and YABX are straight lines.
Angle CBX = 32° and angle EFA = 90°.

Calculate the value of

(i) p,

Answer(a)(i) p = ................................................ [1]

(ii) q,

Answer(a)(ii) q = ................................................ [2]

(iii) t,

Answer(a)(iii) t = ................................................ [1]

(iv) x.

Answer(a)(iv) x = ................................................ [3]

© UCLES 2014 0580/43/M/J/14


11

(b)
R
x° Q

S

NOT TO
SCALE

63°
T P U

P, Q, R and S are points on a circle and PS = SQ.


PR is a diameter and TPU is the tangent to the circle at P.
Angle SPT = 63°.

Find the value of

(i) x,

Answer(b)(i) x = ................................................ [2]

(ii) y.

Answer(b)(ii) y = ................................................ [2]


__________________________________________________________________________________________

© UCLES 2014 0580/43/M/J/14 [Turn over


12

7 2x - 3
8 (a) (i) Show that the equation + = 1 can be simplified to 2x2 + 3x – 6 = 0 .
x+4 2
Answer(a)(i)

[3]

(ii) Solve the equation 2x2 + 3x – 6 = 0 .

Show all your working and give your answers correct to 2 decimal places.

Answer(a)(ii) x = ........................... or x = ........................... [4]

(b) The total surface area of a cone with radius x and slant height 3x is equal to the area of a circle with
radius r.

Show that r = 2x.


[The curved surface area, A, of a cone with radius r and slant height l is A = πrl.]

Answer(b)

[4]
__________________________________________________________________________________________

© UCLES 2014 0580/43/M/J/14


13
–x
9 f(x) = 4 – 3x g(x) = 3

(a) Find f(2x) in terms of x.

Answer(a) f(2x) = ................................................ [1]

(b) Find ff(x) in its simplest form.

Answer(b) ff(x) = ................................................ [2]

(c) Work out gg(–1).


Give your answer as a fraction.

Answer(c) ................................................ [3]

(d) Find f –1(x), the inverse of f(x).

Answer(d) f –1(x) = ................................................ [2]

(e) Solve the equation gf(x) = 1.

Answer(e) x = ................................................ [3]


__________________________________________________________________________________________

© UCLES 2014 0580/43/M/J/14 [Turn over


14

10 (a)

8 cm NOT TO
SCALE

r cm

The three sides of an equilateral triangle are tangents to a circle of radius r cm.
The sides of the triangle are 8 cm long.

Calculate the value of r.


Show that it rounds to 2.3, correct to 1 decimal place.

Answer(a)

[3]

(b)

8 cm
NOT TO
SCALE

12 cm

The diagram shows a box in the shape of a triangular prism of height 12 cm.
The cross section is an equilateral triangle of side 8 cm.

Calculate the volume of the box.

Answer(b) ......................................... cm3 [4]

© UCLES 2014 0580/43/M/J/14


15

(c) The box contains biscuits.


Each biscuit is a cylinder of radius 2.3 centimetres and height 4 millimetres.

Calculate

(i) the largest number of biscuits that can be placed in the box,

Answer(c)(i) ................................................ [3]

(ii) the volume of one biscuit in cubic centimetres,

Answer(c)(ii) ......................................... cm3 [2]

(iii) the percentage of the volume of the box not filled with biscuits.

Answer(c)(iii) ............................................ % [3]


__________________________________________________________________________________________

Question 11 is printed on the next page.

© UCLES 2014 0580/43/M/J/14 [Turn over


16

11

Diagram 1 Diagram 2 Diagram 3

The first three diagrams in a sequence are shown above.


Diagram 1 shows an equilateral triangle with sides of length 1 unit.
1
In Diagram 2, there are 4 triangles with sides of length 2 unit.
1
In Diagram 3, there are 16 triangles with sides of length 4 unit.

(a) Complete this table for Diagrams 4, 5, 6 and n.

Diagram 1 Diagram 2 Diagram 3 Diagram 4 Diagram 5 Diagram 6 Diagram n

1 1
Length of side 1 2 4

Length of side
20 2–1 2–2
as a power of 2
[6]

(b) (i) Complete this table for the number of the smallest triangles in Diagrams 4, 5 and 6.

Diagram 1 Diagram 2 Diagram 3 Diagram 4 Diagram 5 Diagram 6

Number of smallest
1 4 16
triangles
Number of smallest
20 22 24
triangles as a power of 2
[2]

(ii) Find the number of the smallest triangles in Diagram n, giving your answer as a power of 2.

Answer(b)(ii) ................................................ [1]

(c) Calculate the number of the smallest triangles in the diagram where the smallest triangles have sides of
1
length 128 unit.

Answer(c) ................................................ [2]

Permission to reproduce items where third-party owned material protected by copyright is included has been sought and cleared where possible. Every
reasonable effort has been made by the publisher (UCLES) to trace copyright holders, but if any items requiring clearance have unwittingly been included the
publisher will be pleased to make amends at the earliest possible opportunity.

Cambridge International Examinations is part of the Cambridge Assessment Group. Cambridge Assessment is the brand name of University of Cambridge Local
Examinations Syndicate (UCLES), which is itself a department of the University of Cambridge.

© UCLES 2014 0580/43/M/J/14


International General Certificate of Secondary Education
UNIVERSITY OF CAMBRIDGE LOCAL EXAMINATIONS SYNDICATE
MATHEMATICS 0580/4, 0581/4
PAPER 4
OCTOBER/NOVEMBER SESSION 2002 2 hours 30 minutes

Additional materials:
Answer paper
Electronic calculator
Geometrical instruments
Graph paper (1 sheet)
Mathematical tables (optional)
Tracing paper (optional)

TIME 2 hours 30 minutes

INSTRUCTIONS TO CANDIDATES
Write your name, Centre number and candidate number in the spaces provided on the answer paper/
answer booklet.
Answer all questions.
Write your answers on the separate answer paper provided.
All working must be clearly shown. It should be done on the same sheet as the rest of the answer.
Marks will be given for working which shows that you know how to solve the problem even if you get the
answer wrong.
If you use more than one sheet of paper, fasten the sheets together.

INFORMATION FOR CANDIDATES


The number of marks is given in brackets [ ] at the end of each question or part question.
The total of the marks for this paper is 130.
Electronic calculators should be used.
If the degree of accuracy is not specified in the question, and if the answer is not exact, the answer
should be given to three significant figures. Answers in degrees should be given to one decimal place.
For π, use either your calculator value or 3.142.

This question paper consists of 7 printed pages and 1 blank page.


SJF2283/CG S14616/1
© UCLES 2002 [Turn over
2

1 (a) At an athletics meeting, Ben’s time for the 10 000 metres race was 33 minutes exactly and he finished
at 15 17.

(i) At what time did the race start? [1]


(ii) What was Ben’s average speed for the race? Give your answer in kilometres per hour. [2]
(iii) The winner finished 51.2 seconds ahead of Ben.
How long did the winner take to run the 10 000 metres? [1]

(b) The winning distance in the javelin competition was 80 metres.


Otto’s throw was 95% of the winning distance.
Calculate the distance of Otto’s throw. [2]

(c) Pamela won the long jump competition with a jump of 6.16 metres.
This was 10% further than Mona’s jump.
How far did Mona jump? [2]

2 D

NOT TO SCALE
cm
10
6cm

A E
8cm C 5cm

5cm

The diagram shows a sketch of the net of a solid tetrahedron (triangular prism).
The right-angled triangle ABC is its base.
AC = 8 cm, BC = 6 cm and AB = 10 cm. FC = CE = 5 cm.

(a) (i) Show that BE = √61 cm. [1]


(ii) Write down the length of DB. [1]
(iii) Explain why DA = √89 cm. [2]

(b) Calculate the size of angle DBA. [4]

(c) Calculate the area of triangle DBA. [3]

(d) Find the total surface area of the solid. [3]

(e) Calculate the volume of the solid.


[The volume of a tetrahedron is –1 (area of the base) × perpendicular height.] [3]
3

0580/4,0581/4/O/N02
3

3 y

P(6,6)
6

x
-6 -4 -2 0 2 4 6 8 10

Answer the whole of this question on a sheet of graph paper.

(a) Using a scale of 1 cm to represent 1 unit on each axis, draw an x-axis for –6 ⭐ x ⭐ 10 and a y-axis for
–8 ⭐ y ⭐ 8.
Copy the word EXAM onto your grid so that it is exactly as it is in the diagram above.
Mark the point P (6,6). [2]

(b) Draw accurately the following transformations.


(i) Reflect the letter E in the line x = 0. [2]
(ii) Enlarge the letter X by scale factor 3 about centre P (6,6). [2]
(iii) Rotate the letter A 90° anticlockwise about the origin. [2]
(iv) Stretch the letter M vertically with scale factor 2 and x-axis invariant. [2]
→ –3

(c) (i) Mark and label the point Q so that PQ =  2 . [1]
→
(ii) Calculate |PQ | correct to two decimal places. [2]
→
(iii) Mark and label the point S so that PS –4 . [1]
–1
(iv) Mark and label the point R so that PQRS is a parallelogram. [1]

0580/4,0581/4/O/N02 [Turn over


4

3 4
2 5
1 6
10 7
9 8
A wheel is divided into 10 sectors numbered 1 to 10 as shown in the diagram.
The sectors 1, 2, 3 and 4 are shaded.
The wheel is spun and when it stops the fixed arrow points to one of the sectors.
(Each sector is equally likely.)

(a) The wheel is spun once so that one sector is selected. Find the probability that
(i) the number in the sector is even, [1]
(ii) the sector is shaded, [1]
(iii) the number is even or the sector is shaded, [1]
(iv) the number is odd and the sector is shaded. [1]

(b) The wheel is spun twice so that each time a sector is selected. Find the probability that
(i) both sectors are shaded, [2]
(ii) one sector is shaded and one is not, [2]
(iii) the sum of the numbers in the two sectors is greater than 20, [2]
(iv) the sum of the numbers in the two sectors is less than 4, [2]
(v) the product of the numbers in the two sectors is a square number. [3]

0580/4,0581/4/O/N02
5

5 Answer the whole of this question on a sheet of graph paper.


24
(a) The table gives values of f(x) = + x2 for 0.8 ⭐ x ⭐ 6.
x2

x 0.8 1 1.5 2 2.5 3 3.5 4 4.5 5 5.5 6

f(x) 38.1 25 12.9 10 10.1 11.7 l m n 26 31 36.7

Calculate, correct to 1 decimal place, the values of l, m and n. [3]

(b) Using a scale of 2 cm to represent 1 unit on the x-axis and 2 cm to represent 5 units on the y-axis, draw
an x-axis for 0 ⭐ x ⭐ 6 and a y-axis for 0 ⭐ y ⭐ 40.

Draw the graph of y = f(x) for 0.8 ⭐ x ⭐ 6. [6]

(c) Draw the tangent to your graph at x = 1.5 and use it to calculate an estimate of the gradient of the curve
at this point. [4]

(d) (i) Draw a straight line joining the points (0, 20) and (6, 32). [1]
(ii) Write down the equation of this line in the form y = mx + c. [2]
(iii) Use your graph to write down the x-values of the points of intersection of this line and the curve
y = f(x). [2]
(iv) Draw the tangent to the curve which has the same gradient as your line in part d(i). [1]
(v) Write down the equation for the tangent in part d(iv). [2]

6 (a) On 1st January 2000, Ashraf was x years old.


Bukki was 5 years older than Ashraf and Claude was twice as old as Ashraf.

(i) Write down in terms of x, the ages of Bukki and Claude on 1st January 2000. [2]
(ii) Write down in terms of x, the ages of Ashraf, Bukki and Claude on 1st January 2002. [1]
(iii) The product of Claude’s age and Ashraf’s age on 1st January 2002 is the same as the square of
Bukki’s age on 1st January 2000.
Write down an equation in x and show that it simplifies to x2 – 4x – 21 = 0. [4]
(iv) Solve the equation x2 – 4x – 21 = 0. [2]
(v) How old was Claude on 1st January 2002? [1]

(b) Claude’s height, h metres, is one of the solutions of h2 + 8h – 17 = 0.

(i) Solve the equation h2 + 8h – 17 = 0.


Show all your working and give your answers correct to 2 decimal places. [4]
(ii) Write down Claude’s height, to the nearest centimetre. [1]

0580/4,0581/4/O/N02 [Turn over


6

7 (a) A group of students sat an examination. Each student got one of the grades A, B, C or D.
The pie chart shows these results.

D
A

108° NOT TO SCALE

36 students got grade A, shown by an angle of 108°.

(i) Calculate the total number of students who sat the examination. [2]
(ii) How many students did not get grade A? [1]
(iii) The ratio of the number of students getting grades B, C or D is 4 : 5 : 3.
Find the number of students getting each grade. [3]
(iv) Work out the angles in the pie chart for grades B, C and D. [3]
(v) Find the ratio, in its lowest terms,
the number of students with grade A : the number of students with grade B. [1]

(b) A group of children were asked how much money they had saved. The histogram and table show the
results.

Frequency
Density

0 10 20 30 40 50 60 70 80 m
Money saved in dollars (m)

Money saved ($m) 0 < m ⭐ 20 20 < m ⭐ 30 30 < m ⭐ 40 40 < m ⭐ 70


Frequency 25 p q r

Use the histogram to calculate the values of p, q and r. [4]

0580/4,0581/4/O/N02
7

8 NOT TO SCALE

3h 3h

h h
r 3r r 3r

STANDARD A B C

Sarah investigates cylindrical plant pots.


The standard pot has base radius r cm and height h cm.
Pot A has radius 3r and height h. Pot B has radius r and height 3h. Pot C has radius 3r and height 3h.

(a) (i) Write down the volumes of pots A, B and C in terms of π , r and h. [3]
(ii) Find in its lowest terms the ratio of the volumes of A : B : C. [2]
(iii) Which one of the pots A, B or C is mathematically similar to the standard pot?
Explain your answer. [2]
(iv) The surface area of the standard pot is S cm2. Write down in terms of S the surface area of the
similar pot. [2]

(b) Sarah buys a cylindrical plant pot with radius 15 cm and height 20 cm. She wants to paint its outside
surface (base and curved surface area).

(i) Calculate the area she wants to paint. [2]


(ii) Sarah buys a tin of paint which will cover 30 m2.
How many plant pots of this size could be painted on their outside surfaces completely using this
tin of paint? [4]

9 (a) Write down the 10th term and the nth term of the following sequences.

(i) 1, 2, 3, 4, 5 …, …, [1]
(ii) 7, 8, 9, 10, 11 …, …, [1]
(iii) 8, 10, 12, 14, 16 …, … . [3]

(b) Consider the sequence

1(8 – 7), 2(10 – 8), 3(12 – 9), 4(14 – 10), …………, ………… .

(i) Write down the next term and the 10th term of this sequence in the form a(b – c) where a, b and c
are integers. [3]
(ii) Write down the nth term in the form a(b – c) and then simplify your answer. [2]

0580/4,0581/4/O/N02
8

BLANK PAGE

0580/4,0581/4/O/N02
CAMBRIDGE INTERNATIONAL EXAMINATIONS
International General Certificate of Secondary Education
MATHEMATICS 0580/04
0581/04
Paper 4
October/November 2003

2 hours 30 minutes
Additional Materials: Answer Booklet/Paper
Electronic calculator
Geometrical instruments
Graph paper (2 sheets)
Mathematical tables (optional)
Tracing paper (optional)

READ THESE INSTRUCTIONS FIRST

Write your answers and working on the separate Answer Booklet/Paper provided.
Write your Centre number, candidate number and name on all the work you hand in.
Write in dark blue or black pen on both sides of the paper.
You may use a pencil for any diagrams or graphs.
Do not use staples, paper clips, highlighters, glue or correction fluid.

Answer all questions


At the end of the examination, fasten all your work securely together.
The number of marks is given in brackets [ ] at the end of each question or part question.

All working must be clearly shown. It should be done on the same sheet as the rest of the answer.
Marks will be given for working which shows that you know how to solve the problem even if you get the
answer wrong.
The total of the marks for this paper is 130.
Electronic calculators should be used.
If the degree of accuracy is not specified in the question, and if the answer is not exact, give the answer to
three significant figures. Answers in degrees should be given to one decimal place.
For π, use either your calculator value or 3.142.

This document consists of 7 printed pages and 1 blank page.


MCS-UCH161-S40981/3
© CIE 2003 [Turn over
2

1 A train starts its journey with 240 passengers.


144 of the passengers are adults and the rest are children.

(a) Write the ratio Adults : Children in its lowest terms. [2]

(b) At the first stop, 3721 % of the adults and 31 of the children get off the train.
20 adults and x children get onto the train.
The total number of passengers on the train is now 200.

(i) How many children got off the train? [1]


(ii) How many adults got off the train? [1]
(iii) How many adult passengers are on the train as it sets off again? [1]
(iv) What is the value of x? [1]

(c) After a second stop, there are 300 passengers on the train and the ratio
Men : Women : Children is 6 : 5 : 4.
Calculate the number of children now on the train. [2]

(d) On Tuesday the train journey took 7 hours and 20 minutes and began at 13 53.

(i) At what time did the train journey end? [1]


(ii) Tuesday’s time of 7 hours 20 minutes was 10% more than Monday’s journey time.
How many minutes longer was Tuesday’s journey? [2]

2 (a) The surface area of a person’s body, A square metres, is given by the formula


hm
A=
3600
where h is the height in centimetres and m is the mass in kilograms.

(i) Dolores is 167 cm high and has a mass of 70 kg. Calculate the surface area of her body.
[1]
(ii) Erik has a mass of 80 kg. Find his height if A # 1.99. [2]
(iii) Make h the subject of the formula. [3]

(b) Factorise
(i) x2 0 16, [1]
(ii) x2 0 16x, [1]
(iii) x2 0 9x ! 8. [2]

0580/04/0581/04/O/N/03
3

(c) Erik runs a race at an average speed of x ms.


His time is (3x 0 9) seconds and the race distance is (2x2 0 8) metres.

(i) Write down an equation in x and show that it simplifies to


x2 0 9x ! 8 # 0. [2]
(ii) Solve x2 0 9x ! 8 # 0. [2]
(iii) Write down Erik’s time and the race distance. [2]

3 North B
NOT TO
SCALE
m
17

40°
F

32 m

Felipe (F) stands 17 metres from a bridge (B) and 32 metres from a tree (T).
The points F, B and T are on level ground and angle BFT # 40°.

(a) Calculate
(i) the distance BT, [4]
(ii) the angle BTF. [3]

(b) The bearing of B from F is 085°. Find the bearing of


(i) T from F, [1]
(ii) F from T, [1]
(iii) B from T. [1]

(c) The top of the tree is 30 metres vertically above T.


Calculate the angle of elevation of the top of the tree from F. [2]

0580/04/0581/04/O/N/03 [Turn over


4

4 Answer the whole of this question on a sheet of graph paper.

t 0 1 2 3 4 5 6 7

f(t) 0 25 37.5 43.8 46.9 48.4 49.2 49.6

(a) Using a scale of 2 cm to represent 1 unit on the horizontal t-axis and 2 cm to represent 10 units
on the y-axis, draw axes for 0 ≤ t ≤ 7 and 0 ≤ y ≤ 60.
Draw the graph of the curve y # f(t) using the table of values above. [5]

(b) f(t) # 50(1 0 20t).


(i) Calculate the value of f(8) and the value of f(9). [2]
(ii) Estimate the value of f(t) when t is large. [1]

(c) (i) Draw the tangent to y # f(t) at t # 2 and use it to calculate an estimate of the gradient of the
curve at this point. [3]
(ii) The function f(t) represents the speed of a particle at time t.
Write down what quantity the gradient gives. [1]

(d) (i) On the same grid, draw y # g(t) where g(t) # 6t ! 10, for 0 ≤ t ≤ 7. [2]
(ii) Write down the range of values for t where f(t) p g(t). [2]
(iii) The function g(t) represents the speed of a second particle at time t.
State whether the first or second particle travels the greater distance for 0 ≤ t ≤ 7.
You must give a reason for your answer. [2]

5
A D A M D A N I E L

Adam writes his name on four red cards and Daniel writes his name on six white cards.

(a) One of the ten cards is chosen at random. Find the probability that
(i) the letter on the card is D, [1]
(ii) the card is red, [1]
(iii) the card is red or the letter on the card is D, [1]
(iv) the card is red and the letter on the card is D, [1]
(v) the card is red and the letter on the card is N. [1]

0580/04/0581/04/O/N/03
5

(b) Adam chooses a card at random and then Daniel chooses one of the remaining 9 cards at
random.
Giving your answers as fractions, find the probability that the letters on the two cards are
(i) both D, [2]
(ii) both A, [2]
(iii) the same, [2]
(iv) different. [2]

6 (a) Calculate the volume of a cylinder with radius 30 cm and height 50 cm. [2]

(b)
NOT TO
SCALE

O
cm
30
X
A B
12 cm

Y 50 cm

A cylindrical tank, radius 30 cm and length 50 cm, lies on its side.


It is partially filled with water.
The shaded segment AXBY in the diagram shows the cross-section of the water.
The greatest depth, XY, is 12 cm.
OA # OB # 30 cm.

(i) Write down the length of OX. [1]


(ii) Calculate the angle AOB correct to two decimal places, showing all your working. [3]

(c) Using angle AOB # 106.3°, find


(i) the area of the sector AOBY, [3]
(ii) the area of triangle AOB, [2]
(iii) the area of the shaded segment AXBY. [1]

(d) Calculate the volume of water in the cylinder, giving your answer

(i) in cubic centimetres, [2]


(ii) in litres. [1]

(e) How many more litres must be added to make the tank half full? [2]

0580/04/0581/04/O/N/03 [Turn over


6

7 (a) y
6

C
4

2
D T A

–4 –2 0 2 4 6 8 10 12 x

–2
F
E
–4

Use one of the letters A, B, C, D, E or F to answer the following questions.

(i) Which triangle is T mapped onto by a translation? Write down the translation vector. [2]
(ii) Which triangle is T mapped onto by a reflection? Write down the equation of the mirror
line. [2]
(iii) Which triangle is T mapped onto by a rotation? Write down the coordinates of the centre
of rotation. [2]
(iv) Which triangle is T mapped onto by a stretch with the x-axis invariant?
Write down the scale factor of the stretch. [2]

1 4
(v) M =
 
0 1
. Which triangle is T mapped onto by M?

Write down the name of this transformation. [2]



−1


1 3
(b) P =
5 7  
,Q # ( 01 02),R # (123),S = 2 .
3
Only some of the following matrix operations are possible with matrices P, Q, R and S above.
PQ, QP, P ! Q, PR, RS
Write down and calculate each matrix operation that is possible. [6]

0580/04/0581/04/O/N/03
7

8 Answer the whole of this question on a sheet of graph paper.


120 passengers on an aircraft had their baggage weighed. The results are shown in the table.

Mass of baggage (M kg) 0 ` M ≤ 10 10 ` M ≤ 15 15 ` M ≤ 20 20 ` M ≤ 25 25 ` M ≤ 40


Number of passengers 12 32 28 24 24

(a) (i) Write down the modal class. [1]


(ii) Calculate an estimate of the mean mass of baggage for the 120 passengers. Show all your
working. [4]
(iii) Sophia draws a pie chart to show the data.
What angle should she have in the 0 ` M ≤ 10 sector? [1]

(b) Using a scale of 2 cm to represent 5 kg, draw a horizontal axis for 0 ` M ≤ 40.
Using an area scale of 1 cm2 to represent 1 passenger, draw a histogram for this data. [7]

9 In each of the diagrams below, triangle ABC is an isosceles right-angled triangle.


AB # AC # 6 cm.
A straight line or a circular arc divides the triangle into two parts, one of which is shaded.

B B B B B

3 3
6 6 6 x° x°
3 3
A C A C A C A C A C
3 3 3 3 6 3 3 6

Diagram 1 Diagram 2 Diagram 3 Diagram 4 Diagram 5

(a) Which diagram has a shaded region showing all the points in the triangle which are
(i) closer to BC than to BA, [1]
(ii) more than 3 cm from A, [1]
(iii) closer to C than to A? [1]

(b) For each of the five diagrams, calculate the shaded area. [11]

0580/04/0581/04/O/N/03
8

BLANK PAGE

0580/04/0581/04/O/N/03
UNIVERSITY OF CAMBRIDGE INTERNATIONAL EXAMINATIONS
International General Certificate of Secondary Education

MATHEMATICS 0580/04
0581/04
Paper 4 (Extended)
October/November 2004
Additional Materials: Answer Booklet/Paper 2 hours 30 minutes
Electronic calculator
Geometrical instruments
Graph paper (2 sheets)
Mathematical tables (optional)
Tracing paper (optional)

READ THESE INSTRUCTIONS FIRST

Write your answers and working on the separate Answer Booklet/Paper provided.
Write your Centre number, candidate number and name on all the work you hand in.
Write in dark blue or black pen on both sides of the paper.
You may use a soft pencil for any diagrams or graphs.
Do not use staples, paper clips, highlighters, glue or correction fluid.

Answer all questions.


At the end of the examination, fasten all your work securely together.
The number of marks is given in brackets [ ] at the end of each question or part question.

All working must be clearly shown. It should be done on the same sheet as the rest of the answer.
Marks will be given for working which shows that you know how to solve the problem even if you get the
answer wrong.
The total of the marks for this paper is 130.
Electronic calculators should be used.
If the degree of accuracy is not specified in the question, and if the answer is not exact, give the answer to
three significant figures.
Answers in degrees should be given to one decimal place.
For π use either your calculator value or 3.142.

This document consists of 8 printed pages.


IB04 11_0580_04/2RP
 UCLES 2004 [Turn over
2

1 The population of Newtown is 45 000.


The population of Villeneuve is 39 000.

(a) Calculate the ratio of these populations in its simplest form. [1]

(b) In Newtown, 28% of the population are below the age of twenty.
Calculate how many people in Newtown are below the age of twenty. [2]

(c) In Villeneuve, 16 000 people are below the age of twenty.


Calculate the percentage of people in Villeneuve below the age of twenty. [2]

(d) The population of Newtown is 125% greater than it was fifty years ago.
Calculate the population of Newtown fifty years ago. [2]

(e) The two towns are combined and made into one city called Monocity.
In Monocity the ratio of
men : women :children is 12 : 13 : 5.
Calculate the number of children in Monocity. [2]

2 Answer the whole of this question on a sheet of graph paper.

12
(a) f(x) =
x +1

x 0 1 2 3 4 5 6 7 8 9 10 11

f(x) p 6 4 3 2.4 2 1.71 q 1.33 r 1.09 1

(i) Calculate the values of p, q and r. [3]

(ii) Draw the graph of y = f(x) for 0 x 11.


Use a scale of 1cm to 1 unit on each axis. [5]

(iii) By drawing a suitable line, find an estimate of the gradient of the graph at the point (3, 3). [3]

(b) On the same grid draw the graph of y = 8 − x for 0 x 8. [2]

(c) (i) Show that the equation f(x) = 8 − x simplifies to x2 − 7 x + 4 = 0 . [2]

(ii) Use your graph to solve this equation, giving your answers correct to 1 decimal place. [2]

© UCLES 2004 0580/04/O/N/04


3

3 Water flows through a pipe into an empty cylindrical tank.


The tank has a radius of 40 cm and a height of 110 cm.

(a) Calculate the volume of the tank. [2]

(b) The pipe has a cross-sectional area of 1.6 cm2.


The water comes out of the pipe at a speed of 14 cm/s.
How long does it take to fill the tank?
Give your answer in hours and minutes, correct to the nearest minute. [4]

(c) All the water from the tank is added to a pond which has a surface area of 70 m2.
Work out the increase in the depth of water in the pond.
Give your answer in millimetres, correct to the nearest millimetre. [4]

4 Answer the whole of this question on a sheet of graph paper.

(a) Draw x- and y-axes from –8 to 8 using a scale of 1cm to 1 unit.


Draw triangle ABC with A (2, 2), B (5, 2) and C (5, 4). [2]

−9
(b) Draw the image of triangle ABC under a translation of .
3
Label it A1B1C1. [2]

(c) Draw the image of triangle ABC under a reflection in the line y = −1.
Label it A2B2C2. [2]

(d) Draw the image of triangle ABC under an enlargement, scale factor 2, centre (6,0).
Label it A3B3C3. [2]

0 −1
(e) The matrix represents a transformation.
−1 0

(i) Draw the image of triangle ABC under this transformation. Label it A4B4C4. [2]

(ii) Describe fully this single transformation. [2]

(f) (i) Draw the image of triangle ABC under a stretch, factor 1.5, with the y-axis invariant.
Label it A5B5C5. [2]

(ii) Find the 2 by 2 matrix which represents this transformation. [2]

© UCLES 2004 0580/04/O/N/04 [Turn over


4

5
y

NOT TO
SCALE
D
North

54o C
o
32
B 45 m

40 m 70 m

x
O

(a) During a soccer match a player runs from A to B and then from B to C as shown in the diagram.
AB = 40 m, BC = 45 m and AC = 70 m.

(i) Show by calculation that angle BAC = 37°, correct to the nearest degree. [3]

(ii) The bearing of C from A is 051°. Find the bearing of B from A. [1]

(iii) Calculate the area of triangle ABC. [3]

(b) x- and y-axes are shown in the diagram.


 p
=   , where p and q are measured in metres.
q

(i) Show that p = 54.4. [2]

(ii) Find the value of q. [2]

(c) Another player is standing at D.


BC = 45 m, angle BCD = 54° and angle DBC = 32°.
Calculate the length of BD. [4]

© UCLES 2004 0580/04/O/N/04


5

6 (a) Students are given marks 0, 1, 2, 3 or 4 for a piece of work.


The table shows the number of students getting each mark.

Mark 0 1 2 3 4

Frequency 3 10 12 9 x

(i) The mean mark is 2.125.


Find the value of x. [4]

(ii) Write down the lower quartile mark. [1]

(b) The heights (h centimetres) of flowers in a shop are shown in the histogram below.
All the flowers are less than 60 cm high.
One bar has not been drawn on the histogram.

3
Frequency
density
2

0 10 20 30 40 50 60
Height (h cm)

(i) There are 25 flowers in the interval 20< h 25.


How many flowers are there in the intervals

(a) 25 < h 30, [1]

(b) 10 < h 20? [1]

(ii) There are 42 flowers in the interval 30 < h 60.


This can be shown by a single bar on the histogram.
Calculate the height of this bar. [2]

(iii) Calculate an estimate of the mean height of the flowers. [3]

© UCLES 2004 0580/04/O/N/04 [Turn over


6

2
7 A sketch of the graph of the quadratic function y = px + qx + r is shown in the diagram.

line of
symmetry

x
K L

The graph cuts the x-axis at K and L.


The point M lies on the graph and on the line of symmetry.

(a) When p = 1, q = −2, r = −3 , find

(i) the y-coordinate of the point where x = 4, [1]

(ii) the coordinates of K and L, [3]

(iii) the coordinates of M. [2]

(b) Describe how the above sketch of the graph would change in each of the following cases.

(i) p is negative. [1]

(ii) p = 1, q = r = 0 . [1]

2
(c) Another quadratic function is y = ax + bx + c .

(i) Its graph passes through the origin.


Write down the value of c. [1]

(ii) The graph also passes through the points (3, 0) and (4, 8).
Find the values of a and b. [4]

© UCLES 2004 0580/04/O/N/04


7

8 (a) The technical data of a car includes the following information.

Type of road Petrol used per 100 km

Main roads 9.2 litres

Other roads 8.0 litres

(i) How much petrol is used on a journey of 350 km on a main road? [1]

(ii) On other roads, how far can the car travel on 44 litres of petrol? [1]

(iii) A journey consists of 200 km on a main road and 160 km on other roads.

(a) How much petrol is used? [2]

(b) Work out the amount of petrol used per 100 km of this journey. [1]

(b) A model of a car has a scale of 1 : 25.

(i) The length of the car is 3.95 m.


Calculate the length of the model.
Give your answer in centimetres. [3]

(ii) The painted surface area of the model is 128 cm2.


Calculate the painted surface area of the car, giving your answer in square centimetres. [2]

(iii) The size of the luggage space of the car is 250 litres.
Calculate the size of the luggage space of the model, giving your answer in millilitres. [3]

9 (a) f(x) = 2 – 3x and g(x) = x2.

(i) Solve the equation f(x) = 7 – x. [2]

(ii) Find f—1 (x). [2]

(iii) Find the value of gf(2) – fg(2). [3]

(iv) Find fg(x). [1]

(b) h(x) = xx.

(i) Find the value of h(2). [1]

(ii) Find the value of h(−3), giving your answer as a fraction. [1]

(iii) Find the value of h(7.5), giving your answer in standard form. [2]

(iv) h(−0.5) is not a real number. Explain why. [1]

(v) Find the integer value for which h(x) = 3125. [1]

© UCLES 2004 0580/04/O/N/04 [Turn over


8

10 Quadrilaterals P and Q each have diagonals which

• are unequal,
• intersect at right angles.

P has two lines of symmetry. Q has one line of symmetry.

(a) (i) Sketch quadrilateral P.


Write down its geometrical name. [2]

(ii) Sketch quadrilateral Q.


Write down its geometrical name. [2]

(b) In quadrilateral P, an angle between one diagonal and a side is x°.


Write down, in terms of x, the four angles of quadrilateral P. [2]

(c) The diagonals of quadrilateral Q have lengths 20 cm and 12 cm.


Calculate the area of quadrilateral Q. [2]

(d) Quadrilateral P has the same area as quadrilateral Q.


The lengths of the diagonals and sides of quadrilateral P are all integer values.
Find the length of a side of quadrilateral P. [3]

Every reasonable effort has been made to trace all copyright holders. The publishers would be pleased to hear from anyone whose rights we have unwittingly infringed.

University of Cambridge International Examinations is part of the University of Cambridge Local Examinations Syndicate (UCLES), which is itself a department of the
University of Cambridge.

© UCLES 2004 0580/04/O/N/04


UNIVERSITY OF CAMBRIDGE INTERNATIONAL EXAMINATIONS
International General Certificate of Secondary Education

MATHEMATICS 0580/04
0581/04
Paper 4 (Extended)
October/November 2005

Additional Materials: Answer Booklet/Paper 2 hours 30 minutes


Electronic calculator
Geometrical instruments
Graph paper (3 sheets)
Mathematical tables (optional)
Tracing paper (optional)

READ THESE INSTRUCTIONS FIRST

Write your answers and working on the separate Answer Booklet/Paper provided.
Write your name, Centre number and candidate number on all the work you hand in.
Write in dark blue or black pen on both sides of the paper.
You may use a soft pencil for any diagrams or graphs.
Do not use staples, paper clips, highlighters, glue or correction fluid.

Answer all questions.


At the end of the examination, fasten all your work securely together.
The number of marks is given in brackets [ ] at the end of each question or part question.

All working must be clearly shown. It should be done on the same sheet as the rest of the answer.
Marks will be given for working which shows that you know how to solve the problem even if you get the
answer wrong.
The total of the marks for this paper is 130.
Electronic calculators should be used.
If the degree of accuracy is not specified in the question, and if the answer is not exact, give the answer to
three significant figures.
Give answers in degrees to one decimal place.
For π use either your calculator value or 3.142.

This document consists of 8 printed pages.


IB05 11_0580_04/4RP
 UCLES 2005 [Turn over
2

1 A Spanish family went to Scotland for a holiday.

(a) The family bought 800 pounds (£) at a rate of £1 = 1.52 euros (€).
How much did this cost in euros? [1]

(b) The family returned home with £118 and changed this back into euros.
They received €173.46.
Calculate how many euros they received for each pound. [1]

(c) A toy which costs €11.50 in Spain costs only €9.75 in Scotland.
Calculate, as a percentage of the cost in Spain, how much less it costs in Scotland. [2]

(d) The total cost of the holiday was €4347.00.


In the family there were 2 adults and 3 children.
The cost for one adult was double the cost for one child.
Calculate the cost for one child. [2]

(e) The original cost of the holiday was reduced by 10% to €4347.00.
Calculate the original cost. [2]

(f) The plane took 3 hours 15 minutes to return to Spain.


The length of this journey was 2350 km.
Calculate the average speed of the plane in

(i) kilometres per hour, [2]

(ii) metres per second. [1]

2 Answer the whole of this question on one sheet of graph paper.

(a) Draw and label x and y axes from –8 to +8, using a scale of 1 cm to 1 unit on each axis. [1]

(b) Draw and label triangle ABC with A (2, 2), B (5, 2) and C (5, 4). [1]

(c) On your grid:


 3
(i) translate triangle ABC by the vector   and label this image A1B1C1; [2]
 −9 

(ii) reflect triangle ABC in the line x = −1 and label this image A2B2C2; [2]

(iii) rotate triangle ABC by 180° about (0, 0) and label this image A3B3C3. [2]

1.5 0 
(d) A stretch is represented by the matrix  .
 0 1

(i) Draw the image of triangle ABC under this transformation. Label this image A4B4C4. [3]

1.5 0 
(ii) Work out the inverse of the matrix  . [2]
 0 1

(iii) Describe fully the single transformation represented by this inverse. [3]

© UCLES 2005 0580/04, 0581/04 Nov 2005


3

3
S
North
7 km

30o
NOT TO
P R
SCALE 55o 15 km

14 km

Q
The quadrilateral PQRS shows the boundary of a forest.
A straight 15 kilometre road goes due East from P to R.

(a) The bearing of S from P is 030° and PS = 7 km.

(i) Write down the size of angle SPR. [1]

(ii) Calculate the length of RS. [4]

(b) Angle RPQ = 55º and QR = 14 km.

(i) Write down the bearing of Q from P. [1]

(ii) Calculate the acute angle PQR. [3]

(iii) Calculate the length of PQ. [3]

(c) Calculate the area of the forest, correct to the nearest square kilometre. [4]

© UCLES 2005 0580/04, 0581/04 Nov 2005 [Turn over


4

4 (a) All 24 students in a class are asked whether they like football and whether they like basketball.
Some of the results are shown in the Venn diagram below.

F B
7 12 2

= {students in the class}.


F = {students who like football}.
B = {students who like basketball}.

(i) How many students like both sports? [1]

(ii) How many students do not like either sport? [1]

(iii) Write down the value of n(F∪B). [1]

(iv) Write down the value of n(F ′∩B). [1]

(v) A student from the class is selected at random.


What is the probability that this student likes basketball? [1]

(vi) A student who likes football is selected at random.


What is the probability that this student likes basketball? [1]

(b) Two students are selected at random from a group of 10 boys and 12 girls.
Find the probability that

(i) they are both girls, [2]

(ii) one is a boy and one is a girl. [3]

5 Answer the whole of this question on one sheet of graph paper.

1
f(x) = 1 − , x ≠ 0.
x2
(a)
x −3 −2 −1 −0.5 −0.4 −0.3 0.3 0.4 0.5 1 2 3
f(x) p 0.75 0 −3 −5.25 q q −5.25 −3 0 0.75 p

Find the values of p and q. [2]

(b) (i) Draw an x-axis for −3 x 3 using 2 cm to represent 1 unit and a y-axis for −11 y 2
using 1 cm to represent 1 unit. [1]

(ii) Draw the graph of y = f(x) for −3 x −0.3 and for 0.3 x 3. [5]

(c) Write down an integer k such that f(x) = k has no solutions. [1]

© UCLES 2005 0580/04, 0581/04 Nov 2005


5

(d) On the same grid, draw the graph of y = 2x – 5 for –3 x 3. [2]

1
(e) (i) Use your graphs to find solutions of the equation 1 − = 2x − 5 . [3]
x2
1
(ii) Rearrange 1 − = 2 x − 5 into the form ax 3 + bx 2 + c = 0 , where a, b and c are integers. [2]
x2

(f) (i) Draw a tangent to the graph of y = f(x) which is parallel to the line y = 2 x − 5 . [1]

(ii) Write down the equation of this tangent. [2]

6
P

D C
3 cm
NOT TO 5 cm
SCALE M
F

A 6 cm B

The diagram shows a pyramid on a rectangular base ABCD, with AB = 6 cm and AD = 5 cm.
The diagonals AC and BD intersect at F.
The vertical height FP = 3 cm.

(a) How many planes of symmetry does the pyramid have? [1]

(b) Calculate the volume of the pyramid.


1
[The volume of a pyramid is 3
× area of base × height.] [2]

(c) The mid-point of BC is M.


Calculate the angle between PM and the base. [2]

(d) Calculate the angle between PB and the base. [4]

(e) Calculate the length of PB. [2]

© UCLES 2005 0580/04, 0581/04 Nov 2005 [Turn over


6

7
35 m

1.1 m
NOT TO
24 m
SCALE

D C

2.5 m
B

The diagram shows a swimming pool of length 35 m and width 24 m.


A cross-section of the pool, ABCD, is a trapezium with AD = 2.5 m and BC = 1.1 m.

(a) Calculate

(i) the area of the trapezium ABCD, [2]

(ii) the volume of the pool, [2]

(iii) the number of litres of water in the pool, when it is full. [1]

(b) AB = 35.03 m correct to 2 decimal places.


The sloping rectangular floor of the pool is painted.
It costs $2.25 to paint one square metre.

(i) Calculate the cost of painting the floor of the pool. [2]

(ii) Write your answer to part (b)(i) correct to the nearest hundred dollars. [1]

(c) (i) Calculate the volume of a cylinder, radius 12.5 cm and height 14 cm. [2]

(ii) When the pool is emptied, the water flows through a cylindrical pipe of radius 12.5 cm.
The water flows along this pipe at a rate of 14 centimetres per second.
Calculate the time taken to empty the pool.
Give your answer in days and hours, correct to the nearest hour. [4]

© UCLES 2005 0580/04, 0581/04 Nov 2005


7

8 (a) (i) The cost of a book is $x.


Write down an expression in terms of x for the number of these books which are bought for
$40. [1]

(ii) The cost of each book is increased by $2.


The number of books which are bought for $40 is now one less than before.
Write down an equation in x and show that it simplifies to x 2 + 2 x − 80 = 0 . [4]

(iii) Solve the equation x 2 + 2 x − 80 = 0 . [2]

(iv) Find the original cost of one book. [1]

(b) Magazines cost $m each and newspapers cost $n each.


One magazine costs $2.55 more than one newspaper.
The cost of two magazines is the same as the cost of five newspapers.

(i) Write down two equations in m and n to show this information. [2]

(ii) Find the values of m and n. [3]

QUESTION 9 is on page 8.

© UCLES 2005 0580/04, 0581/04 Nov 2005 [Turn over


8

9
Answer the whole of this question on one sheet of graph paper.

The heights (h cm) of 270 students in a school are measured and the results are shown in the table.

h Frequency
120 < h 130 15
130 < h 140 24
140 < h 150 36
150 < h 160 45
160 < h 170 50
170 < h 180 43
180 < h 190 37
190 < h 200 20

(a) Write down the modal group. [1]

(b) (i) Calculate an estimate of the mean height. [4]

(ii) Explain why the answer to part (b)(i) is an estimate. [1]

(c) The following table shows the cumulative frequencies for the heights of the students.

h Cumulative frequency
h 120 0
h 130 p
h 140 q
h 150 r
h 160 120
h 170 170
h 180 213
h 190 250
h 200 270

Write down the values of p, q and r. [2]

(d) Using a scale of 1cm to 5 units, draw a horizontal h-axis, starting at h = 120.
Using a scale of 1cm to 20 units on the vertical axis, draw a cumulative frequency diagram. [5]

(e) Use your diagram to find

(i) the median height, [1]

(ii) the upper quartile, [1]

(iii) the inter-quartile range, [1]

(iv) the 60th percentile. [1]

(f) All the players in the school’s basketball team are chosen from the 30 tallest students.
Use your diagram to find the least possible height of any player in the basketball team. [2]

Permission to reproduce items where third-party owned material protected by copyright is included has been sought and cleared where possible. Every reasonable effort has
been made by the publisher (UCLES) to trace copyright holders, but if any items requiring clearance have unwittingly been included, the publisher will be pleased to make
amends at the earliest possible opportunity.

University of Cambridge International Examinations is part of the University of Cambridge Local Examinations Syndicate (UCLES), which is itself a department of the
University of Cambridge.

© UCLES 2005 0580/04, 0581/04 Nov 2005


UNIVERSITY OF CAMBRIDGE INTERNATIONAL EXAMINATIONS
International General Certificate of Secondary Education

MATHEMATICS 0580/04
0581/04
Paper 4 (Extended)
October/November 2006

Additional Materials: Answer Booklet/Paper 2 hours 30 minutes


Electronic calculator
Geometrical instruments
Graph paper (2 sheets)
Mathematical tables (optional)
Tracing paper (optional)

READ THESE INSTRUCTIONS FIRST

Write your answers and working on the separate Answer Booklet/Paper provided.
Write your name, Centre number and candidate number on all the work you hand in.
Write in dark blue or black pen on both sides of the paper.
You may use a soft pencil for any diagrams or graphs.
Do not use staples, paperclips, highlighters, glue or correction fluid.

Answer all questions.


At the end of the examination, fasten all your work securely together.
The number of marks is given in brackets [ ] at the end of each question or part question.

All working must be clearly shown. It should be done on the same sheet as the rest of the answer.
Marks will be given for working which shows that you know how to solve the problem even if you get the
answer wrong.
The total of the marks for this paper is 130.
Electronic calculators should be used.
If the degree of accuracy is not specified in the question, and if the answer is not exact, give the answer to
three significant figures.
Give answers in degrees to one decimal place.
For π use either your calculator value or 3.142.

This document consists of 8 printed pages.


IB06 11_0580_04/4RP
 UCLES 2006 [Turn over
2

1 Maria, Carolina and Pedro receive $800 from their grandmother in the ratio

Maria: Carolina: Pedro = 7:5:4.

(a) Calculate how much money each receives. [3]

2
(b) Maria spends 7
of her money and then invests the rest for two years

at 5% per year simple interest.


How much money does Maria have at the end of the two years? [3]

(c) Carolina spends all of her money on a hi-fi set and two years later sells it at a loss of 20%.
How much money does Carolina have at the end of the two years? [2]

(d) Pedro spends some of his money and at the end of the two years he has $100.
Write down and simplify the ratio of the amounts of money Maria, Carolina and Pedro have at
the end of the two years. [2]

(e) Pedro invests his $100 for two years at a rate of 5% per year compound interest.
Calculate how much money he has at the end of these two years. [2]

2
North L

0 km
140 H
1600 km NOT TO
13º SCALE
W 36º

95º
J

The diagram shows the positions of four cities in Africa, Windhoek (W), Johannesburg (J), Harari (H) and
Lusaka (L).
WL = 1400 km and WH = 1600 km.
Angle LWH = 13°, angle HWJ = 36° and angle WJH = 95°.

(a) Calculate the distance LH. [4]

(b) Calculate the distance WJ. [4]

(c) Calculate the area of quadrilateral WJHL. [3]

(d) The bearing of Lusaka from Windhoek is 060°.


Calculate the bearing of

(i) Harari from Windhoek, [1]

(ii) Windhoek from Johannesburg. [1]

(e) On a map the distance between Windhoek and Harari is 8 cm.


Calculate the scale of the map in the form 1:n. [2]

© UCLES 2006 0580/04/N/06


3

3 Workmen dig a trench in level ground.

D 1.4 m C
NOT TO
SCALE
0.7 m

A B
1.1 m

(a) The cross-section of the trench is a trapezium ABCD with parallel sides of length 1.1 m and
1.4 m and a vertical height of 0.7 m.
Calculate the area of the trapezium. [2]

(b) The trench is 500 m long.


Calculate the volume of soil removed. [2]

(c) One cubic metre of soil has a mass of 4.8 tonnes.


Calculate the mass of soil removed, giving your answer in tonnes and in standard form. [2]

(d) Change your answer to part (c) into grams. [1]

D C
NOT TO
SCALE 0.2 m
m
500
A B

(e) The workmen put a cylindrical pipe, radius 0.2 m and length 500 m, along the bottom of the
trench, as shown in the diagram.
Calculate the volume of the cylindrical pipe. [2]

(f) The trench is then refilled with soil.


Calculate the volume of soil put back into the trench as a percentage of the original amount of
soil removed. [3]

© UCLES 2006 0580/04/N/06 [Turn Over


4

4 Answer the whole of this question on a sheet of graph paper.

1
f(x) = 3x − +3, x ≠ 0.
x2
(a) The table shows some values of f(x).

x −3 −2.5 −2 −1.5 −1 −0.5 −0.4 −0.3 0.3 0.4 0.5 1 1.5 2 2.5 3

f(x) p −4.7 −3.3 −1.9 −1 −2.5 −4.5 −9.0 −7.2 −2.1 0.5 q 7.1 8.8 10.3 r

Find the values of p, q and r. [3]

(b) Draw axes using a scale of 1 cm to represent 0.5 units for −3 x 3 and 1 cm to represent
2 units for −10 y 12. [1]

(c) On your grid, draw the graph of y = f(x) for −3 x −0.3 and 0.3 x 3. [5]

(d) Use your graph to solve the equations

1
(i) 3x − +3 = 0, [1]
x2

1
(ii) 3x − +7 = 0. [3]
x2

(e) g(x) = 3x + 3.
On the same grid, draw the graph of y = g(x) for −3 x 3. [2]

(f) (i) Describe briefly what happens to the graphs of y = f(x) and y = g(x) for large positive or
negative values of x. [1]

(ii) Estimate the gradient of y = f(x) when x = 100. [1]

5 Give your answers to this question as fractions.


2
(a) The probability that it rains today is 3
.

3
If it rains today, the probability that it will rain tomorrow is 4
.

1
If it does not rain today, the probability that it will rain tomorrow is 6
.

The tree diagram below shows this information.

Today Tomorrow
3
4
rain
2
3 rain
t no rain
1
6 rain
s no rain
u no rain

© UCLES 2006 0580/04/N/06


5

(i) Write down, as fractions, the values of s, t and u. [3]


(ii) Calculate the probability that it rains on both days. [2]
(iii) Calculate the probability that it will not rain tomorrow. [2]

1
(b) Each time Christina throws a ball at a target, the probability that she hits the target is 3
.

She throws the ball three times.


Find the probability that she hits the target

(i) three times, [2]


(ii) at least once. [2]

1
(c) Each time Eduardo throws a ball at the target, the probability that he hits the target is 4
.

He throws the ball until he hits the target.


Find the probability that he first hits the target with his

(i) 4th throw, [2]


(ii) nth throw. [1]

6 (a)
Q M

NOT TO
SCALE q L

O p P

OPMQ is a parallelogram and O is the origin.


= p and = q.
L is on PQ so that PL : LQ = 2:1.

Find the following vectors in terms of p and q. Write your answers in their simplest form.

(i) PQ, [1]


(ii) PL , [1]
(iii) ML , [2]
(iv) the position vector of L. [2]
 3 
(b) R is the point (1,2). It is translated onto the point S by the vector   .
 − 4
(i) Write down the co-ordinates of S. [1]
(ii) Write down the vector which translates S onto R. [1]

 0 1
(c) The matrix   represents a single transformation.
 −1 0
(i) Describe fully this transformation. [3]
(ii) Find the co-ordinates of the image of the point (5, 3) after this transformation. [1]

(d) Find the matrix which represents a reflection in the line y = x. [2]

© UCLES 2006 0580/04/N/06 [Turn Over


6

7 (a) The quiz scores of a class of n students are shown in the table.

Quiz score 6 7 8 9
Frequency (number of students) 9 3 a 5

The mean score is 7.2. Find

(i) a, [3]

(ii) n, [1]

(iii) the median score. [1]

(b) 200 students take a mathematics test.


The cumulative frequency diagram shows the results.

200

180

160

140

120

Cumulative
100
frequency
( x marks)
80

60

40

20

0
10 20 30 40 50
Mark (x)
Write down

(i) the median mark, [1]

(ii) the lower quartile, [1]

(iii) the upper quartile, [1]

(iv) the inter-quartile range, [1]

(v) the lowest possible mark scored by the top 40 students, [1]

(vi) the number of students scoring more than 25 marks. [1]

© UCLES 2006 0580/04/N/06


7

(c) Another group of students takes an English test.


The results are shown in the histogram.

Frequency 3
density
2

0
10 20 30 40 50 60 70 80 90 100
Mark (x)
100 students score marks in the range 50 < x 75.

(i) How many students score marks in the range 0 < x 50? [1]

(ii) How many students score marks in the range 75 < x 100? [1]

(iii) Calculate an estimate of the mean mark of this group of students. [4]

8 (a) The surface area, A, of a cylinder, radius r and height h, is given by the formula
A = 2πrh + 2πr2.
(i) Calculate the surface area of a cylinder of radius 5 cm and height 9 cm. [2]

(ii) Make h the subject of the formula. [2]

(iii) A cylinder has a radius of 6 cm and a surface area of 377 cm2.


Calculate the height of this cylinder. [2]

(iv) A cylinder has a surface area of 1200 cm2 and its radius and height are equal.
Calculate the radius. [3]

(b) (i) On Monday a shop receives $60.30 by selling bottles of water at 45 cents each.
How many bottles are sold? [1]

(ii) On Tuesday the shop receives x cents by selling bottles of water at 45 cents each.
In terms of x, how many bottles are sold? [1]

(iii) On Wednesday the shop receives (x – 75) cents by selling bottles of water at 48 cents each.
In terms of x, how many bottles are sold? [1]

(iv) The number of bottles sold on Tuesday was 7 more than the number of bottles sold on
Wednesday.
Write down an equation in x and solve your equation. [4]

Question 9 is on page 8

© UCLES 2006 0580/04/N/06 [Turn Over


8

9 Answer the whole of this question on a sheet of graph paper.

Tiago does some work during the school holidays.


In one week he spends x hours cleaning cars and y hours repairing cycles.
The time he spends repairing cycles is at least equal to the time he spends cleaning cars.
This can be written as y x.

He spends no more than 12 hours working.


He spends at least 4 hours cleaning cars.

(a) Write down two more inequalities in x and/or y to show this information. [3]

(b) Draw x and y axes from 0 to 12, using a scale of 1 cm to represent 1 unit on each axis. [1]

(c) Draw three lines to show the three inequalities. Shade the unwanted regions. [5]

(d) Tiago receives $3 each hour for cleaning cars and $1.50 each hour for repairing cycles.

(i) What is the least amount he could receive? [2]

(ii) What is the largest amount he could receive? [2]

Permission to reproduce items where third-party owned material protected by copyright is included has been sought and cleared where possible. Every
reasonable effort has been made by the publisher (UCLES) to trace copyright holders, but if any items requiring clearance have unwittingly been included, the
publisher will be pleased to make amends at the earliest possible opportunity.

University of Cambridge International Examinations is part of the University of Cambridge Local Examinations Syndicate (UCLES), which is itself a department
of the University of Cambridge.

© UCLES 2006 0580/04/N/06


UNIVERSITY OF CAMBRIDGE INTERNATIONAL EXAMINATIONS
International General Certificate of Secondary Education

MATHEMATICS 0580/04, 0581/04


Paper 4 (Extended) October/November 2007
2 hours 30 minutes
Additional Materials: Answer Booklet/Paper Electronic calculator
Geometrical instruments Graph paper (1 sheet)
*0356263045*

Mathematical tables (optional) Tracing paper (optional)

READ THESE INSTRUCTIONS FIRST

If you have been given an Answer Booklet, follow the instructions on the front cover of the Booklet.
Write your Centre number, candidate number and name on all the work you hand in.
Write in dark blue or black pen.
You may use a soft pencil for any diagrams or graphs.
Do not use staples, paper clips, highlighters, glue or correction fluid.

Answer all questions.


All working must be clearly shown. It should be done on the same sheet as the rest of the answer.
Marks will be given for working which shows that you know how to solve the problem even if you get the
answer wrong.
Electronic calculators should be used.
If the degree of accuracy is not specified in the question, and if the answer is not exact, give the answer to
three significant figures.
Give answers in degrees to one decimal place.
For π use either your calculator value or 3.142.

At the end of the examination, fasten all your work securely together.
The number of marks is given in brackets [ ] at the end of each question or part question.
The total of the marks for this paper is 130.

This document consists of 10 printed pages and 2 blank pages.

IB07 11_0580_04/3RP
© UCLES 2007 [Turn over
2

DO NOT WRITE YOUR ANSWERS ON THIS QUESTION PAPER.

WRITE ALL YOUR WORKING AND ANSWERS ON THE SEPARATE ANSWER BOOK OR
PAPER PROVIDED.

1 Each year a school organises a concert.

(a) (i) In 2004 the cost of organising the concert was $ 385.

In 2005 the cost was 10% less than in 2004.

Calculate the cost in 2005. [2]

(ii) The cost of $ 385 in 2004 was 10% more than the cost in 2003.

Calculate the cost in 2003. [2]

(b) (i) In 2006 the number of tickets sold was 210.

The ratio

Number of adult tickets : Number of student tickets was 23 : 19.

How many adult tickets were sold? [2]

(ii) Adult tickets were $ 2.50 each and student tickets were $ 1.50 each.

Calculate the total amount received from selling the tickets. [2]

(iii) In 2006 the cost of organising the concert was $ 410.

Calculate the percentage profit in 2006. [2]

(c) In 2007, the number of tickets sold was again 210.

Adult tickets were $ 2.60 each and student tickets were $ 1.40 each.

The total amount received from selling the 210 tickets was $ 480.

How many student tickets were sold? [4]

© UCLES 2007 0580/04/O/N/07


3

2 (a)

Grade 1 2 3 4 5 6 7

Number of students 1 2 4 7 4 8 2

The table shows the grades gained by 28 students in a history test.

(i) Write down the mode. [1]

(ii) Find the median. [1]

(iii) Calculate the mean. [3]

(iv) Two students are chosen at random.

Calculate the probability that they both gained grade 5. [2]

(v) From all the students who gained grades 4 or 5 or 6 or 7, two are chosen at random.

Calculate the probability that they both gained grade 5. [2]

(vi) Students are chosen at random, one by one, from the original 28, until the student
chosen has a grade 5.

Calculate the probability that this is the third student chosen. [2]

(b) Claude goes to school by bus.

The probability that the bus is late is 0.1 .

If the bus is late, the probability that Claude is late to school is 0.8 .

If the bus is not late, the probability that Claude is late to school is 0.05 .

(i) Calculate the probability that the bus is late and Claude is late to school. [1]

(ii) Calculate the probability that Claude is late to school. [3]

(iii) The school term lasts 56 days.

How many days would Claude expect to be late? [1]

© UCLES 2007 0580/04/O/N/07 [Turn over


4

3
y

NOT TO
SCALE

B
C

x
0

The diagram shows a sketch of y = x2 + 1 and y = 4 – x.

(a) Write down the co-ordinates of

(i) the point C, [1]

(ii) the points of intersection of y = 4 – x with each axis. [2]

(b) Write down the gradient of the line y = 4 – x. [1]

(c) Write down the range of values of x for which the gradient of the graph of y = x2 + 1 is negative. [1]

(d) The two graphs intersect at A and B.

Show that the x co-ordinates of A and B satisfy the equation x2 + x – 3 = 0. [1]

(e) Solve the equation x2 + x – 3 = 0, giving your answers correct to 2 decimal places. [4]

(f) Find the co-ordinates of the mid-point of the straight line AB. [2]

© UCLES 2007 0580/04/O/N/07


5

4 [The surface area of a sphere of radius r is 4πr2 and the volume is 4 πr 3 .]


3

(a) A solid metal sphere has a radius of 3.5 cm.

One cubic centimetre of the metal has a mass of 5.6 grams.

Calculate

(i) the surface area of the sphere, [2]

(ii) the volume of the sphere, [2]

(iii) the mass of the sphere. [2]

(b)

NOT TO
SCALE

h
8 cm

16 cm 16 cm
Diagram 1 Diagram 2

Diagram 1 shows a cylinder with a diameter of 16 cm.

It contains water to a depth of 8 cm.

Two spheres identical to the sphere in part (a) are placed in the water. This is shown in Diagram 2.

Calculate h, the new depth of water in the cylinder. [4]

(c) A different metal sphere has a mass of 1 kilogram.

One cubic centimetre of this metal has a mass of 4.8 grams.

Calculate the radius of this sphere. [3]

© UCLES 2007 0580/04/O/N/07 [Turn over


6

S
NOT TO
SCALE
D C
8 cm

P 6 cm N R

7 cm
M
A B

The diagram above shows the net of a pyramid.

The base ABCD is a rectangle 8 cm by 6 cm.

All the sloping edges of the pyramid are of length 7 cm.

M is the mid-point of AB and N is the mid-point of BC.

(a) Calculate the length of

(i) QM, [2]

(ii) RN. [1]

(b) Calculate the surface area of the pyramid. [2]

© UCLES 2007 0580/04/O/N/07


7

(c)
P
7 cm

D G C

NOT TO
H N 6 cm SCALE
X

M
A 8 cm B

The net is made into a pyramid, with P, Q, R and S meeting at P.

The mid-point of CD is G and the mid-point of DA is H.

The diagonals of the rectangle ABCD meet at X.

(i) Show that the height, PX, of the pyramid is 4.90 cm, correct to 2 decimal places. [2]

(ii) Calculate angle PNX. [2]

(iii) Calculate angle HPN. [2]

(iv) Calculate the angle between the edge PA and the base ABCD. [3]

(v) Write down the vertices of a triangle which is a plane of symmetry of the pyramid. [1]

© UCLES 2007 0580/04/O/N/07 [Turn over


8

6
18 m
D C

NOT TO
SCALE

30 m

80º
A B
26 m

The diagram shows the plan of a garden.

The garden is a trapezium with AB = 26 metres, DC = 18 metres and angle DAB = 80°.

A straight path from B to D has a length of 30 metres.

(a) (i) Using a scale of 1 : 200, draw an accurate plan of the garden. [3]

(ii) Measure and write down the size of angle ADB and the size of angle DCB. [2]

(iii) A second path is such that all points on it are equidistant from AB and from AD.

Using a straight edge and compasses only, construct this path on your plan. [2]

(iv) A third path is such that all points on it are equidistant from A and from D.

Using a straight edge and compasses only, construct this path on your plan. [2]

(v) In the garden, vegetables are grown in the region which is nearer to AB than to AD and
nearer to A than to D.

Shade this region on your plan. [1]

(b) Use trigonometry, showing all your working, to calculate

(i) angle ADB, [3]

(ii) the length of BC, [4]

(iii) the area of the garden. [3]

© UCLES 2007 0580/04/O/N/07


9

7 Answer the whole of this question on a sheet of graph paper.

(a) Draw x and y axes from 0 to 12 using a scale of 1 cm to 1 unit on each axis. [1]

(b) Draw and label triangle T with vertices (8, 6), (6, 10) and (10, 12). [1]

(c) Triangle T is reflected in the line y = x.

(i) Draw the image of triangle T. Label this image P. [2]

(ii) Write down the matrix which represents this reflection. [2]

1 0
(d) A transformation is represented by the matrix 2 1
0 2

(i) Draw the image of triangle T under this transformation. Label this image Q. [2]

(ii) Describe fully this single transformation. [3]

1
(e) Triangle T is stretched with the y-axis invariant and a stretch factor of .
2

Draw the image of triangle T. Label this image R. [2]

8 f(x) = 2x – 1, g(x) = 3 +1 , h(x) = 2x.


x

(a) Find the value of fg(6). [1]

(b) Write, as a single fraction, gf(x) in terms of x. [3]

(c) Find g−1(x). [3]

(d) Find hh(3). [2]

(e) Find x when h(x) = g − 24 [2]


7

Question 9 is printed on the next page.

© UCLES 2007 0580/04/O/N/07 [Turn over


10

9 The table shows some terms of several sequences.

Term 1 2 3 4 8

Sequence P 7 5 3 1 p

Sequence Q 1 8 27 64 q

1 2 3 4
Sequence R 5 r
2 3 4

Sequence S 4 9 16 25 s

Sequence T 1 3 9 27 t

Sequence U 3 6 7 −2 u

(a) Find the values of p, q, r, s, t and u. [6]

(b) Find the nth term of sequence

(i) P, [1]

(ii) Q, [1]

(iii) R, [1]

(iv) S, [1]

(v) T, [1]

(vi) U. [1]

(c) Which term in sequence P is equal to −777? [2]

(d) Which term in sequence T is equal to 177 147? [2]

© UCLES 2007 0580/04/O/N/07


11

BLANK PAGE

© UCLES 2007 0580/04/O/N/07


12

BLANK PAGE

Permission to reproduce items where third-party owned material protected by copyright is included has been sought and cleared where possible. Every
reasonable effort has been made by the publisher (UCLES) to trace copyright holders, but if any items requiring clearance have unwittingly been included, the
publisher will be pleased to make amends at the earliest possible opportunity.

University of Cambridge International Examinations is part of the Cambridge Assessment Group. Cambridge Assessment is the brand name of University of
Cambridge Local Examinations Syndicate (UCLES), which is itself a department of the University of Cambridge.

© UCLES 2007 0580/04/O/N/07


UNIVERSITY OF CAMBRIDGE INTERNATIONAL EXAMINATIONS
International General Certificate of Secondary Education

MATHEMATICS 0580/04, 0581/04


Paper 4 (Extended) October/November 2008
2 hours 30 minutes
Additional Materials: Answer Booklet/Paper Electronic calculator
Geometrical instruments Graph paper (1 sheet)
*1500050716*

Mathematical tables (optional) Tracing paper (optional)

READ THESE INSTRUCTIONS FIRST

If you have been given an Answer Booklet, follow the instructions on the front cover of the Booklet.
Write your Centre number, candidate number and name on all the work you hand in.
Write in dark blue or black pen.
You may use a pencil for any diagrams or graphs.
Do not use staples, paper clips, highlighters, glue or correction fluid.

Answer all questions.


All working must be clearly shown. It should be done on the same sheet as the rest of the answer.
Marks will be given for working which shows that you know how to solve the problem even if you get the
answer wrong.
Electronic calculators should be used.
If the degree of accuracy is not specified in the question, and if the answer is not exact, give the answer to
three significant figures.
Give answers in degrees to one decimal place.
For π use either your calculator value or 3.142.

At the end of the examination, fasten all your work securely together.
The number of marks is given in brackets [ ] at the end of each question or part question.
The total of the marks for this paper is 130.

This document consists of 11 printed pages and 1 blank page.

IB08 11_0580_04/4RP
© UCLES 2008 [Turn over
2

DO NOT DO ANY WORKING ON THIS QUESTION PAPER


USE THE ANSWER BOOK OR PAPER PROVIDED

1 Beatrice has an income of $40 000 in one year.

(a) She pays:

no tax on the first $10 000 of her income;

10 % tax on the next $10 000 of her income;

25 % tax on the rest of her income.

Calculate

(i) the total amount of tax Beatrice pays, [2]

(ii) the total amount of tax as a percentage of the $40 000. [2]

(b) Beatrice pays a yearly rent of $10 800.

After she has paid her tax, rent and bills, she has $12 000.

Calculate how much Beatrice spends on bills. [1]

(c) Beatrice divides the $12 000 between shopping and saving in the ratio

shopping : saving = 5 : 3.

(i) Calculate how much Beatrice spends on shopping in one year. [2]

(ii) What fraction of the original $40 000 does Beatrice save?

Give your answer in its lowest terms. [1]

(d) The rent of $10 800 is an increase of 25 % on her previous rent.

Calculate her previous rent. [2]

© UCLES 2008 0580/04/O/N/08


3

2
C

NOT TO
SCALE
x cm

y
A (x + 4) cm B

(a) When the area of triangle ABC is 48 cm2,

(i) show that x2 + 4x − 96 = 0, [2]

(ii) solve the equation x2 + 4x − 96 = 0, [2]

(iii) write down the length of AB. [1]

(b) When tan y = 1 , find the value of x. [2]


6

(c) When the length of AC is 9 cm,

(i) show that 2x2 + 8x − 65 = 0, [2]

(ii) solve the equation 2x2 + 8x − 65 = 0,

(Show your working and give your answers correct to 2 decimal places.) [4]

(iii) calculate the perimeter of triangle ABC. [1]

© UCLES 2008 0580/04/O/N/08 [Turn over


4

3 Answer the whole of this question on a sheet of graph paper.

The table shows some of the values of the function f(x) = x2 − 1 , x ≠ 0.


x

x −3 −2 −1 −0.5 −0.2 0.2 0.5 1 2 3

y 9.3 4.5 2.0 2.3 p −5.0 −1.8 q 3.5 r

(a) Find the values of p, q and r, correct to 1 decimal place. [3]

(b) Using a scale of 2 cm to represent 1 unit on the x-axis and 1 cm to represent 1 unit on the y-axis, draw
an x-axis for −3 Y x Y 3 and a y-axis for –6 Y y Y 10.

Draw the graph of y = f(x) for −3 Y x Y − 0.2 and 0.2 Y x Y 3. [6]

(c) (i) By drawing a suitable straight line, find the three values of x where f(x) = −3x. [3]

1
(ii) x2 – = –3x can be written as x3 + ax2 + b = 0.
x
Find the values of a and b. [2]

(d) Draw a tangent to the graph of y = f(x) at the point where x = –2.

Use it to estimate the gradient of y = f(x) when x = –2. [3]

© UCLES 2008 0580/04/O/N/08


5

4
NOT TO
SCALE D
D

E C
E C O

15 cm

A B
A B 50 cm

Diagram 1 Diagram 2

Diagram 1 shows a solid wooden prism of length 50 cm.

The cross-section of the prism is a regular pentagon ABCDE.

The prism is made by removing 5 identical pieces of wood from a solid wooden cylinder.

Diagram 2 shows the cross-section of the cylinder, centre O, radius 15 cm.

(a) Find the angle AOB. [1]

(b) Calculate

(i) the area of triangle AOB, [2]

(ii) the area of the pentagon ABCDE, [1]

(iii) the volume of wood removed from the cylinder. [4]

(c) Calculate the total surface area of the prism. [4]

© UCLES 2008 0580/04/O/N/08 [Turn over


6

5
North
North

40 km B
80° 115°
A
NOT TO
SCALE

60 km

Island

To avoid an island, a ship travels 40 kilometres from A to B and then 60 kilometres from B to C.

The bearing of B from A is 080° and angle ABC is 115°.

(a) The ship leaves A at 11 55.

It travels at an average speed of 35 km / h.

Calculate, to the nearest minute, the time it arrives at C. [3]

(b) Find the bearing of

(i) A from B, [1]

(ii) C from B. [1]

(c) Calculate the straight line distance AC. [4]

(d) Calculate angle BAC. [3]

(e) Calculate how far C is east of A. [3]

© UCLES 2008 0580/04/O/N/08


7

6 (a) Each student in a class is given a bag of sweets.

The students note the number of sweets in their bag.

The results are shown in the table, where 0 Y x I 10.

Number of sweets 30 31 32

Frequency (number of bags) 10 7 x

(i) State the mode. [1]

(ii) Find the possible values of the median. [3]

(iii) The mean number of sweets is 30.65.

Find the value of x. [3]

(b) The mass, m grams, of each of 200 chocolates is noted and the results are shown in the table.

Mass (m grams) 10 I m Y 20 20 I m Y=22 22 I m Y24 24 I m Y30

Frequency 35 115 26 24

(i) Calculate an estimate of the mean mass of a chocolate. [4]

(ii) On a histogram, the height of the column for the 20 I m Y=22 interval is 11.5 cm.

Calculate the heights of the other three columns.

Do not draw the histogram. [5]

© UCLES 2008 0580/04/O/N/08 [Turn over


8

7
y

6
Q T
4
U S
2

x
–8 –6 –4 –2 0 2 4 6 8
–2

–4
P
–6
R

–8

The diagram shows triangles P, Q, R, S, T and U.

(a) Describe fully the single transformation which maps triangle

(i) T onto P, [2]

(ii) Q onto T, [2]

(iii) T onto R, [2]

(iv) T onto S, [3]

(v) U onto Q. [3]

(b) Find the 2 by 2 matrix representing the transformation which maps triangle

(i) T onto R, [2]

(ii) U onto Q. [2]

© UCLES 2008 0580/04/O/N/08


9

8
D C
y° x°

E

NOT TO
SCALE

36°
78°
A B

ABCDE is a pentagon.
A circle, centre O, passes through the points A, C, D and E.
Angle EAC = 36°, angle CAB = 78° and AB is parallel to DC.

(a) Find the values of x, y and z, giving a reason for each. [6]

(b) Explain why ED is not parallel to AC. [1]

(c) Find the value of angle EOC. [1]

(d) AB = AC.
Find the value of angle ABC. [1]

© UCLES 2008 0580/04/O/N/08 [Turn over


10

9 In a survey, 100 students are asked if they like basketball (B), football (F) and swimming (S).

The Venn diagram shows the results.

F
B

25 q
20

17
12 p

8
r

42 students like swimming.

40 students like exactly one sport.

(a) Find the values of p, q and r. [3]

(b) How many students like

(i) all three sports, [1]

(ii) basketball and swimming but not football? [1]

(c) Find

(i) n(B′ ), [1]

(ii) n((B∪F )∩S ′ ). [1]

(d) One student is chosen at random from the 100 students.


Find the probability that the student

(i) only likes swimming, [1]

(ii) likes basketball but not swimming. [1]

(e) Two students are chosen at random from those who like basketball.

Find the probability that they each like exactly one other sport. [3]

© UCLES 2008 0580/04/O/N/08


11

10 1 + 2 + 3 + 4 + 5 + ……………………………. + n = n(n +1)


2

(a) (i) Show that this formula is true for the sum of the first 8 natural numbers. [2]

(ii) Find the sum of the first 400 natural numbers. [1]

(b) (i) Show that 2 + 4 + 6 + 8 + ………………... + 2n = n(n + 1). [1]

(ii) Find the sum of the first 200 even numbers. [1]

(iii) Find the sum of the first 200 odd numbers. [1]

(c) (i) Use the formula at the beginning of the question to find the sum of the first 2n natural
numbers. [1]

(ii) Find a formula, in its simplest form, for

1 + 3 + 5 + 7 + 9 + …………………... + (2n – 1).

Show your working. [2]

© UCLES 2008 0580/04/O/N/08


12

BLANK PAGE

Permission to reproduce items where third-party owned material protected by copyright is included has been sought and cleared where possible. Every
reasonable effort has been made by the publisher (UCLES) to trace copyright holders, but if any items requiring clearance have unwittingly been included, the
publisher will be pleased to make amends at the earliest possible opportunity.

University of Cambridge International Examinations is part of the Cambridge Assessment Group. Cambridge Assessment is the brand name of University of
Cambridge Local Examinations Syndicate (UCLES), which is itself a department of the University of Cambridge.

0580/04/O/N/08
w
w
w
.X
tr
me
eP
ap
UNIVERSITY OF CAMBRIDGE INTERNATIONAL EXAMINATIONS

er
s
International General Certificate of Secondary Education

.c
om
*8557670727*

MATHEMATICS 0580/04
Paper 4 (Extended) October/November 2009
2 hours 30 minutes
Candidates answer on the Question Paper.
Additional Materials: Electronic calculator Geometrical instruments
Mathematical tables (optional) Tracing paper (optional)

READ THESE INSTRUCTIONS FIRST

Write your Centre number, candidate number and name on all the work you hand in.
Write in dark blue or black pen.
You may use a soft pencil for any diagrams or graphs.
Do not use staples, paper clips, highlighters, glue or correction fluid.

Answer all questions.


If working is needed for any question it must be shown below that question.
Electronic calculators should be used.
If the degree of accuracy is not specified in the question, and if the answer is not exact, give the answer to
three significant figures. Give answers in degrees to one decimal place.
For π use either your calculator value or 3.142.

At the end of the examination, fasten all your work securely together.
The number of marks is given in brackets [ ] at the end of each question or part question.
The total of the marks for this paper is 130.

For Examiner's Use

This document consists of 21 printed pages and 3 blank pages.

IB09 11_0580_04/7RP
© UCLES 2009 [Turn over
2

1 Chris goes to a shop to buy meat, vegetables and fruit. For


Examiner's
Use
(a) (i) The costs of the meat, vegetables and fruit are in the ratio

meat : vegetables : fruit = 2 : 2 : 3.

The cost of the meat is $2.40.

Calculate the total cost of the meat, vegetables and fruit.

Answer(a)(i) $ [2]

(ii) Chris pays with a $20 note.

What percentage of the $20 has he spent?

Answer(a)(ii) % [2]

(b) The masses of the meat, vegetables and fruit are in the ratio

meat : vegetables : fruit = 1 : 8 : 3.

The total mass is 9 kg.

Calculate the mass of the vegetables.

Answer(b) kg [2]

© UCLES 2009 0580/04/O/N/09


3

(c) Calculate the cost per kilogram of the fruit. For


Examiner's
Use

Answer(c) $ [3]

(d) The cost of the meat, $2.40, is an increase of 25% on the cost the previous week.

Calculate the cost of the meat the previous week.

Answer(d) $ [2]

© UCLES 2009 0580/04/O/N/09 [Turn over


4

2 For
y Examiner's
Use
8

6
X
5

2
U T
1

–6 –5 –4 –3 –2 –1 0 1 2 3 4 5 6 7 x

–1
V
–2
W
–3

–4

–5

–6

(a) Describe fully the single transformation which maps

(i) triangle T onto triangle U,

Answer(a)(i) [2]

(ii) triangle T onto triangle V,

Answer(a)(ii) [3]

© UCLES 2009 0580/04/O/N/09


5

(iii) triangle T onto triangle W, For


Examiner's
Use

Answer(a)(iii) [3]

(iv) triangle U onto triangle X.

Answer(a)(iv) [3]

(b) Find the matrix representing the transformation which maps

(i) triangle U onto triangle V,

Answer(b)(i) [2]

(ii) triangle U onto triangle X.

Answer(b)(ii) [2]

© UCLES 2009 0580/04/O/N/09 [Turn over


6

3 For
Examiner's
Use

1 1 6 7 11 12

Six cards are numbered 1, 1, 6, 7, 11 and 12.

In this question, give all probabilities as fractions.

(a) One of the six cards is chosen at random.

1
(i) Which number has a probability of being chosen of ?
3
Answer(a)(i) [1]

(ii) What is the probability of choosing a card with a number which is


smaller than at least three of the other numbers?

Answer(a)(ii) [1]

(b) Two of the six cards are chosen at random, without replacement.

Find the probability that

(i) they are both numbered 1,

Answer(b)(i) [2]

(ii) the total of the two numbers is 18,

Answer(b)(ii) [3]

© UCLES 2009 0580/04/O/N/09


7

(iii) the first number is not a 1 and the second number is a 1. For
Examiner's
Use

Answer(b)(iii) [2]

(c) Cards are chosen, without replacement, until a card numbered 1 is chosen.

Find the probability that this happens before the third card is chosen.

Answer(c) [2]

(d) A seventh card is added to the six cards shown in the diagram.
The mean value of the seven numbers on the cards is 6.

Find the number on the seventh card.

Answer(d) [2]

© UCLES 2009 0580/04/O/N/09 [Turn over


8

4 For
C Examiner's
Use
NOT TO
SCALE
140 m 180 m

A 240 m B

The boundary of a park is in the shape of a triangle ABC.


AB = 240 m, BC = 180 m and CA = 140 m.

In part (a), show clearly all your construction arcs.

(a) (i) Using a scale of 1 centimetre to represent 20 metres, construct an accurate scale drawing
of triangle ABC. The line AB has already been drawn for you.

A B

[2]
(ii) Using a straight edge and compasses only, construct the bisector of angle ACB.

Label the point D, where this bisector meets AB. [2]

(iii) Using a straight edge and compasses only, construct the locus of points, inside the triangle,
which are equidistant from A and from D. [2]

(iv) Flowers are planted in the park so that they are nearer to AC than to BC and nearer
to D than to A.

Shade the region inside your triangle which shows where the flowers are planted. [1]

© UCLES 2009 0580/04/O/N/09


9

In part (b), use trigonometry. For


You must show your working and must NOT use any measurements from your construction in Examiner's
Use
part (a).

(b) (i) Show clearly that angle ACB is 96.4°.

Answer(b)(i)

[3]

(ii) Calculate the area of the park.

Answer(b)(ii) m2 [2]

(iii) Use the sine rule to calculate angle ABC.

Answer(b)(iii) Angle ABC = [3]

© UCLES 2009 0580/04/O/N/09 [Turn over


10

5 For
NOT TO Examiner's
D C SCALE Use

S R

(x + 3) cm
x cm

A (2x + 5) cm B P (x + 4) cm Q

The diagram shows two rectangles ABCD and PQRS.

AB = (2x + 5) cm, AD = (x + 3) cm, PQ = (x + 4) cm and PS = x cm.

(a) For one value of x, the area of rectangle ABCD is 59 cm2 more than the area of rectangle PQRS.

(i) Show that x2 + 7x − 44 = 0.

Answer(a)(i)

[3]
(ii) Factorise x2 + 7x − 44.

Answer(a)(ii) [2]

(iii) Solve the equation x2 + 7x − 44 = 0.

Answer(a)(iii) x = or x = [1]

(iv) Calculate the size of angle DBA.

Answer(a)(iv) Angle DBA = [2]

© UCLES 2009 0580/04/O/N/09


11

(b) For a different value of x, the rectangles ABCD and PQRS are similar. For
Examiner's
Use
(i) Show that this value of x satisfies the equation x2 − 2x − 12 = 0.

Answer(b)(i)

[3]

(ii) Solve the equation x2 − 2x − 12 = 0, giving your answers correct to 2 decimal places.

Answer(b)(ii) x = or x = [4]

(iii) Calculate the perimeter of the rectangle PQRS.

Answer(b)(iii) cm [1]

© UCLES 2009 0580/04/O/N/09 [Turn over


12

6 For
y Examiner's
Use
50

A 40
y = f(x)

30

20

y = g(x)
10 B

x
–5 –4 –3 –2 –1 0 1 2

–10

–20

–30

The graphs of y = f(x) and y = g(x) are shown above.

(a) Find the value of

(i) f(−2),

Answer(a)(i) [1]

(ii) g(0).

Answer(a)(ii) [1]

© UCLES 2009 0580/04/O/N/09


13

(b) Use the graphs to solve For


Examiner's
Use
(i) the equation f(x) = 20,

Answer(b)(i) x = or x = [2]

(ii) the equation f(x) = g(x),

Answer(b)(ii) x = or x = [2]

(iii) the inequality f(x) < g(x).

Answer(b)(iii) [1]

(c) Use the points A and B to find the gradient of y = g(x) as an exact fraction.

Answer(c) [2]

(d) On the grid, draw the graph of y = g(x) − 10. [2]

(e) (i) Draw the tangent to the graph of y = f(x) at ( −3, −27 ). [1]

(ii) Write down the equation of this tangent.

Answer(e)(ii) [1]

(f) A region, R, contains points whose co-ordinates satisfy the inequalities

−3 Y x Y −2, y Y 40 and y [ g(x).

On the grid, draw suitable lines and label this region R. [2]

© UCLES 2009 0580/04/O/N/09 [Turn over


14

7 For
A Examiner's
NOT TO Use
SCALE

60° O

24 cm

(a) The sector of a circle, centre O, radius 24 cm, has angle AOB = 60°.

Calculate

(i) the length of the arc AB,

Answer(a)(i) cm [2]

(ii) the area of the sector OAB.

Answer(a)(ii) cm2 [2]

(b) The points A and B of the sector are joined together to make a hollow cone as shown in the
diagram. The arc AB of the sector becomes the circumference of the base of the cone.

O
NOT TO
SCALE

24 cm

A
B

© UCLES 2009 0580/04/O/N/09


15

Calculate For
Examiner's
Use
(i) the radius of the base of the cone,

Answer(b)(i) cm [2]

(ii) the height of the cone,

Answer(b)(ii) cm [2]

(iii) the volume of the cone.


[The volume, V, of a cone of radius r and height h is V = 1 πr2h.]
3

Answer(b)(iii) cm3 [2]

(c) A different cone, with radius x and height y, has a volume W.

Find, in terms of W, the volume of

(i) a similar cone, with both radius and height 3 times larger,

Answer(c)(i) [1]

(ii) a cone of radius 2x and height y.

Answer(c)(ii) [1]

© UCLES 2009 0580/04/O/N/09 [Turn over


16

8 Fifty students are timed when running one kilometre. For


Examiner's
Use
The results are shown in the table.

Time 4.0 < t Y 4.5 4.5 < t Y 5.0 5.0 < t Y 5.5 5.5 < t Y 6.0 6.0 < t Y 6.5 6.5 < t Y 7.0
(t minutes)

Frequency 2 7 8 18 10 5

(a) Write down the modal time interval.

Answer(a) min [1]

(b) Calculate an estimate of the mean time.

Answer(b) min [4]

(c) A new frequency table is made from the results shown in the table above.

Time 4.0 < t Y 5.5 5.5 < t Y 6.0 6.0 < t Y 7.0
(t minutes)

Frequency 18

(i) Complete the table by filling in the two empty boxes. [1]

© UCLES 2009 0580/04/O/N/09


17

(ii) On the grid below, complete an accurate histogram to show the information in this new For
table. Examiner's
Use

40

30
Frequency
density

20

10

0 t
4 5 6 7 8
Time (minutes)

[3]

(iii) Find the number of students represented by 1 cm2 on the histogram.

Answer(c)(iii) [1]

© UCLES 2009 0580/04/O/N/09 [Turn over


18

m _3 m +4 _ For
9 (a) Solve the equation + = 7. Examiner's
4 3 Use

Answer(a) m = [4]

3 _ 2
(b) (i) y =
x _1 x +3

Find the value of y when x = 5.

Answer(b)(i) [1]

3 _ 2
(ii) Write as a single fraction.
x _1 x +3

Answer(b)(ii) [2]

© UCLES 2009 0580/04/O/N/09


19

3 _ 2 1 For
(iii) Solve the equation _ = . Examiner's
x 1 x +3 x Use

Answer(b)(iii) x = [3]

t
(c) p=
q_ 1

Find q in terms of p and t.

Answer(c) q = [3]

© UCLES 2009 0580/04/O/N/09 [Turn over


20

10 For
Examiner's
Total Use

Row 1 1 = 1

Row 2 3 + 5 = 8

Row 3 7 + 9 + 11 = 27

Row 4 13 + 15 + 17 + 19 = 64

Row 5

Row 6

The rows above show sets of consecutive odd numbers and their totals.

(a) Complete Row 5 and Row 6. [2]

(b) What is the special name given to the numbers 1, 8, 27, 64…?

Answer(b) [1]

(c) Write down in terms of n,

(i) how many consecutive odd numbers there are in Row n,

Answer(c)(i) [1]

(ii) the total of these numbers.

Answer(c)(ii) [1]

(d) The first number in Row n is given by n2 − n + 1.

Show that this formula is true for Row 4.

Answer(d)

[1]

© UCLES 2009 0580/04/O/N/09


21

(e) The total of Row 3 is 27. This can be calculated by (3 × 7) + 2 + 4. For


Examiner's
Use
The total of Row 4 is 64. This can be calculated by (4 × 13) + 2 + 4 + 6.

The total of Row 7 is 343. Show how this can be calculated in the same way.

Answer(e)

[1]

(f) The total of the first n even numbers is n(n + 1).

Write down a formula for the total of the first (n – 1) even numbers.

Answer(f) [1]

(g) Use the results of parts (d), (e) and (f) to show clearly that the total of the numbers in Row n
gives your answer to part (c)(ii).

Answer(g)

[2]

© UCLES 2009 0580/04/O/N/09


22

BLANK PAGE

0580/04/O/N/09
23

BLANK PAGE

0580/04/O/N/09
24

BLANK PAGE

Permission to reproduce items where third-party owned material protected by copyright is included has been sought and cleared where possible. Every
reasonable effort has been made by the publisher (UCLES) to trace copyright holders, but if any items requiring clearance have unwittingly been included, the
publisher will be pleased to make amends at the earliest possible opportunity.

University of Cambridge International Examinations is part of the Cambridge Assessment Group. Cambridge Assessment is the brand name of University of
Cambridge Local Examinations Syndicate (UCLES), which is itself a department of the University of Cambridge.

0580/04/O/N/09
UNIVERSITY OF CAMBRIDGE INTERNATIONAL EXAMINATIONS
International General Certificate of Secondary Education
*7157806085*

MATHEMATICS 0580/41
Paper 4 (Extended) October/November 2010
2 hours 30 minutes
Candidates answer on the Question Paper.
Additional Materials: Electronic calculator Geometrical instruments
Mathematical tables (optional) Tracing paper (optional)

READ THESE INSTRUCTIONS FIRST

Write your Centre number, candidate number and name on all the work you hand in.
Write in dark blue or black pen.
You may use a pencil for any diagrams or graphs.
Do not use staples, paper clips, highlighters, glue or correction fluid.
DO NOT WRITE IN ANY BARCODES.

Answer all questions.


If working is needed for any question it must be shown below that question.
Electronic calculators should be used.
If the degree of accuracy is not specified in the question, and if the answer is not exact, give the answer to
three significant figures. Give answers in degrees to one decimal place.
For π use either your calculator value or 3.142.

At the end of the examination, fasten all your work securely together.
The number of marks is given in brackets [ ] at the end of each question or part question.
The total of the marks for this paper is 130.

This document consists of 16 printed pages.

IB10 11_0580_41/5RP
© UCLES 2010 [Turn over
2

1 (a) In 2008 the total number of tickets sold for an athletics meeting was 3136. For
The ratio child tickets sold : adult tickets sold = 17 : 32. Examiner's
Use

(i) How many child tickets were sold?

Answer(a)(i) [2]

(ii) Child tickets cost $2 each and adult tickets cost $4.50 each.

Show that the total amount received from the sale of the tickets in 2008 was $11 392.

Answer(a)(ii)

[2]

(b) In 2009 the amount received from the sale of tickets for the athletics meeting was $12 748.

Calculate the percentage increase in the amount received from 2008 to 2009.

Answer(b) % [3]

(c) In 2008 the amount of $11 392 was 28% more than the amount received in 2007.

Calculate how much was received in 2007.

Answer(c) $ [3]

© UCLES 2010 0580/41/O/N/10


3

2 (a) For
y Examiner's
Use
5

2
A
1

x
–5 –4 –3 –2 –1 0 1 2 3 4 5
–1

–2

–3

–4

–5

(i) Draw the image when triangle A is reflected in the line y = 0.


Label the image B. [2]

(ii) Draw the image when triangle A is rotated through 90U anticlockwise about the origin.
Label the image C. [2]

(iii) Describe fully the single transformation which maps triangle B onto triangle C.

Answer(a)(iii) [2]

(b) Rotation through 90U anticlockwise about the origin is represented by the matrix M = 
0 −1 
.
1 0
(i) Find M–1, the inverse of matrix M.

 
–1  
Answer(b)(i) M =  
 
  [2]

(ii) Describe fully the single transformation represented by the matrix M–1.

Answer(b)(ii) [2]

© UCLES 2010 0580/41/O/N/10 [Turn over


4

3 For
Wall Examiner's
Use
NOT TO
SCALE
Enclosure x

A farmer makes a rectangular enclosure for his animals.


He uses a wall for one side and a total of 72 metres of fencing for the other three sides.

The enclosure has width x metres and area A square metres.

(a) Show that A = 72x – 2x2.

Answer (a)

[2]

(b) Factorise completely 72x – 2x2.

Answer(b) [2]

(c) Complete the table for A = 72x – 2x2.

x 0 5 10 15 20 25 30 35

A 0 310 520 550 360

[3]

(d) Draw the graph of A = 72x – 2x2 for 0 Y x Y 35 on the grid opposite.

© UCLES 2010 0580/41/O/N/10


5

A For
Examiner's
700 Use

600

500

400

300

200

100

x
0
5 10 15 20 25 30 35
[4]

(e) Use your graph to find

(i) the values of x when A = 450,

Answer(e)(i) x = or x = [2]

(ii) the maximum area of the enclosure.

Answer(e)(ii) m2 [1]

(f) Each animal must have at least 12 m2 for grazing.

Calculate the greatest number of animals that the farmer can keep in an enclosure which has an
area of 500 m2.

Answer(f) [2]

© UCLES 2010 0580/41/O/N/10 [Turn over


6

4 For
Examiner's
Use

NOT TO
SCALE
4m

1.5 m

2m
l

An open water storage tank is in the shape of a cylinder on top of a cone.


The radius of both the cylinder and the cone is 1.5 m.
The height of the cylinder is 4 m and the height of the cone is 2 m.

(a) Calculate the total surface area of the outside of the tank.
[The curved surface area, A, of a cone with radius r and slant height l is A = πrl. ]

Answer(a) m2 [6]

(b) The tank is completely full of water.

(i) Calculate the volume of water in the tank and show that it rounds to 33 m3, correct to the
nearest whole number.
1
[The volume, V, of a cone with radius r and height h is V = πr2h.]
3
Answer(b)(i)

[4]

© UCLES 2010 0580/41/O/N/10


7

(ii) For
Examiner's
Use

0.5 m NOT TO
SCALE

The cross-section of an irrigation channel is a semi-circle of radius 0.5 m.


The 33 m3 of water from the tank completely fills the irrigation channel.

Calculate the length of the channel.

Answer(b)(ii) m [3]

(c) (i) Calculate the number of litres in a full tank of 33 m3.

Answer(c)(i) litres [1]

(ii) The water drains from the tank at a rate of 1800 litres per minute.

Calculate the time, in minutes and seconds, taken to empty the tank.

Answer(c)(ii) min s [2]

© UCLES 2010 0580/41/O/N/10 [Turn over


8

5 The cumulative frequency table shows the distribution of heights, h centimetres, of 200 students. For
Examiner's
Use

Height (h cm) Y130 Y140 Y150 Y160 Y165 Y170 Y180 Y190

Cumulative frequency 0 10 50 95 115 145 180 200

(a) Draw a cumulative frequency diagram to show the information in the table.

200

160

120
Cumulative
frequency

80

40

0
130 140 150 160 170 180 190
Height (h cm)
[4]
(b) Use your diagram to find

(i) the median,


Answer(b)(i) cm [1]

(ii) the upper quartile,


Answer(b)(ii) cm [1]

(iii) the interquartile range.


Answer(b)(iii) cm [1]

(c) (i) One of the 200 students is chosen at random.

Use the table to find the probability that the height of this student is greater than 170 cm.
Give your answer as a fraction.

Answer(c)(i) [1]

© UCLES 2010 0580/41/O/N/10


9

(ii) One of the 200 students is chosen at random and then a second student is chosen at random For
from the remaining students. Examiner's
Use

Calculate the probability that one has a height greater than 170 cm and the other has a
height of 140 cm or less.
Give your answer as a fraction.

Answer(c)(ii) [3]

(d) (i) Complete this frequency table which shows the distribution of the heights of the 200
students.

Height (h cm) 130IhY140 140IhY150 150IhY160 160IhY165 165IhY170 170IhY180 180IhY190

Frequency 10 40 45 20

[2]
(ii) Complete this histogram to show the distribution of the heights of the 200 students.

Frequency
3
density

0
130 140 150 160 170 180 190

Height (h cm)
[3]

© UCLES 2010 0580/41/O/N/10 [Turn over


10

6 (a) For
A Examiner's
Use
P

19.5 cm NOT TO
16.5 cm 11 cm SCALE

Q R
B C

The diagram shows a toy boat.


AC = 16.5 cm, AB = 19.5 cm and PR = 11 cm.
Triangles ABC and PQR are similar.

(i) Calculate PQ.

Answer(a)(i) PQ = cm [2]

(ii) Calculate BC.

Answer(a)(ii) BC = cm [3]

(iii) Calculate angle ABC.

Answer(a)(iii) Angle ABC = [2]

© UCLES 2010 0580/41/O/N/10


11

(iv) The toy boat is mathematically similar to a real boat. For


The length of the real boat is 32 times the length of the toy boat. Examiner's
Use
The fuel tank in the toy boat holds 0.02 litres of diesel.

Calculate how many litres of diesel the fuel tank of the real boat holds.

Answer(a)(iv) litres [2]

(b)
E

F
32°
143°
NOT TO
67 m SCALE
105 m
70°

D G

The diagram shows a field DEFG, in the shape of a quadrilateral, with a footpath along the
diagonal DF.
DF = 105 m and FG = 67 m.
Angle EDF = 70U, angle EFD = 32U and angle DFG = 143U.

(i) Calculate DG.

Answer(b)(i) DG = m [4]

(ii) Calculate EF.

Answer(b)(ii) EF = m [4]

© UCLES 2010 0580/41/O/N/10 [Turn over


12

7 (a) For
A Examiner's
Use
y
NOT TO
SCALE
D x B
w 62°

A, B, C and D are points on the circumference of a circle centre O.


AC is a diameter.
BD = BC and angle DBC = 62U.

Work out the values of w, x, y and z.


Give a reason for each of your answers.

w= because [2]

x= because [2]

y= because [2]

z= because [2]

(b)

B (4,4)
NOT TO
SCALE

A (2,1)
x
O

(i) Write down as a column vector.

 
 
Answer(b)(i) =  
 
  [1]

© UCLES 2010 0580/41/O/N/10


13

=   .
0 For
(ii) Examiner's
7 Use
Work out as a column vector.

 
 
Answer(b)(ii) =  
 
  [2]

(c)
R

NOT TO
r SCALE
P

O Q T
t

= r and = t.
P is on RT such that RP : PT = 2 : 1.
2
Q is on OT such that OQ = OT.
3

Write the following in terms of r and/or t.


Simplify your answers where possible.

(i)

Answer(c)(i) = [1]

(ii)

Answer(c)(ii) = [2]

(iii)

Answer(c)(iii) = [2]

(iv) Write down two conclusions you can make about the line segment QP.

Answer(c)(iv)

[2]

© UCLES 2010 0580/41/O/N/10 [Turn over


14

8 (a) f(x) = 2x – 1 g(x) = x2 For


Examiner's
Use
Work out

(i) f(2),
Answer(a)(i) [1]

(ii) g( – 2),
Answer(a)(ii) [1]

(iii) ff(x) in its simplest form,

Answer(a)(iii) ff(x) = [2]

(iv) f –1(x), the inverse of f(x),

Answer(a)(iv) f –1(x) = [2]

(v) x when gf(x) = 4.

Answer(a)(v) x = or x = [4]

(b) y is inversely proportional to x and y = 8 when x = 2.

Find,

(i) an equation connecting y and x,

Answer(b)(i) [2]

1
(ii) y when x = .
2

Answer(b)(ii) y = [1]

© UCLES 2010 0580/41/O/N/10


15

9 (a) The first five terms P1, P2, P3, P4 and P5 of a sequence are given below. For
Examiner's
Use
1 = 1 = P1

1+2 = 3 = P2

1+2+3 = 6 = P3

1+2+3+4 = 10 = P4

1+2+3+4+5 = 15 = P5

(i) Write down the next term, P6, in the sequence 1, 3, 6, 10, 15…

Answer(a)(i) [1]

(ii) The formula for the nth term of this sequence is

1
Pn = n(n + 1).
2

Show this formula is true when n = 6.

Answer (a)(ii)

[1]

(iii) Use the formula to find P50, the 50th term of this sequence.

Answer(a)(iii) [1]

(iv) Use your answer to part (iii) to find 3 + 6 + 9 + 12 +15 + ………… + 150.

Answer(a)(iv) [1]

(v) Find 1 + 2 + 3 + 4 + 5 +………… + 150.

Answer(a)(v) [1]

(vi) Use your answers to parts (iv) and (v) to find the sum of the numbers less than 150 which
are not multiples of 3.

Answer(a)(vi) [1]

This question continues on the next page.

© UCLES 2010 0580/41/O/N/10 [Turn over


16

(b) The first five terms, S1, S2, S3, S4 and S5 of a different sequence are given below. For
Examiner's
Use
(1 × 1) = 1 = S1

(1 × 2) + (2 × 1) = 4 = S2

(1 × 3) + (2 × 2) + (3 × 1) = 10 = S3

(1 × 4) + (2 × 3) + (3 × 2) + (4 × 1) = 20 = S4

(1 × 5) + (2 × 4) + (3 × 3) + (4 × 2) + (5 × 1) = 35 = S5

(i) Work out the next term, S6, in the sequence 1, 4, 10, 20, 35…

Answer(b)(i) [2]

(ii) The formula for the nth term of this sequence is

1
Sn = n(n + 1)(n + 2).
6

Show this formula is true for n = 6.

Answer(b)(ii)

[1]

(iii) Find (1 × 20) + (2 × 19) + (3 × 18) ………… + (20 × 1) .

Answer(b)(iii) [1]

(c) Show that S6 – S5 = P6, where P6 is your answer to part (a)(i).

Answer(c)

[1]
1
(d) Show by algebra that Sn – Sn – 1 = Pn . [Pn = n(n + 1)]
2
Answer(d)

[3]

Permission to reproduce items where third-party owned material protected by copyright is included has been sought and cleared where possible. Every
reasonable effort has been made by the publisher (UCLES) to trace copyright holders, but if any items requiring clearance have unwittingly been included, the
publisher will be pleased to make amends at the earliest possible opportunity.

University of Cambridge International Examinations is part of the Cambridge Assessment Group. Cambridge Assessment is the brand name of University of
Cambridge Local Examinations Syndicate (UCLES), which is itself a department of the University of Cambridge.

© UCLES 2010 0580/41/O/N/10


UNIVERSITY OF CAMBRIDGE INTERNATIONAL EXAMINATIONS
International General Certificate of Secondary Education
*0866018584*

MATHEMATICS 0580/42
Paper 4 (Extended) October/November 2010
2 hours 30 minutes
Candidates answer on the Question Paper.
Additional Materials: Electronic calculator Geometrical instruments
Mathematical tables (optional) Tracing paper (optional)

READ THESE INSTRUCTIONS FIRST

Write your Centre number, candidate number and name on all the work you hand in.
Write in dark blue or black pen.
You may use a pencil for any diagrams or graphs.
Do not use staples, paper clips, highlighters, glue or correction fluid.
DO NOT WRITE IN ANY BARCODES.

Answer all questions.


If working is needed for any question it must be shown below that question.
Electronic calculators should be used.
If the degree of accuracy is not specified in the question, and if the answer is not exact, give the answer to
three significant figures. Give answers in degrees to one decimal place.
For π use either your calculator value or 3.142.

At the end of the examination, fasten all your work securely together.
The number of marks is given in brackets [ ] at the end of each question or part question.
The total of the marks for this paper is 130.

This document consists of 18 printed pages and 2 blank pages.

IB10 11_0580_42/3RP
© UCLES 2010 [Turn over
2

1 (a) Hansi and Megan go on holiday. For


The costs of their holidays are in the ratio Hansi : Megan = 7 : 4. Examiner's
Use
Hansi’s holiday costs $756.
Find the cost of Megan’s holiday.

Answer(a) $ [2]

(b) In 2008, Hansi earned $7800.

(i) He earned 15% more in 2009.


Calculate how much he earned in 2009.

Answer(b)(i) $ [2]

(ii) In 2010, he earns 10% more than in 2009.


Calculate the percentage increase in his earnings from 2008 to 2010.

Answer(b)(ii) % [3]

(c) Megan earned $9720 in 2009. This was 20% more than she earned in 2008.
How much did she earn in 2008?

Answer(c) $ [3]

(d) Hansi invested $500 at a rate of 4% per year compound interest.


Calculate the final amount he had after three years.

Answer(d) $ [3]

© UCLES 2010 0580/42/O/N/10


3

2 f(x) = 6 + x2 g(x) = 4x –1 For


Examiner's
Use
(a) Find

(i) g(3),

Answer(a)(i) [1]

(ii) f (–4 ).

Answer(a)(ii) [1]

(b) Find the inverse function g–1(x).

Answer(b) g–1(x) = [2]

(c) Find fg(x) in its simplest form.

Answer(c) fg(x) = [3]

(d) Solve the equation gg(x) = 3.

Answer(d) x = [3]

© UCLES 2010 0580/42/O/N/10 [Turn over


4

3 80 boys each had their mass, m kilograms, recorded. For


The cumulative frequency diagram shows the results. Examiner's
Use

80

60

Cumulative
40
frequency

20

0 m
30 40 50 60 70 80 90
Mass (kg)

(a) Find

(i) the median,

Answer(a)(i) kg [1]

(ii) the lower quartile,

Answer(a)(ii) kg [1]

(iii) the interquartile range.

Answer(a)(iii) kg [1]

(b) How many boys had a mass greater than 60kg?

Answer(b) [2]

© UCLES 2010 0580/42/O/N/10


5

(c) (i) Use the cumulative frequency graph to complete this frequency table. For
Examiner's
Use

Mass, m Frequency

30 I m Y 40 8

40 I m Y 50

50 I m Y 60 14

60 I m Y 70 22

70 I m Y 80

80 I m Y 90 10
[2]

(ii) Calculate an estimate of the mean mass.

Answer(c)(ii) kg [4]

© UCLES 2010 0580/42/O/N/10 [Turn over


6

4 (a) For
Examiner's
4 cm Use
NOT TO
SCALE

13 cm

The diagram shows a cone of radius 4 cm and height 13 cm.


It is filled with soil to grow small plants.
Each cubic centimetre of soil has a mass of 2.3g.

(i) Calculate the volume of the soil inside the cone.


1
[The volume, V, of a cone with radius r and height h is V = π r 2 h .]
3

Answer(a)(i) cm3 [2]

(ii) Calculate the mass of the soil.

Answer(a)(ii) g [1]

(iii) Calculate the greatest number of these cones which can be filled completely using 50 kg
of soil.

Answer(a)(iii) [2]

(b) A similar cone of height 32.5 cm is used for growing larger plants.

Calculate the volume of soil used to fill this cone.

Answer(b) cm3 [3]

© UCLES 2010 0580/42/O/N/10


7

(c) For
Examiner's
Use
NOT TO
SCALE
12 cm

Some plants are put into a cylindrical container with height 12 cm and volume 550 cm3.

Calculate the radius of the cylinder.

Answer(c) cm [3]

© UCLES 2010 0580/42/O/N/10 [Turn over


8

5 (a) For
A Examiner's
Use
NOT TO
SCALE
17 cm
x cm

B (x + 7) cm C

In the right-angled triangle ABC, AB = x cm, BC = (x + 7) cm and AC = 17 cm.

(i) Show that x2 + 7x – 120 = 0.

Answer(a)(i)

[3]

(ii) Factorise x2 + 7x – 120.

Answer(a)(ii) [2]

(iii) Write down the solutions of x2 + 7x – 120 = 0.

Answer(a)(iii) x = or x = [1]

(iv) Write down the length of BC.

Answer(a)(iv) BC = cm [1]

© UCLES 2010 0580/42/O/N/10


9

(b) For
Examiner's
NOT TO Use
SCALE

3x cm
(2x + 3) cm

(2x – 1) cm (2x + 3) cm

The rectangle and the square shown in the diagram above have the same area.

(i) Show that 2x2 – 15x – 9 = 0.

Answer(b)(i)

[3]

(ii) Solve the equation 2x2 – 15x – 9 = 0.


Show all your working and give your answers correct to 2 decimal places.

Answer(b)(ii) x = or x = [4]

(iii) Calculate the perimeter of the square.

Answer(b)(iii) cm [1]

© UCLES 2010 0580/42/O/N/10 [Turn over


10

6 For
L 5480 km Examiner's
Use
D
165° NOT TO
3300 km SCALE

The diagram shows the positions of London (L), Dubai (D) and Colombo (C).

(a) (i) Show that LC is 8710 km correct to the nearest kilometre.

Answer(a)(i)

[4]

(ii) Calculate the angle CLD.

Answer(a)(ii) Angle CLD = [3]

© UCLES 2010 0580/42/O/N/10


11

(b) A plane flies from London to Dubai and then to Colombo. For
It leaves London at 01 50 and the total journey takes 13 hours and 45 minutes. Examiner's
Use
The local time in Colombo is 7 hours ahead of London.
Find the arrival time in Colombo.

Answer(b) [2]

(c) Another plane flies the 8710 km directly from London to Colombo at an average speed of
800 km/h.
How much longer did the plane in part (b) take to travel from London to Colombo?
Give your answer in hours and minutes, correct to the nearest minute.

Answer(c) h min [4]

© UCLES 2010 0580/42/O/N/10 [Turn over


12

2 For
7 (a) Complete the table for the function f(x) = −x 2. Examiner's
x Use

x –3 –2 –1 –0.5 –0.2 0.2 0.5 1 2 3


f(x) –9.7 –5 –10.0 10.0 3.75 1 –8.3
[3]
(b) On the grid draw the graph of y = f(x) for –3 Y x Y –0.2 and 0.2 Y x Y 3.
y
10

x
–3 –2 –1 0 1 2 3

–2

–4

–6

–8

–10 [5]

© UCLES 2010 0580/42/O/N/10


13

(c) Use your graph to For


Examiner's
Use
(i) solve f(x) = 2,

Answer(c)(i) x = [1]

(ii) find a value for k so that f(x) = k has 3 solutions.

Answer(c)(ii) k = [1]

2
(d) Draw a suitable line on the grid and use your graphs to solve the equation − x 2 = 5x.
x

Answer(d) x = or x = [3]

(e) Draw the tangent to the graph of y = f(x) at the point where x = –2.

Use it to calculate an estimate of the gradient of y = f(x) when x = –2.

Answer(e) [3]

© UCLES 2010 0580/42/O/N/10 [Turn over


14

8 (a) For
y Examiner's
Use
8

4
A

A
2

x
–8 –6 –4 –2 0 2 4 6 8

–2

–4

–6

–8

Draw the images of the following transformations on the grid above.

(i) Translation of triangle A by the vector 


3
− 7
 . Label the image B. [2]
 

(ii) Reflection of triangle A in the line x = 3. Label the image C. [2]

(iii) Rotation of triangle A through 90° anticlockwise around the point (0, 0).
Label the image D. [2]

(iv) Enlargement of triangle A by scale factor –4, with centre (0, 1).
Label the image E. [2]

© UCLES 2010 0580/42/O/N/10


15

(b) The area of triangle E is k × area of triangle A. For


Write down the value of k. Examiner's
Use

Answer(b) k = [1]

(c)
y
5

1
F
x
–5 –4 –3 –2 –1 0 1 2 3 4 5
–1

–2

–3

–4

–5

(i) Draw the image of triangle F under the transformation represented by the
 1 3
matrix M =   . [3]
 0 1

(ii) Describe fully this single transformation.

Answer(c)(ii)

[3]

(iii) Find M–1, the inverse of the matrix M.

 
 
Answer(c)(iii)  
 
  [2]

© UCLES 2010 0580/42/O/N/10 [Turn over


16

9 A bag contains 7 red sweets and 4 green sweets. For


Aimee takes out a sweet at random and eats it. Examiner's
Use
She then takes out a second sweet at random and eats it.

(a) Complete the tree diagram.

First sweet Second sweet

6
red
10

7 red
11
green
..........

.......... red

.......... green

green
..........
[3]

(b) Calculate the probability that Aimee has taken

(i) two red sweets,

Answer(b)(i) [2]

(ii) one sweet of each colour.

Answer(b)(ii) [3]

© UCLES 2010 0580/42/O/N/10


17

(c) Aimee takes a third sweet at random. For


Calculate the probability that she has taken Examiner's
Use

(i) three red sweets,

Answer(c)(i) [2]

(ii) at least one red sweet.

Answer(c)(ii) [3]

© UCLES 2010 0580/42/O/N/10 [Turn over


18

10 In all the following sequences, after the first two terms, the rule is to add the previous two terms to For
find the next term. Examiner's
Use

(a) Write down the next two terms in this sequence.

1 1 2 3 5 8 13 [1]

(b) Write down the first two terms of this sequence.

3 11 14 [2]

(c) (i) Find the value of d and the value of e.

2 d e 10

Answer(c)(i) d =

e= [3]

(ii) Find the value of x, the value of y and the value of z.

O33 x y z 18

Answer(c)(ii) x =

y=

z= [5]

© UCLES 2010 0580/42/O/N/10


19

BLANK PAGE

© UCLES 2010 0580/42/O/N/10


20

BLANK PAGE

Permission to reproduce items where third-party owned material protected by copyright is included has been sought and cleared where possible. Every
reasonable effort has been made by the publisher (UCLES) to trace copyright holders, but if any items requiring clearance have unwittingly been included, the
publisher will be pleased to make amends at the earliest possible opportunity.

University of Cambridge International Examinations is part of the Cambridge Assessment Group. Cambridge Assessment is the brand name of University of
Cambridge Local Examinations Syndicate (UCLES), which is itself a department of the University of Cambridge.

© UCLES 2010 0580/42/O/N/10


UNIVERSITY OF CAMBRIDGE INTERNATIONAL EXAMINATIONS
International General Certificate of Secondary Education
*8742282716*

MATHEMATICS 0580/43
Paper 4 (Extended) October/November 2010
2 hours 30 minutes
Candidates answer on the Question Paper.
Additional Materials: Electronic calculator Geometrical instruments
Mathematical tables (optional) Tracing paper (optional)

READ THESE INSTRUCTIONS FIRST

Write your Centre number, candidate number and name on all the work you hand in.
Write in dark blue or black pen.
You may use a pencil for any diagrams or graphs.
Do not use staples, paper clips, highlighters, glue or correction fluid.
DO NOT WRITE IN ANY BARCODES.

Answer all questions.


If working is needed for any question it must be shown below that question.
Electronic calculators should be used.
If the degree of accuracy is not specified in the question, and if the answer is not exact, give the answer to
three significant figures. Give answers in degrees to one decimal place.
For π use either your calculator value or 3.142.

At the end of the examination, fasten all your work securely together.
The number of marks is given in brackets [ ] at the end of each question or part question.
The total of the marks for this paper is 130.

This document consists of 19 printed pages and 1 blank page.

IB10 11_0580_43/4RP
© UCLES 2010 [Turn over
2

1 Thomas, Ursula and Vanessa share $200 in the ratio For


Examiner's
Use
Thomas : Ursula : Vanessa = 3 : 2 : 5.

(a) Show that Thomas receives $60 and Ursula receives $40.

Answer(a)

[2]

(b) Thomas buys a book for $21.


What percentage of his $60 does Thomas have left?

Answer(b) % [2]

(c) Ursula buys a computer game for $36.80 in a sale.


The sale price is 20% less than the original price.
Calculate the original price of the computer game.

Answer(c) $ [3]

(d) Vanessa buys some books and some pencils.


Each book costs $12 more than each pencil.
The total cost of 5 books and 2 pencils is $64.20.
Find the cost of one pencil.

Answer(d) $ [3]

© UCLES 2010 0580/43/O/N/10


3

2 For
R 4 km Examiner's
Q Use
NOT TO
SCALE
7 km
4.5 km
85°
S 40°
P

The diagram shows five straight roads.


PQ = 4.5 km, QR = 4 km and PR = 7 km.
Angle RPS = 40° and angle PSR = 85°.

(a) Calculate angle PQR and show that it rounds to 110.7°.

Answer(a)

[4]

(b) Calculate the length of the road RS and show that it rounds to 4.52 km.

Answer(b)

[3]

(c) Calculate the area of the quadrilateral PQRS.


[Use the value of 110.7° for angle PQR and the value of 4.52 km for RS.]

Answer(c) km2 [5]

© UCLES 2010 0580/43/O/N/10 [Turn over


4

3 (a) Expand the brackets and simplify. For


Examiner's
x ( x + 3) + 4 x ( x −1) Use

Answer(a) [2]

(b) Simplify (3 x 3 ) 3 .

Answer(b) [2]

(c) Factorise the following completely.

(i) 7 x 7 + 14 x14

Answer(c)(i) [2]

(ii) xy + xw + 2ay + 2aw

Answer(c)(ii) [2]

(iii) 4 x 2 − 49

Answer(c)(iii) [1]

© UCLES 2010 0580/43/O/N/10


5

(d) Solve the equation. For


2 Examiner's
2x + 5x + 1 = 0
Use

Show all your working and give your answers correct to 2 decimal places.

Answer(d) x = or x = [4]

© UCLES 2010 0580/43/O/N/10 [Turn over


6

4 (a) For
Examiner's
 2 3  2
A=   B=   C = (1 2 ) Use
 4 5 7

Find the following matrices.

(i) AB

Answer(a)(i) [2]

(ii) CB

Answer(a)(ii) [2]

(iii) A-1, the inverse of A

Answer(a)(iii) [2]

1 0
(b) Describe fully the single transformation represented by the matrix  _ .
0 1 

Answer(b) [2]

(c) Find the 2 by 2 matrix that represents an anticlockwise rotation of 90° about the origin.

 
 
 
Answer(c)  
 
 
  [2]

© UCLES 2010 0580/43/O/N/10


7

5 For
Examiner's
Use
C

The diagram shows an area of land ABCD used for a shop, a car park and gardens.

(a) Using a straight edge and compasses only, construct

(i) the locus of points equidistant from C and from D, [2]

(ii) the locus of points equidistant from AD and from AB. [2]

(b) The shop is on the land nearer to D than to C and nearer to AD than to AB.

Write the word SHOP in this region on the diagram. [1]

(c) (i) The scale of the diagram is 1 centimetre to 20 metres.


The gardens are the part of the land less than 100 m from B.
Draw the boundary for the gardens. [1]

(ii) The car park is the part of the land not used for the shop and not used for the gardens.
Shade the car park region on the diagram. [1]

© UCLES 2010 0580/43/O/N/10 [Turn over


8

6 Sacha either walks or cycles to school. For


3 Examiner's
On any day, the probability that he walks to school is . Use
5

(a) (i) A school term has 55 days.

Work out the expected number of days Sacha walks to school.

Answer(a)(i) [1]

(ii) Calculate the probability that Sacha walks to school on the first 5 days of the term.

Answer(a)(ii) [2]

1
(b) When Sacha walks to school, the probability that he is late is .
4
1
When he cycles to school, the probability that he is late is .
8

(i) Complete the tree diagram by writing the probabilities in the four spaces provided.

1
late
4

3 walks
5
not late
..........

.......... late

.......... cycles

not late
..........

[3]

© UCLES 2010 0580/43/O/N/10


9

(ii) Calculate the probability that Sacha cycles to school and is late. For
Examiner's
Use

Answer(b)(ii) [2]

(iii) Calculate the probability that Sacha is late to school.

Answer(b)(iii) [2]

© UCLES 2010 0580/43/O/N/10 [Turn over


10

x3 For
7 (a) Complete the table for the function f(x) = +1. Examiner's
10 Use

x –4 –3 –2 –1 0 1 2 3

f(x) –1.7 0.2 0.9 1 1.1 1.8

[2]

(b) On the grid, draw the graph of y = f(x) for –4 Y x Y=3.

x
–4 –3 –2 –1 0 1 2 3
–1

–2

–3

–4

–5

–6

–7

–8

[4]

4
(c) Complete the table for the function g(x) = , x ≠ 0.
x

x –4 –3 –2 –1 1 2 3

g(x) –1 –1.3 2 1.3


[2]

© UCLES 2010 0580/43/O/N/10


11

(d) On the grid, draw the graph of y = g(x) for –4 Y x Y –1 and 1 Y x Y 3. [3] For
Examiner's
Use

x3 4
(e) (i) Use your graphs to solve the equation +1 = .
10 x

Answer(e)(i) x = or x = [2]

x3 4
(ii) The equation +1 = can be written as x 4 + ax + b = 0 .
10 x

Find the values of a and b.

Answer(e)(ii) a =

b= [2]

© UCLES 2010 0580/43/O/N/10 [Turn over


12

8 For
Examiner's
NOT TO Use
SCALE

3 cm

12 cm

The diagram shows a solid made up of a hemisphere and a cylinder.


The radius of both the cylinder and the hemisphere is 3 cm.
The length of the cylinder is 12 cm.

(a) (i) Calculate the volume of the solid.


4
[ The volume, V, of a sphere with radius r is V = πr 3 .]
3

Answer(a)(i) cm3 [4]

(ii) The solid is made of steel and 1 cm3 of steel has a mass of 7.9 g.
Calculate the mass of the solid.
Give your answer in kilograms.

Answer(a)(ii) kg [2]

© UCLES 2010 0580/43/O/N/10


13

(iii) The solid fits into a box in the shape of a cuboid, 15 cm by 6 cm by 6 cm. For
Calculate the volume of the box not occupied by the solid. Examiner's
Use

Answer(a)(iii) cm3 [2]

(b) (i) Calculate the total surface area of the solid.


You must show your working.
[ The surface area, A, of a sphere with radius r is A = 4πr 2 .]

Answer(b)(i) cm2 [5]

(ii) The surface of the solid is painted.


The cost of the paint is $0.09 per millilitre.
One millilitre of paint covers an area of 8 cm2.
Calculate the cost of painting the solid.

Answer(b)(ii) $ [2]

© UCLES 2010 0580/43/O/N/10 [Turn over


14

9 (a) For
y Examiner's
Use
3
A
2

1
x
–4 –3 –2 –1 0 1 2 3 4 5
–1
C
–2

–3
B
–4

The points A (5, 3), B (1, –4) and C (–4, –2) are shown in the diagram.

(i) Write as a column vector.

 
 
 
Answer(a)(i) =  
 
 
  [1]

(ii) Find – as a single column vector.

 
 
 
Answer(a)(ii)  
 
 
  [2]

(iii) Complete the following statement.

– = [1]

(iv) Calculate .

Answer(a)(iv) [2]

© UCLES 2010 0580/43/O/N/10


15

(b) For
D u C Examiner's
Use
NOT TO
SCALE
t
M

A B

1
ABCD is a trapezium with DC parallel to AB and DC = AB.
2

M is the midpoint of BC.

= t and = u.

Find the following vectors in terms of t and / or u.

Give each answer in its simplest form.

(i)

Answer(b)(i) = [1]

(ii)

Answer(b)(ii) = [2]

(iii)

Answer(b)(iii) = [2]

© UCLES 2010 0580/43/O/N/10 [Turn over


16

10 (a) For a set of six integers, the mode is 8, the median is 9 and the mean is 10. For
Examiner's
Use
The smallest integer is greater than 6 and the largest integer is 16.

Find the two possible sets of six integers.

Answer(a) First set , , , , ,

Second set , , , , , [5]

(b) One day Ahmed sells 160 oranges.


He records the mass of each orange.
The results are shown in the table.

Mass (m grams) 50 < m Y 80 80 < m Y 90 90 < m Y 100 100 < m Y 120 120 < m Y 150

Frequency 30 35 40 40 15

(i) Calculate an estimate of the mean mass of the 160 oranges.

Answer(b)(i) g [4]

© UCLES 2010 0580/43/O/N/10


17

(ii) On the grid, complete the histogram to show the information in the table. For
Examiner's
Use
5

3
Frequency
density
2

0 m
50 60 70 80 90 100 110 120 130 140 150

Mass (grams)
[4]

Question 11 is printed on the next page.

© UCLES 2010 0580/43/O/N/10 [Turn over


18

11 For
Examiner's
Use

Diagram 1 Diagram 2 Diagram 3 Diagram 4

The first four Diagrams in a sequence are shown above.


Each Diagram is made from dots and one centimetre lines.
The area of each small square is 1 cm2.

(a) Complete the table for Diagrams 5 and 6.

Diagram 1 2 3 4 5 6

Area (cm2) 2 6 12 20

Number of dots 6 12 20 30

Number of one centimetre lines 7 17 31 49


[4]

(b) The area of Diagram n is n(n + 1) cm2.

(i) Find the area of Diagram 50.

Answer(b)(i) cm2 [1]

(ii) Which Diagram has an area of 930 cm2?

Answer(b)(ii) [1]

(c) Find, in terms of n, the number of dots in Diagram n.

Answer(c) [1]

© UCLES 2010 0580/43/O/N/10


19

(d) The number of one centimetre lines in Diagram n is 2n 2 + pn + 1 . For


Examiner's
Use
(i) Show that p = 4.

Answer(d)(i)

[2]

(ii) Find the number of one centimetre lines in Diagram 10.

Answer(d)(ii) [1]

(iii) Which Diagram has 337 one centimetre lines?

Answer(d)(iii) [3]

(e) For each Diagram, the number of squares of area 1 cm2 is A, the number of dots is D and the
number of one centimetre lines is L.

Find a connection between A, D and L that is true for each Diagram.

Answer(e) [1]

© UCLES 2010 0580/43/O/N/10


20

BLANK PAGE

Permission to reproduce items where third-party owned material protected by copyright is included has been sought and cleared where possible. Every
reasonable effort has been made by the publisher (UCLES) to trace copyright holders, but if any items requiring clearance have unwittingly been included, the
publisher will be pleased to make amends at the earliest possible opportunity.

University of Cambridge International Examinations is part of the Cambridge Assessment Group. Cambridge Assessment is the brand name of University of
Cambridge Local Examinations Syndicate (UCLES), which is itself a department of the University of Cambridge.

© UCLES 2010 0580/43/O/N/10


w
w
w
.X
tr
me
eP
ap
UNIVERSITY OF CAMBRIDGE INTERNATIONAL EXAMINATIONS

er
s
International General Certificate of Secondary Education

.c
om
*1019432694*

MATHEMATICS 0580/41
Paper 4 (Extended) October/November 2011
2 hours 30 minutes
Candidates answer on the Question Paper.
Additional Materials: Electronic calculator Geometrical instruments
Mathematical tables (optional) Tracing paper (optional)

READ THESE INSTRUCTIONS FIRST

Write your Centre number, candidate number and name on all the work you hand in.
Write in dark blue or black pen.
You may use a pencil for any diagrams or graphs.
Do not use staples, paper clips, highlighters, glue or correction fluid.
DO NOT WRITE IN ANY BARCODES.

Answer all questions.


If working is needed for any question it must be shown below that question.
Electronic calculators should be used.
If the degree of accuracy is not specified in the question, and if the answer is not exact, give the answer to
three significant figures. Give answers in degrees to one decimal place.
For π use either your calculator value or 3.142.

At the end of the examination, fasten all your work securely together.
The number of marks is given in brackets [ ] at the end of each question or part question.
The total of the marks for this paper is 130.

This document consists of 16 printed pages.

IB11 11_0580_41/6RP
© UCLES 2011 [Turn over
2

1 (a) Abdullah and Jasmine bought a car for $9000. For


Abdullah paid 45% of the $9000 and Jasmine paid the rest. Examiner's
Use

(i) How much did Jasmine pay towards the cost of the car?

Answer(a)(i) $ [2]

(ii) Write down the ratio of the payments Abdullah : Jasmine in its simplest form.

Answer(a)(ii) : [1]

(b) Last year it cost $2256 to run the car.


Abdullah, Jasmine and their son Henri share this cost in the ratio 8 : 3 : 1.
Calculate the amount each paid to run the car.

Answer(b) Abdullah $

Jasmine $

Henri $ [3]

(c) (i) A new truck costs $15 000 and loses 23% of its value each year.
Calculate the value of the truck after three years.

Answer(c)(i) $ [3]

(ii) Calculate the overall percentage loss of the truck’s value after three years.

Answer(c)(ii) % [3]

© UCLES 2011 0580/41/O/N/11


3

2 (a) Find the integer values for x which satisfy the inequality –3 I 2x –1 Y 6 . For
Examiner's
Use

Answer(a) [3]

x 2 + 3 x − 10
(b) Simplify .
x 2 − 25

Answer(b) [4]

5 2
(c) (i) Show that + = 3 can be simplified to 3x2 – 13x – 8 = 0.
x − 3 x +1

Answer(c)(i)

[3]

(ii) Solve the equation 3x2 – 13x – 8 = 0.

Show all your working and give your answers correct to two decimal places.

Answer(c)(ii) x = or x = [4]

© UCLES 2011 0580/41/O/N/11 [Turn over


4

3 The table shows information about the heights of 120 girls in a swimming club. For
Examiner's
Use
Height (h metres) Frequency
1.3 I h Y 1.4 4
1.4 I h Y 1.5 13
1.5 I h Y 1.6 33
1.6 I h Y 1.7 45
1.7 I h Y 1.8 19
1.8 I h Y 1.9 6

(a) (i) Write down the modal class.

Answer(a)(i) m [1]

(ii) Calculate an estimate of the mean height. Show all of your working.

Answer(a)(ii) m [4]

(b) Girls from this swimming club are chosen at random to swim in a race.
Calculate the probability that

(i) the height of the first girl chosen is more than 1.8 metres,

Answer(b)(i) [1]

(ii) the heights of both the first and second girl chosen are 1.8 metres or less.

Answer(b)(ii) [3]

© UCLES 2011 0580/41/O/N/11


5

(c) (i) Complete the cumulative frequency table for the heights. For
Examiner's
Use
Height (h metres) Cumulative frequency
h Y 1.3 0
h Y 1.4 4
h Y 1.5 17
h Y 1.6 50
h Y 1.7
h Y 1.8 114
h Y 1.9
[1]
(ii) Draw the cumulative frequency graph on the grid.

120

110

100

90

80

70
Cumulative
frequency 60

50

40

30

20

10

0 h
1.3 1.4 1.5 1.6 1.7 1.8 1.9
Height (m) [3]
(d) Use your graph to find

(i) the median height,


Answer(d)(i) m [1]

(ii) the 30th percentile.


Answer(d)(ii) m [1]

© UCLES 2011 0580/41/O/N/11 [Turn over


6

4 For
r Examiner's
Use

NOT TO
8 cm s SCALE
2.7 cm

20 cm

The diagram shows a plastic cup in the shape of a cone with the end removed.
The vertical height of the cone in the diagram is 20 cm.
The height of the cup is 8 cm.
The base of the cup has radius 2.7 cm.

(a) (i) Show that the radius, r, of the circular top of the cup is 4.5 cm.

Answer(a)(i)

[2]

(ii) Calculate the volume of water in the cup when it is full.


1
[The volume, V, of a cone with radius r and height h is V = πr2h.]
3

Answer(a)(ii) cm3 [4]

© UCLES 2011 0580/41/O/N/11


7

(b) (i) Show that the slant height, s, of the cup is 8.2 cm. For
Examiner's
Use
Answer(b)(i)

[3]

(ii) Calculate the curved surface area of the outside of the cup.
[The curved surface area, A, of a cone with radius r and slant height l is A = πrl.]

Answer(b)(ii) cm2 [5]

© UCLES 2011 0580/41/O/N/11 [Turn over


8

x3 For
5 (a) Complete the table for the function f(x) = − 3x − 1 . Examiner's
2 Use

x –3 –2 –1.5 –1 0 1 1.5 2 3 3.5

f(x) –5.5 1.8 1.5 –3.5 –3.8 –3 9.9


[3]

(b) On the grid draw the graph of y = f(x) for O3 Y x Y 3.5 .

y
10

x
–3 –2 –1 0 1 2 3 4

–2

–4

–6
[4]

© UCLES 2011 0580/41/O/N/11


9

(c) Use your graph to For


Examiner's
Use
(i) solve f(x) = 0.5,

Answer(c)(i) x = or x = or x = [3]

(ii) find the inequalities for k, so that f(x) = k has only 1 answer.

Answer(c)(ii) k I

kK [2]

(d) (i) On the same grid, draw the graph of y = 3x – 2 for O1 Y x Y 3.5 . [3]

x3
(ii) The equation − 3 x − 1 = 3x – 2 can be written in the form x3 + ax + b = 0.
2
Find the values of a and b.

Answer(d)(ii) a = and b = [2]

x3
(iii) Use your graph to find the positive answers to − 3 x − 1 = 3x – 2 for O3 Y x Y 3.5 .
2

Answer(d)(iii) x = or x = [2]

© UCLES 2011 0580/41/O/N/11 [Turn over


10

6 For
C Examiner's
Use
B 26° NOT TO
SCALE

79 m 95 m

77°
A
120 m D

The quadrilateral ABCD represents an area of land.


There is a straight road from A to C.
AB = 79 m, AD = 120 m and CD = 95 m.
Angle BCA = 26° and angle CDA = 77°.

(a) Show that the length of the road, AC, is 135 m correct to the nearest metre.

Answer(a)

[4]

(b) Calculate the size of the obtuse angle ABC.

Answer(b) Angle ABC = [4]

© UCLES 2011 0580/41/O/N/11


11

(c) A straight path is to be built from B to the nearest point on the road AC. For
Examiner's
Use
Calculate the length of this path.

Answer(c) m [3]

(d) Houses are to be built on the land in triangle ACD.


Each house needs at least 180 m2 of land.

Calculate the maximum number of houses which can be built.


Show all of your working.

Answer(d) [4]

© UCLES 2011 0580/41/O/N/11 [Turn over


12

7 For
y Examiner's
Use
8

B
2

x
–8 –6 –4 –2 0 2 4 6 8

–2

–4

D
–6
A

–8

(a) Describe fully the single transformation which maps

(i) triangle A onto triangle B,

Answer(a)(i) [2]

(ii) triangle A onto triangle C,

Answer(a)(ii) [3]

(iii) triangle A onto triangle D.

Answer(a)(iii) [3]

© UCLES 2011 0580/41/O/N/11


13

(b) Draw the image of For


− 5  Examiner's
(i) triangle B after a translation of   , [2] Use
 2

1 0
(ii) triangle B after a transformation by the matrix   . [3]
0 2

1 0
(c) Describe fully the single transformation represented by the matrix   .
0 2

Answer(c)

[3]

© UCLES 2011 0580/41/O/N/11 [Turn over


14

8 Mr Chang hires x large coaches and y small coaches to take 300 students on a school trip. For
Large coaches can carry 50 students and small coaches 30 students. Examiner's
Use
There is a maximum of 5 large coaches.

(a) Explain clearly how the following two inequalities satisfy these conditions.

(i) x Y 5

Answer(a)(i) [1]

(ii) 5x + 3y [ 30

Answer(a)(ii)

[2]

Mr Chang also knows that x + y Y 10.

(b) On the grid, show the information above by drawing three straight lines and shading the
unwanted regions.

10

x
0
2 4 6 8 10
[5]

© UCLES 2011 0580/41/O/N/11


15

(c) A large coach costs $450 to hire and a small coach costs $350. For
Examiner's
Use
(i) Find the number of large coaches and the number of small coaches that would give the
minimum hire cost for this school trip.

Answer(c)(i) Large coaches

Small coaches [2]

(ii) Calculate this minimum cost.

Answer(c)(ii) $ [1]

9 (a) 72 = 2 × 2 × 2 × 3 × 3 written as a product of prime factors.

(i) Write the number 126 as a product of prime factors.

Answer(a)(i) 126 = [2]

(ii) Find the value of the highest common factor of 72 and 126.

Answer(a)(ii) [1]

(iii) Find the value of the lowest common multiple of 72 and 126.

Answer(a)(iii) [2]

The rest of question 9 is printed on the next page.

© UCLES 2011 0580/41/O/N/11 [Turn over


16

(b) John wants to estimate the value of π. For


He measures the circumference of a circular pizza as 105 cm and its diameter as 34 cm, both Examiner's
Use
correct to the nearest centimetre.

Calculate the lower bound of his estimate of the value of π.


Give your answer correct to 3 decimal places.

Answer(b) [4]

(c) The volume of a cylindrical can is 550 cm3, correct to the nearest 10 cm3.
The height of the can is 12 cm correct to the nearest centimetre.

Calculate the upper bound of the radius of the can.


Give your answer correct to 3 decimal places.

Answer(c) cm [5]

Permission to reproduce items where third-party owned material protected by copyright is included has been sought and cleared where possible. Every
reasonable effort has been made by the publisher (UCLES) to trace copyright holders, but if any items requiring clearance have unwittingly been included, the
publisher will be pleased to make amends at the earliest possible opportunity.

University of Cambridge International Examinations is part of the Cambridge Assessment Group. Cambridge Assessment is the brand name of University of
Cambridge Local Examinations Syndicate (UCLES), which is itself a department of the University of Cambridge.

© UCLES 2011 0580/41/O/N/11


w
w
w
.X
tr
me
eP
ap
UNIVERSITY OF CAMBRIDGE INTERNATIONAL EXAMINATIONS

er
s
International General Certificate of Secondary Education

.c
om
*1108636051*

MATHEMATICS 0580/42
Paper 4 (Extended) October/November 2011
2 hours 30 minutes
Candidates answer on the Question Paper.
Additional Materials: Electronic calculator Geometrical instruments
Mathematical tables (optional) Tracing paper (optional)

READ THESE INSTRUCTIONS FIRST

Write your Centre number, candidate number and name on all the work you hand in.
Write in dark blue or black pen.
You may use a pencil for any diagrams or graphs.
Do not use staples, paper clips, highlighters, glue or correction fluid.
DO NOT WRITE IN ANY BARCODES.

Answer all questions.


If working is needed for any question it must be shown below that question.
Electronic calculators should be used.
If the degree of accuracy is not specified in the question, and if the answer is not exact, give the answer to
three significant figures. Give answers in degrees to one decimal place.
For π use either your calculator value or 3.142.

At the end of the examination, fasten all your work securely together.
The number of marks is given in brackets [ ] at the end of each question or part question.
The total of the marks for this paper is 130.

This document consists of 19 printed pages and 1 blank page.

IB11 11_0580_42/5RP
© UCLES 2011 [Turn over
2

1 Children go to camp on holiday. For


Examiner's
Use
(a) Fatima buys bananas and apples for the camp.

(i) Bananas cost $0.85 per kilogram.

Fatima buys 20kg of bananas and receives a discount of 14%.

How much does she spend on bananas?

Answer(a)(i) $ [3]

(ii) Fatima spends $16.40 on apples after a discount of 18%.

Calculate the original price of the apples.

Answer(a)(ii) $ [3]

(iii) The ratio number of bananas : number of apples = 4 : 5.

There are 108 bananas.

Calculate the number of apples.

Answer(a)(iii) [2]

© UCLES 2011 0580/42/O/N/11


3

(b) The cost to hire a tent consists of two parts. For


Examiner's
Use

$c + $d per day

The total cost for 4 days is $27.10 and for 7 days is $34.30.

Write down two equations in c and d and solve them.

Answer(b) c=

d= [4]

(c) The children travel 270 km to the camp, leaving at 07 43 and arriving at 15 13.

Calculate their average speed in km/h.

Answer(c) km/h [3]

(d) Two years ago $540 was put in a savings account to pay for the holiday.

The account paid compound interest at a rate of 6% per year.

How much is in the account now?

Answer(d) $ [2]

© UCLES 2011 0580/42/O/N/11 [Turn over


4

2 For
f(x) = 4x O 2 Examiner's
Use

2
g(x) = +1
x

h(x) = x2 + 3

(a) (i) Find the value of hf(2).

Answer(a)(i) [2]

(ii) Write fg(x) in its simplest form.

Answer(a)(ii) fg(x) = [2]

(b) Solve g(x) = 0.2.

Answer(b) x = [2]

(c) Find the value of gg(3).

Answer(c) [2]

© UCLES 2011 0580/42/O/N/11


5

(d) (i) Show that f(x) = g(x) can be written as 4x2 – 3x – 2 = 0. For
Examiner's
Use
Answer (d)(i)

[1]

(ii) Solve the equation 4x2 – 3x – 2 = 0.

Show all your working and give your answers correct to 2 decimal places.

Answer(d)(ii) x = or x = [4]

© UCLES 2011 0580/42/O/N/11 [Turn over


6

3 For
y Examiner's
Use
9

2
A T
1

x
–9 –8 –7 –6 –5 –4 –3 –2 –1 0 1 2 3 4 5 6 7 8 9
–1

–2

–3

–4

–5

–6

–7

–8

–9

Triangles T and A are drawn on the grid above.

(a) Describe fully the single transformation that maps triangle T onto triangle A.

Answer(a) [2]

(b) (i) Draw the image of triangle T after a rotation of 90° anticlockwise about the point (0,0).

Label the image B. [2]

(ii) Draw the image of triangle T after a reflection in the line x + y = 0.

Label the image C. [2]

(iii) Draw the image of triangle T after an enlargement with centre (4, 5) and scale factor 1.5.

Label the image D. [2]

© UCLES 2011 0580/42/O/N/11


7

(c) (i) Triangle T has its vertices at co-ordinates (2, 1), (6, 1) and (6, 3). For
Examiner's
Use
1 0 
Transform triangle T by the matrix  .
1 1 

Draw this image on the grid and label it E.

[3]
1 0 
(ii) Describe fully the single transformation represented by the matrix  .
1 1 

Answer(c)(ii) [3]

(d) Write down the matrix that transforms triangle B onto triangle T.

 
 
Answer(d)   [2]
 
 

© UCLES 2011 0580/42/O/N/11 [Turn over


8

4 Boris has a recipe which makes 16 biscuits. For


Examiner's
Use
The ingredients are

160 g flour,

160 g sugar,

240 g butter,

200 g oatmeal.

(a) Boris has only 350 grams of oatmeal but plenty of the other ingredients.

(i) How many biscuits can he make?

Answer(a)(i) [2]

(ii) How many grams of butter does he need to make this number of biscuits?

Answer(a)(ii) g [2]

(b) The ingredients are mixed together to make dough.

This dough is made into a sphere of volume 1080 cm3.

Calculate the radius of this sphere.

4
[The volume, V, of a sphere of radius r is V = πr3.]
3

Answer(b) cm [3]

© UCLES 2011 0580/42/O/N/11


9

(c) For
20 cm Examiner's
Use

1.8 cm NOT TO
30 cm
SCALE

The 1080 cm3 of dough is then rolled out to form a cuboid 20 cm × 30 cm × 1.8 cm.

Boris cuts out circular biscuits of diameter 5 cm.

(i) How many whole biscuits can he cut from this cuboid?

Answer(c)(i) [1]

(ii) Calculate the volume of dough left over.

Answer(c)(ii) cm3 [3]

© UCLES 2011 0580/42/O/N/11 [Turn over


10

5 (a) The times, t seconds, for 200 people to solve a problem are shown in the table. For
Examiner's
Use
Time (t seconds) Frequency
0 I t Y 20 6
20 I t Y 40 12
40 I t Y 50 20
50 I t Y 60 37
60 I t Y 70 42
70 I t Y 80 50
80 I t Y 90 28
90 I t Y 100 5

Calculate an estimate of the mean time.

Answer(a) s [4]

(b) (i) Complete the cumulative frequency table for this data.

Time
t Y 20 t Y 40 t Y 50 t Y 60 t Y 70 t Y 80 t Y 90 t Y 100
(t seconds)
Cumulative
6 18 38 167
Frequency

[2]

(ii) Draw the cumulative frequency graph on the grid opposite to show this data. [4]

(c) Use your cumulative frequency graph to find

(i) the median time,


Answer(c)(i) s [1]

(ii) the lower quartile,


Answer(c)(ii) s [1]

(iii) the inter-quartile range,


Answer(c)(iii) s [1]

(iv) how many people took between 65 and 75 seconds to solve the problem,

Answer(c)(iv) [1]

(v) how many people took longer than 45 seconds to solve the problem.

Answer(c)(v) [2]

© UCLES 2011 0580/42/O/N/11


11

200 For
Examiner's
Use

180

160

140

120

Cumulative
frequency 100

80

60

40

20

t
0
20 40 60 80 100
Time (seconds)

© UCLES 2011 0580/42/O/N/11 [Turn over


12

6 For
Examiner's
NOT TO Use

10 cm SCALE
h cm

9 cm

A solid cone has diameter 9 cm, slant height 10 cm and vertical height h cm.

(a) (i) Calculate the curved surface area of the cone.


[The curved surface area, A, of a cone, radius r and slant height l is A = πrl.]

Answer(a)(i) cm2 [2]

(ii) Calculate the value of h, the vertical height of the cone.

Answer(a)(ii) h = [3]

(b)

NOT TO
SCALE

9 cm 3 cm

Sasha cuts off the top of the cone, making a smaller cone with diameter 3 cm.
This cone is similar to the original cone.

(i) Calculate the vertical height of this small cone.

Answer(b)(i) cm [2]

© UCLES 2011 0580/42/O/N/11


13

(ii) Calculate the curved surface area of this small cone. For
Examiner's
Use

Answer(b)(ii) cm2 [2]

(c)

NOT TO
SCALE

12 cm

9 cm

The shaded solid from part (b) is joined to a solid cylinder with diameter 9 cm
and height 12 cm.
Calculate the total surface area of the whole solid.

Answer(c) cm2 [5]

© UCLES 2011 0580/42/O/N/11 [Turn over


14

1 For
7 The diagram shows the accurate graph of y = f(x) where f(x) = + x2 for 0 I x Y 3. Examiner's
x Use
y
10

x
–3 –2 –1 0 1 2 3

–2

–4

–6

–8

–10

© UCLES 2011 0580/42/O/N/11


15

1 For
(a) Complete the table for f(x) = + x2 . Examiner's
x Use

x O3 O2 O1 O0.5 O0.3 O0.1

f(x) 3.5 0 O1.8

[3]

(b) On the grid, draw the graph of y = f(x) for O3 Y x I 0. [3]

(c) By drawing a tangent, work out an estimate of the gradient of the graph where x = 2.

Answer(c) [3]

(d) Write down the inequality satisfied by k when f(x) = k has three answers.

Answer(d) [1]

(e) (i) Draw the line y = 1 – x on the grid for O3 Y x Y 3. [2]

1
(ii) Use your graphs to solve the equation 1–x= + x2 .
x

Answer(e)(ii) x = [1]

1
(f) (i) Rearrange x3 O x2 – 2x + 1 = 0 into the form + x2 = ax + b, where a and b are integers.
x
Answer(f)(i)

[2]

(ii) Write down the equation of the line that could be drawn on the graph
to solve x3 O x2 – 2x + 1 = 0 .

Answer(f)(ii) y = [1]

© UCLES 2011 0580/42/O/N/11 [Turn over


16

8 For
D Examiner's
Use

NOT TO
SCALE

5m

45°
B 3m A

Parvatti has a piece of canvas ABCD in the shape of an irregular quadrilateral.

AB = 3 m, AC = 5 m and angle BAC = 45°.

(a) (i) Calculate the length of BC and show that it rounds to 3.58 m, correct to 2 decimal places.

You must show all your working.

Answer(a)(i)

[4]

(ii) Calculate angle BCA.

Answer(a)(ii) Angle BCA = [3]

© UCLES 2011 0580/42/O/N/11


17

(b) AC = CD and angle CDA = 52°. For


Examiner's
Use
(i) Find angle DCA.

Answer(b)(i) Angle DCA = [1]

(ii) Calculate the area of the canvas.

Answer(b)(ii) m2 [3]

(c) Parvatti uses the canvas to give some shade.

She attaches corners A and D to the top of vertical poles, AP and DQ, each of height 2 m.

Corners B and C are pegged to the horizontal ground.

AB is a straight line and angle BPA = 90°.

A D

3m NOT TO
2m SCALE
2m
B C

P Q

Calculate angle PAB.

Answer(c) Angle PAB = [2]

© UCLES 2011 0580/42/O/N/11 [Turn over


18

9 (a) Emile lost 2 blue buttons from his shirt. For


Examiner's
Use
A bag of spare buttons contains 6 white buttons and 2 blue buttons.

Emile takes 3 buttons out of the bag at random without replacement.

Calculate the probability that

(i) all 3 buttons are white,

Answer(a)(i) [3]

(ii) exactly one of the 3 buttons is blue.

Answer(a)(ii) [3]

© UCLES 2011 0580/42/O/N/11


19

(b) There are 25 buttons in another bag. For


Examiner's
Use
This bag contains x blue buttons.

Two buttons are taken at random without replacement.


7
The probability that they are both blue is .
100

(i) Show that x2 O x O 42 = 0.

Answer (b)(i)

[4]
(ii) Factorise x2 O x O 42.

Answer(b)(ii) [2]

(iii) Solve the equation x2 O x O 42 = 0.

Answer(b)(iii) x = or x = [1]

(iv) Write down the number of buttons in the bag which are not blue.

Answer(b)(iv) [1]

© UCLES 2011 0580/42/O/N/11


20

BLANK PAGE

Permission to reproduce items where third-party owned material protected by copyright is included has been sought and cleared where possible. Every
reasonable effort has been made by the publisher (UCLES) to trace copyright holders, but if any items requiring clearance have unwittingly been included, the
publisher will be pleased to make amends at the earliest possible opportunity.

University of Cambridge International Examinations is part of the Cambridge Assessment Group. Cambridge Assessment is the brand name of University of
Cambridge Local Examinations Syndicate (UCLES), which is itself a department of the University of Cambridge.

© UCLES 2011 0580/42/O/N/11


w
w
w
.X
tr
me
eP
ap
UNIVERSITY OF CAMBRIDGE INTERNATIONAL EXAMINATIONS

er
s
International General Certificate of Secondary Education

.c
om
*8178142556*

MATHEMATICS 0580/43
Paper 4 (Extended) October/November 2011
2 hours 30 minutes
Candidates answer on the Question Paper.
Additional Materials: Electronic calculator Geometrical instruments
Mathematical tables (optional) Tracing paper (optional)

READ THESE INSTRUCTIONS FIRST

Write your Centre number, candidate number and name on all the work you hand in.
Write in dark blue or black pen.
You may use a pencil for any diagrams or graphs.
Do not use staples, paper clips, highlighters, glue or correction fluid.
DO NOT WRITE IN ANY BARCODES.

Answer all questions.


If working is needed for any question it must be shown below that question.
Electronic calculators should be used.
If the degree of accuracy is not specified in the question, and if the answer is not exact, give the answer to
three significant figures. Give answers in degrees to one decimal place.
For π use either your calculator value or 3.142.

At the end of the examination, fasten all your work securely together.
The number of marks is given in brackets [ ] at the end of each question or part question.
The total of the marks for this paper is 130.

This document consists of 20 printed pages.

IB11 11_0580_43/4RP
© UCLES 2011 [Turn over
2

1 For
0.8 m Examiner's
Use

0.5 m NOT TO
SCALE
1.2 m

1.2 m
d

0.4 m

A rectangular tank measures 1.2 m by 0.8 m by 0.5 m.

(a) Water flows from the full tank into a cylinder at a rate of 0.3 m3/min.

Calculate the time it takes for the full tank to empty.


Give your answer in minutes and seconds.

Answer(a) min s [3]

© UCLES 2011 0580/43/O/N/11


3

(b) The radius of the cylinder is 0.4 m. For


Examiner's
Use
Calculate the depth of water, d, when all the water from the rectangular tank is in the cylinder.

Answer(b) d = m [3]

(c) The cylinder has a height of 1.2 m and is open at the top.
The inside surface is painted at a cost of $2.30 per m2.

Calculate the cost of painting the inside surface.

Answer(c) $ [4]

© UCLES 2011 0580/43/O/N/11 [Turn over


4

2 (a) Complete the table of values for y = 2x. For


Examiner's
Use
x –2 –1 0 1 2 3

y 0.25 1 2 8
[2]

(b) On the grid, draw the graph of y = 2x for O2 Y x Y 3.

y
10

x
–2 –1 0 1 2 3
–1
[3]

© UCLES 2011 0580/43/O/N/11


5

For
Examiner's
Use
(c) (i) On the grid, draw the straight line which passes through the points (0, 2) and (3, 8). [1]

(ii) The equation of this line is y = mx + 2.

Show that the value of m is 2.

Answer(c)(ii)

[1]
(iii) One answer to the equation 2x =2x + 2 is x = 3.

Use your graph to find the other answer.

Answer(c)(iii) x = [1]

(d) Draw the tangent to the curve at the point where x = 1.

Use this tangent to calculate an estimate of the gradient of y = 2x when x = 1.

Answer(d) [3]

© UCLES 2011 0580/43/O/N/11 [Turn over


6

3 (a) For
D Examiner's
NOT TO Use
SCALE
C

40° 30°
A B E

ABCD is a quadrilateral with angle BAD = 40°.


AB is extended to E and angle EBC = 30°.
AB = AD and BD = BC.

(i) Calculate angle BCD.

Answer(a)(i) Angle BCD = [3]

(ii) Give a reason why DC is not parallel to AE.

Answer(a)(ii) [1]

(b) A regular polygon has n sides.


5n
Each exterior angle is degrees.
2
Find the value of n.

Answer(b) n = [3]

© UCLES 2011 0580/43/O/N/11


7

(c) For
Examiner's
Use

NOT TO
SCALE

O
C
25°

A B

The diagram shows a circle centre O.


A, B and C are points on the circumference.
OC is parallel to AB.
Angle OCA = 25°.

Calculate angle OBC.

Answer(c)Angle OBC = [3]

© UCLES 2011 0580/43/O/N/11 [Turn over


8

4 For
y Examiner's
Use
6

P
2
W

x
–6 –4 –2 0 2 4 6

–2

–4

–6

(a) Draw the reflection of shape P in the line y = x. [2]

− 2 
(b) Draw the translation of shape P by the vector   . [2]
 1

(c) (i) Describe fully the single transformation that maps shape P onto shape W.

Answer(c)(i) [3]

(ii) Find the 2 by 2 matrix which represents this transformation.

 
 
Answer(c)(ii)   [2]
 
 

1 0
(d) Describe fully the single transformation represented by the matrix   .
0 2

Answer(d) [3]

© UCLES 2011 0580/43/O/N/11


9

5 (a) The cost of a bottle of juice is 5 cents more than the cost of a bottle of water. For
Mohini buys 3 bottles of water and 6 bottles of juice. Examiner's
Use
The total cost is $5.25.

Find the cost of a bottle of water.


Give your answer in cents.

Answer(a) cents [4]

(b) The cost of a biscuit is x cents.


The cost of a cake is (x + 3) cents.
The number of biscuits Roshni can buy for 72 cents is 2 more than the number of cakes she can
buy for 72 cents.

(i) Show that x2 + 3x O 108 = 0.

Answer(b)(i)

[3]

(ii) Solve the equation x2 + 3x O 108 = 0.

Answer(b)(ii) x = or x = [3]

(iii) Find the total cost of 2 biscuits and 1 cake.

Answer(b)(iii) cents [1]

© UCLES 2011 0580/43/O/N/11 [Turn over


10

6 For
Q P Examiner's
Use

NOT TO
SCALE
3 cm

D
C

4 cm

A 12 cm B

The diagram shows a triangular prism of length 12 cm.

The rectangle ABCD is horizontal and the rectangle DCPQ is vertical.

The cross-section is triangle PBC in which angle BCP = 90°, BC = 4 cm and CP = 3 cm.

(a) (i) Calculate the length of AP.

Answer(a)(i) AP = cm [3]

(ii) Calculate the angle of elevation of P from A.

Answer(a)(ii) [2]

© UCLES 2011 0580/43/O/N/11


11

(b) (i) Calculate angle PBC. For


Examiner's
Use

Answer(b)(i) Angle PBC = [2]

(ii) X is on BP so that angle BXC = 120°.

Calculate the length of XC.

Answer(b)(ii) XC = cm [3]

© UCLES 2011 0580/43/O/N/11 [Turn over


12

7 The times, t minutes, taken for 200 students to cycle one kilometre are shown in the table. For
Examiner's
Use

Time (t minutes) 0 I=t=Y=2 2 I=t=Y=3 3 I=t=Y=4 4 I=t=Y=8

Frequency 24 68 72 36

(a) Write down the class interval that contains the median.

Answer(a) [1]

(b) Calculate an estimate of the mean.


Show all your working.

Answer(b) min [4]

© UCLES 2011 0580/43/O/N/11


13

(c) (i) Use the information in the table opposite to complete the cumulative frequency table. For
Examiner's
Use
Time (t minutes) tY2 tY3 tY4 tY8

Cumulative frequency 24 200


[1]

(ii) On the grid, draw a cumulative frequency diagram.

200

180

160
Cumulative frequency

140

120

100

80

60

40

20

t
0
1 2 3 4 5 6 7 8
Time (minutes)
[3]

(iii) Use your diagram to find the median, the lower quartile and the inter-quartile range.

Answer(c)(iii) Median = min

Lower quartile = min

Inter-quartile range = min [3]

© UCLES 2011 0580/43/O/N/11 [Turn over


14

For
8 f(x) = x2 + x O1 g(x) = 1 O 2x h(x) = 3x Examiner's
Use

(a) Find the value of hg(–2).

Answer(a) [2]

(b) Find g –1(x).

Answer(b) g 1(x) =
O
[2]

(c) Solve the equation f(x) = 0.


Show all your working and give your answers correct to 2 decimal places.

Answer(c) x = or x = [4]

(d) Find fg(x).


Give your answer in its simplest form.

Answer(d) fg(x) = [3]

(e) Solve the equation h – 1(x) = 2.

Answer(e) x = [1]

© UCLES 2011 0580/43/O/N/11


15

9 For
Examiner's
Set A S U M S Use

Set B M I N U S
The diagram shows two sets of cards.

(a) One card is chosen at random from Set A and replaced.

(i) Write down the probability that the card chosen shows the letter M.

Answer(a)(i) [1]

(ii) If this is carried out 100 times, write down the expected number of times the card chosen
shows the letter M.

Answer(a)(ii) [1]

(b) Two cards are chosen at random, without replacement, from Set A.

Find the probability that both cards show the letter S.

Answer(b) [2]

(c) One card is chosen at random from Set A and one card is chosen at random from Set B.

Find the probability that exactly one of the two cards shows the letter U.

Answer(c) [3]

(d) A card is chosen at random, without replacement, from Set B until the letter shown is either
I or U.

Find the probability that this does not happen until the 4th card is chosen.

Answer(d) [2]

© UCLES 2011 0580/43/O/N/11 [Turn over


16

10 Hassan stores books in large boxes and small boxes. For


Each large box holds 20 books and each small box holds 10 books. Examiner's
Use
He has x large boxes and y small boxes.

(a) Hassan must store at least 200 books.

Show that 2x + y [ 20.

Answer(a)

[1]

(b) Hassan must not use more than 15 boxes.


He must use at least 3 small boxes.
The number of small boxes must be less than or equal to the number of large boxes.

Write down three inequalities to show this information.

Answer(b)

[3]

(c) On the grid, show the information in part (a) and part (b) by drawing four straight lines and
shading the unwanted regions.
y
20

18

16

14

12

10

x
0
2 4 6 8 10 12 14 16 18 20
[6]

© UCLES 2011 0580/43/O/N/11


17

(d) A large box costs $5 and a small box costs $2. For
Examiner's
Use
(i) Find the least possible total cost of the boxes.

Answer(d)(i) $ [1]

(ii) Find the number of large boxes and the number of small boxes which give this least
possible cost.

Answer(d)(ii) Number of large boxes =

Number of small boxes = [2]

Question 11 is printed on the next page.

© UCLES 2011 0580/43/O/N/11 [Turn over


18

11 (a) For
y Examiner's
Use
5

3
Q
2
P
1

x
–3 –2 –1 0 1 2 3 4 5

The points P and Q have co-ordinates (–3, 1) and (5, 2).

(i) Write as a column vector.

 
 
Answer(a)(i) =   [1]
 
 

− 1 
(ii) = 2 
 1
Mark the point R on the grid. [1]

(iii) Write down the position vector of the point P.

 
 
Answer(a)(iii)   [1]
 
 

© UCLES 2011 0580/43/O/N/11


19

(b) For
U Examiner's
Use
L
NOT TO
u SCALE
M K

O V
v

In the diagram, = u and = v.


2 3
K is on UV so that = and L is on OU so that = .
3 4
M is the midpoint of KL.

Find the following in terms of u and v, giving your answers in their simplest form.

(i)

Answer(b)(i) = [4]

(ii)

Answer(b)(ii) = [2]

Question 12 is printed on the next page.

© UCLES 2011 0580/43/O/N/11 [Turn over


20

12 (a) The nth term of a sequence is n(n+1). For


Examiner's
Use

(i) Write the two missing terms in the spaces. 2, 6, , 20, [2]

(ii) Write down an expression in terms of n for the (n + 1)th term.

Answer(a)(ii) [1]

(iii) The difference between the nth term and the (n + 1)th term is pn + q.

Find the values of p and q.

Answer(a)(iii) p =

q= [2]

(iv) Find the positions of the two consecutive terms which have a difference of 140.

Answer(a)(iv) and [2]

(b) A sequence u1, u2, u3, u4, …………. is given by the following rules.

u1 = 2, u2 = 3 and un = 2u n − 2 + u n −1 for n [ 3.

For example, the third term is u3 and u3 = 2u1 + u2 = 2 × 2 + 3 = 7.


So, the sequence is 2, 3, 7, u4, u5, …..

(i) Show that u4 = 13.

Answer(b)(i) [1]

(ii) Find the value of u5 .

Answer(b)(ii) u5 = [1]

(iii) Two consecutive terms of the sequence are 3413 and 6827 .

Find the term before and the term after these two given terms.

Answer(b)(iii) , 3413, 6827, [2]

Permission to reproduce items where third-party owned material protected by copyright is included has been sought and cleared where possible. Every
reasonable effort has been made by the publisher (UCLES) to trace copyright holders, but if any items requiring clearance have unwittingly been included, the
publisher will be pleased to make amends at the earliest possible opportunity.

University of Cambridge International Examinations is part of the Cambridge Assessment Group. Cambridge Assessment is the brand name of University of
Cambridge Local Examinations Syndicate (UCLES), which is itself a department of the University of Cambridge.

© UCLES 2011 0580/43/O/N/11


w
w
w
.X
tr
me
eP
ap
UNIVERSITY OF CAMBRIDGE INTERNATIONAL EXAMINATIONS

er
s
International General Certificate of Secondary Education

.c
om
*5830631420*

MATHEMATICS 0580/41
Paper 4 (Extended) October/November 2012
2 hours 30 minutes
Candidates answer on the Question Paper.
Additional Materials: Electronic calculator Geometrical instruments
Mathematical tables (optional) Tracing paper (optional)

READ THESE INSTRUCTIONS FIRST

Write your Centre number, candidate number and name on all the work you hand in.
Write in dark blue or black pen.
You may use a pencil for any diagrams or graphs.
Do not use staples, paper clips, highlighters, glue or correction fluid.
DO NOT WRITE IN ANY BARCODES.

Answer all questions.


If working is needed for any question it must be shown below that question.
Electronic calculators should be used.
If the degree of accuracy is not specified in the question, and if the answer is not exact, give the answer to
three significant figures. Give answers in degrees to one decimal place.
For π use either your calculator value or 3.142.

At the end of the examination, fasten all your work securely together.
The number of marks is given in brackets [ ] at the end of each question or part question.
The total of the marks for this paper is 130.

This document consists of 16 printed pages.

IB12 11_0580_41/4RP
© UCLES 2012 [Turn over
2

1 For
Examiner's
Use
B, C or D NOT TO
A or A* SCALE
A or A*

72°
(x + 18)°
60° E, F or G
x° B, C or D

E, F or G

Girls Boys

The pie charts show information on the grades achieved in mathematics by the girls and boys at a
school.

(a) For the Girls’ pie chart, calculate

(i) x,

Answer(a)(i) x = [2]

(ii) the angle for grades B, C or D.

Answer(a)(ii) [1]

(b) Calculate the percentage of the Boys who achieved grades E, F or G.

Answer(b) % [2]

(c) There were 140 girls and 180 boys.

(i) Calculate the percentage of students (girls and boys) who achieved grades A or A*.

Answer(c)(i) % [3]

© UCLES 2012 0580/41/O/N/12


3

(ii) How many more boys than girls achieved grades B, C or D? For
Examiner's
Use

Answer(c)(ii) [2]

(d) The table shows information about the times, t minutes, taken by 80 of the girls to complete
their mathematics examination.

Time taken (t minutes) 40 I t Y 60 60 I t Y 80 80 I t Y 120 120 I t Y 150

Frequency 5 14 29 32

(i) Calculate an estimate of the mean time taken by these 80 girls to complete the examination.

Answer(d)(i) min [4]

(ii) On a histogram, the height of the column for the interval 60 I t Y 80 is 2.8 cm.

Calculate the heights of the other three columns.


Do not draw the histogram.

Answer(d)(ii) 40 I t Y 60 column height = cm

80 I t Y 120 column height = cm

120 I t Y 150 column height = cm [4]

© UCLES 2012 0580/41/O/N/12 [Turn over


4

1 For
2 (a) (i) Complete the table of values for y= x3 + x2 – 7x. Examiner's
2 Use

x –5 –4 –3 –2 –1 0 1 2 3 4

y –2.5 12 16.5 7.5 0 –6 1.5


[3]

1
(ii) On the grid, draw the graph of y= x3 + x2 – 7x for –5 Y x Y 4 .
2

y
22

20

18

16

14

12

10

x
–5 –4 –3 –2 –1 0 1 2 3 4
–2

–4

–6

–8

[4]

1
(b) Use your graph to solve the equation x3 + x2 – 7x = 2 .
2

Answer(b) x = or x = or x = [3]

© UCLES 2012 0580/41/O/N/12


5

(c) By drawing a suitable tangent, calculate an estimate of the gradient of the graph where x = O4 . For
Examiner's
Use

Answer(c) [3]

(d) (i) On the grid draw the line y = 10 – 5x for O2 Y x Y 3 . [3]

1
(ii) Use your graphs to solve the equation x3 + x2 – 7x = 10 – 5x.
2

Answer(d)(ii) x = [1]

© UCLES 2012 0580/41/O/N/12 [Turn over


6

3 90 students are asked which school clubs they attend. For


Examiner's
Use
D = {students who attend drama club}
M = {students who attend music club}
S = { students who attend sports club}

39 students attend music club.


26 students attend exactly two clubs.
35 students attend drama club.

D M

10
........ 13

........ ........
........
23

(a) Write the four missing values in the Venn diagram. [4]

(b) How many students attend

(i) all three clubs,

Answer(b)(i) [1]

(ii) one club only?

Answer(b)(ii) [1]

(c) Find

(i) n(D ∩ M ),

Answer(c)(i) [1]

(ii) n((D ∩ M ) ∩ S' ).

Answer(c)(ii) [1]

© UCLES 2012 0580/41/O/N/12


7

(d) One of the 90 students is chosen at random. For


Examiner's
Use
Find the probability that the student

(i) only attends music club,

Answer(d)(i) [1]

(ii) attends both music and drama clubs.

Answer(d)(ii) [1]

(e) Two of the 90 students are chosen at random without replacement.

Find the probability that

(i) they both attend all three clubs,

Answer(e)(i) [2]

(ii) one of them attends sports club only and the other attends music club only.

Answer(e)(ii) [3]

© UCLES 2012 0580/41/O/N/12 [Turn over


8

4 (a) Solve the equations. For


Examiner's
Use
(i) 4x – 7 = 8 – 2x

Answer(a)(i) x = [2]

x−7
(ii) =2
3

Answer(a)(ii) x = [2]

(b) Simplify the expressions.

(i) (3xy 4)3

Answer(b)(i) [2]

1
(ii) (16a6b2) 2

Answer(b)(ii) [2]

x2 − 7x − 8
(iii)
x 2 − 64

Answer(b)(iii) [4]

© UCLES 2012 0580/41/O/N/12


9

5 (a) For
NOT TO Examiner's
Use
SCALE

20 cm

24 cm
46 cm

Jose has a fish tank in the shape of a cuboid measuring 46 cm by 24 cm by 20 cm.

Calculate the length of the diagonal shown in the diagram.

Answer(a) cm [3]

(b) Maria has a fish tank with a volume of 20 000 cm3.

Write the volume of Maria’s fish tank as a percentage of the volume of Jose’s fish tank.

Answer(b) % [3]

(c) Lorenzo’s fish tank is mathematically similar to Jose’s and double the volume.

Calculate the dimensions of Lorenzo’s fish tank.

Answer(c) cm by cm by cm [3]

(d) A sphere has a volume of 20 000 cm3. Calculate its radius.


4
[The volume, V, of a sphere with radius r is V = πr3.]
3

Answer(d) cm [3]

© UCLES 2012 0580/41/O/N/12 [Turn over


10

 − 2  2  − 10  For
6 (a) a=   b=   c=   Examiner's
 3 − 7  21 Use

(i) Find 2a + b.

 
 
Answer(a)(i)   [1]
 
 

(ii) Find ö=b ö.

Answer(a)(ii) [2]

(iii) ma + nb = c

Find the values of m and n.


Show all your working.

Answer(a)(iii) m =

n= [6]

© UCLES 2012 0580/41/O/N/12


11

(b) For
P Examiner's
X Use

NOT TO
SCALE
O

Y
Q

In the diagram, OX : XP = 3 : 2 and OY : YQ = 3 : 2 .


= p and = q.

(i) Write in terms of p and q.

Answer(b)(i) = [1]

(ii) Write in terms of p and q.

Answer(b)(ii) = [1]

(iii) Complete the following sentences.

The lines XY and PQ are

The triangles OXY and OPQ are

The ratio of the area of triangle OXY to the area of triangle OPQ is : [3]

© UCLES 2012 0580/41/O/N/12 [Turn over


12

7 For
W A X Examiner's
Use
NOT TO
SCALE

E B
O

7 cm

Z D C Y

The vertices A, B, C, D and E of a regular pentagon lie on the circumference of a circle, centre O,
radius 7 cm.
They also lie on the sides of a rectangle WXYZ.

(a) Show that

(i) angle DOC = 72°,

Answer(a)(i)

[1]

(ii) angle DCB = 108°,

Answer(a)(ii)

[2]

(iii) angle CBY = 18°.

Answer(a)(iii)

[1]

© UCLES 2012 0580/41/O/N/12


13

(b) Show that the length CD of one side of the pentagon is 8.23 cm correct to three significant For
figures. Examiner's
Use

Answer(b)

[3]
(c) Calculate

(i) the area of the triangle DOC,

Answer(c)(i) cm2 [2]

(ii) the area of the pentagon ABCDE,

Answer(c)(ii) cm2 [1]


(iii) the area of the sector ODC,

Answer(c)(iii) cm2 [2]


(iv) the length XY.

Answer(c)(iv) cm [2]

(d) Calculate the ratio


area of the pentagon ABCDE : area of the rectangle WXYZ.

Give your answer in the form 1 : n.

Answer(d) 1 : [5]

© UCLES 2012 0580/41/O/N/12 [Turn over


14

8 A rectangular piece of card has a square of side 2 cm removed from each corner. For
Examiner's
2 cm Use

2 cm

NOT TO
(2x + 3) cm SCALE

(x + 5) cm

(a) Write expressions, in terms of x, for the dimensions of the rectangular card before the squares
are removed from the corners.

Answer(a) cm by cm [2]

(b) The diagram shows a net for an open box.


Show that the volume, V cm3, of the open box is given by the formula V = 4x2 + 26x + 30 .

Answer(b)

[3]

© UCLES 2012 0580/41/O/N/12


15

(c) (i) Calculate the values of x when V = 75. For


Show all your working and give your answers correct to two decimal places. Examiner's
Use

Answer(c)(i) x = or x = [5]

(ii) Write down the length of the longest edge of the box.

Answer(c)(ii) cm [1]

Question 9 is printed on the next page.

© UCLES 2012 0580/41/O/N/12 [Turn over


16

9 Distances from the Sun can be measured in astronomical units, AU. For
Earth is a distance of 1 AU from the Sun. Examiner's
Use
One AU is approximately 1.496 × 108 km.

The table shows distances from the Sun.

Name Distance from the Sun in AU Distance from the Sun in kilometres

Earth 1 1.496 × 108

Mercury 0.387

Jupiter 7.79 × 108

Pluto 5.91 × 109

(a) Complete the table. [3]

(b) Light travels at approximately 300 000 kilometres per second.

(i) How long does it take light to travel from the Sun to Earth?
Give your answer in seconds.

Answer(b)(i) s [2]

(ii) How long does it take light to travel from the Sun to Pluto?
Give your answer in minutes.

Answer(b)(ii) min [2]

(c) One light year is the distance that light travels in one year (365 days).

How far is one light year in kilometres?


Give your answer in standard form.

Answer(c) km [3]

(d) How many astronomical units (AU) are equal to one light year?

Answer(d) AU [2]
Permission to reproduce items where third-party owned material protected by copyright is included has been sought and cleared where possible. Every
reasonable effort has been made by the publisher (UCLES) to trace copyright holders, but if any items requiring clearance have unwittingly been included, the
publisher will be pleased to make amends at the earliest possible opportunity.

University of Cambridge International Examinations is part of the Cambridge Assessment Group. Cambridge Assessment is the brand name of University of
Cambridge Local Examinations Syndicate (UCLES), which is itself a department of the University of Cambridge.

© UCLES 2012 0580/41/O/N/12


w
w
w
.X
tr
me
eP
ap
UNIVERSITY OF CAMBRIDGE INTERNATIONAL EXAMINATIONS

er
s
International General Certificate of Secondary Education

.c
om
*4849274249*

MATHEMATICS 0580/42
Paper 4 (Extended) October/November 2012
2 hours 30 minutes
Candidates answer on the Question Paper.
Additional Materials: Electronic calculator Geometrical instruments
Mathematical tables (optional) Tracing paper (optional)

READ THESE INSTRUCTIONS FIRST

Write your Centre number, candidate number and name on all the work you hand in.
Write in dark blue or black pen.
You may use a pencil for any diagrams or graphs.
Do not use staples, paper clips, highlighters, glue or correction fluid.
DO NOT WRITE IN ANY BARCODES.

Answer all questions.


If working is needed for any question it must be shown below that question.
Electronic calculators should be used.
If the degree of accuracy is not specified in the question, and if the answer is not exact, give the answer to
three significant figures. Give answers in degrees to one decimal place.
For π use either your calculator value or 3.142.

At the end of the examination, fasten all your work securely together.
The number of marks is given in brackets [ ] at the end of each question or part question.
The total of the marks for this paper is 130.

This document consists of 20 printed pages.

IB12 11_0580_42/2RP
© UCLES 2012 [Turn over
2

1 A factory produces bird food made with sunflower seed, millet and maize. For
Examiner's
Use
(a) The amounts of sunflower seed, millet and maize are in the ratio

sunflower seed : millet : maize = 5 : 3 : 1 .

(i) How much millet is there in 15 kg of bird food?

Answer(a)(i) kg [2]

(ii) In a small bag of bird food there is 60 g of sunflower seed.

What is the mass of bird food in a small bag?

Answer(a)(ii) g [2]

(b) Sunflower seeds cost $204.50 for 30 kg from Jon’s farm or €96.40 for 20 kg from Ann’s farm.
The exchange rate is $1 = €0.718.

Which farm has the cheapest price per kilogram?


You must show clearly all your working.

Answer(b) [4]

© UCLES 2012 0580/42/O/N/12


3

(c) Bags are filled with bird food at a rate of 420 grams per second. For
Examiner's
Use
How many 20 kg bags can be completely filled in 4 hours?

Answer(c) [3]

(d) Brian buys bags of bird food from the factory and sells them in his shop for $15.30 each.
He makes 12.5% profit on each bag.

How much does Brian pay for each bag of bird food?

Answer(d) $ [3]

(e) Brian orders 600 bags of bird food.


1
The probability that a bag is damaged is.
50
How many bags would Brian expect to be damaged?

Answer(e) [1]

© UCLES 2012 0580/42/O/N/12 [Turn over


4

2 For
A Examiner's
32 m Use

B NOT TO
SCALE

43 m 64 m

C D

The diagram represents a field in the shape of a quadrilateral ABCD.


AB = 32 m, BC = 43 m and AC = 64 m.

(a) (i) Show clearly that angle CAB = 37.0° correct to one decimal place.

Answer(a)(i)

[4]

(ii) Calculate the area of the triangle ABC.

Answer(a)(ii) m2 [2]

(b) CD = 70 m and angle DAC = 55°.

Calculate the perimeter of the whole field ABCD.

Answer(b) m [6]

© UCLES 2012 0580/42/O/N/12


5

3 (a) (i) Factorise completely the expression 4x2 O 18x O 10 . For


Examiner's
Use

Answer(a)(i) [3]

(ii) Solve 4x2 O 18x O 10 = 0.

Answer(a)(ii) x = or x = [1]

(b) Solve the equation 2x2 O 7x O 10 = 0 .

Show all your working and give your answers correct to two decimal places.

Answer(b) x = or x = [4]

6 2
(c) Write O= as a single fraction in its simplest form.
3x − 1 x−2

Answer(c) [3]

© UCLES 2012 0580/42/O/N/12 [Turn over


6

4 (a) For
B Examiner's
A 32° Use

143°
NOT TO
SCALE
D
O

Points A, C and D lie on a circle centre O.


BA and BC are tangents to the circle.
Angle ABC = 32° and angle DAB = 143°.

(i) Calculate angle AOC in quadrilateral AOCB.

Answer(a)(i) Angle AOC = [2]

(ii) Calculate angle ADC.

Answer(a)(ii) Angle ADC = [1]

(iii) Calculate angle OCD.

Answer(a)(iii) Angle OCD = [2]

(iv) OA = 6 cm.

Calculate the length of AB.

Answer(a)(iv) AB = cm [3]

© UCLES 2012 0580/42/O/N/12


7

(b) For
B Examiner's
Use

17°
NOT TO
SCALE
A 39°

C
D

A, B, C and D are on the circumference of the circle centre O.


AC is a diameter.
Angle CAB = 39° and angle ABD = 17°.

(i) Calculate angle ACB.

Answer(b)(i) Angle ACB = [2]

(ii) Calculate angle BXC.

Answer(b)(ii) Angle BXC = [2]

(iii) Give the reason why angle DOA is 34°.

Answer(b)(iii) [1]

(iv) Calculate angle BDO.

Answer(b)(iv) Angle BDO = [1]

(v) The radius of the circle is 12 cm. Calculate the length of major arc ABCD.

Answer(b)(v) Arc ABCD = cm [3]

© UCLES 2012 0580/42/O/N/12 [Turn over


8

5 (a) A farmer takes a sample of 158 potatoes from his crop. He records the mass of each potato and For
the results are shown in the table. Examiner's
Use

Mass (m grams) Frequency

0 I m Y 40 6

40 I m Y 80 10

80 I m Y 120 28

120 I m Y 160 76

160 I m Y 200 22

200 I m Y 240 16

Calculate an estimate of the mean mass.


Show all your working.

Answer(a) g [4]

(b) A new frequency table is made from the results shown in the table in part (a).

Mass (m grams) Frequency

0 I m Y 80

80 I m Y 200

200 I m Y 240 16

(i) Complete the table above. [2]

(ii) On the grid opposite, complete the histogram to show the information in this new table.

© UCLES 2012 0580/42/O/N/12


9

1.2 For
Examiner's
Use

1.0

0.8

Frequency
0.6
density

0.4

0.2

m
0
40 80 120 160 200 240
Mass (grams) [3]

(c) A bag contains 15 potatoes which have a mean mass of 136 g.


The farmer puts 3 potatoes which have a mean mass of 130 g into the bag.

Calculate the mean mass of all the potatoes in the bag.

Answer(c) g [3]

© UCLES 2012 0580/42/O/N/12 [Turn over


10

 3 For
6 (a) Calculate the magnitude of the vector  . Examiner's
 − 5 Use

Answer(a) [2]

(b)
y
16

14

12

10

8
R P
6

x
0
2 4 6 8 10 12 14 16 18

(i) The points P and R are marked on the grid above.

 3
=  . Draw the vector on the grid above. [1]
 − 5

(ii) Draw the image of vector after rotation by 90° anticlockwise about R. [2]

(c) = 2a + b and = 3b O a.

Find in terms of a and b. Write your answer in its simplest form.

Answer(c) = [2]

© UCLES 2012 0580/42/O/N/12


11

 − 2  5 For
(d) =  and = . Examiner's
 5  − 1 Use

Write as a column vector.

 
 
Answer(d) =   [2]
 
 

(e)
A
NOT TO
M SCALE

X
C

= b and = c.

(i) Find in terms of b and c.

Answer(e)(i) = [1]

(ii) X divides CB in the ratio 1 : 3 .


M is the midpoint of AB.

Find in terms of b and c.


Show all your working and write your answer in its simplest form.

Answer(e)(ii) = [4]

© UCLES 2012 0580/42/O/N/12 [Turn over


12

7 Jay makes wooden boxes in two sizes. He makes x small boxes and y large boxes. For
He makes at least 5 small boxes. Examiner's
Use
The greatest number of large boxes he can make is 8.
The greatest total number of boxes is 14.
The number of large boxes is at least half the number of small boxes.
(a) (i) Write down four inequalities in x and y to show this information.

Answer(a)(i)

[4]
(ii) Draw four lines on the grid and write the letter R in the region which represents these
inequalities.
y
15

14

13

12

11

10

x
0
1 2 3 4 5 6 7 8 9 10 11 12 13 14 15
[5]

© UCLES 2012 0580/42/O/N/12


13

(b) The price of the small box is $20 and the price of the large box is $45. For
Examiner's
Use
(i) What is the greatest amount of money he receives when he sells all the boxes he has made?

Answer(b)(i) $ [2]

(ii) For this amount of money, how many boxes of each size did he make?

Answer(b)(ii) small boxes and large boxes [1]

© UCLES 2012 0580/42/O/N/12 [Turn over


14

8 The graph of y = f(x) is drawn on the grid for 0 Y x Y 3.2 . For


Examiner's
Use
y
5

y = f(x)
4

x
0
1 2 3

(a) (i) Draw the tangent to the curve y = f(x) at x = 2.5 . [1]

(ii) Use your tangent to estimate the gradient of the curve at x = 2.5 .

Answer(a)(ii) [2]

(b) Use the graph to solve f(x) = 2, for 0 Y x Y 3.2 .

Answer(b) x = or x = [2]

© UCLES 2012 0580/42/O/N/12


15

x 2 For
(c) g(x) = + 2
x ≠ 0. Examiner's
2 x Use

(i) Complete the table for values of g(x), correct to 1 decimal place.

x 0.7 1 1.5 2 2.5 3

g(x) 1.6 1.6 1.7


[2]

(ii) On the grid opposite, draw the graph of y = g(x) for 0.7 Y x Y 3 . [3]

(iii) Solve f(x) = g(x) for 0.7 Y x Y 3.

Answer(c) (iii) x = or x = or x = [3]

© UCLES 2012 0580/42/O/N/12 [Turn over


16

9 (a) = {25 students in a class} For


Examiner's
F = {students who study French} Use

S = {students who study Spanish}

16 students study French and 18 students study Spanish.

2 students study neither of these.

(i) Complete the Venn diagram to show this information.

F S

..... ..... ..... .....

[2]
(ii) Find n(F ').
Answer(a)(ii) [1]

(iii) Find n(F ∩ S)'.


Answer(a)(iii) [1]

(iv) One student is chosen at random.

Find the probability that this student studies both French and Spanish.

Answer(a)(iv) [1]

(v) Two students are chosen at random without replacement.

Find the probability that they both study only Spanish.

Answer(a)(v) [2]

© UCLES 2012 0580/42/O/N/12


17

(b) In another class the students all study at least one language from French, German and Spanish. For
Examiner's
Use
No student studies all three languages.

The set of students who study German is a proper subset of the set of students who study
French.

4 students study both French and German.

12 students study Spanish but not French.

9 students study French but not Spanish.

A total of 16 students study French.

(i) Draw a Venn diagram to represent this information.

[4]

(ii) Find the total number of students in this class.

Answer(b)(ii) [1]

© UCLES 2012 0580/42/O/N/12 [Turn over


18

10 Consecutive integers are set out in rows in a grid. For


Examiner's
Use
(a) This grid has 5 columns.

1 2 3 4 5

6 7 8 9 10 a b

11 12 13 14 15 n

16 17 18 19 20 c d

21 22 23 24 25

26 27 28 29 30

31 32 33 34 35

The shape drawn encloses five numbers 7, 9, 13, 17 and 19. This is the n = 13 shape.

In this shape, a = 7, b = 9, c = 17 and d = 19.

(i) Calculate bc O ad for the n = 13 shape.

Answer(a)(i) [1]

(ii) For the 5 column grid, a = n O 6.

Write down b, c and d in terms of n for this grid.

Answer(a)(ii) b =

c=

d= [2]

(iii) Write down bc O ad in terms of n.


Show clearly that it simplifies to 20.

Answer(a)(iii)

[2]

© UCLES 2012 0580/42/O/N/12


19

(b) This grid has 6 columns. The shape is drawn for n = 10. For
Examiner's
Use
1 2 3 4 5 6 a b

7 8 9 10 11 12 n

13 14 15 16 17 18 c d

19 20 21 22 23 24

25 26 27 28 29 30

31 32 33 34 35 36

(i) Calculate the value of bc O ad for n = 10.

Answer(b)(i) [1]

(ii) Without simplifying, write down bc O ad in terms of n for this grid.

Answer(b)(ii) [2]

(c) This grid has 7 columns.

1 2 3 4 5 6 7 a b

8 9 10 11 12 13 14 n

15 16 17 18 19 20 21 c d

22 23 24 25 26 27 28

29 30 31 32 33 34 35

Show clearly that bc O ad = 28 for n = 17.

Answer(c)

[1]

Question 10 continues on the next page.

© UCLES 2012 0580/42/O/N/12 [Turn over


20

(d) Write down the value of bc O ad when there are t columns in the grid. For
Examiner's
Use
Answer(d) [1]

(e) Find the values of c, d and bc O ad for this shape.

2 3 4

16

c d

Answer (e) c =

d=

bc O ad = [2]

Permission to reproduce items where third-party owned material protected by copyright is included has been sought and cleared where possible. Every
reasonable effort has been made by the publisher (UCLES) to trace copyright holders, but if any items requiring clearance have unwittingly been included, the
publisher will be pleased to make amends at the earliest possible opportunity.

University of Cambridge International Examinations is part of the Cambridge Assessment Group. Cambridge Assessment is the brand name of University of
Cambridge Local Examinations Syndicate (UCLES), which is itself a department of the University of Cambridge.

© UCLES 2012 0580/42/O/N/12


w
w
w
.X
tr
me
eP
ap
UNIVERSITY OF CAMBRIDGE INTERNATIONAL EXAMINATIONS

er
s
International General Certificate of Secondary Education

.c
om
*5306291564*

MATHEMATICS 0580/43
Paper 4 (Extended) October/November 2012
2 hours 30 minutes
Candidates answer on the Question Paper.
Additional Materials: Electronic calculator Geometrical instruments
Mathematical tables (optional) Tracing paper (optional)

READ THESE INSTRUCTIONS FIRST

Write your Centre number, candidate number and name on all the work you hand in.
Write in dark blue or black pen.
You may use a pencil for any diagrams or graphs.
Do not use staples, paper clips, highlighters, glue or correction fluid.
DO NOT WRITE IN ANY BARCODES.

Answer all questions.


If working is needed for any question it must be shown below that question.
Electronic calculators should be used.
If the degree of accuracy is not specified in the question, and if the answer is not exact, give the answer to
three significant figures. Give answers in degrees to one decimal place.
For π use either your calculator value or 3.142.

At the end of the examination, fasten all your work securely together.
The number of marks is given in brackets [ ] at the end of each question or part question.
The total of the marks for this paper is 130.

This document consists of 19 printed pages and 1 blank page.

IB12 11_0580_43/6RP
© UCLES 2012 [Turn over
2

1 (a) The Martinez family travels by car to Seatown. For


The distance is 92 km and the journey takes 1 hour 25 minutes. Examiner's
Use

(i) The family leaves home at 07 50.


Write down the time they arrive at Seatown.

Answer(a)(i) [1]

(ii) Calculate the average speed for the journey.

Answer(a)(ii) km/h [2]

(iii) During the journey, the family stops for 10 minutes.

Calculate 10 minutes as a percentage of 1 hour 25 minutes.

Answer(a)(iii) % [1]

(iv) 92 km is 15% more than the distance from Seatown to Deecity.

Calculate the distance from Seatown to Deecity.

Answer(a)(iv) km [3]

© UCLES 2012 0580/43/O/N/12


3

(b) The Martinez family spends $150 in the ratio For


Examiner's
Use
fuel : meals : gifts = 11 : 16 : 3 .

(i) Show that $15 is spent on gifts.

Answer (b)(i)

[2]

(ii) The family buys two gifts.


The first gift costs $8.25.

Find the ratio

cost of first gift : cost of second gift.

Give your answer in its simplest form.

Answer(b)(ii) : [2]

© UCLES 2012 0580/43/O/N/12 [Turn over


4

2 (a) For
y Examiner's
Use
8

7
X
6

x
–8 –7 –6 –5 –4 –3 –2 –1 0 1 2 3 4 5 6 7 8
–1

–2
Y
–3

–4

–5

–6

–7

–8

–9

 − 11
(i) Draw the translation of triangle X by the vector  . [2]
 − 1

1
(ii) Draw the enlargement of triangle Y with centre (–6, – 4) and scale factor . [2]
2

© UCLES 2012 0580/43/O/N/12


5

(b) For
y Examiner's
Use
8
7
W X
6
5
4
3
2
1
x
–8 –7 –6 –5 –4 –3 –2 –1 0 1 2 3 4 5 6 7 8
–1
–2
Y
–3
–4
Z
–5
–6
–7
–8
–9
Describe fully the single transformation that maps

(i) triangle X onto triangle Z,

Answer(b)(i) [2]

(ii) triangle X onto triangle Y,

Answer(b)(ii) [3]

(iii) triangle X onto triangle W.

Answer(b)(iii) [3]

(c) Find the matrix that represents the transformation in part (b)(iii).

 
 
Answer(c)   [2]
 
 

© UCLES 2012 0580/43/O/N/12 [Turn over


6

3 A metal cuboid has a volume of 1080 cm3 and a mass of 8 kg. For
Examiner's
Use
(a) Calculate the mass of one cubic centimetre of the metal.
Give your answer in grams.

Answer(a) g [1]

(b) The base of the cuboid measures 12 cm by 10 cm.

Calculate the height of the cuboid.

Answer(b) cm [2]

(c) The cuboid is melted down and made into a sphere with radius r cm.

(i) Calculate the value of r.


4
[The volume, V, of a sphere with radius r is V = πr 3.]
3

Answer(c)(i) r = [3]

© UCLES 2012 0580/43/O/N/12


7

(ii) Calculate the surface area of the sphere. For


Examiner's
[The surface area, A, of a sphere with radius r is A = 4πr 2.] Use

Answer(c)(ii) cm2 [2]

(d) A larger sphere has a radius R cm.


The surface area of this sphere is double the surface area of the sphere with radius r cm in
part (c).
R
Find the value of .
r

Answer(d) [2]

© UCLES 2012 0580/43/O/N/12 [Turn over


8

2 For
4 f(x) = 2
O 3x, x ≠ 0 Examiner's
x Use
(a) Complete the table.

x O3 O2.5 O2 O1.5 O1 O0.5 0.5 1 1.5 2 2.5 3

f(x) 9.2 7.8 6.5 5.4 9.5 6.5 O3.6 O5.5 O7.2 O8.8

[2]
(b) On the grid, draw the graph of y = f(x), for O3 Y x Y O0.5 and 0.5 Y x Y 3 .
y
10

x
–3 –2 –1 0 1 2 3
–1

–2

–3

–4

–5

–6

–7

–8

–9 [5]

© UCLES 2012 0580/43/O/N/12


9

(c) Use your graph to solve the equations. For


Examiner's
Use
(i) f(x) = 4

Answer(c)(i) x = [1]

(ii) f(x) = 3x

Answer(c)(ii) x = [2]

(d) The equation f(x) = 3x can be written as x3 = k.

Find the value of k.

Answer(d) k = [2]

(e) (i) Draw the straight line through the points (–1, 5) and (3, –9). [1]

(ii) Find the equation of this line.

Answer(e)(ii) [3]

(iii) Complete the statement.

The straight line in part (e)(ii) is a to the graph of y = f(x). [1]

© UCLES 2012 0580/43/O/N/12 [Turn over


10

5 (a) Marcos buys 2 bottles of water and 3 bottles of lemonade. For


The total cost is $3.60. Examiner's
Use
The cost of one bottle of lemonade is $0.25 more than the cost of one bottle of water.
Find the cost of one bottle of water.

Answer(a) $ [4]

(b)

NOT TO
y cm 6 cm2 Y cm
5 cm2 SCALE
x cm (x + 2) cm

The diagram shows two rectangles.


The first rectangle measures x cm by y cm and has an area of 5 cm2.
The second rectangle measures (x + 2) cm by Y cm and has an area of 6 cm2.

(i) When y + Y = 1, show that x2 O 9x O 10 = 0 .

Answer (b)(i)

[4]

(ii) Factorise x2 O 9x O 10 .

Answer(b)(ii) [2]

(iii) Calculate the perimeter of the first rectangle.

Answer(b)(iii) cm [2]

© UCLES 2012 0580/43/O/N/12


11

(c) For
Examiner's
Use

(2x + 3) cm NOT TO
5 cm
SCALE

(x + 3) cm

The diagram shows a right-angled triangle with sides of length 5 cm, (x + 3) cm and (2x + 3) cm.

(i) Show that 3x2 + 6x O 25 = 0 .

Answer (c)(i)

[4]

(ii) Solve the equation 3x2 + 6x O 25 = 0 .


Show all your working and give your answers correct to 2 decimal places.

Answer(c)(ii) x = or x = [4]

(iii) Calculate the area of the triangle.

Answer(c)(iii) cm2 [2]

© UCLES 2012 0580/43/O/N/12 [Turn over


12

6 For
A Examiner's
Use

16 cm NOT TO
SCALE

B 25 cm C

The area of triangle ABC is 130 cm2.


AB = 16 cm and BC = 25 cm.

(a) Show clearly that angle ABC = 40.5°, correct to one decimal place.

Answer (a)

[3]

(b) Calculate the length of AC.

Answer(b) AC = cm [4]

(c) Calculate the shortest distance from A to BC.

Answer(c) cm [2]

© UCLES 2012 0580/43/O/N/12


13

7 (a) For
Examiner's
1 2 2 3 4 Use

Two discs are chosen at random without replacement from the five discs shown in the diagram.

(i) Find the probability that both discs are numbered 2 .

Answer(a)(i) [2]

(ii) Find the probability that the numbers on the two discs have a total of 5 .

Answer(a)(ii) [3]

(iii) Find the probability that the numbers on the two discs do not have a total of 5.

Answer(a)(iii) [1]

(b) A group of international students take part in a survey on the nationality of their parents.

E = {students with an English parent} E F


F = {students with a French parent}

n( ) = 50, n(E) = 15, n(F ) = 9 and n(E ∪ F )' = 33 .

(i) Find n(E ∩ F ).


Answer(b)(i) [1]

(ii) Find n(E' ∪ F ).


Answer(b)(ii) [1]

(iii) A student is chosen at random.


Find the probability that this student has an English parent and a French parent.

Answer(b)(iii) [1]

(iv) A student who has a French parent is chosen at random.


Find the probability that this student also has an English parent.

Answer(b)(iv) [1]

© UCLES 2012 0580/43/O/N/12 [Turn over


14

8 (a) For
D Examiner's
Use

NOT TO
SCALE
A

52°
28°
X

A, B, C and D lie on a circle.


The chords AC and BD intersect at X.
Angle BAC = 28° and angle AXD = 52°.
Calculate angle XCD.

Answer(a)Angle XCD = [3]

(b)
S

NOT TO
SCALE
P

R 25x° 22x° O

PQRS is a cyclic quadrilateral in the circle, centre O.


Angle QOS = 22x° and angle QRS = 25x°.
Find the value of x.

Answer(b) x = [3]

© UCLES 2012 0580/43/O/N/12


15

(c) For
L Examiner's
Use

NOT TO
8 cm SCALE

44°
O K M

In the diagram OKL is a sector of a circle, centre O and radius 8 cm.


OKM is a straight line and ML is a tangent to the circle at L.
Angle LOK = 44°.

Calculate the area shaded in the diagram.

Answer(c) cm2 [5]

© UCLES 2012 0580/43/O/N/12 [Turn over


16

9 200 students take a Mathematics examination. For


The cumulative frequency diagram shows information about the times taken, t minutes, to complete Examiner's
Use
the examination.

200

190

180

170

160

150

140

130

120

110
Cumulative
100
frequency

90

80

70

60

50

40

30

20

10

t
0
30 40 50 60 70 80 90
Time (minutes)

© UCLES 2012 0580/43/O/N/12


17

(a) Find For


Examiner's
Use
(i) the median,

Answer(a)(i) min [1]

(ii) the lower quartile,

Answer(a)(ii) min [1]

(iii) the inter-quartile range,

Answer(a)(iii) min [1]

(iv) the number of students who took more than 1 hour.

Answer(a)(iv) [2]

(b) (i) Use the cumulative frequency diagram to complete the grouped frequency table.

Time,
30 I t Y 40 40 I t Y 50 50 I t Y 60 60 I t Y 70 70 I t Y 80 80 I t Y 90
t minutes

Frequency 9 16 28 108 28

[1]

(ii) Calculate an estimate of the mean time taken by the 200 students to complete the
examination.
Show all your working.

Answer(b)(ii) min [4]

© UCLES 2012 0580/43/O/N/12 [Turn over


18

10 (a) Complete the table for the 6 th term and the n th term in each sequence. For
Examiner's
Use
Sequence 6 th term n th term

A 11, 9, 7, 5, 3

B 1, 4, 9, 16, 25

C 2, 6, 12, 20, 30

D 3, 9, 27, 81, 243

E 1, 3, 15, 61, 213

[12]

(b) Find the value of the 100 th term in

(i) Sequence A,

Answer(b)(i) [1]

(ii) Sequence C.

Answer(b)(ii) [1]

© UCLES 2012 0580/43/O/N/12


19

(c) Find the value of n in Sequence D when the n th term is equal to 6561. For
Examiner's
Use

Answer(c) n = [1]

(d) Find the value of the 10 th term in Sequence E.

Answer(d) [1]

© UCLES 2012 0580/43/O/N/12


20

BLANK PAGE

Permission to reproduce items where third-party owned material protected by copyright is included has been sought and cleared where possible. Every
reasonable effort has been made by the publisher (UCLES) to trace copyright holders, but if any items requiring clearance have unwittingly been included, the
publisher will be pleased to make amends at the earliest possible opportunity.

University of Cambridge International Examinations is part of the Cambridge Assessment Group. Cambridge Assessment is the brand name of University of
Cambridge Local Examinations Syndicate (UCLES), which is itself a department of the University of Cambridge.

© UCLES 2012 0580/43/O/N/12


w
w
w
.X
tr
me
eP
ap
UNIVERSITY OF CAMBRIDGE INTERNATIONAL EXAMINATIONS

er
s
International General Certificate of Secondary Education

.c
om
*2039242007*

MATHEMATICS 0580/43
Paper 4 (Extended) October/November 2013
2 hours 30 minutes
Candidates answer on the Question Paper.
Additional Materials: Electronic calculator Geometrical instruments
Tracing paper (optional)

READ THESE INSTRUCTIONS FIRST

Write your Centre number, candidate number and name on all the work you hand in.
Write in dark blue or black pen.
You may use a pencil for any diagrams or graphs.
Do not use staples, paper clips, highlighters, glue or correction fluid.
DO NOT WRITE IN ANY BARCODES.

Answer all questions.


If working is needed for any question it must be shown below that question.
Electronic calculators should be used.
If the degree of accuracy is not specified in the question, and if the answer is not exact, give the answer to
three significant figures. Give answers in degrees to one decimal place.
For π, use either your calculator value or 3.142.

At the end of the examination, fasten all your work securely together.
The number of marks is given in brackets [ ] at the end of each question or part question.
The total of the marks for this paper is 130.

This document consists of 20 printed pages.

IB13 11_0580_43/2RP
© UCLES 2013 [Turn over
2
For
1 (a) (i) In a camera magazine, 63 pages are used for adverts. Examiner′s
The ratio number of pages of adverts : number of pages of reviews = 7 : 5 . Use

Calculate the number of pages used for reviews.

Answer(a)(i) ............................................... [2]

(ii) In another copy of the magazine, 56 pages are used for reviews and for photographs.
The ratio number of pages of reviews : number of pages of photographs = 9 : 5 .

Calculate the number of pages used for photographs.

Answer(a)(ii) ............................................... [2]

(iii) One copy of the magazine costs $4.90 .


An annual subscription costs $48.80 for 13 copies.

Calculate the percentage discount by having an annual subscription.

Answer(a)(iii) ........................................... % [3]

© UCLES 2013 0580/43/O/N/13


3
For
(b) In a car magazine, 25% of the pages are used for selling second-hand cars, Examiner′s
1
62 2 % of the remaining pages are used for features, Use
and the other 36 pages are used for reviews.

Work out the total number of pages in the magazine.

Answer(b) ............................................... [4]


_____________________________________________________________________________________

© UCLES 2013 0580/43/O/N/13 [Turn over


4
For
2 A field, ABCD, is in the shape of a quadrilateral. Examiner′s
A footpath crosses the field from A to C. Use

26°
B
NOT TO
SCALE
55 m

65°
32° 122°
A 62 m D

(a) Use the sine rule to calculate the distance AC and show that it rounds to 119.9 m,
correct to 1 decimal place.

Answer(a)

[3]

(b) Calculate the length of BC.

Answer(b) BC = ........................................... m [4]

© UCLES 2013 0580/43/O/N/13


5
For
(c) Calculate the area of triangle ACD. Examiner′s
Use

Answer(c) .......................................... m2 [2]

(d) The field is for sale at $4.50 per square metre.

Calculate the cost of the field.

Answer(d) $ ............................................... [3]


_____________________________________________________________________________________

© UCLES 2013 0580/43/O/N/13 [Turn over


6
For
3 A rectangular metal sheet measures 9 cm by 7 cm. Examiner′s
A square, of side x cm, is cut from each corner. Use
The metal is then folded to make an open box of height x cm.

9 cm

NOT TO
x cm SCALE
7 cm
x cm
x cm

(a) Write down, in terms of x, the length and width of the box.

Answer(a) Length = ...............................................

Width = ............................................... [2]

(b) Show that the volume, V, of the box is 4x3 – 32x2 + 63x.

Answer(b)

[2]

(c) Complete this table of values for V = 4x3 – 32x2 + 63x.

x 0 0.5 1 1.5 2 2.5 3 3.5

V 0 35 36 30 9 0
[2]

(d) On the grid opposite, draw the graph of V = 4x3 – 32x2 + 63x for 0 Y x Y 3.5 .
Three of the points have been plotted for you.

© UCLES 2013 0580/43/O/N/13


7
For
V Examiner′s
Use
40

35

30

25

20

15

10

x
0
0.5 1 1.5 2 2.5 3 3.5
[3]

(e) The volume of the box is at least 30 cm3.


Write down, as an inequality, the possible values of x.

Answer(e) ............................................... [2]

(f) (i) Write down the maximum volume of the box.

Answer(f)(i) ........................................ cm3 [1]

(ii) Write down the value of x which gives the maximum volume.

Answer(f)(ii) ............................................... [1]


_____________________________________________________________________________________

© UCLES 2013 0580/43/O/N/13 [Turn over


8
For
4 (a) One angle of an isosceles triangle is 48°. Examiner′s
Use
Write down the possible pairs of values for the remaining two angles.

Answer(a) .................. and ..................

.................. and .................. [2]

(b) Calculate the sum of the interior angles of a pentagon.

Answer(b) ............................................... [2]

(c) Calculate the sum of the angles a, b, c, d, e, f and g shown in this diagram.

g b

NOT TO
SCALE
f c

e d

Answer(c) ............................................... [2]

© UCLES 2013 0580/43/O/N/13


9
For
(d) The trapezium, ABCD, has four angles as shown. Examiner′s
All the angles are in degrees. Use

B C
3y – 20 4x – 5
NOT TO
SCALE

2x + 5 x + y – 10
A D

(i) Show that 7x + 4y = 390 .

Answer(d)(i)

[1]

(ii) Show that 2x + 3y = 195 .

Answer(d)(ii)

[1]

(iii) Solve these simultaneous equations.

Answer(d)(iii) x = ...............................................

y = ............................................... [4]

(iv) Use your answer to part (d)(iii) to find the sizes of all four angles of the trapezium.

Answer(d)(iv) ................. , ................. , ................. , ................. [1]


_____________________________________________________________________________________

© UCLES 2013 0580/43/O/N/13 [Turn over


10
For
5 (a) 80 students were asked how much time they spent on the internet in one day. Examiner′s
This table shows the results. Use

Time (t hours) 0<tY1 1<tY2 2<tY3 3<tY5 5<tY7 7 < t Y 10

Number of students 15 11 10 19 13 12

(i) Calculate an estimate of the mean time spent on the internet by the 80 students.

Answer(a)(i) ..................................... hours [4]

(ii) On the grid, complete the histogram to show this information.

16

14

12

10
Frequency
density 8

t
0
1 2 3 4 5 6 7 8 9 10
Time (hours)
[4]

© UCLES 2013 0580/43/O/N/13


11

3 For
(b) The probability that Chaminda uses the internet on any day is 5 . Examiner′s
Use
3
The probability that Niluka uses the internet on any day is 4 .

(i) Complete the tree diagram.

Chaminda Niluka
3
4 Uses the
internet
Uses the
3 internet
5 Does not
........ use the
internet

........ Uses the


........ internet
Does not
use the
internet Does not
........ use the
internet
[2]

(ii) Calculate the probability, that on any day, at least one of the two students uses the internet.

Answer(b)(ii) ............................................... [3]

(iii) Calculate the probability that Chaminda uses the internet on three consecutive days.

Answer(b)(iii) ............................................... [2]


_____________________________________________________________________________________

© UCLES 2013 0580/43/O/N/13 [Turn over


12
For
6 Sandra has designed this open container. Examiner′s
The height of the container is 35 cm. Use

NOT TO
SCALE
35 cm

The cross section of the container is designed from three semi-circles with diameters 17.5 cm, 6.5 cm
and 24 cm.

17.5 cm 6.5 cm
NOT TO
SCALE

(a) Calculate the area of the cross section of the container.

Answer(a) ........................................ cm2 [3]

(b) Calculate the external surface area of the container, including the base.

Answer(b) ........................................ cm2 [4]

© UCLES 2013 0580/43/O/N/13


13
For
(c) The container has a height of 35 cm. Examiner′s
Use
Calculate the capacity of the container.
Give your answer in litres.

Answer(c) ...................................... litres [3]

(d) Sandra’s container is completely filled with water.


All the water is then poured into another container in the shape of a cone.
The cone has radius 20 cm and height 40 cm.

20 cm

NOT TO
r
SCALE
40 cm
h

(i) The diagram shows the water in the cone.


h .
Show that r=
2
Answer(d)(i)

[1]

(ii) Find the height, h, of the water in the cone.


1
[The volume, V, of a cone with radius r and height h is V = 3 πr 2h.]

Answer(d)(ii) h = ......................................... cm [3]


_____________________________________________________________________________________

© UCLES 2013 0580/43/O/N/13 [Turn over


14
For
7 (a) The co-ordinates of P are (–4, –4) and the co-ordinates of Q are (8, 14). Examiner′s
Use
(i) Find the gradient of the line PQ.

Answer(a)(i) ............................................... [2]

(ii) Find the equation of the line PQ.

Answer(a)(ii) ............................................... [2]

(iii) Write as a column vector.

Answer(a)(iii) = f p [1]

(iv) Find the magnitude of .

Answer(a)(iv) ............................................... [2]

© UCLES 2013 0580/43/O/N/13


15
For
(b) Examiner′s
T Use

A NOT TO
SCALE
R
4a

O 3b B

In the diagram, = 4a and = 3b.


1
R lies on AB such that = 5 (12a + 6b).
3
T is the point such that = 2 .

(i) Find the following in terms of a and b, giving each answer in its simplest form.

(a)

Answer(b)(i)(a) = ............................................... [1]

(b)

Answer(b)(i)(b) = ............................................... [2]

(c)

Answer(b)(i)(c) = ............................................... [1]

(ii) Complete the following statement.

The points O, R and T are in a straight line because ................................................................

........................................................................................................................................... [1]

(iii) Triangle OAR and triangle TBR are similar.


area of triangle TBR
Find the value of .
area of triangle OAR

Answer(b)(iii) ............................................... [2]


_____________________________________________________________________________________

© UCLES 2013 0580/43/O/N/13 [Turn over


16
For
1
8 (a) Rearrange s = ut + 2 at2 to make a the subject. Examiner′s
Use

Answer(a) a = ............................................... [3]

(b) The formula v = u + at can be used to calculate the speed, v, of a car.

u = 15, a = 2 and t = 8, each correct to the nearest integer.

Calculate the upper bound of the speed v.

Answer(b) ............................................... [3]

© UCLES 2013 0580/43/O/N/13


17
For
(c) The diagram shows the speed-time graph for a car travelling between two sets of traffic lights. Examiner′s
Use

16

Speed
(m/s)

0
10 20 25
Time (seconds)

(i) Calculate the deceleration of the car for the last 5 seconds of the journey.

Answer(c)(i) ....................................... m/s2 [1]

(ii) Calculate the average speed of the car between the two sets of traffic lights.

Answer(c)(ii) ........................................ m/s [4]


_____________________________________________________________________________________

© UCLES 2013 0580/43/O/N/13 [Turn over


18
For
9 The first four diagrams in a sequence are shown below. Examiner′s
Use

Diagram 1 Diagram 2 Diagram 3 Diagram 4

The diagrams are made from dots ( ) and squares ( ) joined by lines.

(a) Complete the table.

Diagram 1 2 3 4 5 n

Number of dots 6 9 12

Number of squares 0 1 3 1
n(n – 1)
2
Number of triangles 4 9 16

Number of lines 9 18 30 45 63 3
(n + 1)(n + 2)
2
[9]

(b) Which diagram has 360 lines?

Answer(b) ............................................... [2]

© UCLES 2013 0580/43/O/N/13


19
For
(c) The total number of lines in the first n diagrams is Examiner′s
Use
1
2
n3 + pn2 + qn.

1
(i) When n = 1, show that p + q = 8 2 .

Answer(c)(i)

[1]

(ii) By choosing another value of n and using the equation in part (c)(i), find the values
of p and q.

Answer(c)(ii) p = ...............................................

q = ............................................... [5]
_____________________________________________________________________________________

Question 10 is printed on the next page.

© UCLES 2013 0580/43/O/N/13 [Turn over


20
For
10 (a) Simplify. Examiner′s
x2 - 3x Use
x2 - 9

Answer(a) ............................................... [3]

(b) Solve.
15 – 20
=2
x x +1

Answer(b) x = ............... or x = ............... [7]

Permission to reproduce items where third-party owned material protected by copyright is included has been sought and cleared where possible. Every
reasonable effort has been made by the publisher (UCLES) to trace copyright holders, but if any items requiring clearance have unwittingly been included the
publisher will be pleased to make amends at the earliest possible opportunity.

University of Cambridge International Examinations is part of the Cambridge Assessment Group. Cambridge Assessment is the brand name of University of
Cambridge Local Examinations Syndicate (UCLES), which is itself a department of the University of Cambridge.

© UCLES 2013 0580/43/O/N/13


w
w
w
.X
tr
me
eP
ap
UNIVERSITY OF CAMBRIDGE INTERNATIONAL EXAMINATIONS

er
s
International General Certificate of Secondary Education

.c
om
*6296615356*

MATHEMATICS 0580/41
Paper 4 (Extended) October/November 2013
2 hours 30 minutes
Candidates answer on the Question Paper.
Additional Materials: Electronic calculator Geometrical instruments
Tracing paper (optional)

READ THESE INSTRUCTIONS FIRST

Write your Centre number, candidate number and name on all the work you hand in.
Write in dark blue or black pen.
You may use a pencil for any diagrams or graphs.
Do not use staples, paper clips, highlighters, glue or correction fluid.
DO NOT WRITE IN ANY BARCODES.

Answer all questions.


If working is needed for any question it must be shown below that question.
Electronic calculators should be used.
If the degree of accuracy is not specified in the question, and if the answer is not exact, give the answer to
three significant figures. Give answers in degrees to one decimal place.
For π, use either your calculator value or 3.142.

At the end of the examination, fasten all your work securely together.
The number of marks is given in brackets [ ] at the end of each question or part question.
The total of the marks for this paper is 130.

This document consists of 19 printed pages and 1 blank page.

IB13 11_0580_41/2RP
© UCLES 2013 [Turn over
2
For
1 David sells fruit at the market. Examiner′s
Use
(a) In one week, David sells 120 kg of tomatoes and 80 kg of grapes.

(i) Write 80 kg as a fraction of the total mass of tomatoes and grapes.


Give your answer in its lowest terms.

Answer(a)(i) ............................................... [1]

(ii) Write down the ratio mass of tomatoes : mass of grapes.


Give your answer in its simplest form.

Answer(a)(ii) ...................... : ...................... [1]

(b) (i) One day he sells 28 kg of oranges at $1.56 per kilogram.


He also sells 35 kg of apples.
The total he receives from selling the oranges and the apples is $86.38 .

Calculate the price of 1 kilogram of apples.

Answer(b)(i) $ ............................................... [2]

(ii) The price of 1 kilogram of oranges is $1.56 .


This is 20% more than the price two weeks ago.

Calculate the price two weeks ago.

Answer(b)(ii) $ ............................................... [3]

(c) On another day, David received a total of $667 from all the fruit he sold.
The cost of the fruit was $314.20 .
1
David worked for 10 2 hours on this day.

Calculate David’s rate of profit in dollars per hour.

Answer(c) ................................ dollars/h [2]


_____________________________________________________________________________________

© UCLES 2013 0580/41/O/N/13


3
For
2 Emily cycles along a path for 2 minutes. Examiner′s
She starts from rest and accelerates at a constant rate until she reaches a speed of 5 m/s after 40 seconds. Use
She continues cycling at 5 m/s for 60 seconds.
She then decelerates at a constant rate until she stops after a further 20 seconds.

(a) On the grid, draw a speed-time graph to show Emily’s journey.

3
Speed
(m/s)
2

0
10 20 30 40 50 60 70 80 90 100 110 120
Time (seconds)
[2]

(b) Find Emily’s acceleration.

Answer(b) ....................................... m/s2 [1]

(c) Calculate Emily’s average speed for the journey.

Answer(c) ........................................ m/s [4]


_____________________________________________________________________________________

© UCLES 2013 0580/41/O/N/13 [Turn over


4
For
3 Examiner′s
Use

NOT TO
SCALE
13 cm
h

5 cm

(a) The diagram shows a cone of radius 5 cm and slant height 13 cm.

(i) Calculate the curved surface area of the cone.


[The curved surface area, A, of a cone with radius r and slant height l is A = πrl.]

Answer(a)(i) ........................................ cm2 [2]

(ii) Calculate the perpendicular height, h, of the cone.

Answer(a)(ii) h = ......................................... cm [3]

(iii) Calculate the volume of the cone.


πr2h.]
1
[The volume, V, of a cone with radius r and height h is V = 3

Answer(a)(iii) ........................................ cm3 [2]

(iv) Write your answer to part (a)(iii) in cubic metres.


Give your answer in standard form.

Answer(a)(iv) .......................................... m3 [2]

© UCLES 2013 0580/41/O/N/13


5
For
(b) Examiner′s
A Use

O
NOT TO
SCALE

13 cm
h
O

5 cm

The cone is now cut along a slant height and it opens out to make the sector AOB of a circle.

Calculate angle AOB.

Answer(b) Angle AOB = ............................................... [4]


_____________________________________________________________________________________

© UCLES 2013 0580/41/O/N/13 [Turn over


6
For
4 Examiner′s
D C Use
32°

70 m
NOT TO
SCALE
40°
A B
55 m

The diagram shows a school playground ABCD.


ABCD is a trapezium.
AB = 55 m, BD = 70 m, angle ABD = 40° and angle BCD = 32°.

(a) Calculate AD.

Answer(a) AD = ........................................... m [4]

(b) Calculate BC.

Answer(b) BC = ........................................... m [4]

© UCLES 2013 0580/41/O/N/13


7
For
(c) (i) Calculate the area of the playground ABCD. Examiner′s
Use

Answer(c)(i) .......................................... m2 [3]

(ii) An accurate plan of the school playground is to be drawn to a scale of 1: 200 .

Calculate the area of the school playground on the plan.


Give your answer in cm2.

Answer(c)(ii) ........................................ cm2 [2]

(d) A fence, BD, divides the playground into two areas.

Calculate the shortest distance from A to BD.

Answer(d) ........................................... m [2]


_____________________________________________________________________________________

© UCLES 2013 0580/41/O/N/13 [Turn over


8
For
5 (a) Examiner′s
y Use

10

2
T U
1

x
0
1 2 3 4 5 6 7 8 9 10

(i) Draw the reflection of triangle T in the line y = 5. [2]

(ii) Draw the rotation of triangle T about the point (4, 2) through 180°. [2]

(iii) Describe fully the single transformation that maps triangle T onto triangle U.

Answer(a)(iii) .................................................................................................................... [3]

(iv) Find the 2 × 2 matrix which represents the transformation in part (a)(iii).

Answer(a)(iv) f p [2]

© UCLES 2013 0580/41/O/N/13


9
For
(b) Examiner′s
P Q Use

NOT TO
SCALE
p R

O s S

In the pentagon OPQRS, OP is parallel to RQ and OS is parallel to PQ.


PQ = 2OS and OP = 2RQ.
O is the origin, = p and = s.

Find, in terms of p and s, in their simplest form,

(i) the position vector of Q,

Answer(b)(i) ............................................... [2]

(ii) .

Answer(b)(ii) = ............................................... [2]

(c) Explain what your answers in part (b) tell you about the lines OQ and SR.

Answer(c) .................................................................................................................................. [1]


_____________________________________________________________________________________

© UCLES 2013 0580/41/O/N/13 [Turn over


10
For
6 (a) Examiner′s
y Use

x
–3 –2 –1 0 1 2 3
–1

–2

–3

–4

–5

The diagram shows the graph of y = f(x) for –3 Ğ x Ğ 3.

(i) Find f(2).

Answer(a)(i) ............................................... [1]

(ii) Solve the equation f(x) = 0.

Answer(a)(ii) x = ............................................... [1]

(iii) Write down the value of the largest integer, k, for which the equation f(x) = k has 3 solutions.

Answer(a)(iii) k = ............................................... [1]

(iv) By drawing a suitable straight line, solve the equation f(x) = x.

Answer(a)(iv) x = ..................... or x = ..................... or x = ..................... [3]

© UCLES 2013 0580/41/O/N/13


11
For
(b) g(x) = 1 – 2x h(x) = x2 – 1 Examiner′s
Use
(i) Find gh(3).

Answer(b)(i) ............................................... [2]

(ii) Find g–1(x).

Answer(b)(ii) g–1(x) = ............................................... [2]

(iii) Solve the equation h(x) = 3.

Answer(b)(iii) x = .................... or x = .................... [3]

(iv) Solve the equation g(3x) = 2x.

Answer(b)(iv) x = ............................................... [3]


_____________________________________________________________________________________

© UCLES 2013 0580/41/O/N/13 [Turn over


12
For
7 120 students are asked to answer a question. Examiner′s
The time, t seconds, taken by each student to answer the question is measured. Use
The frequency table shows the results.

Time 0 < t Y 10 10 < t Y 20 20 < t Y 30 30 < t Y 40 40 < t Y 50 50 < t Y 60

Frequency 6 44 40 14 10 6

(a) Calculate an estimate of the mean time.

Answer(a) ............................................ s [4]

(b) (i) Complete the cumulative frequency table.

Time t Y 10 t Y 20 t Y 30 t Y 40 t Y 50 t Y 60

Cumulative frequency 6 104 120


[2]

(ii) On the grid below, draw a cumulative frequency diagram to show this information.

120

100

80
Cumulative
60
frequency

40

20

t
0
10 20 30 40 50 60
Time (seconds)
[3]

© UCLES 2013 0580/41/O/N/13


13
For
(iii) Use your cumulative frequency diagram to find the median, the lower quartile and Examiner′s
the 60th percentile. Use

Answer(b)(iii) Median ............................................ s

Lower quartile ............................................ s

60th percentile ............................................ s [4]

(c) The intervals for the times taken are changed.

(i) Use the information in the frequency table on the opposite page to complete this new table.

Time 0 < t Y 20 20 < t Y 30 30 < t Y 60

Frequency 40
[2]

(ii) On the grid below, complete the histogram to show the information in the new table.
One column has already been drawn for you.

3.5

2.5
Frequency
density 2

1.5

0.5

t
0
10 20 30 40 50 60
Time (seconds)
[3]
_____________________________________________________________________________________

© UCLES 2013 0580/41/O/N/13 [Turn over


14
For
8 (a) Solve the equation 8x2 – 11x – 11 = 0. Examiner′s
Show all your working and give your answers correct to 2 decimal places. Use

Answer(a) x = ........................ or x = ........................ [4]

(b) y varies directly as the square root of x.


y = 18 when x = 9.

Find y when x = 484.

Answer(b) y = ........................... [3]

© UCLES 2013 0580/41/O/N/13


15
For
(c) Sara spends $x on pens which cost $2.50 each. Examiner′s
She also spends $(x – 14.50) on pencils which cost $0.50 each. Use
The total of the number of pens and the number of pencils is 19.

Write down and solve an equation in x.

Answer(c) x = ............................................... [6]


_____________________________________________________________________________________

© UCLES 2013 0580/41/O/N/13 [Turn over


16
For
9 Examiner′s
y Use
L2
5

3
R

2 L1

x
0
1 2 3 4 5 6 7 8 9 10 L3

(a) Find the equations of the lines L1, L2 and L3.

Answer(a) L1 ...............................................

L2 ...............................................

L3 ............................................... [5]

(b) Write down the three inequalities that define the shaded region, R.

Answer(b) ................................................

................................................

................................................ [3]

© UCLES 2013 0580/41/O/N/13


17
For
(c) A gardener buys x bushes and y trees. Examiner′s
The cost of a bush is $30 and the cost of a tree is $200. Use
The shaded region R shows the only possible numbers of bushes and trees the gardener can buy.

(i) Find the number of bushes and the number of trees when the total cost is $720.

Answer(c)(i) ...................................... bushes

...................................... trees [2]

(ii) Find the number of bushes and the number of trees which give the greatest possible total cost.
Write down this greatest possible total cost.

Answer(c)(ii) .......................................... bushes

.......................................... trees

Greatest possible total cost = $ ............................................... [3]


_____________________________________________________________________________________

© UCLES 2013 0580/41/O/N/13 [Turn over


18
For
10 (a) 1 =1 Examiner′s
Use
1+2 =3

1+2+3 =6

1+2+3+4 = 10

(i) Write down the next line of this pattern.

Answer(a)(i) ...................................................................................................................... [1]


n
(ii) The sum of the first n integers is (n + 1).
k
Show that k = 2.

Answer(a)(ii)

[2]

(iii) Find the sum of the first 60 integers.

Answer(a)(iii) ............................................... [1]

(iv) Find n when the sum of the first n integers is 465.

Answer(a)(iv) n = ............................................... [2]

(n - 8) (n - 7)
(v) 1 + 2 + 3 + 4 + ....... + x =
2
Write x in terms of n.

Answer(a)(v) x = ............................................... [1]

© UCLES 2013 0580/41/O/N/13


19
For
(b) 13 =1 Examiner′s
Use
13 + 23 =9

13 + 23 + 33 = 36

13 + 23 + 33 + 43 = 100

(i) Complete the statement.

13 + 23 + 33 + 43 + 53 = ...................... = (......................)2 [2]

(ii) The sum of the first n integers is n (n + 1).


2
Find an expression, in terms of n, for the sum of the first n cubes.

Answer(b)(ii) ............................................... [1]

(iii) Find the sum of the first 19 cubes.

Answer(b)(iii) ............................................... [2]


_____________________________________________________________________________________

© UCLES 2013 0580/41/O/N/13 [Turn over


20

BLANK PAGE

Permission to reproduce items where third-party owned material protected by copyright is included has been sought and cleared where possible. Every
reasonable effort has been made by the publisher (UCLES) to trace copyright holders, but if any items requiring clearance have unwittingly been included the
publisher will be pleased to make amends at the earliest possible opportunity.

University of Cambridge International Examinations is part of the Cambridge Assessment Group. Cambridge Assessment is the brand name of University of
Cambridge Local Examinations Syndicate (UCLES), which is itself a department of the University of Cambridge.

© UCLES 2013 0580/41/O/N/13


w
w
w
.X
tr
me
eP
ap
UNIVERSITY OF CAMBRIDGE INTERNATIONAL EXAMINATIONS

er
s
International General Certificate of Secondary Education

.c
om
*2635490223*

MATHEMATICS 0580/42
Paper 4 (Extended) October/November 2013
2 hours 30 minutes
Candidates answer on the Question Paper.
Additional Materials: Electronic calculator Geometrical instruments
Tracing paper (optional)

READ THESE INSTRUCTIONS FIRST

Write your Centre number, candidate number and name on all the work you hand in.
Write in dark blue or black pen.
You may use a pencil for any diagrams or graphs.
Do not use staples, paper clips, highlighters, glue or correction fluid.
DO NOT WRITE IN ANY BARCODES.

Answer all questions.


If working is needed for any question it must be shown below that question.
Electronic calculators should be used.
If the degree of accuracy is not specified in the question, and if the answer is not exact, give the answer to
three significant figures. Give answers in degrees to one decimal place.
For π, use either your calculator value or 3.142.

At the end of the examination, fasten all your work securely together.
The number of marks is given in brackets [ ] at the end of each question or part question.
The total of the marks for this paper is 130.

This document consists of 16 printed pages.

IB13 11_0580_42/RP
© UCLES 2013 [Turn over
2
For
1 Last year Mukthar earned $18 900 . Examiner′s
He did not pay tax on $5500 of his earnings. Use
He paid 24% tax on his remaining earnings.

(a) (i) Calculate how much tax Mukthar paid last year.

Answer(a)(i) $ ............................................... [2]

(ii) Calculate how much Mukthar earned each month after tax had been paid.

Answer(a)(ii) $ ............................................... [2]

(b) This year Mukthar now earns $19 750.50 .

Calculate the percentage increase from $18 900.

Answer(b) ........................................... % [2]

(c) Mukthar has $1500 to invest in one of the following ways.

● Account A paying simple interest at a rate of 4.1% per year

● Account B paying compound interest at a rate of 3.3% per year

Which account will be worth more after 3 years and by how much?

Answer(c) Account ........ by $ ............................................... [5]


_____________________________________________________________________________________

© UCLES 2013 0580/42/O/N/13


3
For
2 Examiner′s
B 2.4 m C Use
NOT TO
6.46 m SCALE
1.8 m

A D
8.6 m

The diagram shows the cross section, ABCD, of a ramp.

(a) Calculate angle DBC.

Answer(a) Angle DBC = ............................................... [2]

(b) (i) Show that BD is exactly 3 m.

Answer(b)(i)

[2]

(ii) Use the cosine rule to calculate angle ABD.

Answer(b)(ii) Angle ABD = ............................................... [4]

(c) The ramp is a prism of width 4 m.

Calculate the volume of this prism.

Answer(c) .......................................... m3 [3]


_____________________________________________________________________________________

© UCLES 2013 0580/42/O/N/13 [Turn over


4
For
3 (a) Write as a single fraction in its simplest form. Examiner′s
Use
2x - 1 3x + 1
-
2 5

Answer(a) ............................................... [3]

(b) Expand and simplify.


(2x – 3)2 – 3x(x – 4)

Answer(b) ............................................... [4]

(c) (i) Factorise.


2x2 + 5x – 3

Answer(c)(i) ............................................... [2]

(ii) Simplify.
2x2 + 5x - 3
2x2 - 18

Answer(c)(ii) ............................................... [3]


_____________________________________________________________________________________

© UCLES 2013 0580/42/O/N/13


5
For
4 Examiner′s
O 8 cm A Use
42°
NOT TO
8 cm SCALE
B

h cm

A wedge of cheese in the shape of a prism is cut from a cylinder of cheese of height h cm.
The radius of the cylinder, OA, is 8 cm and the angle AOB = 42°.

(a) (i) The volume of the wedge of cheese is 90 cm3.

Show that the value of h is 3.84 cm correct to 2 decimal places.

Answer(a)(i)

[4]

(ii) Calculate the total surface area of the wedge of cheese.

Answer(a)(ii) ........................................ cm2 [5]

(b) A mathematically similar wedge of cheese has a volume of 22.5 cm3.

Calculate the height of this wedge.

Answer(b) ......................................... cm [3]


_____________________________________________________________________________________

© UCLES 2013 0580/42/O/N/13 [Turn over


6
For
2 1 Examiner′s
5 (a) Complete the table of values for y = 2 - - 3x .
x x Use

x –3 –2 –1 –0.5 –0.3 0.3 0.5 1 2 3

y 9.6 6 26.5 18.0 –2 –6 –9.1


[3]

2 1
(b) Draw the graph of y = - - 3x for –3 Y x Y –0.3 and 0.3 Y x Y 3 .
x2 x
y
30

25

20

15

10

x
–3 –2 –1 0 1 2 3

–5

–10

[5]

© UCLES 2013 0580/42/O/N/13


7
For
(c) Use your graph to solve these equations. Examiner′s
Use
2 1
(i) 2 - - 3x = 0
x x
Answer(c)(i) x = ............... [1]

2 1
(ii) 2 - - 3x - 7.5 = 0
x x

Answer(c)(ii) x = ............... or x = ............... or x = ............... [3]

2 1
(d) (i) By drawing a suitable straight line on the graph, solve the equation - - 3x = 10 - 3x .
x2 x

Answer(d)(i) x = ............... or x = ............... [4]

2 1
(ii) The equation - - 3x = 10 - 3x can be written in the form ax2 + bx + c = 0 where
x2 x
a, b and c are integers.

Find the values of a, b and c.

Answer(d)(ii) a = ............... , b = ............... , c = ............... [3]


_____________________________________________________________________________________

© UCLES 2013 0580/42/O/N/13 [Turn over


8
For
6 Examiner′s
Use
E N L A R G E M E N T

Prettie picks a card at random from the 11 cards above and does not replace it.
She then picks a second card at random and does not replace it.

(a) Find the probability that she picks

(i) the letter L and then the letter G,

Answer(a)(i) ............................................... [2]

(ii) the letter E twice,

Answer(a)(ii) ............................................... [2]

(iii) two letters that are the same.

Answer(a)(iii) ............................................... [2]

© UCLES 2013 0580/42/O/N/13


9
For
(b) Prettie now picks a third card at random. Examiner′s
Use
Find the probability that the three letters

(i) are all the same,

Answer(b)(i) ............................................... [2]

(ii) do not include a letter E,

Answer(b)(ii) ............................................... [2]

(iii) include exactly two letters that are the same.

Answer(b)(iii) ............................................... [5]


_____________________________________________________________________________________

© UCLES 2013 0580/42/O/N/13 [Turn over


10
For
7 Noma flies from Johannesburg to Hong Kong. Examiner′s
Her plane leaves Johannesburg at 18 45 and arrives in Hong Kong 13 hours and 25 minutes later. Use
The local time in Hong Kong is 6 hours ahead of the time in Johannesburg.

(a) At what time does Noma arrive in Hong Kong?

Answer(a) ............................................... [2]

(b) Noma sleeps for part of the journey.


The time that she spends sleeping is given by the ratio

sleeping : awake = 3 : 4 .

Calculate how long Noma sleeps during the journey.


Give your answer in hours and minutes.

Answer(b) ................... h ................... min [2]

© UCLES 2013 0580/42/O/N/13


11
For
(c) (i) The distance from Hong Kong to Johannesburg is 10 712 km. Examiner′s
The time taken for the journey is 13 hours and 25 minutes. Use

Calculate the average speed of the plane for this journey.

Answer(c)(i) ...................................... km/h [2]

(ii) The plane uses fuel at the rate of 1 litre for every 59 metres travelled.

Calculate the number of litres of fuel used for the journey from Johannesburg to Hong Kong.
Give your answer in standard form.

Answer(c)(ii) ...................................... litres [4]

(d) The cost of Noma’s journey is 10 148 South African Rand (R).
This is an increase of 18% on the cost of the journey one year ago.

Calculate the cost of the same journey one year ago.

Answer(d) R ............................................... [3]


_____________________________________________________________________________________

© UCLES 2013 0580/42/O/N/13 [Turn over


12
For
7 2 Examiner′s
8 f(x) = 4x + 3 g(x) = (x ¸ –1) h(x) = x + 5x
x +1 Use

(a) Work out

(i) h(–3),

Answer(a)(i) ............................................... [1]

(ii) hg(13).

Answer(a)(ii) ............................................... [2]

(b) Find f –1(x).

Answer(b) f –1(x) = ............................................... [2]

© UCLES 2013 0580/42/O/N/13


13
For
(c) (i) Solve the equation f(x) = 23. Examiner′s
Use

Answer(c)(i) x = ............................................... [2]

(ii) Solve the equation h(x) = 7.

Show all your working and give your answers correct to 2 decimal places.

Answer(c)(ii) x = ......................... or x = ......................... [5]


_____________________________________________________________________________________

© UCLES 2013 0580/42/O/N/13 [Turn over


14
For
9 Examiner′s
y Use

4
D
C
3

2
B A
1

x
–8 –7 –6 –5 –4 –3 –2 –1 0 1 2 3 4 5 6 7 8
–1

–2

–3

–4

–5

–6

–7

–8

(a) Describe fully the single transformation that maps triangle A onto

(i) triangle B,

Answer(a)(i) ...................................................................................................................... [2]

(ii) triangle C,

Answer(a)(ii) ..................................................................................................................... [2]

(iii) triangle D.

Answer(a)(iii) .................................................................................................................... [3]

© UCLES 2013 0580/42/O/N/13


15
For
(b) On the grid, draw Examiner′s
Use
(i) the rotation of triangle A about (6, 0) through 90° clockwise, [2]

(ii) the enlargement of triangle A by scale factor –2 with centre (0, –1), [2]

(iii) the shear of triangle A by shear factor –2 with the y-axis invariant. [2]

(c) Find the matrix that represents the transformation in part (b)(iii).

Answer(c) f p [2]

_____________________________________________________________________________________

Question 10 is printed on the next page.

© UCLES 2013 0580/42/O/N/13 [Turn over


16
For
10 Complete the table for the following sequences. Examiner′s
The first row has been completed for you. Use

Sequence Next two terms nth term

1 5 9 13 17 21 4n – 3

(a) 12 21 30 39 [3]

(b) 80 74 68 62 [3]

(c) 1 8 27 64 [2]

(d) 2 10 30 68 [2]

Permission to reproduce items where third-party owned material protected by copyright is included has been sought and cleared where possible. Every
reasonable effort has been made by the publisher (UCLES) to trace copyright holders, but if any items requiring clearance have unwittingly been included the
publisher will be pleased to make amends at the earliest possible opportunity.

University of Cambridge International Examinations is part of the Cambridge Assessment Group. Cambridge Assessment is the brand name of University of
Cambridge Local Examinations Syndicate (UCLES), which is itself a department of the University of Cambridge.

© UCLES 2013 0580/42/O/N/13


Cambridge International Examinations
Cambridge International General Certificate of Secondary Education
*3415255704*

MATHEMATICS 0580/41
Paper 4 (Extended) May/June 2014
2 hours 30 minutes
Candidates answer on the Question Paper.
Additional Materials: Electronic calculator Geometrical instruments
Tracing paper (optional)

READ THESE INSTRUCTIONS FIRST

Write your Centre number, candidate number and name on all the work you hand in.
Write in dark blue or black pen.
You may use an HB pencil for any diagrams or graphs.
Do not use staples, paper clips, glue or correction fluid.
DO NOT WRITE IN ANY BARCODES.

Answer all questions.


If working is needed for any question it must be shown below that question.
Electronic calculators should be used.
If the degree of accuracy is not specified in the question, and if the answer is not exact, give the answer to
three significant figures. Give answers in degrees to one decimal place.
For π, use either your calculator value or 3.142.

At the end of the examination, fasten all your work securely together.
The number of marks is given in brackets [ ] at the end of each question or part question.
The total of the marks for this paper is 130.

The syllabus is approved for use in England, Wales and Northern Ireland as a Cambridge International Level 1/Level 2 Certificate.

This document consists of 19 printed pages and 1 blank page.

IB14 06_0580_41/4RP
© UCLES 2014 [Turn over
2

A= f p C= e o D= f p
3 2 -2 2 0
1 B = (–2 5)
-1 1 5 0 2

(a) Work out, when possible, each of the following.


If it is not possible, write ‘not possible’ in the answer space.

(i) 2A

Answer(a)(i) [1]

(ii) B + C

Answer(a)(ii) [1]

(iii) AD

Answer(a)(iii) [2]

(iv) A–1, the inverse of A.

Answer(a)(iv) [2]

(b) Explain why it is not possible to work out CD.

Answer(b) ........................................................................................................................................... [1]

(c) Describe fully the single transformation represented by the matrix D.

Answer(c) ............................................................................................................................................

............................................................................................................................................................. [3]
__________________________________________________________________________________________

© UCLES 2014 0580/41/M/J/14


3

2 Ali leaves home at 10 00 to cycle to his grandmother’s house. He arrives at 13 00.


The distance-time graph represents his journey.

40

30
Distance from
home (km)

20

10

0
10 00 11 00 12 00 13 00 14 00 15 00 16 00 17 00
Time

(a) Calculate Ali’s speed between 10 00 and 11 30.


Give your answer in kilometres per hour.

Answer(a) ...................................... km/h [2]

(b) Show that Ali’s average speed for the whole journey to his grandmother’s house is 12 km/h.

Answer(b)

[2]

(c) Change 12 kilometres per hour into metres per minute.

Answer(c) ..................................... m/min [2]

(d) Ali stays for 45 minutes at his grandmother’s house and then returns home.
He arrives home at 16 42.

Complete the distance-time graph. [2]


__________________________________________________________________________________________

© UCLES 2014 0580/41/M/J/14 [Turn over


4

3 (a) The running costs for a papermill are $75 246.


This amount is divided in the ratio labour costs : materials = 5 : 1.

Calculate the labour costs.

Answer(a) $ ................................................ [2]

(b) In 2012 the company made a profit of $135 890.


In 2013 the profit was $150 675.

Calculate the percentage increase in the profit from 2012 to 2013.

Answer(b) ............................................ % [3]

(c) The profit of $135 890 in 2012 was an increase of 7% on the profit in 2011.

Calculate the profit in 2011.

Answer(c) $ ................................................ [3]

(d)
2 cm
NOT TO
SCALE

21 cm

30 cm

Paper is sold in cylindrical rolls.


There is a wooden cylinder of radius 2 cm and height 21 cm in the centre of each roll.
The outer radius of a roll of paper is 30 cm.

(i) Calculate the volume of paper in a roll.

Answer(d)(i) ......................................... cm3 [3]

© UCLES 2014 0580/41/M/J/14


5

(ii) The paper is cut into sheets which measure 21 cm by 29.7 cm.
The thickness of each sheet is 0.125 mm.

(a) Change 0.125 millimetres into centimetres.

Answer(d)(ii)(a) .......................................... cm [1]

(b) Work out how many whole sheets of paper can be cut from a roll.

Answer(d)(ii)(b) ................................................ [4]


__________________________________________________________________________________________

© UCLES 2014 0580/41/M/J/14 [Turn over


6

T B

11 9

x
6–x

In the Venn diagram, = {children in a nursery}

B = {children who received a book for their birthday}


T = {children who received a toy for their birthday}
P = {children who received a puzzle for their birthday}

x children received a book and a toy and a puzzle.


6 children received a toy and a puzzle.

(a) 4 children received a book and a toy.


5 children received a book and a puzzle.
7 children received a puzzle but not a book and not a toy.

Complete the Venn diagram above. [3]

(b) There are 40 children in the nursery.

Using the Venn diagram, write down and solve an equation in x.

Answer(b)

[3]

© UCLES 2014 0580/41/M/J/14


7

(c) Work out

(i) the probability that a child, chosen at random, received a book but not a toy and not a puzzle,

Answer(c)(i) ................................................ [1]

(ii) the number of children who received a book and a puzzle but not a toy,

Answer(c)(ii) ................................................ [1]

(iii) n(B),

Answer(c)(iii) ................................................ [1]

(iv) n(B ∪ P),

Answer(c)(iv) ................................................ [1]

(v) n(B ∪ T ∪ P)'.

Answer(c)(v) ................................................ [1]

(d)

T B

Shade the region B ∩ (T ∪ P)'. [1]


__________________________________________________________________________________________

© UCLES 2014 0580/41/M/J/14 [Turn over


8

5
S

North

Scale: 2 cm to 3 km

In the scale drawing, P is a port, L is a lighthouse and S is a ship.


The scale is 2 centimetres represents 3 kilometres.

(a) Measure the bearing of S from P.

Answer(a) ................................................ [1]

(b) Find the actual distance of S from L.

Answer(b) .......................................... km [2]

(c) The bearing of L from S is 160°.

Calculate the bearing of S from L.

Answer(c) ................................................ [1]

© UCLES 2014 0580/41/M/J/14


9

(d) Work out the scale of the map in the form 1 : n.

Answer(d) 1 : ................................................ [2]

(e) A boat B is

● equidistant from S and L


and
● equidistant from the lines PS and SL.

On the diagram, using a straight edge and compasses only, construct the position of B. [5]

(f) The lighthouse stands on an island of area 1.5 cm2 on the scale drawing.

Work out the actual area of the island.

Answer(f) ......................................... km2 [2]


__________________________________________________________________________________________

© UCLES 2014 0580/41/M/J/14 [Turn over


10

6 (a) A square spinner is biased.


The probabilities of obtaining the scores 1, 2, 3 and 4 when it is spun are given in the table.

Score 1 2 3 4

Probability 0.1 0.2 0.4 0.3

(i) Work out the probability that on one spin the score is 2 or 3.

Answer(a)(i) ................................................ [2]

(ii) In 5000 spins, how many times would you expect to score 4 with this spinner?

Answer(a)(ii) ................................................ [1]

(iii) Work out the probability of scoring 1 on the first spin and 4 on the second spin.

Answer(a)(iii) ................................................ [2]

(b) In a bag there are 7 red discs and 5 blue discs.


From the bag a disc is chosen at random and not replaced.
A second disc is then chosen at random.

Work out the probability that at least one of the discs is red.
Give your answer as a fraction.

Answer(b) ................................................ [3]


__________________________________________________________________________________________

© UCLES 2014 0580/41/M/J/14


11

7
y
4

3
A
2

x
–6 –5 –4 –3 –2 –1 0 1 2 3 4 5 6
–1

–2

–3

–4

–5

(a) On the grid,

(i) draw the image of shape A after a translation by the vector e o ,


-5
[2]
-4
(ii) draw the image of shape A after a rotation through 90° clockwise about the origin. [2]

(b) (i) On the grid, draw the image of shape A after the transformation represented by the matrix f p.
2 0
0 1

[3]

(ii) Describe fully the single transformation represented by the matrix f p.


2 0
0 1
Answer(b)(ii) ...............................................................................................................................

..................................................................................................................................................... [3]
__________________________________________________________________________________________

© UCLES 2014 0580/41/M/J/14 [Turn over


12

8 (a) Complete the table of values for y = x3 – 3x + 1 .

x –2.5 –2 –1.5 –1 –0.5 0 0.5 1 1.5 2 2.5

y –7.125 –1 3 1 –0.375 –1 –0.125 3 9.125

[2]

(b) Draw the graph of y = x3 – 3x + 1 for –2.5 Ğ x Ğ 2.5 .

y
10

x
–3 –2 –1 0 1 2 3
–1

–2

–3

–4

–5

–6

–7

–8
[4]

© UCLES 2014 0580/41/M/J/14


13

(c) By drawing a suitable tangent, estimate the gradient of the curve at the point where x = 2.

Answer(c) ................................................ [3]

(d) Use your graph to solve the equation x3 – 3x + 1 = 1 .

Answer(d) x = ..................... or x = ..................... or x = ..................... [2]

(e) Use your graph to complete the inequality in k for which the equation

x3 – 3x + 1 = k has three different solutions.

Answer(e) ........................ < k < ........................ [2]


__________________________________________________________________________________________

© UCLES 2014 0580/41/M/J/14 [Turn over


14

9
80

70

60

50
Cumulative
frequency
40

30

20

10

t
0
10 20 30 40 50
Time (minutes)

The times (t minutes) taken by 80 people to complete a charity swim were recorded.
The results are shown in the cumulative frequency diagram above.

(a) Find

(i) the median,

Answer(a)(i) ......................................... min [1]

(ii) the inter-quartile range,

Answer(a)(ii) ......................................... min [2]

© UCLES 2014 0580/41/M/J/14


15

(iii) the 70th percentile.

Answer(a)(iii) ......................................... min [2]

(b) The times taken by the 80 people are shown in this grouped frequency table.

Time (t minutes) 0 < t Ğ 20 20 < t Ğ 30 30 < t Ğ 45 45 < t Ğ 50

Frequency 12 21 33 14

(i) Calculate an estimate of the mean time.

Answer(b)(i) ......................................... min [4]

(ii) Draw a histogram to represent the grouped frequency table.

Frequency
density
2

t
0
10 20 30 40 50
Time (minutes)
[4]
__________________________________________________________________________________________

© UCLES 2014 0580/41/M/J/14 [Turn over


16

1
10 (a) f(x) = 2x – 3 g(x) = +2 h(x) = 3x
x +1
(i) Work out f(4).

Answer(a)(i) ................................................ [1]

(ii) Work out fh(–1).

Answer(a)(ii) ................................................ [2]

(iii) Find f –1(x), the inverse of f(x).

Answer(a)(iii) f –1(x) = ................................................ [2]

(iv) Find ff(x) in its simplest form.

Answer(a)(iv) ff(x) = ................................................ [2]

© UCLES 2014 0580/41/M/J/14


17

(v) Show that the equation f(x) = g(x) simplifies to 2x2 – 3x – 6 = 0 .

Answer(a)(v)

[3]

(vi) Solve the equation 2x2 – 3x – 6 = 0 .

Give your answers correct to 2 decimal places.


Show all your working.

Answer(a)(vi) x = ..................... or x = ..................... [4]


x2 - 3x + 2
(b) Simplify .
x2 + 3x - 10

Answer(b) ................................................ [4]


__________________________________________________________________________________________

© UCLES 2014 0580/41/M/J/14 [Turn over


18

=e o
-3
11 (a)
4
(i) P is the point (–2, 3).

Work out the co-ordinates of Q.

Answer(a)(i) (............. , .............) [1]

(ii) Work out  , the magnitude of .

Answer(a)(ii) ................................................ [2]

© UCLES 2014 0580/41/M/J/14


19

(b)
C

Y
NOT TO
A SCALE

N
a
B

b
O

OACB is a parallelogram.
= a and = b.
2
AN : NB = 2 : 3 and AY = 5 AC.

(i) Write each of the following in terms of a and/or b.


Give your answers in their simplest form.

(a)

Answer(b)(i)(a) = ................................................ [2]

(b)

Answer(b)(i)(b) = ................................................ [2]

(ii) Write down two conclusions you can make about the line segments NY and BC.

Answer(b)(ii) ...............................................................................................................................

..................................................................................................................................................... [2]
__________________________________________________________________________________________

© UCLES 2014 0580/41/M/J/14 [Turn over


20

BLANK PAGE

Permission to reproduce items where third-party owned material protected by copyright is included has been sought and cleared where possible. Every
reasonable effort has been made by the publisher (UCLES) to trace copyright holders, but if any items requiring clearance have unwittingly been included the
publisher will be pleased to make amends at the earliest possible opportunity.

Cambridge International Examinations is part of the Cambridge Assessment Group. Cambridge Assessment is the brand name of University of Cambridge Local
Examinations Syndicate (UCLES), which is itself a department of the University of Cambridge.

© UCLES 2014 0580/41/M/J/14


Cambridge International Examinations
Cambridge International General Certificate of Secondary Education
*0048847567*

MATHEMATICS 0580/42
Paper 4 (Extended) May/June 2014
2 hours 30 minutes
Candidates answer on the Question Paper.
Additional Materials: Electronic calculator Geometrical instruments
Tracing paper (optional)

READ THESE INSTRUCTIONS FIRST

Write your Centre number, candidate number and name on all the work you hand in.
Write in dark blue or black pen.
You may use an HB pencil for any diagrams or graphs.
Do not use staples, paper clips, glue or correction fluid.
DO NOT WRITE IN ANY BARCODES.

Answer all questions.


If working is needed for any question it must be shown below that question.
Electronic calculators should be used.
If the degree of accuracy is not specified in the question, and if the answer is not exact, give the answer to
three significant figures. Give answers in degrees to one decimal place.
For π, use either your calculator value or 3.142.

At the end of the examination, fasten all your work securely together.
The number of marks is given in brackets [ ] at the end of each question or part question.
The total of the marks for this paper is 130.

The syllabus is approved for use in England, Wales and Northern Ireland as a Cambridge International Level 1/Level 2 Certificate.

This document consists of 16 printed pages.

IB14 06_0580_42/2RP
© UCLES 2014 [Turn over
2

1 Jane and Kate share $240 in the ratio 5 : 7 .

(a) Show that Kate receives $140.

Answer(a)

[2]

(b) Jane and Kate each spend $20.

Find the new ratio Jane’s remaining money : Kate’s remaining money.
Give your answer in its simplest form.

Answer(b) ....................... : ....................... [2]

(c) Kate invests $120 for 5 years at 4% per year simple interest.

Calculate the total amount Kate has after 5 years.

Answer(c) $ ................................................ [3]

(d) Jane invests $80 for 3 years at 4% per year compound interest.

Calculate the total amount Jane has after 3 years.


Give your answer correct to the nearest cent.

Answer(d) $ ................................................ [3]

(e) An investment of $200 for 2 years at 4% per year compound interest is the same as an investment of
$200 for 2 years at r % per year simple interest.

Find the value of r.

Answer(e) r = ................................................ [3]


__________________________________________________________________________________________

© UCLES 2014 0580/42/M/J/14


3

1
2 f(x) = – 2x , x ≠ 0
x2
(a) Complete the table of values for f(x).

x –3 –2.5 –2 –1.5 –1 –0.5 0.4 0.5 1 1.5 2

f(x) 6.1 5.2 4.3 3.4 5 5.5 –2.6 –3.8


[3]

(b) On the grid, draw the graph of y = f(x) for –3 Y x Y – 0.5 and 0.4 Y x Y 2 .

y
7

x
–3 –2 –1 0 1 2
–1

–2

–3

–4
[5]

(c) Solve the equation f(x) = 2 .

Answer(c) x = ................................................ [1]

(d) Solve the equation f(x) = 2x + 3 .

Answer(d) x = ................................................ [3]

(e) (i) Draw the tangent to the graph of y = f(x) at the point where x = –1.5 . [1]

(ii) Use the tangent to estimate the gradient of the graph of y = f(x) where x = –1.5 .

Answer(e)(ii) ................................................ [2]


__________________________________________________________________________________________

© UCLES 2014 0580/42/M/J/14 [Turn over


4

3
C
90 m
D

NOT TO
80 m SCALE
95 m

49°
A
55°
B

The diagram shows a quadrilateral ABCD.


Angle BAD = 49° and angle ABD = 55°.
BD = 80 m, BC = 95 m and CD = 90 m.

(a) Use the sine rule to calculate the length of AD.

Answer(a) AD = ............................................ m [3]

(b) Use the cosine rule to calculate angle BCD.

Answer(b) Angle BCD = ................................................ [4]

© UCLES 2014 0580/42/M/J/14


5

(c) Calculate the area of the quadrilateral ABCD.

Answer(c) ........................................... m2 [3]

(d) The quadrilateral represents a field.


Corn seeds are sown across the whole field at a cost of $3250 per hectare.

Calculate the cost of the corn seeds used.


1 hectare = 10 000 m2

Answer(d) $ ................................................ [3]


__________________________________________________________________________________________

© UCLES 2014 0580/42/M/J/14 [Turn over


6

4
y
8

3
Q
2

x
–8 –7 –6 –5 –4 –3 –2 –1 0 1 2 3 4 5 6 7 8
–1

–2

–3

–4

–5

–6

–7

–8

(a) Draw the reflection of shape Q in the line x = –1 . [2]

(b) (i) Draw the enlargement of shape Q, centre (0, 0), scale factor –2 . [2]

(ii) Find the 2 × 2 matrix that represents an enlargement, centre (0, 0), scale factor –2 .

Answer(b)(ii) f p [2]

© UCLES 2014 0580/42/M/J/14


7

(c) (i) Draw the stretch of shape Q, factor 2, x-axis invariant. [2]

(ii) Find the 2 × 2 matrix that represents a stretch, factor 2, x-axis invariant.

Answer(c)(ii) f p [2]

(iii) Find the inverse of the matrix in part (c)(ii).

Answer(c)(iii) f p [2]

(iv) Describe fully the single transformation represented by the matrix in part (c)(iii).

Answer(c)(iv) ..............................................................................................................................

..................................................................................................................................................... [3]
__________________________________________________________________________________________

© UCLES 2014 0580/42/M/J/14 [Turn over


8

5
8 cm

12 cm NOT TO
SCALE

10 cm

4 cm

The diagram shows a cylinder with radius 8 cm and height 12 cm which is full of water.
A pipe connects the cylinder to a cone.
The cone has radius 4 cm and height 10 cm.

(a) (i) Calculate the volume of water in the cylinder.


Show that it rounds to 2410 cm3 correct to 3 significant figures.

Answer(a)(i)

[2]

(ii) Change 2410 cm3 into litres.

Answer(a)(ii) ....................................... litres [1]

© UCLES 2014 0580/42/M/J/14


9

(b) Water flows from the cylinder along the pipe into the cone at a rate of 2 cm3 per second.

Calculate the time taken to fill the empty cone.


Give your answer in minutes and seconds correct to the nearest second.
1
[The volume, V, of a cone with radius r and height h is V = 3 πr 2h.]

Answer(b) .................. min .................. s [4]

(c) Find the number of empty cones which can be filled completely from the full cylinder.

Answer(c) ................................................ [3]


__________________________________________________________________________________________

© UCLES 2014 0580/42/M/J/14 [Turn over


10

NOT TO
S
SCALE
21°
R
117°
T
y° Q

(a) The chords PR and SQ of the circle intersect at T.


Angle RST = 21° and angle STR = 117°.

(i) Find the values of x and y.

Answer(a)(i) x = ................................................

y = ................................................ [2]

(ii) SR = 8.23 cm, RT = 3.31 cm and PQ = 9.43 cm.

Calculate the length of TQ.

Answer(a)(ii) TQ = .......................................... cm [2]

© UCLES 2014 0580/42/M/J/14


11

(b) EFGH is a cyclic quadrilateral.


G H
EF is a diameter of the circle. NOT TO
KE is the tangent to the circle at E. SCALE
GH is parallel to FE and angle KEG = 115°.

F 115° E

Calculate angle GEH. K

Answer(b) Angle GEH = ................................................ [4]

(c) A, B, C and D are points on the circle centre O.


C
Angle AOB = 140° and angle OAC = 14°.
AD = DC.
D NOT TO
SCALE
O
14° 140°

A B

Calculate angle ACD.

Answer(c) Angle ACD = ................................................ [5]


__________________________________________________________________________________________

© UCLES 2014 0580/42/M/J/14 [Turn over


12

7 (a)
1.0

0.8

Frequency 0.6
density
0.4

0.2

m
0
10 20 30 40 50 60 70 80 90 100
Mass (grams)

The histogram shows some information about the masses (m grams) of 39 apples.

(i) Show that there are 12 apples in the interval 70 < m Y 100 .

Answer(a)(i)

[1]

(ii) Calculate an estimate of the mean mass of the 39 apples.

Answer(a)(ii) ............................................. g [5]

(b) The mean mass of 20 oranges is 70 g.


One orange is eaten.
The mean mass of the remaining oranges is 70.5 g.

Find the mass of the orange that was eaten.

Answer(b) ............................................. g [3]


__________________________________________________________________________________________

© UCLES 2014 0580/42/M/J/14


13

8 The distance a train travels on a journey is 600 km.

(a) Write down an expression, in terms of x, for the average speed of the train when

(i) the journey takes x hours,

Answer(a)(i) ....................................... km/h [1]

(ii) the journey takes (x + 1) hours.

Answer(a)(ii) ....................................... km/h [1]

(b) The difference between the average speeds in part(a)(i) and part(a)(ii) is 20 km/h.

(i) Show that x 2 + x – 30 = 0 .

Answer(b)(i)

[3]

(ii) Find the average speed of the train for the journey in part(a)(ii).
Show all your working.

Answer(b)(ii) ....................................... km/h [4]


__________________________________________________________________________________________

© UCLES 2014 0580/42/M/J/14 [Turn over


14

1
9 If the weather is fine the probability that Carlos is late arriving at school is 10 .
1
If the weather is not fine the probability that he is late arriving at school is 3 .
3
The probability that the weather is fine on any day is 4 .

(a) Complete the tree diagram to show this information.

Weather Arriving at school

1
10 Late

3 Fine
4 Not late
........

........ Late
........ Not fine
Not late
........
[3]

(b) In a school term of 60 days, find the number of days the weather is expected to be fine.

Answer(b) ................................................ [1]

(c) Find the probability that the weather is fine and Carlos is late arriving at school.

Answer(c) ................................................ [2]

(d) Find the probability that Carlos is not late arriving at school.

Answer(d) ................................................ [3]

(e) Find the probability that the weather is not fine on at least one day in a school week of 5 days.

Answer(e) ................................................ [2]


__________________________________________________________________________________________

© UCLES 2014 0580/42/M/J/14


15

1
10 f(x) = x , x ≠ 0 g(x) = 1 – x h(x) = x 2 + 1

(a) Find fg` 2 j .


1

Answer(a) ................................................ [2]

(b) Find g–1(x), the inverse of g(x).

Answer(b) g–1(x) = ................................................ [1]

(c) Find hg(x), giving your answer in its simplest form.

Answer(c) hg(x) = ................................................ [3]

(d) Find the value of x when g(x) = 7 .

Answer(d) x = ................................................ [1]

(e) Solve the equation h(x) = 3x.


Show your working and give your answers correct to 2 decimal places.

Answer(e) x = ......................... or x = ......................... [4]

(f) A function k(x) is its own inverse when k –1(x) = k(x).

For which of the functions f(x) , g(x) and h(x) is this true?

Answer(f) ................................................ [1]


__________________________________________________________________________________________

Question 11 is printed on the next page.

© UCLES 2014 0580/42/M/J/14 [Turn over


16

11 The total area of each of the following shapes is X.


The area of the shaded part of each shape is kX.

For each shape, find the value of k and write your answer below each diagram.

NOT TO NOT TO
SCALE J SCALE
NOT TO F
SCALE I
O 72°

K
G
A B C D
H
AB = BC = CD Angle JOK = 72° EF = FG and EI = IH

k = ..................................... k = ..................................... k = .....................................

A
NOT TO NOT TO
SCALE SCALE

O B

The shape is a regular hexagon. The diagram shows a sector of a circle centre O.
Angle AOB = 90°

k = ..................................... k = .....................................
[10]

Permission to reproduce items where third-party owned material protected by copyright is included has been sought and cleared where possible. Every
reasonable effort has been made by the publisher (UCLES) to trace copyright holders, but if any items requiring clearance have unwittingly been included the
publisher will be pleased to make amends at the earliest possible opportunity.

Cambridge International Examinations is part of the Cambridge Assessment Group. Cambridge Assessment is the brand name of University of Cambridge Local
Examinations Syndicate (UCLES), which is itself a department of the University of Cambridge.

© UCLES 2014 0580/42/M/J/14


Cambridge International Examinations
Cambridge International General Certificate of Secondary Education
*9468931136*

MATHEMATICS 0580/43
Paper 4 (Extended) May/June 2014
2 hours 30 minutes
Candidates answer on the Question Paper.
Additional Materials: Electronic calculator Geometrical instruments
Tracing paper (optional)

READ THESE INSTRUCTIONS FIRST

Write your Centre number, candidate number and name on all the work you hand in.
Write in dark blue or black pen.
You may use an HB pencil for any diagrams or graphs.
Do not use staples, paper clips, glue or correction fluid.
DO NOT WRITE IN ANY BARCODES.

Answer all questions.


If working is needed for any question it must be shown below that question.
Electronic calculators should be used.
If the degree of accuracy is not specified in the question, and if the answer is not exact, give the answer to
three significant figures. Give answers in degrees to one decimal place.
For π, use either your calculator value or 3.142.

At the end of the examination, fasten all your work securely together.
The number of marks is given in brackets [ ] at the end of each question or part question.
The total of the marks for this paper is 130.

The syllabus is approved for use in England, Wales and Northern Ireland as a Cambridge International Level 1/Level 2 Certificate.

This document consists of 16 printed pages.

IB14 06_0580_43/2RP
© UCLES 2014 [Turn over
2

1 In July, a supermarket sold 45 981 bottles of fruit juice.

(a) The cost of a bottle of fruit juice was $1.35 .

Calculate the amount received from the sale of the 45 981 bottles.
Give your answer correct to the nearest hundred dollars.

Answer(a) $ ................................................ [2]

(b) The number of bottles sold in July was 17% more than the number sold in January.

Calculate the number of bottles sold in January.

Answer(b) ................................................ [3]

(c) There were 3 different flavours of fruit juice.


The number of bottles sold in each flavour was in the ratio apple : orange : cherry = 3 : 4 : 2.
The total number of bottles sold was 45 981.

Calculate the number of bottles of orange juice sold.

Answer(c) ................................................ [2]

(d) One bottle contains 1.5 litres of fruit juice.

Calculate the number of 330 ml glasses that can be filled completely from one bottle.

Answer(d) ................................................ [3]


5
(e) 9
of the 45 981 bottles are recycled.

Calculate the number of bottles that are recycled.

Answer(e) ................................................ [2]


__________________________________________________________________________________________

© UCLES 2014 0580/43/M/J/14


3

2
4

Frequency
density
2

0
10 20 30 40 50 60
Amount ($x)

A survey asked 90 people how much money they gave to charity in one month.
The histogram shows the results of the survey.

(a) Complete the frequency table for the six columns in the histogram.

Amount ($x) 0 < x Y 10

Frequency 4

[5]

(b) Use your frequency table to calculate an estimate of the mean amount these 90 people gave to charity.

Answer(b) $ ................................................ [4]


__________________________________________________________________________________________

© UCLES 2014 0580/43/M/J/14 [Turn over


4

3 (a)
P

12 cm NOT TO
SCALE
X 17 cm

The diagram shows triangle PQR with PQ = 12 cm and PR = 17 cm.


The area of triangle PQR is 97 cm2 and angle QPR is acute.

(i) Calculate angle QPR.

Answer(a)(i) Angle QPR = ................................................ [3]

(ii) The midpoint of PQ is X.

Use the cosine rule to calculate the length of XR.

Answer(a)(ii) XR = .......................................... cm [4]

© UCLES 2014 0580/43/M/J/14


5

(b)
NOT TO
9.4 cm 42° a cm SCALE

37°

Calculate the value of a.

Answer(b) a = ................................................ [4]

(c) sin x = cos 40°, 0° Y x Y 180°

Find the two values of x.

Answer(c) x = .................. or x = .................. [2]


__________________________________________________________________________________________

© UCLES 2014 0580/43/M/J/14 [Turn over


6

1
4 The table shows some values for the function y = + x , x ≠ 0.
x2

x –3 –2 –1 –0.5 0.5 1 2 3 4

y –2.89 –1.75 3.5 2 2.25 4.06

(a) Complete the table of values. [3]


1
(b) On the grid, draw the graph of y = + x for –3 Y x Y – 0.5 and 0.5 Y x Y 4.
x2
y
5

x
–3 –2 –1 0 1 2 3 4

–1

–2

–3
[5]

© UCLES 2014 0580/43/M/J/14


7

1
(c) Use your graph to solve the equation +x–3=0.
x2

Answer(c) x = ..................... or x = ..................... or x = ..................... [3]

1
(d) Use your graph to solve the equation + x = 1 – x.
x2

Answer(d) x = ................................................ [3]

(e) By drawing a suitable tangent, find an estimate of the gradient of the curve at the point where x = 2.

Answer(e) ................................................ [3]


3 -1 .
(f) Using algebra, show that you can use the graph at y = 0 to find

Answer(f)

[3]
__________________________________________________________________________________________

© UCLES 2014 0580/43/M/J/14 [Turn over


8

5 (a)
y
5
A
4
3
2
B
1
x
0
1 2 3 4 5 6 7 8

(i) Write down the position vector of A.

Answer(a)(i) f p [1]

(ii) Find ì ì , the magnitude of .

Answer(a)(ii) ................................................ [2]

(b)
S
NOT TO
Q SCALE

O p P R

O is the origin, = p and = q.


OP is extended to R so that OP = PR.
OQ is extended to S so that OQ = QS.

(i) Write down in terms of p and q.

Answer(b)(i) = ................................................ [1]

(ii) PS and RQ intersect at M and RM = 2MQ.

Use vectors to find the ratio PM : PS, showing all your working.

Answer(b)(ii) PM : PS = ....................... : ....................... [4]


__________________________________________________________________________________________

© UCLES 2014 0580/43/M/J/14


9

6 In this question, give all your answers as fractions.

N A T I O N

The letters of the word NATION are printed on 6 cards.

(a) A card is chosen at random.

Write down the probability that

(i) it has the letter T printed on it,

Answer(a)(i) ................................................ [1]

(ii) it does not have the letter N printed on it,

Answer(a)(ii) ................................................ [1]

(iii) the letter printed on it has no lines of symmetry.

Answer(a)(iii) ................................................ [1]

(b) Lara chooses a card at random, replaces it, then chooses a card again.

Calculate the probability that only one of the cards she chooses has the letter N printed on it.

Answer(b) ................................................ [3]

(c) Jacob chooses a card at random and does not replace it.
He continues until he chooses a card with the letter N printed on it.

Find the probability that this happens when he chooses the 4th card.

Answer(c) ................................................ [3]


__________________________________________________________________________________________

© UCLES 2014 0580/43/M/J/14 [Turn over


10

7 (a)
E D
t° x°
NOT TO
SCALE
x° C

q° p° 32°
Y A B X

ABCDEF is a hexagon.
AB is parallel to ED and BC is parallel to FE.
YFE and YABX are straight lines.
Angle CBX = 32° and angle EFA = 90°.

Calculate the value of

(i) p,

Answer(a)(i) p = ................................................ [1]

(ii) q,

Answer(a)(ii) q = ................................................ [2]

(iii) t,

Answer(a)(iii) t = ................................................ [1]

(iv) x.

Answer(a)(iv) x = ................................................ [3]

© UCLES 2014 0580/43/M/J/14


11

(b)
R
x° Q

S

NOT TO
SCALE

63°
T P U

P, Q, R and S are points on a circle and PS = SQ.


PR is a diameter and TPU is the tangent to the circle at P.
Angle SPT = 63°.

Find the value of

(i) x,

Answer(b)(i) x = ................................................ [2]

(ii) y.

Answer(b)(ii) y = ................................................ [2]


__________________________________________________________________________________________

© UCLES 2014 0580/43/M/J/14 [Turn over


12

7 2x - 3
8 (a) (i) Show that the equation + = 1 can be simplified to 2x2 + 3x – 6 = 0 .
x+4 2
Answer(a)(i)

[3]

(ii) Solve the equation 2x2 + 3x – 6 = 0 .

Show all your working and give your answers correct to 2 decimal places.

Answer(a)(ii) x = ........................... or x = ........................... [4]

(b) The total surface area of a cone with radius x and slant height 3x is equal to the area of a circle with
radius r.

Show that r = 2x.


[The curved surface area, A, of a cone with radius r and slant height l is A = πrl.]

Answer(b)

[4]
__________________________________________________________________________________________

© UCLES 2014 0580/43/M/J/14


13
–x
9 f(x) = 4 – 3x g(x) = 3

(a) Find f(2x) in terms of x.

Answer(a) f(2x) = ................................................ [1]

(b) Find ff(x) in its simplest form.

Answer(b) ff(x) = ................................................ [2]

(c) Work out gg(–1).


Give your answer as a fraction.

Answer(c) ................................................ [3]

(d) Find f –1(x), the inverse of f(x).

Answer(d) f –1(x) = ................................................ [2]

(e) Solve the equation gf(x) = 1.

Answer(e) x = ................................................ [3]


__________________________________________________________________________________________

© UCLES 2014 0580/43/M/J/14 [Turn over


14

10 (a)

8 cm NOT TO
SCALE

r cm

The three sides of an equilateral triangle are tangents to a circle of radius r cm.
The sides of the triangle are 8 cm long.

Calculate the value of r.


Show that it rounds to 2.3, correct to 1 decimal place.

Answer(a)

[3]

(b)

8 cm
NOT TO
SCALE

12 cm

The diagram shows a box in the shape of a triangular prism of height 12 cm.
The cross section is an equilateral triangle of side 8 cm.

Calculate the volume of the box.

Answer(b) ......................................... cm3 [4]

© UCLES 2014 0580/43/M/J/14


15

(c) The box contains biscuits.


Each biscuit is a cylinder of radius 2.3 centimetres and height 4 millimetres.

Calculate

(i) the largest number of biscuits that can be placed in the box,

Answer(c)(i) ................................................ [3]

(ii) the volume of one biscuit in cubic centimetres,

Answer(c)(ii) ......................................... cm3 [2]

(iii) the percentage of the volume of the box not filled with biscuits.

Answer(c)(iii) ............................................ % [3]


__________________________________________________________________________________________

Question 11 is printed on the next page.

© UCLES 2014 0580/43/M/J/14 [Turn over


16

11

Diagram 1 Diagram 2 Diagram 3

The first three diagrams in a sequence are shown above.


Diagram 1 shows an equilateral triangle with sides of length 1 unit.
1
In Diagram 2, there are 4 triangles with sides of length 2 unit.
1
In Diagram 3, there are 16 triangles with sides of length 4 unit.

(a) Complete this table for Diagrams 4, 5, 6 and n.

Diagram 1 Diagram 2 Diagram 3 Diagram 4 Diagram 5 Diagram 6 Diagram n

1 1
Length of side 1 2 4

Length of side
20 2–1 2–2
as a power of 2
[6]

(b) (i) Complete this table for the number of the smallest triangles in Diagrams 4, 5 and 6.

Diagram 1 Diagram 2 Diagram 3 Diagram 4 Diagram 5 Diagram 6

Number of smallest
1 4 16
triangles
Number of smallest
20 22 24
triangles as a power of 2
[2]

(ii) Find the number of the smallest triangles in Diagram n, giving your answer as a power of 2.

Answer(b)(ii) ................................................ [1]

(c) Calculate the number of the smallest triangles in the diagram where the smallest triangles have sides of
1
length 128 unit.

Answer(c) ................................................ [2]

Permission to reproduce items where third-party owned material protected by copyright is included has been sought and cleared where possible. Every
reasonable effort has been made by the publisher (UCLES) to trace copyright holders, but if any items requiring clearance have unwittingly been included the
publisher will be pleased to make amends at the earliest possible opportunity.

Cambridge International Examinations is part of the Cambridge Assessment Group. Cambridge Assessment is the brand name of University of Cambridge Local
Examinations Syndicate (UCLES), which is itself a department of the University of Cambridge.

© UCLES 2014 0580/43/M/J/14


Cambridge International Examinations
Cambridge International General Certificate of Secondary Education
*1651372849*

MATHEMATICS 0580/41
Paper 4 (Extended) October/November 2014
2 hours 30 minutes
Candidates answer on the Question Paper.
Additional Materials: Electronic calculator Geometrical instruments
Tracing paper (optional)

READ THESE INSTRUCTIONS FIRST

Write your Centre number, candidate number and name on all the work you hand in.
Write in dark blue or black pen.
You may use an HB pencil for any diagrams or graphs.
Do not use staples, paper clips, glue or correction fluid.
DO NOT WRITE IN ANY BARCODES.

Answer all questions.


If working is needed for any question it must be shown below that question.
Electronic calculators should be used.
If the degree of accuracy is not specified in the question, and if the answer is not exact, give the answer to
three significant figures. Give answers in degrees to one decimal place.
For π, use either your calculator value or 3.142.

At the end of the examination, fasten all your work securely together.
The number of marks is given in brackets [ ] at the end of each question or part question.
The total of the marks for this paper is 130.

The syllabus is approved for use in England, Wales and Northern Ireland as a Cambridge International Level 1/Level 2 Certificate.

This document consists of 19 printed pages and 1 blank page.

IB14 11_0580_41/2RP
© UCLES 2014 [Turn over
www.egyptigstudentroom.com
2

1 (a) A company makes compost by mixing loam, sand and coir in the following ratio.

loam : sand : coir = 7 : 2 : 3

(i) How much loam is there in a 72 litre bag of the compost?

Answer(a)(i) ....................................... litres [2]

(ii) In a small bag of the compost there are 13.5 litres of coir.

How much compost is in a small bag?

Answer(a)(ii) ....................................... litres [2]

(iii) The price of a large bag of compost is $8.40 .


This is an increase of 12% on the price last year.

Calculate the price last year.

Answer(a)(iii) $ ................................................ [3]

(b) Teresa builds a raised garden bed in the shape of a hexagonal prism.

2m
NOT TO
SCALE

45 cm

The garden bed has a height of 45 cm.


The cross section of the inside of the garden bed is a regular hexagon of side 2 m.

www.egyptigstudentroom.com
© UCLES 2014 0580/41/O/N/14
3

(i) Show that the area of the cross section of the inside of the garden bed is 10.4 m2, correct to
3 significant figures.

Answer(b)(i)

[3]

(ii) Calculate the volume of soil needed to fill the garden bed.

Answer(b)(ii) ........................................... m3 [2]

(iii) Teresa wants to fill the garden bed with organic top soil.
She sees this advertisement in the local garden centre.

ORGANIC TOP SOIL Number of tonnes purchased


1 to 5 6 to 10 Over 10
Cost per tonne $47.00 $45.50 $44.00

Organic top soil is sold in one tonne bags.


1 m3 of organic top soil has a mass of 1250 kg.

Calculate the cost of the organic top soil needed to fill the garden bed completely.
[1 tonne = 1000 kg]

Answer(b)(iii) $ ................................................ [4]


__________________________________________________________________________________________

www.egyptigstudentroom.com
© UCLES 2014 0580/41/O/N/14 [Turn over
4

2 (a) Rearrange the formula v2 = u2 – 2as to make u the subject.

Answer(a) u = ................................................ [2]

(b) Chuck cycles along Skyline Drive.


He cycles 60 km at an average speed of x km/h.
He then cycles a further 45 km at an average speed of (x + 4) km/h.
His total journey time is 6 hours.

(i) Write down an equation in x and show that it simplifies to 2x2 – 27x – 80 = 0.

Answer(b)(i)

[4]

(ii) Solve 2x2 – 27x – 80 = 0 to find the value of x.

Answer(b)(ii) x = ................................................ [3]

www.egyptigstudentroom.com
© UCLES 2014 0580/41/O/N/14
5

(c) The diagram shows the speed-time graph for a car travelling along a road for T seconds.

Speed NOT TO
(m/s) SCALE

0 T
20 110
Time (seconds)

To begin with the car accelerated at 0.75 m/s2 for 20 seconds to reach a speed of v m/s.

(i) Show that the speed, v, of the car is 15 m/s.

Answer(c)(i)

[1]

(ii) The total distance travelled is 1.8 kilometres.

Calculate the total time, T, of the journey.

Answer(c)(ii) ................................... seconds [4]

(d) Asma runs 22 kilometres, correct to the nearest kilometre.


She takes 2 12 hours, correct to the nearest half hour.

Calculate the upper bound of Asma’s speed.

Answer(d) ....................................... km/h [3]


__________________________________________________________________________________________

www.egyptigstudentroom.com
© UCLES 2014 0580/41/O/N/14 [Turn over
6

3
y
8

2
A
1

x
–8 –7 –6 –5 –4 –3 –2 –1 0 1 2 3 4 5 6 7 8
–1

–2

–3
B
–4

–5

–6

–7

–8

(a) Draw the image when triangle A is reflected in the line x = 0. [1]

(b) Draw the image when triangle A is rotated through 90° anticlockwise about (–4, 0). [2]

(c) (i) Describe fully the single transformation that maps triangle A onto triangle B.

Answer(c)(i) ................................................................................................................................

..................................................................................................................................................... [3]

(ii) Complete the following statement.

Area of triangle A : Area of triangle B = .................... : .................... [2]

www.egyptigstudentroom.com
© UCLES 2014 0580/41/O/N/14
7

(d) Write down the matrix that represents a stretch, factor 4 with the y-axis invariant.

Answer(d) f p [2]

(e) (i) On the grid, draw the image of triangle A after the transformation represented by the

matrix e o.
1 0
2 1

[3]

(ii) Describe fully this single transformation.

Answer(e)(ii) ...............................................................................................................................

..................................................................................................................................................... [3]

(iii) Find the inverse of the matrix e o.


1 0
2 1

Answer(e)(iii) f p [2]

__________________________________________________________________________________________

www.egyptigstudentroom.com
© UCLES 2014 0580/41/O/N/14 [Turn over
8

4 (a) Expand and simplify.

(i) 4(2x – 1) – 3(3x – 5)

Answer(a)(i) ................................................ [2]

(ii) (2x – 3y)(3x + 4y)

Answer(a)(ii) ................................................ [3]

(b) Factorise.
x3 – 5x

Answer(b) ................................................ [1]

(c) Solve the inequality.


2x + 1 5x - 8
3 Y= 4

Answer(c) ................................................ [3]

www.egyptigstudentroom.com
© UCLES 2014 0580/41/O/N/14
9

(d) (i) x2 – 9x + 12 = (x – p)2 – q

Find the value of p and the value of q.

Answer(d)(i) p = ................................................

q = ................................................ [3]

(ii) Write down the minimum value of x2 – 9x + 12.

Answer(d)(ii) ................................................ [1]

(iii) Write down the equation of the line of symmetry of the graph of y = x2 – 9x + 12.

Answer(d)(iii) ................................................ [1]


__________________________________________________________________________________________

www.egyptigstudentroom.com
© UCLES 2014 0580/41/O/N/14 [Turn over
10

3
5 (a) Complete the table of values for y = x2 + x , x ≠ 0.

x –3 –2 –1 –0.5 0.4 0.6 1 1.5 2 3

y 8 2.5 –5.8 7.7 5.4 4 4.3 10


[2]
3
(b) Draw the graph of y = x2 + x for –3 Y x Y –0.5 and 0.4 Y x Y 3.

y
10

x
–3 –2 –1 0 1 2 3

–2

–4

–6
[5]

www.egyptigstudentroom.com
© UCLES 2014 0580/41/O/N/14
11

3
(c) Use your graph to solve the equation x2 + x = 5.

Answer(c) x = .................... or x = .................... or x = .................... [3]

3
(d) By drawing a suitable straight line, solve the equation x2 + x = x + 5.

Answer(d) x = .................... or x = .................... or x = .................... [4]


__________________________________________________________________________________________

www.egyptigstudentroom.com
© UCLES 2014 0580/41/O/N/14 [Turn over
12

6 A company tested 200 light bulbs to find the lifetime, T hours, of each bulb.
The results are shown in the table.

Lifetime Number
(T hours) of bulbs
0 < T Y 1000 10
1000 < T Y 1500 30
1500 < T Y 2000 55
2000 < T Y 2500 72
2500 < T Y 3500 33

(a) Calculate an estimate of the mean lifetime for the 200 light bulbs.

Answer(a) ...................................... hours [4]

(b) (i) Complete the cumulative frequency table.

Lifetime (T hours) T Y 1000 T Y 1500 T Y 2000 T Y 2500 T Y 3500

Number of bulbs
[2]

www.egyptigstudentroom.com
© UCLES 2014 0580/41/O/N/14
13

(ii) On the grid, draw a cumulative frequency diagram to show this information.

200

150

Cumulative
frequency
100

50

0 T
500 1000 1500 2000 2500 3000 3500
Lifetime (hours)
[3]

(iii) The company says that the average lifetime of a bulb is 2200 hours.

Estimate the number of bulbs that lasted longer than 2200 hours.

Answer(b)(iii) ................................................ [2]

(c) Robert buys one energy saving bulb and one halogen bulb.
9
The probability that the energy saving bulb lasts longer than 3500 hours is 10 .
3
The probability that the halogen bulb lasts longer than 3500 hours is 5 .

Work out the probability that exactly one of the bulbs will last longer than 3500 hours.

Answer(c) ................................................ [4]


__________________________________________________________________________________________

www.egyptigstudentroom.com
© UCLES 2014 0580/41/O/N/14 [Turn over
14

7 (a) The diagram shows a circle with two chords, AB and CD, intersecting at X.

B NOT TO
C SCALE

X
A

(i) Show that triangles ACX and DBX are similar.

Answer(a)(i)

[2]

(ii) AX = 3.2 cm, BX = 12.5 cm, CX = 4 cm and angle AXC = 110°.

(a) Find DX.

Answer(a)(ii)(a) DX = .......................................... cm [2]

(b) Use the cosine rule to find AC.

Answer(a)(ii)(b) AC = .......................................... cm [4]

www.egyptigstudentroom.com
© UCLES 2014 0580/41/O/N/14
15

(c) Find the area of triangle BXD.

Answer(a)(ii)(c) ......................................... cm2 [2]

(b)
D

NOT TO
C
SCALE

30 m

37°
31°
A B

In the diagram, BC represents a building 30 m tall.


A flagpole, DC, stands on top of the building.
From a point, A, the angle of elevation of the top of the building is 31°.
The angle of elevation of the top of the flagpole is 37°.

Calculate the height, DC, of the flagpole.

Answer(b) ............................................ m [5]


__________________________________________________________________________________________

www.egyptigstudentroom.com
© UCLES 2014 0580/41/O/N/14 [Turn over
16

8
B
P
A

NOT TO
SCALE

Q
9b
6a

3c

In the diagram, O is the origin and = 6a, = 9b and = 3c.


The point P lies on AB such that = 3b – 2a.
The point Q lies on BC such that = 2c – 6b.

(a) Find, in terms of b and c, the position vector of Q.


Give your answer in its simplest form.

Answer(a) ................................................ [2]

www.egyptigstudentroom.com
© UCLES 2014 0580/41/O/N/14
17

(b) Find, in terms of a and c, in its simplest form

(i) ,

Answer(b)(i) = ................................................ [1]

(ii) .

Answer(b)(ii) = ................................................ [2]

(c) Explain what your answers in part (b) tell you about PQ and AC.

Answer(c) ............................................................................................................................................

............................................................................................................................................................. [2]
__________________________________________________________________________________________

www.egyptigstudentroom.com
© UCLES 2014 0580/41/O/N/14 [Turn over
18

Diagram 1 Diagram 2 Diagram 3 Diagram 4

The first four diagrams in a sequence are shown above.


The diagrams are drawn using white squares and grey squares .

(a) Complete the columns in the table for Diagram 4 and Diagram n.

Diagram 1 2 3 4 n

Number of white squares 12 20 28

Number of grey squares 0 1 4

Total number of squares 12 21 32 (n + 1)(n + 5)

[6]

(b) Work out the number of the diagram which has a total of 480 squares.

Answer(b) ................................................ [2]

www.egyptigstudentroom.com
© UCLES 2014 0580/41/O/N/14
19

(c) The total number of squares in the first n diagrams is


1
3 n3 + pn2 + qn.

(i) Use n = 1 in this expression to show that p + q = 11 32 .

Answer(c)(i)

[1]

(ii) Use n = 2 in the expression to show that 4p + 2q = 30 13 .

Answer(c)(ii)

[2]

(iii) Find the values of p and q.

Answer(c)(iii) p = ................................................

q = ................................................ [3]
__________________________________________________________________________________________

www.egyptigstudentroom.com
© UCLES 2014 0580/41/O/N/14 [Turn over
20

BLANK PAGE

Permission to reproduce items where third-party owned material protected by copyright is included has been sought and cleared where possible. Every
reasonable effort has been made by the publisher (UCLES) to trace copyright holders, but if any items requiring clearance have unwittingly been included the
publisher will be pleased to make amends at the earliest possible opportunity.

Cambridge International Examinations is part of the Cambridge Assessment Group. Cambridge Assessment is the brand name of University of Cambridge Local
www.egyptigstudentroom.com
Examinations Syndicate (UCLES), which is itself a department of the University of Cambridge.

© UCLES 2014 0580/41/O/N/14


Cambridge International Examinations
Cambridge International General Certificate of Secondary Education
*6498228295*

MATHEMATICS 0580/42
Paper 4 (Extended) October/November 2014
2 hours 30 minutes
Candidates answer on the Question Paper.
Additional Materials: Electronic calculator Geometrical instruments
Tracing paper (optional)

READ THESE INSTRUCTIONS FIRST

Write your Centre number, candidate number and name on all the work you hand in.
Write in dark blue or black pen.
You may use an HB pencil for any diagrams or graphs.
Do not use staples, paper clips, glue or correction fluid.
DO NOT WRITE IN ANY BARCODES.

Answer all questions.


If working is needed for any question it must be shown below that question.
Electronic calculators should be used.
If the degree of accuracy is not specified in the question, and if the answer is not exact, give the answer to
three significant figures. Give answers in degrees to one decimal place.
For π, use either your calculator value or 3.142.

At the end of the examination, fasten all your work securely together.
The number of marks is given in brackets [ ] at the end of each question or part question.
The total of the marks for this paper is 130.

The syllabus is approved for use in England, Wales and Northern Ireland as a Cambridge International Level 1/Level 2 Certificate.

This document consists of 19 printed pages and 1 blank page.

IB14 11_0580_42/RP
© UCLES 2014 [Turn over
www.egyptigstudentroom.com
2

1 (a) Alfonso has $75 to spend on the internet.


He spends some of the money on music, films and books.

(i) The money he spends on music, films and books is in the ratio

music : films : books = 5 : 3 : 7.

He spends $16.50 on music.

Calculate the total amount he spends on music, films and books.

Answer(a)(i) $ ................................................ [3]

(ii) Find this total amount as a percentage of the $75.

Answer(a)(ii) ............................................ % [1]

(b) The download times for the music, films and books are in the ratio

music : films : books = 2 : 9 : 1.

The total download time is 3 hours and 33 minutes.

Calculate the download time for the films.


Give your answer in hours, minutes and seconds.

Answer(b) ................ hours ................ minutes ................ seconds [3]

(c) The cost of $16.50 for the music was a reduction of 12% on the original cost.

Calculate the original cost of the music.

Answer(c) $ ................................................. [3]


__________________________________________________________________________________________

www.egyptigstudentroom.com
© UCLES 2014 0580/42/O/N/14
3

2 (a) Solve the inequality.


7x – 5 > 3(2 – 5x)

Answer(a) ................................................ [3]

(b) (i) Factorise completely.


pq – 2q – 8 + 4p

Answer(b)(i) ................................................ [2]

(ii) Factorise.
9p2 – 25

Answer(b)(ii) ................................................ [1]

(c) Solve this equation by factorising.


5x2 + x – 18 = 0

Answer(c) x = ....................... or x = ....................... [3]


__________________________________________________________________________________________

www.egyptigstudentroom.com
© UCLES 2014 0580/42/O/N/14 [Turn over
4

3 The time, t seconds, taken for each of 50 chefs to cook an omelette is recorded.

Time
20 < t Y 25 25 < t Y 30 30 < t Y 35 35 < t Y 40 40 < t Y 45 45 < t Y 50
(t seconds)
Frequency 2 6 7 19 9 7

(a) Write down the modal time interval.

Answer(a) .............................................. s [1]

(b) Calculate an estimate of the mean time.


Show all your working.

Answer(b) .............................................. s [4]

www.egyptigstudentroom.com
© UCLES 2014 0580/42/O/N/14
5

(c) A new frequency table is made from the results shown in the table opposite.

Time
20 < t Y 35 35 < t Y 40 40 < t Y 50
(t seconds)

Frequency

(i) Complete the table. [1]

(ii) On the grid, draw a histogram to show the information in this new table.

Frequency
density 2

0 t
20 25 30 35 40 45 50
Time (seconds)
[3]
__________________________________________________________________________________________

www.egyptigstudentroom.com
© UCLES 2014 0580/42/O/N/14 [Turn over
6

4
y
8

6
B

4
A
3

x
–8 –7 –6 –5 –4 –3 –2 –1 0 1 2 3 4 5 6 7 8
–1

–2

–3

–4

–5

–6

–7

–8

(a) Describe fully the single transformation that maps triangle A onto triangle B.

Answer(a) ...........................................................................................................................................

............................................................................................................................................................. [3]

www.egyptigstudentroom.com
© UCLES 2014 0580/42/O/N/14
7

(b) On the grid, draw the image of

(i) triangle A after a reflection in the line x = –3, [2]

(ii) triangle A after a rotation about the origin through 270° anticlockwise, [2]

(iii) triangle A after a translation by the vector e o .


-1
[2]
-5

(c) M is the matrix that represents the transformation in part (b)(ii).

(i) Find M.

Answer(c)(i) M = f p [2]

(ii) Describe fully the single transformation represented by M–1, the inverse of M.

Answer(c)(ii) ...............................................................................................................................

..................................................................................................................................................... [2]
__________________________________________________________________________________________

www.egyptigstudentroom.com
© UCLES 2014 0580/42/O/N/14 [Turn over
8

5 f(x) = 5x – 2 g(x) = 7 , x≠3 h(x) = 2x2 + 7x


x-3
(a) Work out

(i) f(2),

Answer(a)(i) ................................................ [1]

(ii) hg(17).

Answer(a)(ii) ................................................ [2]

(b) Solve g(x) = x + 3.

Answer(b) x = ....................... or x = ....................... [3]

www.egyptigstudentroom.com
© UCLES 2014 0580/42/O/N/14
9

(c) Solve h(x) = 11, showing all your working and giving your answers correct to 2 decimal places.

Answer(c) x = ....................... or x = ....................... [5]

(d) Find f –1(x).

Answer(d) f –1(x) = ................................................ [2]

(e) Solve g–1(x) = – 0.5 .

Answer(e) x = ................................................ [1]


__________________________________________________________________________________________

www.egyptigstudentroom.com
© UCLES 2014 0580/42/O/N/14 [Turn over
10

6 f(x) = 5x3 – 8x2 + 10

(a) Complete the table of values.

x –1.5 –1 –0.5 0 0.5 0.75 1 1.5 2

f(x) –24.9 10 8.6 7.6 7 18


[3]

(b) Draw the graph of y = f(x) for –1.5 Y x Y 2.

y
20

15

10

x
–1.5 –1 –0.5 0 0.5 1 1.5 2

–5

–10

–15

–20

–25
[4]
www.egyptigstudentroom.com
© UCLES 2014 0580/42/O/N/14
11

(c) Use your graph to find an integer value of k so that f(x) = k has

(i) exactly one solution,

Answer(c)(i) k = ................................................ [1]

(ii) three solutions.

Answer(c)(ii) k = ................................................ [1]

(d) By drawing a suitable straight line on the graph, solve the equation f(x) = 15x + 2 for –1.5 Y x Y 2.

Answer(d) x = ........................ or x = ........................ [4]

(e) Draw a tangent to the graph of y = f(x) at the point where x = 1.5 .

Use your tangent to estimate the gradient of y = f(x) when x = 1.5 .

Answer(e) ................................................ [3]


__________________________________________________________________________________________

www.egyptigstudentroom.com
© UCLES 2014 0580/42/O/N/14 [Turn over
12

NOT TO
SCALE
75 cm

55 cm
120 cm

The diagram shows a water tank in the shape of a cuboid measuring 120 cm by 55 cm by 75 cm.
The tank is filled completely with water.

(a) Show that the capacity of the water tank is 495 litres.

Answer(a)

[2]

(b) (i) The water from the tank flows into an empty cylinder at a uniform rate of 750 millilitres per second.

Calculate the length of time, in minutes, for the water to be completely emptied from the tank.

Answer(b)(i) ......................................... min [2]

(ii) When the tank is completely empty, the height of the water in the cylinder is 112 cm.

NOT TO
SCALE

112 cm

Calculate the radius of the cylinder.

Answer(b)(ii) .......................................... cm [3]


www.egyptigstudentroom.com
© UCLES 2014 0580/42/O/N/14
13

(c)
x cm
NOT TO
SCALE
75 cm

14 5 cm

55 cm
120 cm

A rod of length 145 cm is placed inside the water tank.


One end of the rod is in the bottom corner of the tank as shown.
The other end of the rod is x cm below the top corner of the tank as shown.

Calculate the value of x.

Answer(c) x = ................................................ [4]

(d) Calculate the angle that the rod makes with the base of the tank.

Answer(d) ................................................ [3]


__________________________________________________________________________________________

www.egyptigstudentroom.com
© UCLES 2014 0580/42/O/N/14 [Turn over
14

8
North

NOT TO
SCALE
P
58 km

North
74 km

A ship sails from port P to port Q.


Q is 74 km from P on a bearing of 142°.
A lighthouse, L, is 58 km from P on a bearing of 110°.

(a) Show that the distance LQ is 39.5 km correct to 1 decimal place.

Answer(a)

[5]

(b) Use the sine rule to calculate angle PQL.

Answer(b) Angle PQL = ................................................ [3]

www.egyptigstudentroom.com
© UCLES 2014 0580/42/O/N/14
15

(c) Find the bearing of

(i) P from Q,

Answer(c)(i) ................................................ [2]

(ii) L from Q.

Answer(c)(ii) ................................................ [1]

(d) The ship takes 2 hours and 15 minutes to sail the 74 km from P to Q.

Calculate the average speed in knots.


[1 knot = 1.85 km/h]

Answer(d) ....................................... knots [3]

(e) Calculate the shortest distance from the lighthouse to the path of the ship.

Answer(e) .......................................... km [3]


__________________________________________________________________________________________

www.egyptigstudentroom.com
© UCLES 2014 0580/42/O/N/14 [Turn over
16

9
Layer 1

Layer 2

Layer 3

The diagrams show layers of white and grey cubes.


Khadega places these layers on top of each other to make a tower.

(a) Complete the table for towers with 5 and 6 layers.

Number of layers 1 2 3 4 5 6

Total number of white cubes 0 1 6 15

Total number of grey cubes 1 5 9 13

Total number of cubes 1 6 15 28


[4]

(b) (i) Find, in terms of n, the total number of grey cubes in a tower with n layers.

Answer(b)(i) ................................................ [2]

(ii) Find the total number of grey cubes in a tower with 60 layers.

Answer(b)(ii) ................................................ [1]

(iii) Khadega has plenty of white cubes but only 200 grey cubes.
How many layers are there in the highest tower that she can build?

Answer(b)(iii) ................................................ [2]

www.egyptigstudentroom.com
© UCLES 2014 0580/42/O/N/14
17

(c) The expression for the total number of white cubes in a tower with n layers is pn2 + qn + 3.

Find the value of p and the value of q.


Show all your working.

Answer(c) p = ................................................

q = ................................................ [5]

(d) Find an expression, in terms of n, for the total number of cubes in a tower with n layers.
Give your answer in its simplest form.

Answer(d) ................................................ [2]


__________________________________________________________________________________________

www.egyptigstudentroom.com
© UCLES 2014 0580/42/O/N/14 [Turn over
18

10 Kenwyn plays a board game.


Two cubes (dice) each have faces numbered 1, 2, 3, 4, 5 and 6.
In the game, a throw is rolling the two fair 6-sided dice and then adding the numbers on their top faces.
This total is the number of spaces to move on the board.
For example, if the numbers are 4 and 3, he moves 7 spaces.

(a) Giving each of your answers as a fraction in its simplest form, find the probability that he moves

(i) two spaces with his next throw,

Answer(a)(i) ................................................ [2]

(ii) ten spaces with his next throw.

Answer(a)(ii) ................................................ [3]

(b) What is the most likely number of spaces that Kenwyn will move with his next throw?
Explain your answer.

Answer(b) .................... because .........................................................................................................

............................................................................................................................................................. [2]

www.egyptigstudentroom.com
© UCLES 2014 0580/42/O/N/14
19

(c)

95 96 97 98 99 100
Go back WIN
3 spaces

To win the game he must move exactly to the 100th space.


Kenwyn is on the 97th space.
If his next throw takes him to 99, he has to move back to 96.
If his next throw takes him over 100, he stays on 97.

Find the probability that he reaches 100 in either of his next two throws.

Answer(c) ................................................ [5]


__________________________________________________________________________________________

www.egyptigstudentroom.com
© UCLES 2014 0580/42/O/N/14 [Turn over
20

BLANK PAGE

Permission to reproduce items where third-party owned material protected by copyright is included has been sought and cleared where possible. Every
reasonable effort has been made by the publisher (UCLES) to trace copyright holders, but if any items requiring clearance have unwittingly been included the
publisher will be pleased to make amends at the earliest possible opportunity.

Cambridge International Examinations is part of the Cambridge Assessment Group. Cambridge Assessment is the brand name of University of Cambridge Local
www.egyptigstudentroom.com
Examinations Syndicate (UCLES), which is itself a department of the University of Cambridge.

© UCLES 2014 0580/42/O/N/14


Cambridge International Examinations
Cambridge International General Certificate of Secondary Education
*2401571107*

MATHEMATICS 0580/43
Paper 4 (Extended) October/November 2014
2 hours 30 minutes
Candidates answer on the Question Paper.
Additional Materials: Electronic calculator Geometrical instruments
Tracing paper (optional)

READ THESE INSTRUCTIONS FIRST

Write your Centre number, candidate number and name on all the work you hand in.
Write in dark blue or black pen.
You may use an HB pencil for any diagrams or graphs.
Do not use staples, paper clips, glue or correction fluid.
DO NOT WRITE IN ANY BARCODES.

Answer all questions.


If working is needed for any question it must be shown below that question.
Electronic calculators should be used.
If the degree of accuracy is not specified in the question, and if the answer is not exact, give the answer to
three significant figures. Give answers in degrees to one decimal place.
For π, use either your calculator value or 3.142.

At the end of the examination, fasten all your work securely together.
The number of marks is given in brackets [ ] at the end of each question or part question.
The total of the marks for this paper is 130.

The syllabus is approved for use in England, Wales and Northern Ireland as a Cambridge International Level 1/Level 2 Certificate.

This document consists of 16 printed pages.

IB14 11_0580_43/RP
© UCLES 2014 [Turn over
www.egyptigstudentroom.com
2

1 (a) ABCD is a trapezium.

A 11 cm B
NOT TO
SCALE
4.7 cm

D C
2.6 cm
17 cm

(i) Calculate the length of AD.

Answer(a)(i) AD = .......................................... cm [2]

(ii) Calculate the size of angle BCD.

Answer(a)(ii) Angle BCD = ................................................ [3]

(iii) Calculate the area of the trapezium ABCD.

Answer(a)(iii) ......................................... cm2 [2]

(b) A similar trapezium has perpendicular height 9.4 cm.

Calculate the area of this trapezium.

Answer(b) ......................................... cm2 [3]


__________________________________________________________________________________________

www.egyptigstudentroom.com
© UCLES 2014 0580/43/O/N/14
3

2 There are three different areas, A, B and C, for seating in a theatre.


The numbers of seats in each area are in the ratio A : B : C = 11 : 8 : 7 .
There are 920 seats in area B.

(a) (i) Show that there are 805 seats in area C.

Answer(a)(i)

[1]

(ii) Write the number of seats in area B as a percentage of the total number of seats.

Answer(a)(ii) ............................................ % [2]

(b) The cost of a ticket for a seat in each area of the theatre is shown in the table.

Area A $11.50
Area B $15
Area C $22.50

3
For a concert 80% of area B tickets were sold and 5 of area C tickets were sold.
The total amount of money taken from ticket sales was $35 834.

Calculate the number of area A tickets that were sold.

Answer(b) ................................................ [5]

(c) The total ticket sales of $35 834 was 5% less than the ticket sales at the previous concert.

Calculate the ticket sales at the previous concert.

Answer(c) $ ................................................. [3]


__________________________________________________________________________________________
www.egyptigstudentroom.com
© UCLES 2014 0580/43/O/N/14 [Turn over
4

3
A
B
NOT TO
52° SCALE

D
O

56°
C E

A, B, C and D are points on a circle, centre O.


CE is a tangent to the circle at C.

(a) Find the sizes of the following angles and give a reason for each answer.

(i) Angle DAC = ..................... because ..........................................................................................

..................................................................................................................................................... [2]

(ii) Angle DOC = ..................... because .........................................................................................

..................................................................................................................................................... [2]

(iii) Angle BCO = ..................... because ..........................................................................................

..................................................................................................................................................... [2]

www.egyptigstudentroom.com
© UCLES 2014 0580/43/O/N/14
5

(b) CE = 8.9 cm and CB = 7 cm.

(i) Calculate the length of BE.

Answer(b)(i) BE = .......................................... cm [4]

(ii) Calculate angle BEC.

Answer(b)(ii) Angle BEC = ................................................ [3]


__________________________________________________________________________________________

www.egyptigstudentroom.com
© UCLES 2014 0580/43/O/N/14 [Turn over
6

4 Yeung and Ariven compete in a triathlon race.


3
The probability that Yeung finishes this race is 5 .
2
The probability that Ariven finishes this race is 3 .

(a) (i) Which of them is more likely to finish this race?


Give a reason for your answer.

Answer(a)(i) ...................................................... because ..........................................................

..................................................................................................................................................... [1]

(ii) Find the probability that they both finish this race.

Answer(a)(ii) ................................................ [2]

(iii) Find the probability that only one of them finishes this race.

Answer(a)(iii) ................................................ [3]

www.egyptigstudentroom.com
© UCLES 2014 0580/43/O/N/14
7

(b) After the first race, Yeung competes in two further triathlon races.

(i) Complete the tree diagram.

First race Second race Third race


7
10 Finishes
6 Finishes
7
Does not
........ finish
Finishes
7
3 10 Finishes
5 ........ Does not
finish
Does not
........ finish
7
10 Finishes
6 Finishes
7
........ Does not
........ finish
Does not
finish 7
10 Finishes
........ Does not
finish
Does not
........ finish
[3]

(ii) Calculate the probability that Yeung finishes all three of his races.

Answer(b)(ii) ................................................ [2]

(iii) Calculate the probability that Yeung finishes at least one of his races.

Answer(b)(iii) ................................................ [3]


__________________________________________________________________________________________

www.egyptigstudentroom.com
© UCLES 2014 0580/43/O/N/14 [Turn over
8

P=f p Q=f p R=e o


0 -1 1 -2 -3
5
1 0 0 1 5
(a) Work out

(i) 4P,

Answer(a)(i) [1]

(ii) P – Q,

Answer(a)(ii) [1]

(iii) P2,

Answer(a)(iii) [2]

(iv) QR.

Answer(a)(iv) [2]

(b) Find the matrix S, so that QS = f p.


1 0
0 1

Answer(b) [3]
__________________________________________________________________________________________
www.egyptigstudentroom.com
© UCLES 2014 0580/43/O/N/14
9

6 (a) Simplify.
3
(i) x3 ÷
x5

Answer(a)(i) ................................................ [1]

(ii) 5xy8 × 3x6y–5

Answer(a)(ii) ................................................ [2]


2

(iii) (64x12) 3

Answer(a)(iii) ................................................ [2]

(b) Solve 3x2 – 7x – 12 = 0.


Show your working and give your answers correct to 2 decimal places.

Answer(b) x = ........................ or x = ........................ [4]

x2 - 25 .
(c) Simplify
x3 - 5x2

Answer(c) ................................................ [3]


__________________________________________________________________________________________

www.egyptigstudentroom.com
© UCLES 2014 0580/43/O/N/14 [Turn over
10

7
A P
NOT TO
SCALE
56° 6.5 cm

8 cm 8 cm
O x

C B Q

The diagram shows a triangle and a sector of a circle.


In triangle ABC, AB = AC = 8 cm and angle BAC = 56°.
Sector OPQ has centre O, sector angle x and radius 6.5 cm.

(a) Show that the area of triangle ABC is 26.5 cm2 correct to 1 decimal place.

Answer(a)

[2]

(b) The area of sector OPQ is equal to the area of triangle ABC.

(i) Calculate the sector angle x.

Answer(b)(i) ................................................ [3]

(ii) Calculate the perimeter of the sector OPQ.

Answer(b)(ii) .......................................... cm [3]

www.egyptigstudentroom.com
© UCLES 2014 0580/43/O/N/14
11

(c) The diagram shows a sector of a circle, radius r cm.

NOT TO
SCALE

r cm

30°

(i) Show that the area of the shaded segment is 4 r 2` 3 π - 1 j cm2.


1 1

Answer(c)(i)

[4]

(ii) The area of the segment is 5 cm2.

Find the value of r.

Answer(c)(ii) r = ................................................ [3]


__________________________________________________________________________________________

www.egyptigstudentroom.com
© UCLES 2014 0580/43/O/N/14 [Turn over
12

8 (a) A straight line joins the points (–1, –4) and (3, 8).

(i) Find the midpoint of this line.

Answer(a)(i) (...................... , ......................) [2]

(ii) Find the equation of this line.


Give your answer in the form y = mx + c.

Answer(a)(ii) y = ................................................ [3]

(b) (i) Factorise x2 + 3x – 10.

Answer(b)(i) ................................................ [2]

(ii) The graph of y = x2 + 3x – 10 is sketched below.

y
NOT TO
SCALE

x
(a, 0) 0 (b, 0)
(0, c)

Write down the values of a, b and c.

Answer(b)(ii) a = ................................................

b = ................................................

c = ................................................ [3]

(iii) Write down the equation of the line of symmetry of the graph of y = x2 + 3x – 10.

Answer(b)(iii) ................................................ [1]

www.egyptigstudentroom.com
© UCLES 2014 0580/43/O/N/14
13

(c) Sketch the graph of y = 18 + 7x – x2 on the axes below.


Indicate clearly the values where the graph crosses the x and y axes.

y
NOT TO
SCALE

x
0

[4]

(d) (i) x2 + 12x – 7 = (x + p)2 – q

Find the value of p and the value of q.

Answer(d)(i) p = ................................................

q = ................................................ [3]

(ii) Write down the minimum value of y for the graph of y = x2 + 12x – 7.

Answer(d)(ii) ................................................ [1]


__________________________________________________________________________________________

www.egyptigstudentroom.com
© UCLES 2014 0580/43/O/N/14 [Turn over
14

9 (a) Ricardo asks some motorists how many litres of fuel they use in one day.
The numbers of litres, correct to the nearest litre, are shown in the table.

Number of litres 16 17 18 19 20
Number of motorists 11 10 p 4 8

(i) For this table, the mean number of litres is 17.7 .

Calculate the value of p.

Answer(a)(i) p = ................................................ [4]

(ii) Find the median number of litres.

Answer(a)(ii) ....................................... litres [1]

(b) Manuel completed a journey of 320 km in his car.


The fuel for the journey cost $1.28 for every 6.4 km travelled.

(i) Calculate the cost of fuel for this journey.

Answer(b)(i) $ ................................................. [2]

(ii) When Manuel travelled 480 km in his car it used 60 litres of fuel.
Manuel’s car used fuel at the same rate for the journey of 320 km.

Calculate the number of litres of fuel the car used for the journey of 320 km.

Answer(b)(ii) ....................................... litres [2]

(iii) Calculate the cost per litre of fuel used for the journey of 320 km.

Answer(b)(iii) $ ................................................. [2]

www.egyptigstudentroom.com
© UCLES 2014 0580/43/O/N/14
15

(c) Ellie drives a car at a constant speed of 30 m/s correct to the nearest 5 m/s.
She maintains this speed for 5 minutes correct to the nearest 10 seconds.

Calculate the upper bound of the distance in kilometres that Ellie could have travelled.

Answer(c) .......................................... km [5]


__________________________________________________________________________________________

10 (a)
(3x – 5) cm
NOT TO
(2x – 3) cm SCALE
(15 – 2x) cm

(2x + 7) cm

(i) Write an expression, in terms of x, for the perimeter of the quadrilateral.


Give your answer in its simplest form.

Answer(a)(i) .......................................... cm [2]

(ii) The perimeter of the quadrilateral is 32 cm.

Find the length of the longest side of the quadrilateral.

Answer(a)(ii) .......................................... cm [3]

Question 10(b) is printed on the next page.


www.egyptigstudentroom.com
© UCLES 2014 0580/43/O/N/14 [Turn over
16

(b)
(5a – 2b) m (6b – a) m
14 m
(7a – 6b) m NOT TO
SCALE
am
13.5 m
(3b + a) m

The triangle has a perimeter of 32.5 m.


The quadrilateral has a perimeter of 39.75 m.

Write two equations in terms of a and b and simplify them.


Use an algebraic method to find the values of a and b.
Show all your working.

Answer(b) a = ................................................

b = ................................................ [6]

Permission to reproduce items where third-party owned material protected by copyright is included has been sought and cleared where possible. Every
reasonable effort has been made by the publisher (UCLES) to trace copyright holders, but if any items requiring clearance have unwittingly been included the
publisher will be pleased to make amends at the earliest possible opportunity.

Cambridge International Examinations is part of the Cambridge Assessment Group. Cambridge Assessment is the brand name of University of Cambridge Local
www.egyptigstudentroom.com
Examinations Syndicate (UCLES), which is itself a department of the University of Cambridge.

© UCLES 2014 0580/43/O/N/14


Cambridge International Examinations
Cambridge International General Certificate of Secondary Education
* 4 6 8 6 6 1 7 2 9 3 *

maThEmaTICs 0580/41
Paper 4 (Extended) may/June 2015
2 hours 30 minutes
Candidates answer on the Question Paper.
Additional Materials: Electronic calculator Geometrical instruments
Tracing paper (optional).

REaD ThEsE INsTRUCTIONs FIRsT

Write your Centre number, candidate number and name on all the work you hand in.
Write in dark blue or black pen.
You may use an HB pencil for any diagrams or graphs.
Do not use staples, paper clips, glue or correction fluid.
DO NOT WRITE IN ANY BARCODES.

Answer all questions.


If working is needed for any question it must be shown below that question.
Electronic calculators should be used.
If the degree of accuracy is not specified in the question, and if the answer is not exact, give the answer to
three significant figures. Give answers in degrees to one decimal place.
For π, use either your calculator value or 3.142.

At the end of the examination, fasten all your work securely together.
The number of marks is given in brackets [ ] at the end of each question or part question.
The total of the marks for this paper is 130.

The syllabus is approved for use in England, Wales and Northern Ireland as a Cambridge International Level 1/Level 2 Certificate.

This document consists of 20 printed pages.

DC (LK/SW) 101617/4
© UCLES 2015 [Turn over
www.egyptigstudentroom.com
2

1 12 000 vehicles drive through a road toll on one day.


The ratio cars : trucks : motorcycles = 13 : 8 : 3.

(a) (i) Show that 6500 cars drive through the road toll on that day.

Answer(a)(i)

[1]

(ii) Calculate the number of trucks that drive through the road toll on that day.

Answer(a)(ii) ................................................. [1]

(b) The toll charges in 2014 are shown in the table.

Vehicle Charge
Cars $2
Trucks $5
Motorcycles $1

Show that the total amount paid in tolls on that day is $34 500.

Answer(b)

[2]

www.egyptigstudentroom.com
© UCLES 2015 0580/41/M/J/15
3

(c) This total amount is a decrease of 8% on the total amount paid on the same day in 2013.

Calculate the total amount paid on that day in 2013.

Answer(c) $ .................................................. [3]

(d) 2750 of the 6500 car drivers pay their toll using a credit card.

Write down, in its simplest terms, the fraction of car drivers who pay using a credit card.

Answer(d) ................................................. [2]

(e) To the nearest thousand, 90 000 cars drive through the road toll in one week.

Write down the lower bound for this number of cars.

Answer(e) ................................................. [1]

www.egyptigstudentroom.com
© UCLES 2015 0580/41/M/J/15 [Turn over
4

1
2 The table shows some values for y = x2 - , x ! 0.
2x

x –2 –1.5 –1 –0.5 –0.25 –0.2 0.2 0.25 0.5 1 1.5 2

y 4.25 2.58 2.06 2.54 –2.46 –1.94 1.92 3.75

(a) Complete the table of values. [4]


1
(b) On the grid, draw the graph of y = x2 – for – 2  x  – 0.2 and 0.2  x  2.
2x
y

x
–2 –1 0 1 2

–1

–2

–3
[5]
1
(c) By drawing a suitable line, use your graph to solve the equation x 2 - = 2.
2x

Answer(c) x = ..................... or x = ..................... or x = ..................... [3]


www.egyptigstudentroom.com
© UCLES 2015 0580/41/M/J/15
5

1
(d) The equation x 2 - = k has only one solution.
2x
Write down the range of values of k for which this is possible.

Answer(d) ................................................. [2]

(e) By drawing a suitable tangent, find an estimate of the gradient of the curve at the point where x = –1.

Answer(e) ................................................. [3]

www.egyptigstudentroom.com
© UCLES 2015 0580/41/M/J/15 [Turn over
6

3
y

7
6
5
4
3
C
2
A
1
x
–12 –11 –10 –9 –8 –7 –6 –5 –4 –3 –2 –1 0 1 2 3 4 5 6
–1
–2
–3
–4
B
–5
–6
–7

(a) Draw the image of

shape A after a translation by f p,


-1
(i) [2]
3

(ii) shape A after a rotation through 180° about the point (0, 0), [2]

shape A after the transformation represented by the matrix f p.


1 0
(iii) [3]
0 -1

(b) Describe fully the single transformation that maps shape A onto shape B.

Answer(b) .................................................................................................................................................

.............................................................................................................................................................. [3]

(c) Find the matrix which represents the transformation that maps shape A onto shape C.

Answer(c) f p [2]

www.egyptigstudentroom.com
© UCLES 2015 0580/41/M/J/15
7

4 30 students were asked if they had a bicycle (B), a mobile phone (M) and a computer (C).
The results are shown in the Venn diagram.


B M
2 4 x

7
1 6

3 2
C

(a) Work out the value of x.

Answer(a) x = ................................................. [1]

(b) Use set notation to describe the shaded region in the Venn diagram.

Answer(b) ................................................. [1]

(c) Find n(C (M B)).

Answer(c) ................................................. [1]

(d) A student is chosen at random.

(i) Write down the probability that the student is a member of the set M  .

Answer(d)(i) ................................................. [1]

(ii) Write down the probability that the student has a bicycle.

Answer(d)(ii) ................................................. [1]

(e) Two students are chosen at random from the students who have computers.

Find the probability that each of these students has a mobile phone but no bicycle.

Answer(e) ................................................ [3]

www.egyptigstudentroom.com
© UCLES 2015 0580/41/M/J/15 [Turn over
8

5 (a) Andrei stands on level horizontal ground, 294 m from the foot of a vertical tower which is 55 m high.

(i) Calculate the angle of elevation of the top of the tower.

Answer(a)(i) ................................................. [2]

(ii) Andrei walks a distance x metres directly towards the tower.


The angle of elevation of the top of the tower is now 24.8°.

Calculate the value of x.

Answer(a)(ii) x = ................................................. [4]

www.egyptigstudentroom.com
© UCLES 2015 0580/41/M/J/15
9

(b) The diagram shows a pyramid with a horizontal rectangular base.

NOT TO
y SCALE

4m

3m

4.8 m

The rectangular base has length 4.8 m and width 3 m and the height of the pyramid is 4 m.

Calculate

(i) y, the length of a sloping edge of the pyramid,

Answer(b)(i) y = ............................................. m [4]

(ii) the angle between a sloping edge and the rectangular base of the pyramid.

Answer(b)(ii) ................................................ [2]

www.egyptigstudentroom.com
© UCLES 2015 0580/41/M/J/15 [Turn over
10

6 The table shows the time, t minutes, that 400 people take to complete a test.

Time taken
0  t  10 10  t  24 24  t  30 30  t  40 40  t  60 60  t  70
(t mins)
Frequency 10 90 135 85 70 10

(a) (i) Write down the modal time interval.

Answer(a)(i) .......................................... min [1]

(ii) Calculate an estimate of the mean time taken to complete the test.

Answer(a)(ii) .......................................... min [4]

(b) (i) Complete the table of cumulative frequencies.

Time taken
t  10 t  24 t  30 t  40 t  60 t  70
(t mins)
Cumulative
10 100 400
frequency
[2]

(ii) On the grid opposite, draw a cumulative frequency diagram to show this information.

www.egyptigstudentroom.com
© UCLES 2015 0580/41/M/J/15
11

400

350

300

250
Cumulative
frequency

200

150

100

50

t
0 10 20 30 40 50 60 70
Time taken (minutes)
[3]
(c) Use your graph to estimate

(i) the median time,


Answer(c)(i) .......................................... min [1]

(ii) the inter-quartile range,

Answer(c)(ii) .......................................... min [2]

(iii) the 15th percentile,

Answer(c)(iii) .......................................... min [2]

(iv) the number of people who took more than 50 minutes.

Answer(c)(iv) ................................................. [2]

www.egyptigstudentroom.com
© UCLES 2015 0580/41/M/J/15 [Turn over
12

7 (a)
P

NOT TO
SCALE

8.4 cm

7.6 cm 62°
R

In the triangle PQR, QR = 7.6 cm and PR = 8.4 cm.


Angle QRP = 62°.

Calculate

(i) PQ,

Answer(a)(i) PQ = ........................................... cm [4]

(ii) the area of triangle PQR.

Answer(a)(ii) .......................................... cm2 [2]

www.egyptigstudentroom.com
© UCLES 2015 0580/41/M/J/15
13

(b)
North

NOT TO
North SCALE

63 km

The diagram shows the positions of three small islands G, H and J.


The bearing of H from G is 045°.
The bearing of J from G is 126°.
The bearing of J from H is 164°.
The distance HJ is 63 km.

Calculate the distance GJ.

Answer(b) GJ = .......................................... km [5]

www.egyptigstudentroom.com
© UCLES 2015 0580/41/M/J/15 [Turn over
14

8 (a) Jamil, Kiera and Luther collect badges.


Jamil has x badges.
Kiera has 12 badges more than Jamil.
Luther has 3 times as many badges as Kiera.
Altogether they have 123 badges.

Form an equation and solve it to find the value of x.

Answer(a) x = ................................................. [3]

(b) Find the integer values of t which satisfy the inequalities.

4t + 7 < 39  7t + 2

Answer(b) ................................................. [3]

(c) Solve the following equations.


21 - x
(i) =4
x+3

Answer(c)(i) x = ................................................. [3]

www.egyptigstudentroom.com
© UCLES 2015 0580/41/M/J/15
15

(ii) 3x2 + 7x – 5 = 0

Show all your working and give your answers correct to 2 decimal places.

Answer(c)(ii) x = ......................... or x = ......................... [4]


__________________________________________________________________________________________

www.egyptigstudentroom.com
© UCLES 2015 0580/41/M/J/15 [Turn over
16

9 (a) The points A, B, C and D lie on a circle.


AC is a diameter of the circle.
ST is the tangent to the circle at A.

S
A
48°
T

27° NOT TO
B x° D SCALE

Find the value of

(i) x,

Answer(a)(i) x = ................................................. [2]

(ii) y.

Answer(a)(ii) y = ................................................. [2]

www.egyptigstudentroom.com
© UCLES 2015 0580/41/M/J/15
17

(b) The diagram shows a shaded shape formed by three semi-circular arcs.
The radius of each semi-circle is shown in the diagram.

NOT TO
SCALE

6 cm
4 cm
2 cm

(i) Calculate the perimeter of the shaded shape.

Answer(b)(i) ........................................... cm [2]

(ii) The shaded shape is made from metal 1.6 mm thick.

Calculate the volume of metal used to make this shape.


Give your answer in cubic millimetres.

Answer(b)(ii) ........................................ mm3 [5]

www.egyptigstudentroom.com
© UCLES 2015 0580/41/M/J/15 [Turn over
18

10 The diagram is a scale drawing of three straight roads, AB, BC and CD.
The scale is 1 : 5000.

A B

Scale 1 : 5000

(a) Find the actual length of the road BC.


Give your answer in metres.

Answer(a) ............................................. m [2]

(b) Another straight road starts at M, the midpoint of AB.


This road is perpendicular to AB and it meets the road CD at X.

Using a straight edge and compasses only, construct MX. [2]

www.egyptigstudentroom.com
© UCLES 2015 0580/41/M/J/15
19

(c) There is a park in the area enclosed by the four roads.

The park is

• less than 290 m from B


and
• nearer to CD than to CB.

Using a ruler and compasses only, construct the boundaries of the park.

Leave in all your construction arcs and label the park P. [5]

Question 11 is printed on the next page.

www.egyptigstudentroom.com
© UCLES 2015 0580/41/M/J/15 [Turn over
20

11 (a) Make x the subject of the formula.


xr
A-x =
t

Answer(a) x = ................................................. [4]

(b) Find the value of a and the value of b when x2 – 16x + a = (x + b)2.

Answer(b) a = ................................................

b = ................................................. [3]

(c) Write as a single fraction in its simplest form.


6 5
-
x - 4 3x - 2

Answer(c) ................................................. [3]

Permission to reproduce items where third-party owned material protected by copyright is included has been sought and cleared where possible. Every reasonable
effort has been made by the publisher (UCLES) to trace copyright holders, but if any items requiring clearance have unwittingly been included, the publisher will
be pleased to make amends at the earliest possible opportunity.

To avoid the issue of disclosure of answer-related information to candidates, all copyright acknowledgements are reproduced online in the Cambridge International
Examinations Copyright Acknowledgements Booklet. This is produced for each series of examinations and is freely available to download at www.cie.org.uk after
the live examination series.

Cambridge International Examinations is part of the Cambridge Assessment Group. Cambridge Assessment is the brand name of University of Cambridge Local

www.egyptigstudentroom.com
Examinations Syndicate (UCLES), which is itself a department of the University of Cambridge.

© UCLES 2015 0580/41/M/J/15


Cambridge International Examinations
Cambridge International General Certificate of Secondary Education

CANDIDATE
NAME

CENTRE CANDIDATE
NUMBER NUMBER
* 8 0 4 2 5 9 9 4 4 0 *

MATHEMATICS 0580/42
Paper 4 (Extended) May/June 2015
2 hours 30 minutes
Candidates answer on the Question Paper.
Additional Materials: Electronic calculator Geometrical instruments
Tracing paper (optional)

READ THESE INSTRUCTIONS FIRST

Write your Centre number, candidate number and name on all the work you hand in.
Write in dark blue or black pen.
You may use an HB pencil for any diagrams or graphs.
Do not use staples, paper clips, glue or correction fluid.
DO NOT WRITE IN ANY BARCODES.

Answer all questions.


If working is needed for any question it must be shown below that question.
Electronic calculators should be used.
If the degree of accuracy is not specified in the question, and if the answer is not exact, give the answer to
three significant figures. Give answers in degrees to one decimal place.
For π, use either your calculator value or 3.142.

At the end of the examination, fasten all your work securely together.
The number of marks is given in brackets [ ] at the end of each question or part question.
The total of the marks for this paper is 130.

The syllabus is approved for use in England, Wales and Northern Ireland as a Cambridge International Level 1/Level 2 Certificate.

This document consists of 20 printed pages.

DC (RW/SG) 103663/3
© UCLES 2015 [Turn over
www.egyptigstudentroom.com
2

1 (a) Last year a golf club charged $1650 for a family membership.
This year the cost increased by 12%.

Calculate the cost of a family membership this year.

Answer(a) $ ................................................ [2]

(b) The golf club runs a competition.


The total prize money is shared in the ratio 1st prize : 2nd prize = 9 : 5.
The 1st prize is $500 more than the 2nd prize.

(i) Calculate the total prize money for the competition.

Answer(b)(i) $ ................................................ [2]

(ii) What percentage of the total prize money is given as the 1st prize?

Answer(b)(ii) .............................................% [1]

(c) For the members of the golf club the ratio men : children = 11 : 2.
The ratio women : children = 10 : 3.

(i) Find the ratio men : women.

Answer(c)(i) ................... : ................... [2]

www.egyptigstudentroom.com
© UCLES 2015 0580/42/M/J/15
3

(ii) The golf club has 24 members who are children.

Find the total number of members.

Answer(c)(ii) ................................................ [3]

(d) The club shop sold a box of golf balls for $20.40 .
The shop made a profit of 20% on the cost price.

Calculate the cost price of the golf balls.

Answer(d) $ ................................................ [3]


__________________________________________________________________________________________

www.egyptigstudentroom.com
© UCLES 2015 0580/42/M/J/15 [Turn over
4

NOT TO
O SCALE
C
E

48°
B A

In the diagram, B, C, D and E lie on the circle, centre O.


AB and AD are tangents to the circle.
Angle BAD = 48°.

(a) Find

(i) angle ABD,

Answer(a)(i) Angle ABD = ................................................ [1]

(ii) angle OBD,

Answer(a)(ii) Angle OBD = ................................................ [1]

(iii) angle BCD,

Answer(a)(iii) Angle BCD = ................................................ [2]

(iv) angle BED.

Answer(a)(iv) Angle BED = ................................................ [1]

(b) The radius of the circle is 15 cm.

Calculate the area of triangle BOD.

Answer(b) ......................................... cm2 [2]

(c) Give a reason why ABOD is a cyclic quadrilateral.

Answer(c) ............................................................................................................................................

............................................................................................................................................................. [1]
__________________________________________________________________________________________

www.egyptigstudentroom.com
© UCLES 2015 0580/42/M/J/15
5

3 On the first part of a journey, Alan drove a distance of x km and his car used 6 litres of fuel.
600
The rate of fuel used by his car was litres per 100 km.
x
(a) Alan then drove another (x + 20) km and his car used another 6 litres of fuel.

(i) Write down an expression, in terms of x, for the rate of fuel used by his car on this part
of the journey.
Give your answer in litres per 100 km.

Answer(a)(i) .............................. litres per 100 km [1]

(ii) On this part of the journey the rate of fuel used by the car decreased by 1.5 litres per 100 km.

Show that x2 + 20x – 8000 = 0.

Answer(a)(ii)

[4]

(b) Solve the equation x2 + 20x – 8000 = 0.

Answer(b) x = ............................... or x = ............................... [3]

(c) Find the rate of fuel used by Alan’s car for the complete journey.
Give your answer in litres per 100 km.

Answer(c) .............................. litres per 100 km [2]


__________________________________________________________________________________________
www.egyptigstudentroom.com
© UCLES 2015 0580/42/M/J/15 [Turn over
6

4 (a) A sector of a circle has radius 12 cm and an angle of 135°.

(i) Calculate the length of the arc of this sector.


Give your answer as a multiple of π. NOT TO
SCALE

135° 12 cm

Answer(a)(i) .......................................... cm [2]

(ii) The sector is used to make a cone.

(a) Calculate the base radius, r. NOT TO


SCALE
h 12 cm

Answer(a)(ii)(a) r = .......................................... cm [2]

(b) Calculate the height of the cone, h.

Answer(a)(ii)(b) h = .......................................... cm [3]

(b) The diagram shows a plant pot.


It is made by removing a small cone from a larger cone and adding a circular base.

NOT TO
SCALE

www.egyptigstudentroom.com
© UCLES 2015 0580/42/M/J/15
7

This is the cross section of the plant pot. 15 cm

(i) Find l.

35 cm

8 cm

NOT TO
SCALE

Answer(b)(i) l = .......................................... cm [3]

(ii) Calculate the total surface area of the outside of the plant pot.
[The curved surface area, A, of a cone with radius r and slant height l is A = πrl.]

Answer(b)(ii) ......................................... cm2 [3]

(c) Some cones are mathematically similar.


For these cones, the mass, M grams, is proportional to the cube of the base radius, r cm.
One of the cones has mass 1458 grams and base radius 4.5 cm.

(i) Find an expression for M in terms of r.

Answer(c)(i) M = ................................................ [2]

(ii) Two of the cones have radii in the ratio 2 : 3.

Write down the ratio of their masses.

Answer(c)(ii) ................. : ................. [1]


__________________________________________________________________________________________
www.egyptigstudentroom.com
© UCLES 2015 0580/42/M/J/15 [Turn over
8

12
5 y = x2 – 2x + x , x ! 0

(a) Complete the table of values.

x –4 –3 –2 –1 –0.5 0.5 1 2 3 4

y 21 11 –9 –22.75 23.25 11 6 11
[2]
12
(b) On the grid, draw the graph of y = x2 – 2x + x for –4  x  –0.5 and 0.5  x  4.
y
25

20

15

10

x
–4 –3 –2 –1 0 1 2 3 4

–5

–10

–15

–20

–25
[5]
www.egyptigstudentroom.com
© UCLES 2015 0580/42/M/J/15
9

(c) By drawing a suitable tangent, find an estimate of the gradient of the graph at the point (1, 11).

Answer(c) ................................................ [3]

12
(d) The equation x2 – 2x + = k has exactly two distinct solutions.
x
Use the graph to find

(i) the value of k,

Answer(d)(i) k = ................................................ [1]


12
(ii) the solutions of x2 – 2x + = k.
x
Answer(d)(ii) x = ........................... or x = ........................... [2]

(e) The equation x3 + ax2 + bx + c = 0 can be solved by drawing the line y = 3x + 1 on the grid.

Find the value of a, the value of b and the value of c.

Answer(e) a = ................................................

b = ................................................

c = ................................................ [3]
__________________________________________________________________________________________

www.egyptigstudentroom.com
© UCLES 2015 0580/42/M/J/15 [Turn over
10

6 The diagram shows the positions of two ships, A and B, and a coastguard station, C.

North

B NOT TO
95.5 km SCALE

83.1 km
101°

(a) Calculate the distance, AB, between the two ships.


Show that it rounds to 138 km, correct to the nearest kilometre.

Answer(a)

[4]

(b) The bearing of the coastguard station C from ship A is 146°.

Calculate the bearing of ship B from ship A.

Answer(b) ................................................ [4]

www.egyptigstudentroom.com
© UCLES 2015 0580/42/M/J/15
11

(c)
L

North 46.2 km

NOT TO
SCALE
21°
45°

At noon, a lighthouse, L, is 46.2 km from ship B on the bearing 021°.


Ship B sails north west.

Calculate the distance ship B must sail from its position at noon to be at its closest distance to the
lighthouse.

Answer(c) .......................................... km [2]


__________________________________________________________________________________________

www.egyptigstudentroom.com
© UCLES 2015 0580/42/M/J/15 [Turn over
12

7 (a) A group of 50 students estimated the mass, M grams, of sweets in a jar.


The results are shown in the table.

Mass (M grams) Number of students

0 < M  200 5

200 < M  300 9

300 < M  350 18

350 < M  400 12

400 < M  500 6

(i) Calculate an estimate of the mean.

Answer(a)(i) ..................................... grams [4]

(ii) Complete this histogram to show the information in the table.

0.4

0.3

Frequency
0.2
density

0.1

M
0
100 200 300 400 500
Mass (grams)
[3]

www.egyptigstudentroom.com
© UCLES 2015 0580/42/M/J/15
13

(b) A group of 50 adults also estimated the mass, M grams, of the sweets in the jar.
The histogram below shows information about their estimates.

Use the histograms to make two comparisons between the distributions of the estimates of the students
and the adults.

0.4

0.3

Frequency
0.2
density

0.1

M
0
100 200 300 400 500
Mass (grams)

Answer(b)

1 ..........................................................................................................................................................

.............................................................................................................................................................

2 ..........................................................................................................................................................

............................................................................................................................................................. [2]
__________________________________________________________________________________________

www.egyptigstudentroom.com
© UCLES 2015 0580/42/M/J/15 [Turn over
14

8 Sima sells x biscuits and y cakes.

(a) (i) She sells at least 100 biscuits.

Write down an inequality in x.

Answer(a)(i) ................................................ [1]

(ii) She sells at least 120 cakes.

Write down an inequality in y.

Answer(a)(ii) ................................................ [1]

(iii) She sells a maximum of 300 biscuits and cakes altogether.

Write down an inequality in x and y.

Answer(a)(iii) ................................................ [1]

(iv) Sima makes a profit of 40 cents on each biscuit and 80 cents on each cake.
Her total profit is at least $160.

Show that x + 2y  400.

Answer(a)(iv)

[1]

www.egyptigstudentroom.com
© UCLES 2015 0580/42/M/J/15
15

(b) On the grid, draw four lines to show the four inequalities and shade the unwanted regions.

y
400

300

200

100

x
0
100 200 300 400 500
[6]

(c) Calculate Sima’s maximum profit.


Give your answer in dollars.

Answer(c) $ ................................................ [2]


__________________________________________________________________________________________

www.egyptigstudentroom.com
© UCLES 2015 0580/42/M/J/15 [Turn over
16

9 (a) Expand and simplify.


3x(x – 2) – 2x(3x – 5)

Answer(a) ................................................ [3]

(b) Factorise the following completely.

(i) 6w + 3wy – 4x – 2xy

Answer(b)(i) ................................................ [2]

(ii) 4x2 – 25y2

Answer(b)(ii) ................................................ [2]

(c) Simplify.
3
16 - 2
c 4m
9x

Answer(c) ................................................ [2]

www.egyptigstudentroom.com
© UCLES 2015 0580/42/M/J/15
17

(d) n is an integer.

(i) Explain why 2n – 1 is an odd number.

Answer(d)(i) ................................................................................................................................

..................................................................................................................................................... [1]

(ii) Write down, in terms of n, the next odd number after 2n – 1.

Answer(d)(ii) ................................................ [1]

(iii) Show that the difference between the squares of two consecutive odd numbers is a multiple of 8.

Answer(d)(iii)

[3]
__________________________________________________________________________________________

www.egyptigstudentroom.com
© UCLES 2015 0580/42/M/J/15 [Turn over
18

5
10 (a) =c m
-8
(i) Find the value of  .

Answer(a)(i)   = ................................................ [2]

(ii) Q is the point (2, –3).

Find the co-ordinates of the point P.

Answer(a)(ii) (...................... , ......................) [1]

(b)
A

NOT TO
a M SCALE

O N B
b

In the diagram, M is the midpoint of AB and L is the midpoint of OM.


The lines OM and AN intersect at L and ON = 13 OB.
= a and = b.

(i) Find, in terms of a and b, in its simplest form,

(a) ,

Answer(b)(i)(a) = ................................................ [2]

(b) ,

Answer(b)(i)(b) = ................................................ [1]

(c) .

Answer(b)(i)(c) = ................................................ [2]

www.egyptigstudentroom.com
© UCLES 2015 0580/42/M/J/15
19

(ii) Find the ratio AL : AN in its simplest form.

Answer(b)(ii) ................ : ................ [3]

(c)
y
4

3
A
2

1
x
–7 –6 –5 –4 –3 –2 –1 0 1 2 3 4 5
–1

–2

–3
B
–4

–5

(i) On the grid, draw the image of triangle A after the transformation represented by the

matrix f p.
- 1.5 0
[3]
0 -1.5

(ii) Find the 2 × 2 matrix which represents the transformation that maps triangle A onto triangle B.

Answer(c)(ii) f p [2]

__________________________________________________________________________________________

Question 11 is printed on the next page.

www.egyptigstudentroom.com
© UCLES 2015 0580/42/M/J/15 [Turn over
20

11 Gareth has 8 sweets in a bag.


4 sweets are orange flavoured, 3 are lemon flavoured and 1 is strawberry flavoured.

(a) He chooses two of the sweets at random.

Find the probability that the two sweets have different flavours.

Answer(a) ................................................ [4]

(b) Gareth now chooses a third sweet.

Find the probability that none of the three sweets is lemon flavoured.

Answer(b) ................................................ [2]


__________________________________________________________________________________________

Permission to reproduce items where third-party owned material protected by copyright is included has been sought and cleared where possible. Every reasonable
effort has been made by the publisher (UCLES) to trace copyright holders, but if any items requiring clearance have unwittingly been included, the publisher will
be pleased to make amends at the earliest possible opportunity.

To avoid the issue of disclosure of answer-related information to candidates, all copyright acknowledgements are reproduced online in the Cambridge International
Examinations Copyright Acknowledgements Booklet. This is produced for each series of examinations and is freely available to download at www.cie.org.uk after
the live examination series.

Cambridge International Examinations is part of the Cambridge Assessment Group. Cambridge Assessment is the brand name of University of Cambridge Local
Examinations Syndicate (UCLES), which is itself a department of the University of Cambridge.

www.egyptigstudentroom.com
© UCLES 2015 0580/42/M/J/15
Cambridge International Examinations
Cambridge International General Certificate of Secondary Education
* 9 2 0 5 5 1 9 9 7 9 *

maThEmaTICs 0580/43
Paper 4 (Extended) may/June 2015
2 hours 30 minutes
Candidates answer on the Question Paper.
Additional Materials: Electronic calculator Geometrical instruments
Tracing paper (optional).

REaD ThEsE INsTRUCTIONs FIRsT

Write your Centre number, candidate number and name on all the work you hand in.
Write in dark blue or black pen.
You may use an HB pencil for any diagrams or graphs.
Do not use staples, paper clips, glue or correction fluid.
DO NOT WRITE IN ANY BARCODES.

Answer all questions.


If working is needed for any question it must be shown below that question.
Electronic calculators should be used.
If the degree of accuracy is not specified in the question, and if the answer is not exact, give the answer to
three significant figures. Give answers in degrees to one decimal place.
For π, use either your calculator value or 3.142.

At the end of the examination, fasten all your work securely together.
The number of marks is given in brackets [ ] at the end of each question or part question.
The total of the marks for this paper is 130.

The syllabus is approved for use in England, Wales and Northern Ireland as a Cambridge International Level 1/Level 2 Certificate.

This document consists of 16 printed pages.

DC (LK/SW) 101619/2
© UCLES 2015 [Turn over
www.egyptigstudentroom.com
2

1
y

5
U
4

3
T
2
V
1

x
–5 –4 –3 –2 –1 0 1 2 3 4 5 6 7
–1

–2

–3

–4

–5

–6

(a) On the grid, draw the image of

(i) triangle T after a reflection in the line x = –1, [2]

(ii) triangle T after a rotation through 180° about (0, 0). [2]

(b) Describe fully the single transformation that maps

(i) triangle T onto triangle U,

Answer(b)(i) ......................................................................................................................................

...................................................................................................................................................... [2]

(ii) triangle T onto triangle V.

Answer(b)(ii) .....................................................................................................................................

...................................................................................................................................................... [3]

www.egyptigstudentroom.com
© UCLES 2015 0580/43/M/J/15
3

2 (a) (i) Eduardo invests $640 at a rate of 2% per year compound interest.

Show that, at the end of 6 years, Eduardo has $721, correct to the nearest dollar.

Answer(a)(i)

[2]

(ii) Manuela also invests $640.


At the end of 4 years, Manuela has $721.

Find the yearly compound interest rate.

Answer(a)(ii) ............................................. % [4]

(b) Carlos buys a motor scooter for $1200.


Each year the value of the scooter decreases by 10% of its value at the beginning of that year.

Find the value of the scooter after 3 years.

Answer(b) $ ................................................. [2]

www.egyptigstudentroom.com
© UCLES 2015 0580/43/M/J/15 [Turn over
4

8 x
3 f (x) = + , x ! 0.
x2 2
(a) Complete the table of values for f(x).

x –5 –4 –3 –2 –1.5 1.5 2 2.5 3 3.5 4 5

f(x) –2.2 –1.5 –0.6 2.8 4.3 2.5 2.4 2.4 2.8

[3]

(b) On the grid, draw the graph of y = f(x) for –5  x  –1.5 and 1.5  x  5.

x
–5 –4 –3 –2 –1 0 1 2 3 4 5

–1

–2

–3
[5]

www.egyptigstudentroom.com
© UCLES 2015 0580/43/M/J/15
5

(c) Solve f(x) = 0.

Answer(c) x = ................................................. [1]

(d) By drawing a suitable line on the grid, solve the equation f(x) = 1 – x.

Answer(d) x = ................................................. [3]

(e) By drawing a tangent at the point (–3, –0.6), estimate the gradient of the graph of y = f(x) when x = –3.

Answer(e) ................................................. [3]

www.egyptigstudentroom.com
© UCLES 2015 0580/43/M/J/15 [Turn over
6

4 The table shows the times, t minutes, taken by 200 students to complete an IGCSE paper.

Time (t minutes) 40  t  60 60  t  70 70  t  75 75  t  90
Frequency 10 50 80 60

(a) By using mid-interval values, calculate an estimate of the mean time.

Answer(a) .......................................... min [3]

(b) On the grid, draw a histogram to show the information in the table.

20

18

16

14

12
Frequency
10
density
8

t
0 40 50 60 70 80 90
Time (minutes)
[4]

www.egyptigstudentroom.com
© UCLES 2015 0580/43/M/J/15
7

A A A A B B C

(a) One of these 7 cards is chosen at random.

Write down the probability that the card

(i) shows the letter A,


Answer(a)(i) ................................................. [1]

(ii) shows the letter A or B,


Answer(a)(ii) ................................................. [1]

(iii) does not show the letter B.


Answer(a)(iii) ................................................. [1]

(b) Two of the cards are chosen at random, without replacement.

Find the probability that

(i) both show the letter A,

Answer(b)(i) ................................................. [2]

(ii) the two letters are different.

Answer(b)(ii) ................................................. [3]

(c) Three of the cards are chosen at random, without replacement.

Find the probability that the cards do not show the letter C.

Answer(c) ................................................. [2]


__________________________________________________________________________________________

www.egyptigstudentroom.com
© UCLES 2015 0580/43/M/J/15 [Turn over
8

6 (a)

E D
120° 140°
NOT TO
SCALE
F
C

A B

In the hexagon ABCDEF, AB is parallel to ED and AF is parallel to CD.


Angle ABC = 90°, angle CDE = 140° and angle DEF = 120°.

Calculate angle EFA.

Answer(a) Angle EFA = ................................................. [4]

(b)

D
C NOT TO
30°
SCALE

100°
B
A

In the cyclic quadrilateral ABCD, angle ABC = 100° and angle BDC = 30°.
The diagonals intersect at X.

(i) Calculate angle ACB.

Answer(b)(i) Angle ACB = ................................................. [2]

(ii) Angle BXC = 89°.

Calculate angle CAD.

Answer(b)(ii) Angle CAD = ................................................. [2]

(iii) Complete the statement.

Triangles AXD and BXC are ............................................................. . [1]


www.egyptigstudentroom.com
© UCLES 2015 0580/43/M/J/15
9

(c)

S R NOT TO
SCALE

Q
P

P, Q, R and S lie on a circle.


PR and QS intersect at Y.
PS = 11 cm, QR = 10 cm and the area of triangle QRY = 23 cm2.

Calculate the area of triangle PYS.

Answer(c) .......................................... cm2 [2]

(d) A regular polygon has n sides.


n
Each exterior angle is equal to degrees.
10
(i) Find the value of n.

Answer(d)(i) n = ................................................. [3]

(ii) Find the size of an interior angle of this polygon.

Answer(d)(ii) ................................................. [2]

www.egyptigstudentroom.com
© UCLES 2015 0580/43/M/J/15 [Turn over
10

7 (a) The total surface area of a cone is given by the formula A = πrl + πr2.

(i) Find A when r = 6.2 cm and l = 10.8 cm.

Answer(a)(i) .......................................... cm2 [2]

(ii) Rearrange the formula to make l the subject.

Answer(a)(ii) l = ................................................. [2]

(b) (i) Irina walks 10 km at 4 km/h and then a further 8 km at 5 km/h.

Calculate Irina’s average speed for the whole journey.

Answer(b)(i) ........................................ km/h [3]

(ii) Dariella walks x km at 5 km/h and then runs (x + 4) km at 10 km/h.


The average speed of this journey is 7 km/h.

Find the value of x.


Show all your working.

Answer(b)(ii) x = ................................................. [5]

www.egyptigstudentroom.com
© UCLES 2015 0580/43/M/J/15
11

(c) (i) Priyantha sells her model car for $19.80 at a profit of 20%.

Calculate the original price of the model car.

Answer(c)(i) $ .................................................. [3]

(ii) Dev sells his model car for $x at a profit of y %.

Find an expression, in terms of x and y, for the original price of this model car.
Write your answer as a single fraction.

Answer(c)(ii) $ .................................................. [3]

www.egyptigstudentroom.com
© UCLES 2015 0580/43/M/J/15 [Turn over
12

8 (a) A cylindrical tank contains 180 000 cm3 of water.


The radius of the tank is 45 cm. 45 cm
NOT TO
Calculate the height of water in the tank. SCALE

Answer(a) ........................................... cm [2]

(b)

D C NOT TO
70 cm SCALE

40 cm
150 cm
A 50 cm B

The diagram shows an empty tank in the shape of a horizontal prism of length 150 cm.
The cross section of the prism is an isosceles trapezium ABCD.
AB = 50 cm, CD = 70 cm and the vertical height of the trapezium is 40 cm.

(i) Calculate the volume of the tank.

Answer(b)(i) .......................................... cm3 [3]

(ii) Write your answer to part (b)(i) in litres.

Answer(b)(ii) ........................................ litres [1]

(c) The 180 000 cm3 of water flows from the tank in part (a) into the tank in part (b) at a rate of 15 cm3/s.

Calculate the time this takes.


Give your answer in hours and minutes.

Answer(c) ................ h ................ min [3]


www.egyptigstudentroom.com
© UCLES 2015 0580/43/M/J/15
13

(d)
D 70 cm C

x cm NOT TO
F E SCALE
40 cm
h cm

A 50 cm B

The 180 000 cm3 of water reaches the level EF as shown above.
EF = x cm and the height of the water is h cm.

(i) Using the properties of similar triangles, show that h = 2(x – 50).

Answer(d)(i)

[2]

(ii) Using h = 2(x – 50), show that the shaded area, in cm2, is x2 – 2500.

Answer(d)(ii)

[1]

(iii) Find the value of x.

Answer(d)(iii) x = ................................................. [2]

(iv) Find the value of h.

Answer(d)(iv) h = ................................................. [1]

www.egyptigstudentroom.com
© UCLES 2015 0580/43/M/J/15 [Turn over
14

2 3 1 2 0 u w 3
9 P=c m Q=c m R=c m S=c m
1 4 0 3 1 v 8 2
(a) Work out PQ.

Answer(a) f p [2]

(b) Find Q –1.

Answer(b) f p [2]

(c) PR = RP

Find the value of u and the value of v.

Answer(c) u = .................................................

v = ................................................. [3]

(d) The determinant of S is 0.

Find the value of w.

Answer(d) w = ................................................. [2]

www.egyptigstudentroom.com
© UCLES 2015 0580/43/M/J/15
15

2
10 f(x) = 2x – 1 g(x) = x2 + x h (x) = , x!0
x
(a) Find ff(3).

Answer(a) ................................................. [2]

(b) Find gf(x), giving your answer in its simplest form.

Answer(b) ................................................. [3]

(c) Find f –1(x).

Answer(c) f –1(x) = ................................................. [2]

(d) Find h(x) + h(x + 2), giving your answer as a single fraction.

Answer(d) ................................................. [4]

Question 11 is printed on the next page.

www.egyptigstudentroom.com
© UCLES 2015 0580/43/M/J/15 [Turn over
16

11 The first four terms of sequences A, B, C and D are shown in the table.

Sequence 1st term 2nd term 3rd term 4th term 5th term nth term

A 1 2 3 4
3 4 5 6

B 3 4 5 6

C –1 0 1 2

D –3 0 5 12

(a) Complete the table.

[8]
36
(b) Which term in sequence A is equal to ?
37

Answer(b) ................................................. [2]

(c) Which term in sequence D is equal to 725?

Answer(c) ................................................. [2]

Permission to reproduce items where third-party owned material protected by copyright is included has been sought and cleared where possible. Every reasonable
effort has been made by the publisher (UCLES) to trace copyright holders, but if any items requiring clearance have unwittingly been included, the publisher will
be pleased to make amends at the earliest possible opportunity.

To avoid the issue of disclosure of answer-related information to candidates, all copyright acknowledgements are reproduced online in the Cambridge International
Examinations Copyright Acknowledgements Booklet. This is produced for each series of examinations and is freely available to download at www.cie.org.uk after
the live examination series.

Cambridge International Examinations is part of the Cambridge Assessment Group. Cambridge Assessment is the brand name of University of Cambridge Local

www.egyptigstudentroom.com
Examinations Syndicate (UCLES), which is itself a department of the University of Cambridge.

© UCLES 2015 0580/43/M/J/15

You might also like